You are on page 1of 927

Dr black house

If

pauhv

Dr Aspirin
amlodipine

hu

Dr Aly Aly

Dr Sameh
mohamed

DrMahmoud
Saad

Dr Assem
Draz

Fatema & Aya H. Murad


A. H. Murad

PasTest 2014 MRCP I index


Cardiology

300

Dermatology

109

endocrinology

335

Gastrology

293

General revision

134

hematology

300

Infectious disease

309

nephrology

317

neurology

318

Ophthalmology

64

Pharmacology

358

Psychiatry

250

Respiratory

351

rheumatology

345

Clinical science

Cell molecular and membrane biology


Genetics
Immunology
Physiology
Statistic
Anatomy
Biochemistry and metabolism

61
81
96
65
132
45
104

You are here: MyPasTest MRCP 1 Online - Sep Exam 2014 Question Browser

Question Browser

next question

or

back to menu

finish session

Which one of the following arrhythmias is unusual in digoxin toxicity?


O Non-paroxysmal atrial tachycardia with varying block
O First-degree heart block
O Type-ll second-degree heart block
O Wenckebach phenomenon
O Bidirectional ventricular tachycardia

or

Which one of the following arrhythmias is unusual in digoxin toxicity?

Non-paroxysmal atrial tachycardia with varying block


First-degree heart block
Type-ll second-degree heart block CORRECT ANSWER
Wenckebach phenomenon
Bidirectional ventricular tachycardia

Associated arrhythmias
Digoxin toxicity can result in any abnormal cardiac rhythm except type-ll second-degree atrioventricular (AV) block
The characteristic arrhythmia includes non-paroxysmal atrial tachycardia with varying block
Other common arrhythmias include
first-degree heart block
Wenckebach (type-l second-degree block) and complete (third degree) heart block
ventricular ectopics and ventricular tachycardia (VT) (uni- and bi-directional)
Ventricular fibrillation (VF) can also rarely occur.
Another unusual arrhythmia is atrial flutter.

Predisposing factors
Factors predisposing to digitalis toxicity are
advanced age
hypoxia
hypokalemia (most common precipitating factor)
hypomagnesaemia
hypercalcaemia
hypothyroidism
amyloidosis
renal failure

Signs and symptoms


The earliest signs of digitalis toxicity include
nausea
vomiting
anorexia

Chronic toxicity causes


exacerbation of heart failure
weight loss
gynaecomastia
yellow vision

A 34-year-old professional footballer is evaluated for symptoms of 'dizziness' during exercise. Physical

examination reveals a laterally displaced apical impulse. On auscultation, there is a 2/6 mid-systolic murmur in
the aortic area that increases on sudden standing. The ECG shows LVH and Q waves in the V2-V5 leads.

What is the most likely diagnosis?

O Young-onset hypertension
O Acute Ml
O Aortic stenosis
O Hypertrophic cardiomyopathy
Atrial septal defect

A 34-year-old professional footballer is evaluated for symptoms of 'dizziness' during exercise. Physical

examination reveals a laterally displaced apical impulse. On auscultation, there is a 2/6 mid-systolic murmur in
the aortic area that increases on sudden standing. The ECG shows LVH and Q waves in the V2-V5 leads.

What is the most likely diagnosis?

Young-onset hypertension
Acute Ml
Aortic stenosis

Hypertrophic cardiomyopathy CORRECT ANSWER


Atrial septal defect

Adverse effects
Hypertrophic cardiomyopathy is the single most common cause of sudden death in young athletes
Syncope and sudden death are associated with severe exertion and competitive sports, which should be avoided
in patients with hypertrophic cardiomyopathy

Symptoms
The majority of patients are asymptomatic or only mildly symptomatic
The most common symptom is dyspnoea

Examination and investigation findings


Typical examination findings include
left ventricular hypertrophy
a loud S4
forceful atrial systole causing a double apical impulse
a possible triple apical impulse due to a late systolic bulge
the carotid pulse demonstrates a late systolic pulse causing the characteristic jerky feature
In contrast to aortic stenosis, the systolic murmur of hypertrophic cardiomyopathy
does not radiate to the carotids
decreases on squatting and passive leg elevation and increases with the Valsalva manoeuvre

ECG
The most common changes seen on the ECG are ST-T wave abnormalities followed by left ventricular hypertrophy
Q waves may also occur in leads II, III, aVp or V2-V0

The exact cause of the Q waves remains unclear

A 68-year-old woman recently diagnosed with multiple myeloma presents to her GP with

progressively increasing breathlessness, exercise intolerance and ankle swelling. On examination,


there is bilateral pitting leg oedema to her thighs, ascites and raised JVP. The apical impulse is
impalpable. An ECG shows diffusely diminished voltage. Chest X-ray is normal and the
echocardiogram shows small thick ventricles and dilated atria with a thickened interatrial septum.
The ventricular myocardium has a granular sparkling texture on echo, and minimal fluid in the
pericardial space is noted. What is the most likely diagnosis leading to symptoms of cardiac
failure?

O Chronic pericardial effusion with tamponade


O Chronic pericardial effusion without tamponade
Constrictive pericarditis
Q Restrictive cardiomyopathy

Congestive heart failure

A 68-year-old woman recently diagnosed with multiple myeloma presents to her GP with

progressively increasing breathlessness, exercise intolerance and ankle swelling. On examination,


there is bilateral pitting leg oedema to her thighs, ascites and raised JVP. The apical impulse is

impalpable. An ECG shows diffusely diminished voltage. Chest X-ray is normal and the
echocardiogram shows small thick ventricles and dilated atria with a thickened interatrial septum.
The ventricular myocardium has a granular sparkling texture on echo, and minimal fluid in the
pericardial space is noted. What is the most likely diagnosis leading to symptoms of cardiac
failure?

Chronic pericardial effusion with tamponade


Chronic pericardial effusion without tamponade
Constrictive pericarditis

Restrictive cardiomyopathy CORRECT ANSWER


Congestive heart failure

Aetiology
Restrictive cardiomyopathy can develop secondary to amyloidosis associated with an immunocyte dyscrasia

Examination and investigation findings


Physical examination reveals right heart failure with a raised JVP, characteristically showing a prominent deep Y
descent

The heart size is often normal.


The physical findings are very similar in constrictive pericarditis (CCP), but the apex is frequently non-palpable due
to the thick pericardium

The chest X-ray may show pericardial calcifications in patients with constrictive pericarditis
Pericardial effusion is common, but rarely causes tamponade
ECG
The most characteristic ECG finding of restrictive cardiomyopathy is diffusely diminished voltages
Echocardiography typically shows small thick ventricles and a thick interatrial septum due to amyloid deposits,
which have a 'granular sparkling' appearance

Adverse effects
Cardiac involvement is the most common cause of death in patients with amyloidosis associated with an
immunocyte dyscrasia - typically as restrictive cardiomyopathy

The epsilon potential is seen on the ECG of patients with which one of the following?

O Hypertrophic cardiomyopathy
Restrictive cardiomyopathy
O Right ventricular dysplasia

O Romano Ward syndrome


O Digoxin toxicity

The epsilon potential is seen on the ECG of patients with which one of the following?

Hypertrophic cardiomyopathy
Restrictive cardiomyopathy

Right ventricular dysplasia CORRECT ANSWER


Romano Ward syndrome

Digoxin toxicity

Right ventricular dysplasia


The epsilon potential is a right-ventricular conduction delay, and appears as a sharp deflection after termination of
the QRS complex during the ST segment or upstroke of the T wave
It is seen in the right ventricular leads, V<i and V2

Fontaine named the waves 'epsilon' since epsilon follows delta in the Greek alphabet
Right ventricular dysplasia is characterised by the displacement of myocytes by fat
This delays the excitation and depolarisation of those viable myocytes enveloped by the fatty tissue, and so leads
to epsilon potentials

A 68-year-old man, although asymptomatic from the cardiac viewpoint, has an ejection systolic murmur best
heard in the aortic area. The murmur radiates to the carotids. Echocardiography confirms severe aortic stenosis
with a gradient of 85 mmHg across the calcified aortic valve.

What is the risk of sudden cardiac death per year in such patients?
O <5%
6-9%
O 10-25%

25-50%

O More than 75%

A 68-year-old man, although asymptomatic from the cardiac viewpoint, has an ejection systolic murmur best
heard in the aortic area. The murmur radiates to the carotids. Echocardiography confirms severe aortic stenosis
with a gradient of 85 mmHg across the calcified aortic valve.

What is the risk of sudden cardiac death per year in such patients?
<5% CORRECT ANSWER
6-9%
10-25%
25-50%
More than 75%

Aortic stenosis
The risk of cardiac death in patients with symptomatic and asymptomatic aortic stenosis according to valve gradient is
calculated using data from epidemiological studies.

Development and prognosis


The natural history of aortic stenosis in adults is characterised by a long latent period, during which there is a
gradually increasing obstruction and an increase in the pressure load on the myocardium while the patient remains
asymptomatic

A gradient of 85 mmHg constitutes a very severe aortic stenosis


Once symptoms appear in patients with an unrelieved obstruction, the prognosis is poor
Although aortic stenosis may be responsible for sudden death, this usually occurs in patients who have previously
been symptomatic

Survival curves have shown that the interval from the onset of symptoms to the time of death is approximately two
years in patients with heart failure, three years in those with syncope and five years in those with angina

Incidence of sudden cardiac death


Before the advent of surgery, sudden cardiac death was quite common in cases of aortic stenosis (in 1968,
Campbell reported that, out of 60 patients with aortic stenosis who died, 44 (73%) of the deaths were sudden)
There is a 6-9% incidence of sudden cardiac death in asymptomatic children with aortic stenosis

A 32-year-old man is brought to the Emergency Department in a collapsed state having sustained a

precordial stab wound. Which of the following cardiac valves is most likely to have been injured?

O Aortic valve
O Tricuspid valve
O Pulmonary valve
O Mitral valve
O Thebesian valve

A 32-year-old man is brought to the Emergency Department in a collapsed state having sustained a

precordial stab wound. Which of the following cardiac valves is most likely to have been injured?

Aortic valve
Tricuspid valve CORRECT ANSWER

Pulmonary valve
Mitral valve
Thebesian valve

Precordial stab wound


The tricuspid valve is the most anterior valve of the human heart and is the most common to be injured during a
stabbing attack

Penetrating injuries may cause lacerations to any of the heart chambers or great vessels, and death may result
from haemorrhage or cardiac tamponade

Late complications include infective pericarditis, valve damage or intracardiac shunts


Echocardiography is useful in diagnosing the underlying problem

A 30-year-old woman with a previous history of deep vein thrombosis is expecting her first child. During which
phase of her pregnancy and puerperium does she have the greatest risk of venous thrombosis?

O First trimester
Second trimester

O Third trimester
During delivery
First 6 weeks after delivery

A 30-year-old woman with a previous history of deep vein thrombosis is expecting her first child. During which

phase of her pregnancy and puerperium does she have the greatest risk of venous thrombosis?

First trimester
Second trimester

Third trimester

During delivery
First 6 weeks after delivery CORRECT ANSWER

Deep vein thrombosis in pregnancy


Thromboembolic complications
There is an increase in thromboembolic complications in the mother in the first 6 weeks after delivery because of
the hypercoagulability that exists postpartum

Management
Anticoagulants may be necessary during pregnancy to prevent or control the following
venous thrombosis
pulmonary embolism
rheumatic mitral valve disease
prosthetic heart valves
peripartum cardiomyopathy
primary pulmonary hypertension
Eisenmenger syndrome

A 28-year-old woman who is known to have a cardiac murmur becomes pregnant. It is noted that the intensity of

her murmur diminishes during her pregnancy.

Which cardiac abnormality is she likely to have?

O Aortic stenosis
O Aortic regurgitation

O Tricuspid stenosis
O Pulmonary stenosis
O Fallot's tetralogy

A 28-year-old woman who is known to have a cardiac murmur becomes pregnant. It is noted that the intensity of

her murmur diminishes during her pregnancy.

Which cardiac abnormality is she likely to have?

Aortic stenosis

Aortic regurgitation CORRECT ANSWER


Tricuspid stenosis

Pulmonary stenosis
Fallot's tetralogy

Aortic regurgitation
Changes in pregnancy
During pregnancy, cardiac output and blood volume increase from the second month up to the thirtieth week to 3050% above the normal levels

The average increase in blood volume during pregnancy amounts to 1600 ml, and there is also an increased
metabolic workload

Symptoms

This produces
warm extremities
a tachycardia with a large-volume pulse
a slight rise in venous pressure

Examination findings
Diastolic blood pressure is lower due to vasodilatation, and this is responsible for the fading of the aortic
regurgitation murmur

The apex beat is displaced, because of cardiomegaly and a raised diaphragm


The increased blood flow may produce a pulmonary systolic murmur and a third heart sound
All stenotic murmurs become more prominent

A 40-year-old woman is admitted with a stroke after a prolonged pyrexial illness. Her GP was

unclear as to the potential cause of her fevers and had prescribed a course of oral Co-amoxiclav in
the expectation it would cover both respiratory and urinary pathogens. On examination she is in
sinus rhythm, has splenomegaly and a pansystolic murmur at the apex. Blood cultures confirm an

infective endocarditis. Which one of the following is the most common causative organism?

O Streptococcus viridans
Staphylococcus aureus

O Streptococcus bovis
Gram-negative bacilli
O Staphylococcus epidermidls

A 40-year-old woman is admitted with a stroke after a prolonged pyrexial illness. Her GP was

unclear as to the potential cause of her fevers and had prescribed a course of oral Co-amoxiclav in

the expectation it would cover both respiratory and urinary pathogens. On examination she is in
sinus rhythm, has splenomegaly and a pansystolic murmur at the apex. Blood cultures confirm an
infective endocarditis. Which one of the following is the most common causative organism?

Streptococcus viridans CORRECT ANSWER


Staphylococcus aureus

Streptococcus bovis
Gram-negative bacilli

Staphylococcus epidermidls

Prevalences of organisms
The prevalences of organisms causing infective endocarditis are shown in the table below
Streptococci

Staphylococci

Viridans group

30-40%

Enterococci

10-15%

Other

20-25%

Staphylococcus aureus 9-27%


Coagulase-negative

1-3%

Gram-negative bacilli Haemophilus spp.

3-8%

Anaerobes

less than 2%

Rickettsia/fungi

Aetiology
Members of the viridans group of streptococci are the commonest cause of subacute endocarditis on native
valves; these commensals of the upper respiratory tract may enter the bloodstream on chewing, tooth brushing or
at the time of dental treatment
Staphylococcus aureus is a common cause of acute endocarditis, originating from skin infections, abscesses,
vascular access sites or intravenous drug misuse
Staphylococcus epidermidis is the most common organism causing postoperative endocarditis following cardiac
surgery

A 38-year-old woman is seen in the Emergency Department with a history of collapse. She recalls

rushing for the bus before feeling faint. Her brother recently died suddenly owing to a heart
problem. On examination she has a 'jerky' pulse, a thrusting cardiac impulse and a mid-systolic

murmur. Which one of the following is the likely diagnosis?


O Dilated cardiomyopathy
O Hypertrophic cardiomyopathy
O Mitral valve prolapse
O Aortic stenosis

0 Pericarditis

A 38-year-old woman is seen in the Emergency Department with a history of collapse. She recalls

rushing for the bus before feeling faint. Her brother recently died suddenly owing to a heart
problem. On examination she has a 'jerky' pulse, a thrusting cardiac impulse and a mid-systolic

murmur. Which one of the following is the likely diagnosis?


Dilated cardiomyopathy

Hypertrophic cardiomyopathy CORRECT ANSWER


Mitral valve prolapse
Aortic stenosis

Pericarditis

Hypertrophic cardiomyopathy
The age of the patient (38) and her family history (a brother who died suddenly as a result of a heart problem)
make hypertrophic cardiomyopathy (HCM) the likely diagnosis
HCM is the commonest form of cardiomyopathy, with a prevalence of about 100 per 100 000
It is a genetic disorder with autosomal-dominant transmission, a high degree of penetrance and variable
expression

Symptoms and signs


Symptoms and signs are similar to those of aortic stenosis, except that the character of the pulse in HCM is jerky
Sudden death can be a presenting symptom of HCM

It typically occurs during or just after vigorous physical activity


Risk factors for sudden death

Risk factors for sudden death in HCM are:


a history of previous cardiac arrest or sustained ventricular tachycardia
recurrent syncope
an adverse genotype and/or family history
exercise-induced hypotension
multiple episodes of non-sustained ventricular tachycardia on ambulatory ECG
a marked increase in the thickness of the left ventricular wall

A 58-year-old-woman suffers a cardiac arrest while on the ward. A rhythm strip shows VF.

What is the strength (in joules) recommended for the monophasic shock used for defibrillation?
O 50 J

100 J
200 J
300 J
360 J

A 58-year-old-woman suffers a cardiac arrest while on the ward. A rhythm strip shows VF.

What is the strength (in joules) recommended for the monophasic shock used for defibrillation?
50 J
100 J

200 J
300 J
360 J CORRECT ANSWER

Cardiac arrest
Aetiology
Three-quarters of cardiac arrests are due to ventricular fibrillation
Only a small proportion is due to pulseless electrical activity (PEA); the rest are due to asystole
PEA may have a potentially reversible cause:
hypovolaemia
hypoxia
hyperkalemia
hypokalaemia
hypothermia
tension pneumothorax
tamponade
toxicity due to drugs
thromboembolism

Defibrillation
Defibrillation is used to convert ventricular fibrillation to sinus rhythm
The recommendation is initially a 360-joule shock

A 42-year-old-man, known to be hypertensive, ran out of his medication 2 days ago. He presented

to the Emergency Department feeling short of breath and dizzy. His blood pressure on admission
was 230/140 mmHg. Fundoscopy showed blurred disc margins. His chest revealed bibasal
crepitations. With therapy, what blood pressure should you aim for in the next 1 hour in such

cases?
< 130/80 mmHg

O < 140/90 mmHg


O Decrease in mean arterial pressure (MAP) by 50%
Decrease in MAP by 25%
O Decrease in MAP by 75%

A 42-year-old-man, known to be hypertensive, ran out of his medication 2 days ago. He presented

to the Emergency Department feeling short of breath and dizzy. His blood pressure on admission

was 230/140 mmHg. Fundoscopy showed blurred disc margins. His chest revealed bibasal
crepitations. With therapy, what blood pressure should you aim for in the next 1 hour in such
cases?
< 130/80 mmHg
< 140/90 mmHg

Decrease in mean arterial pressure (MAP) by 50%


Decrease in MAP by 25% CORRECT ANSWER

Decrease in MAP by 75%

Hypertensive emergency
The patient has a hypertensive emergency with markedly elevated blood pressure and evidence of target organ
damage; this situation requires immediate attention to prevent disability or death
Here, the aim is to reduce the blood pressure promptly but partially, to prevent end-organ damage without
compromising tissue perfusion
The initial target is to lower the mean arterial pressure (MAP) by no more than 25%, or reduce the diastolic blood

pressure by one-third
MAP = diastolic blood pressure + [(systolic blood pressure - diastolic blood pressure )/3]
Even in the presence of heart failure or hypertensive encephalopathy, a controlled reduction, to a level of about
150/90 mmHg, over a period of 24-36 hours is ideal

In most patients, blood pressure can be brought down with bed rest and oral medication
Intravenous labetalol (2 mg/min to a maximum of 200 mg), intravenous glyceryl trinitrate (0.6-1.2 mg/h),
intravenous sodium nitroprusside (0.3-1.0 mg/kg per min) and intramuscular hydralazine (5 or 10 mg repeated at
half-hourly intervals) are all effective but require close monitoring

A 30-year-old-man presents to the outpatient clinic with a 2-month history of progressive effort intolerance. Some
three weeks ago he experienced an episode of shortness of breath at rest, suggestive of paroxysmal nocturnal
dyspnoea. Examination reveals a JVP raised up to his earlobes, a soft tender hepatomegaly and a bilateral pitting
oedema up to his knees. Chest examination reveals bibasal crepitations, and an audible S3 on auscultation of
the heart. The chest X-ray shows cardiomegaly with interstitial infiltrates. Echocardiography shows global left
ventricular hypokinesia with an ejection fraction of 25-30%.

Which one of the following is the least likely aetiological factor?

O Alcohol abuse
O Genetic factor
O Adenovirus
O Eosinophilic states
O HIV infection

A 30-year-old-man presents to the outpatient clinic with a 2-month history of progressive effort intolerance. Some
three weeks ago he experienced an episode of shortness of breath at rest, suggestive of paroxysmal nocturnal
dyspnoea. Examination reveals a JVP raised up to his earlobes, a soft tender hepatomegaly and a bilateral pitting
oedema up to his knees. Chest examination reveals bibasal crepitations, and an audible S3 on auscultation of
the heart. The chest X-ray shows cardiomegaly with interstitial infiltrates. Echocardiography shows global left
ventricular hypokinesia with an ejection fraction of 25-30%.

Which one of the following is the least likely aetiological factor?


Alcohol abuse

Genetic factor

Adenovirus

Eosinophilic states CORRECT ANSWER


HIV infection

Dilated cardiomyopathy
Dilated cardiomyopathy encompasses a heterogeneous group of conditions

Aetiological factors

Alcohol abuse is an important aetiological factor in a significant number of patients


About 25% of cases are inherited as an autosomal-dominant trait

A substantial group is due to a late autoimmune reaction to viral myocarditis


Up to 10% of patients with advanced HIV infection develop dilated cardiomyopathy

Other notes
Eosinophilic states are associated with obliterative cardiomyopathy

A 35-year-old-woman gives a history of progressive exertional dyspnoea and fatigue over the last year.
Examination reveals features of right-sided heart failure with pulmonary hypertension, but there are no crackles
to suggest fibrosis. Pulmonary function testing rules out obstructive airways disease. Lung perfusion scanning
and pulmonary angiography fail to detect pulmonary thromboembolic disease. An echocardiogram shows
enlarged right heart chambers. ANCA testing is negative.

Which one of the following is the likely diagnosis?

O Pulmonary vasculitis
O Mitral valve prolapse
O Primary pulmonary hypertension
O Mitral stenosis

0 Dilated cardiomyopathy

A 35-year-old-woman gives a history of progressive exertional dyspnoea and fatigue over the last year.

Examination reveals features of right-sided heart failure with pulmonary hypertension, but there are no crackles
to suggest fibrosis. Pulmonary function testing rules out obstructive airways disease. Lung perfusion scanning
and pulmonary angiography fail to detect pulmonary thromboembolic disease. An echocardiogram shows
enlarged right heart chambers. ANCA testing is negative.

Which one of the following is the likely diagnosis?

Pulmonary vasculitis
Mitral valve prolapse
Primary pulmonary hypertension CORRECT ANSWER

Mitral stenosis
Dilated cardiomyopathy

Pulmonary hypertension
Pulmonary hypertension can be subdivided into primary (idiopathic) and secondary types

Primary pulmonary hypertension


Primary pulmonary hypertension is a diagnosis of exclusion where no cause of pulmonary hypertension is
discernible

A number of theories have been put forward to explain the origin of primary pulmonary hypertension; none has yet
gained ascendancy

Secondary pulmonary hypertension


Secondary pulmonary hypertension can be caused by:
vasoconstriction due to chronic hypoxia
loss of pulmonary vessels owing to chronic obstructive pulmonary disease, pulmonary fibrosis or vasculitis
pulmonary thromboembolic disease
congenital left-to-right shunting
left heart failure
mitral valve disease
Investigations here largely rule out valve disease or significant cardiac dysfunction, making primary pulmonary
hypertenstion the most likely diagnosis.

A 78-year-old-man presents to the Emergency Department with a history of syncope. An ECG shows complete

heart block.

Which one of the following physical signs is consistent with the diagnosis?

O Cannon 'a' waves on JVP at regular intervals


Soft first heart sound

O Low-volume pulse
Basal systolic murmur

O Loud second heart sound

A 78-year-old-man presents to the Emergency Department with a history of syncope. An ECG shows complete

heart block.

Which one of the following physical signs is consistent with the diagnosis?
Cannon 'a' waves on JVP at regular intervals
Soft first heart sound

Low-volume pulse
Basal systolic murmur CORRECT ANSWER

Loud second heart sound

Complete heart block

Complete heart block produces a slow regular pulse (25-50 bpm) that does not vary with exercise
Usually, there is a compensatory increase in stroke volume with a large-volume pulse and systolic flow murmurs
Cannon 'a' waves are irregularly seen

The intensity of the first and second heart sound varies owing to the loss of atrioventricular synchrony

Which one of the following statements is true regarding pulsus alternans?


O It is found in beriberi heart disease

The pulse is irregular


It is diagnosed electrocardiographically
It is found in association with a third heart sound
It is found in patients with a small pericardial effusion

Which one of the following statements is true regarding pulsus alternans?

It is found in beriberi heart disease


The pulse is irregular

It is diagnosed electrocardiographically
It is found in association with a third heart sound CORRECT ANSWER
It is found in patients with a small pericardial effusion

Pulsus alternans

Pulsus alternans is found in patients with acute left ventricular failure


Alternate weak and strong pulses occur, which are regularly placed
A pathological third heart sound is usually associated

The condition may be associated with heart failure or pericarditis for example, but is not usually seen in
association with pericardial effusion
Beri-beri heart disease leads to long QT, T wave inversion and low voltage complexes

Electricalalternans
Electrical alternans is diagnosed electrocardiographically
The amplitude of QRS complexes varies alternately
It occurs owing to changes in electrical depolarisation, conduction abnormalities or cardiac motion

A 70-year-old-man reverts to atrial fibrillation after several attempts at cardioversion, but remains

symptomatic despite rate control with digoxin and metoprolol. He developed pulmonary fibrosis
with amiodarone. Which of the following will be the next step in the management of this patient?
Switch metoprolol to amlodipine

O Double the dose of digoxin

O Radiofrequency pulmonary vein isolation with ablation


Make another attempt at cardioversion

O Implant a cardioverter defibrillator

A 70-year-old-man reverts to atrial fibrillation after several attempts at cardioversion, but remains

symptomatic despite rate control with digoxin and metoprolol. He developed pulmonary fibrosis
with amiodarone. Which of the following will be the next step in the management of this patient?
Switch metoprolol to amlodipine

Double the dose of digoxin


Radiofrequency pulmonary vein isolation with ablation CORRECT ANSWER
Make another attempt at cardioversion

Implant a cardioverter defibrillator

Management of atrial fibrillation


It is not always possible to restore and maintain sinus rhythm in patients with atrial fibrillation (AF)
If sinus rhythm cannot be maintained, treatment should be directed towards controlling the heart rate with digoxin,
(3-blockers, rate-limiting calcium-channel blockers (verapamil or diltiazem) or amiodarone

Beta blockers and calcium-channel blockers are often more effective than digoxin at controlling the heart rate
during exercise

For patients who remain poorly controlled despite medical therapy, radiofrequency pulmonary vein isolation with
ablation is now seen as the treatment of choice for atrial fibrillation cessation

A 35-year-old woman of African origin presents with a 4-month history of increasing swelling over

her feet and abdominal distension. She has no history of cough, orthopnoea or breathlessness on
exertion. Her heart rate is 98 beats per minute: irregularly irregular. Her JVP is markedly raised and

she has pitting lower limb oedema. The heart sounds are soft, and there are no audible murmurs.
Abdominal examination reveals hepatomegaly along with ascites. Chest X-ray reveals a normal
cardiac size and clear lung fields. A lateral X-ray shows calcification around the heart border.
Urinalysis is normal. Her ECG shows a low QRS voltage and lateral T-wave changes. What is the
likely diagnosis?

O Dilated cardiomyopathy
O Cirrhosis of the liver

q Constrictive pericarditis
O Restrictive cardiomyopathy
O Hypertrophic cardiomyopathy

A 35-year-old woman of African origin presents with a 4-month history of increasing swelling ovei

her feet and abdominal distension. She has no history of cough, orthopnoea or breathlessness or
exertion. Her heart rate is 98 beats per minute: irregularly irregular. Her JVP is markedly raised anc
she has pitting lower limb oedema. The heart sounds are soft, and there are no audible murmurs.
Abdominal examination reveals hepatomegaly along with ascites. Chest X-ray reveals a normal
cardiac size and clear lung fields. A lateral X-ray shows calcification around the heart border.
Urinalysis is normal. Her ECG shows a low QRS voltage and lateral T-wave changes. What is the
likely diagnosis?
Dilated cardiomyopathy
Cirrhosis of the liver
Constrictive pericarditis CORRECT ANSWER
Restrictive cardiomyopathy

Hypertrophic cardiomyopathy

Constrictive pericarditis
Diagnosis
This patient has signs of severe right heart failure but the chest X-ray reveals a normal heart size
The possibilities are constrictive pericarditis and restrictive cardiomyopathy
The presence of calcification around the heart favours constrictive pericarditis

Other notes
Causes of restrictive cardiomyopathy include cardiac amyloidosis, haemachromatosis, endomyocardial fibrosis,
systemic sclerosis, carcinoid syndrome and malignancy

Cardiac amyloidosis is usually associated with myeloma


It is more common in males in their sixth or seventh decades

Which one of the following is the commonest cardiovascular abnormality seen in an adult patient
with Marfan syndrome?

O Aortic regurgitation
O Aortic root dilatation
Mitral regurgitation

Q Mitral annular calcification

Aortic dissection

Which one of the following is the commonest cardiovascular abnormality seen in an adult patient
with Marfan syndrome?

Aortic regurgitation
Aortic root dilatation CORRECT ANSWER

Mitral regurgitation

Mitral annular calcification


Aortic dissection

Marfan syndrome
Marfan syndrome is a connective tissue disorder that is inherited as an autosomal-dominant trait
There is considerable variation in its clinical manifestations
Systems involved

Ocular
dislocation of the lens
Skeletal
arachnodactyly

joint hypermobility
scoliosis
chest deformity
high-arched palate
Cardiovascular systems
aortic root dilatation (70%)
mitral valve prolapse (60%)

Complications
Weakening of the aortic media leads to a fusiform ascending aortic aneurysm, which may be complicated by aortic
regurgitation and aortic dissection

Mitral regurgitation can result from mitral valve prolapse, dilatation of a mitral valve annulus or mitral annular
calcification

Pregnancy is particularly hazardous

Treatment
Treatment with (3-blockers reduces the rate of aortic dilatation and the risk of rupture

An elderly man is admitted to the ICU and put on intermittent positive-pressure ventilation. Which
one of the following statements is true when compared with spontaneous ventilation?

O Lung volumes are decreased

Pulmonary vascular resistance is decreased

O Systemic blood pressure rises


O Venous return and cardiac output fall
O Intrathoracic pressure is decreased

An elderly man is admitted to the ICU and put on intermittent positive-pressure ventilation. Which
one of the following statements is true when compared with spontaneous ventilation?

Lung volumes are decreased


Pulmonary vascular resistance is decreased
Systemic blood pressure rises
Venous return and cardiac output fall CORRECT ANSWER
Intrathoracic pressure is decreased

Intermittent positive-pressure ventilation


Effects of increased lung volume
During intermittent positive-pressure ventilation (IPPV), lung volumes are significantly increased when compared
with spontaneous ventilation
A large tidal volume causes a rise in pulmonary vascular resistance, which may lead to
pulmonary hypertension
right ventricular compromise.

The over-inflated alveoli cause compression of the alveolar blood vessels


Moreover, the resultant increase in right ventricle volume may impede left ventricle (LV) filling (ventricular
interdependence)

Hyperinflation also leads to prostaglandin release which may be a protective mechanism against lung injury

Effects of increasedintrathoracic pressure


The intrathoracic pressure is increased at all points in the respiratory cycle
Inspiration during IPPV increases intrathoracic pressure and so increases right atrial pressure relative to
atmospheric pressure, therefore leading to decreased venous return
The increased intrathoracic pressure also decreases the gradient across the LV that it has to work against, which
results in a decreased afterload. Both these effects reduce intrathoracic blood volume

A 20-year-old-man attends the Emergency Department with palpitations described as 'regular rapid

beating of the heart'. An ECG shows a regular rhythm with a rate of 200 beats per minute and a
QRS duration of 80 ms. The tachycardia spontaneously resolves. An ECG in sinus rhythm reveals a
PR interval of 60 ms and a QRS duration of 120 ms, with a positive delta in V-j. Which one of the
following statements is true regarding this man's tachycardia?

Carotid sinus massage will be ineffective

O Intravenous adenosine is of no use


O Digoxin should be used as a prophylactic agent
Verapamil is contraindicated

O Atrial fibrillation is well tolerated in such patients

A 20-year-old-man attends the Emergency Department with palpitations described as 'regular rapid

beating of the heart'. An ECG shows a regular rhythm with a rate of 200 beats per minute and a

QRS duration of 80 ms. The tachycardia spontaneously resolves. An ECG in sinus rhythm reveals a
PR interval of 60 ms and a QRS duration of 120 ms, with a positive delta in V-j. Which one of the
following statements is true regarding this man's tachycardia?

Carotid sinus massage will be ineffective

Intravenous adenosine is of no use

Digoxin should be used as a prophylactic agent


Verapamil is contra indicated CORRECT ANSWER
Atrial fibrillation is well tolerated in such patients

Wolff-Parkinson-White syndrome
This man has type A Wolff-Parkinson-White (WPW) syndrome, in which an abnormal band of atrial tissue
connects the atria to the ventricle, bypassing the atrioventricular (AV) node.

As the AV node and bypass tract have different conduction speeds and refractory periods, a re-entry circuit can be
formed that results in tachycardia

Treatment
Carotid sinus massage or intravenous adenosine will often terminate an episode of tachycardia, although
adenosine is usually avoided because of the risk of precipitating VF in a small percentage of patients with WPW
Prophylactic drug therapy is indicated for symptomatic patients
Agents used in an attempt to slow the conduction rate and prolong the refractory period of the bypass tract are

flecainide
disopyramide
amiodarone
Digoxin and verapamil are contraindicated as they increase conduction in the bypass tract
Atrial fibrillation is poorly tolerated and is a life-threatening arrhythmia in such patients
the bypass tract lacks the rate-limiting properties of the normal AV node and ventricular fibrillation ensues
This is treated as a medical emergency with DC cardioversion

A 22-year-old woman presents to the Emergency Department with a 4-day history of chest pain. She

has been unwell with an influenza-like illness for the last week. The ECG shows widespread ST
elevation in the inferior, anterior and lateral leads. Which one of these ECG changes would you

expect to see in the next week or two?


Development of deep Q waves in all leads

ST depression in inferior and lateral leads

T-wave inversion in all leads CORRECT ANSWER


Tall and peaked T waves in all leads

Loss of R waves in all leads

Interpreting an ECG

The ECG changes in acute pericarditis consist of ST elevation with concavity upward in all leads facing the epicardial
surface, ie anterior inferior and lateral.
Only 'cavity' leads avr, V-j and, rarely, V2 show ST depression.
This is followed by the return of ST segments to baseline and flat T waves.
T waves then become inverted without the loss of R waves or development of Q waves.
As the illness improves, T waves become normal but may occasionally persist in patients with chronic pericarditis.
Rhythm and conduction abnormalities are not typical in pericarditis unless the myocardium is involved.

Causes of acute idiopathic pericarditis:

Infections
viral infections (eg coxsackievirus B)
tuberculosis
other bacteria
fungi

Inflammatory; post-myocardial infarction/cardiotomy


autoimmune rheumatic disorder

Others
neoplasia
uraemia

trauma

aortic dissection
hypothyroidism
irradiation
drugs, eg hydralazine

Which one of the following statements is true of raised cardiac troponin levels in the blood?

O Are commonly seen to levels above that for Ml diagnosis after DC cardioversion
Remain elevated for up to two days after myocardial damage

O Are seen in patients with NSTEMI


Can be used to distinguish non-Q from Q Ml

O Can be found in patients with hypertrophic cardiomyopathy

Which one of the following statements is true of raised cardiac troponin levels in the blood?

Are commonly seen to levels above that for Ml diagnosis after DC cardioversion
Remain elevated for up to two days after myocardial damage
Are seen in patients with NSTEMI CORRECT ANSWER
Can be used to distinguish non-Q from Q Ml
Can be found in patients with hypertrophic cardiomyopathy

Cardiac troponins

The most sensitive markers of myocardial cell damage are the cardiac troponins T and I
Troponins are regulatory elements of the contractile apparatus in muscle
They are released within 4-6 hours of myocardial cell damage, and remain elevated for up to two weeks
Unstable angina and ST segment myocardial infarction are different ends of the spectrum of myocardial damage,

with greater rises in troponin seen towards the more severe end of the spectrum
Angina associated with a troponin rise is essentially an NSTEMI
Cardioversion can give rise to raised creatine kinase (CK) levels owing to skeletal muscle damage
Non-Q and Q myocardial infarction are diagnosed using ECG

Indications of raised troponin levels


About 30% of patients with unstable angina have elevated troponin levels, and these patients have a poorer
prognosis

Raised troponin levels indicate heart muscle damage: the commonest cause being ischaemic damage
Myocarditis or myocardial contusion can also cause raised troponin levels

A 32-year-old woman was cross-country skiing when she fell down a water-filled gully and became

trapped beneath an ice-sheet. Frantic efforts were made to extract her, but after 40 minutes all

movements ceased, just before she was extracted. Which one of the following statements is true?

Her pulseless state is, in all likelihood, due to ventricular fibrillation

O Defibrillation at the scene is likely to succeed


O Metabolic alkalosis will quickly set in
Aspirated seawater is less likely to produce pulmonary oedema than fresh water

O It is important to lift her out of water in the prone position if possible

A 32-year-old woman was cross-country skiing when she fell down a water-filled gully and became

trapped beneath an ice-sheet. Frantic efforts were made to extract her, but after 40 minutes all

movements ceased, just before she was extracted. Which one of the following statements is true?

Her pulseless state is, in all likelihood, due to ventricular fibrillation


Defibrillation at the scene is likely to succeed
Metabolic alkalosis will quickly set in

Aspirated seawater is less likely to produce pulmonary oedema than fresh water
It is important to lift her out of water in the prone position if possible CORRECT ANSWER

Cold-water drowning
Management
The management of patients who nearly drown in cold water is quite different from that for routine cardiopulmonary
arrests

Re-warming such patients should be undertaken in a hospital that has extracorporeal re-warming facilities
Head-out upright immersion in water at body temperature results in a 32-66% increase in cardiac output because
of the pressure of the surrounding water

Resistance to circulation is suddenly removed as the person leaves the water, which when added to venous
pooling, can cause circulatory collapse

This is believed to be the cause of death in many individuals


To counter this effect, patients should be lifted out of the water in the prone position
Continuous chest compression should be applied throughout transportation, which is as effective as chest
compression with expired air resuscitation
Because hypothermia may render the carotid pulse impalpable, it is important to commence chest compression
with firm evidence of cardiac arrest
Electrocardiographic monitoring should be available
Defibrillation is ineffective if the myocardium is cold

Complications
Metabolic acidosis can develop and should be corrected with adequate oxygenation and plasma expansion

Reference
www.bmj.com/content/327/7427/1336.full.pdf

A 79 year-old man known to have chronic congestive heart failure is readmitted with worsening heart failure. His

furosemide (frusemide) dosage is increased to 200 mg/day to aid the relief of his symptoms. His other
medications are bendrofluazide, ramipril and bisoprolol.

Which one of the following effects is most likely to be encountered?

O Hyperkalaemia
O Hypercalciuria
O Hypermagnesaemia
O Hyperuricaemia

q Hypoalbuminaemia

A 79 year-old man known to have chronic congestive heart failure is readmitted with worsening heart failure. His

furosemide (frusemide) dosage is increased to 200 mg/day to aid the relief of his symptoms. His other
medications are bendrofluazide, ramipril and bisoprolol.

Which one of the following effects is most likely to be encountered?

Hyperkalaemia
Hypercalciuria
Hypermagnesaemia
Hyperuricaemia CORRECT ANSWER

Hypoalbuminaemia

Use of diuretics

Tubular handling of uric acid is complex, with both reabsorption and secretion occurring in the proximal tubule
Diuretics can interfere with either of these processes thereby causing hyperuricaemia
The effect is usually dose-dependent and frequently asymptomatic
Clinical gout is more likely if the patient is also extracellular fluid volume-depleted

Diuretics can cause various other electrolyte disturbances, eg hyponatraemia, hypomagnesaemia and
hypokalaemia

Metabolic alkalosis results from chloride loss and extracellular fluid volume contraction
Extracellular volume depletion and raised urea concentrations can occur owing to renal hypoperfusion
Typically, however, serum urea concentrations are unchanged or do not increase by more than 10-20% of
baseline values

Side-effects
Ototoxicity is an uncommon side-effect occurring with high-dose loop diuretic therapy. It has been noted in patients
with renal failure who are given high-dose (> 2 g/day) infusion therapy

A 50-year-old professor of economics presents with a 6-week history of progressive breathlessness

and bilateral ankle swelling. The ECG shows inverted p in

and partial LBBB. Echocardiography

confirms dilated cardiomyopathy. Which one of the following statements is true?

O There is no relevance of history of alcohol abuse.


O A family history of a similar problem is unrelated

Past cytotoxic drug therapy is relevant


O Viral illness in the past is unrelated
O History of pulmonary tuberculosis in the past is relevant

A 50-year-old professor of economics presents with a 6-week history of progressive breathlessness

and bilateral ankle swelling. The ECG shows inverted p in

and partial LBBB. Echocardiography

confirms dilated cardiomyopathy. Which one of the following statements is true?

There is no relevance of history of alcohol abuse.

A family history of a similar problem is unrelated


Past cytotoxic drug therapy is relevant CORRECT ANSWER

Viral illness in the past is unrelated

History of pulmonary tuberculosis in the past is relevant

Dilated cardiomyopathy
Dilated cardiomyopathy (DCM) is characterised by dilatation and impaired systolic function of the left and/or right
ventricle

Aetiology
The aetiology is idiopathic in the majority of cases
DCM is familial (autosomal dominant) in at least 20% of cases and a role of autoimmunity is proposed in the
pathogenesis of this disease
About 30-40% of patients with DCM have organ-specific antibodies and these may become negative with disease
progression

There is an association with viral (coxsackievirus or HIV) infection, which may be immune-related

Features of DCM
Many patients with systemic heart disease present with features of DCM
alcohol abuse
cytotoxic drug therapy, eg doxorubicin, cyclophosphamide
primary heart muscle diseases, eg amyloidosis
end-stage cardiovascular disease, eg ischaemic, rheumatic, congenital, systemic hypertension
generalised disease, eg haemachromatosis, sarcoidosis
connective tissue disorders, eg systemic sclerosis, systemic lupus erythematosus

A 68-year-old man with AF is admitted electively for DC cardioversion, to be performed as a day-

case procedure. However, the procedure is postponed to a later date. Which one of the following
reasons could be responsible for the delay?
He had discontinued digoxin for the last 2 days

O He was taking amiodarone


O His INR 3 weeks ago was 1.6
His serum potassium level was 4.2 mEq/l

O He had an episode of angina 2 days ago

A 68-year-old man with AF is admitted electively for DC cardioversion, to be performed as a day-

case procedure. However, the procedure is postponed to a later date. Which one of the following
reasons could be responsible for the delay?

He had discontinued digoxin for the last 2 days


He was taking amiodarone
His INR 3 weeks ago was 1.6 CORRECT ANSWER
His serum potassium level was 4.2 mEq/l
He had an episode of angina 2 days ago

Cardioversion

External cardioversion is a safe and effective method for restoring sinus rhythm, and should be attempted at
least once in every patient with chronic atrial fibrillation (AF)

Overt congestive heart failure, hypokalaemia and hyperthyroidism should be controlled as much as possible

before cardioversion

Acute myocardial infarction is not a contraindication to cardioversion


If the patient has a slow ventricular response of AF in the absence of anti-arrhythmic drugs, cardioversion
should be performed after the insertion of a temporary transvenous-pacing catheter

Success

Electrical cardioversion is initially successful in 70-94% cases, but relapse is frequent (25-50% at 1 month and
70-90% at 1 year)

The success depends on the duration of AF, transthoracic impedance, left atrial size and the age of the patient

Before the procedure

Although the international normalised ratio (INR) on the day of cardioversion is important, the INR should be
optimal in the preceding 3-4 weeks prior to cardioversion
Digoxin should be withheld on the day of cardioversion
However, if digoxin toxicity is suspected, the problem should be resolved before cardioversion is attempted
Pre-treatment with amiodarone or sotalol may prevent early recurrence
In elective cases, patients should be established on warfarin to give an INR of between 2 and 3 for a minimum of
three weeks prior to cardioversion

Procedure
The initial shock strength should be 100 J, followed by a second 200-J shock and a third 360-J shock
If AF persists, a second 360-J shock with the paddles in the anteroposterior position can be attempted
Immediate DC cardioversion, after the administration of intravenous heparin, is appropriate in an emergency or if
AF has been present for less than 48 hours
Anticoagulation should be continued for four weeks after successful cardioversion

A 44-year-old man presents with a 2-hour history of severe central chest pain, which worsened significantly in the
40 minutes before admission to the Emergency Department. ECG shows ST elevation in the anterior leads. He
was recently discharged following a laparotomy for intestinal obstruction.

What would be the best line of treatment for him?

O Aspirin and clopidogrel


Streptokinase

O Coronary angioplasty
Intravenous heparin

O Alteplase

A 44-year-old man presents with a 2-hour history of severe central chest pain, which worsened significantly in the
40 minutes before admission to the Emergency Department. ECG shows ST elevation in the anterior leads. He

was recently discharged following a laparotomy for intestinal obstruction.

What would be the best line of treatment for him?


Aspirin and clopidogrel
Streptokinase
Coronary angioplasty CORRECT ANSWER

Intravenous heparin
Alteplase

Myocardial infarction
The symptoms and investigations suggest an acute anterior myocardial infarction
The most appropriate approach here is angioplasty
Given the superiority of angioplasty over thrombolysis in the general Ml population, and the contraindication to
thrombolysis because of recent surgery, the correct answer is obvious

Which one of the following is a characteristic feature of troponin?

It is an integral component of pericardial cells


Levels rise immediately or even prior to the onset of chest pain due to myocardial infarction
About 30% of infarct patients show a rise in levels at 12 hours from the onset of symptoms

1 ng/ml is the cut-off above which a myocardial infarction is indicated

O Levels act as a prognostic factor following an acute coronary syndrome

Which one of the following is a characteristic feature of troponin?

It is an integral component of pericardial cells


Levels rise immediately or even prior to the onset of chest pain due to myocardial infarction
About 30% of infarct patients show a rise in levels at 12 hours from the onset of symptoms
1 ng/ml is the cut-off above which a myocardial infarction is indicated

Levels act as a prognostic factor following an acute coronary syndrome CORRECT ANSWER

Troponin levels

The troponin complex is part of the cardiac myofibril and is released in myocardial damage
Levels rise about 4 hours after the onset of chest pain
One-hundred percent of patients are positive for troponin at 12 hours after the onset of pain

A level of > 0.1 ng/ml is considered as a significant rise


Levels of troponin have a strong relationship to clinical outcomes, such as progression to myocardial infarction and
death

A 36-year-old woman who is 8 weeks' pregnant presents with a swollen left leg. Doppler studies confirm a deep

vein thrombosis.

What would be the management in this case?


O Commence intravenous heparin

O Start subcutaneous heparin throughout pregnancy


O Oral anticoagulation with warfarin daily throughout pregnancy and the postpartum period

Aspirin 300 mg daily throughout pregnancy and the postpartum period


O Elastic band compress of her left leg, bedrest and foot elevation

A 36-year-old woman who is 8 weeks' pregnant presents with a swollen left leg. Doppler studies confirm a deep

vein thrombosis.

What would be the management in this case?


Commence intravenous heparin
Start subcutaneous heparin throughout pregnancy CORRECT ANSWER
Oral anticoagulation with warfarin daily throughout pregnancy and the postpartum period
Aspirin 300 mg daily throughout pregnancy and the postpartum period

Elastic band compress of her left leg, bedrest and foot elevation

Treatment for deep vein thrombosis in pregnancy

Anticoagulation is essential in this condition


Warfarin is contraindicated in the first trimester as it is teratogenic

Heparin is relatively safer and should be the drug of choice for anticoagulation
Either unfractionated or low molecular weight heparin can be used throughout pregnancy
Warfarin may be given in the postpartum period
High doses of aspirin are contraindicated in pregnancy as it can cause premature ductal closure.

References
See SIGN guidelines on anti-thrombotic therapy for full discussion of this topic:
www.sign.ac.uk/guidelines/fulltext/36/section2.html

A young computer programmer suddenly develops dysphasia and right-sided weakness. Cardiac examination is

normal and he is afebrile.

Which investigation would confirm the underlying cardiological diagnosis?

O Chest X-ray
12-lead ECG

O 2-D echocardiography
Carotid Doppler study

O Transoesophageal echocardiogram

A young computer programmer suddenly develops dysphasia and right-sided weakness. Cardiac examination is

normal and he is afebrile.

Which investigation would confirm the underlying cardiological diagnosis?


Chest X-ray

12-lead ECG

2-D echocardiography
Carotid Doppler study
Transoesophageal echocardiogram CORRECT ANSWER

Patent foramen ovale


A young, otherwise healthy person who suddenly develops a stroke is likely to have a paradoxical embolism due to
a patent foramen ovale (PFO or ostium secundum defect)
PFOs are relatively common and may be present in 30% of the general population
PFOs greater than 4 mm and associated with shunting are more likely to be associated with systemic emboli

Diagnosis
A chest X-ray may show enlargement of the heart and pulmonary artery as well as pulmonary plethora
ECG and echocardiography
Incomplete right bundle-branch block is seen on ECG in sinus rhythm
Echocardiography may demonstrate the defect and show right ventricular dilatation and hypertrophy and pulmonary
artery dilatation
However, PFOs may be missed on 2-D echocardiogram
The precise size and location of the defect can be shown on transoesophageal echocardiography
A transoesophageal echocardiogram with Doppler colour-flow imaging would be the investigation of choice in this
case

Right ventricular myocardial infarction is most likely to be associated with which one of the
following?

ST-segment elevation in leads II, III and aVF with Q waves and T-wave inversion in these leads

O Occlusion of the left coronary artery


Marked pulmonary vascular congestion

Q A rise in systolic blood pressure


Absent Kussmaul's sign

Right ventricular myocardial infarction is most likely to be associated with which one of the
following?
ST-segment elevation in leads II, III and aVF with Q waves and T-wave inversion in these leads

CORRECT ANSWER
Occlusion of the left coronary artery

Marked pulmonary vascular congestion


A rise in systolic blood pressure
Absent Kussmaul's sign

Right ventricular myocardial infarction


Right ventricular myocardial infarction usually occurs in association with an inferior-wall left ventricular infarction,
as revealed by an ECG

There is usually a right coronary occlusion


Characteristic clinical features include a low cardiac output syndrome with jugular venous distension but no
pulmonary vascular congestion

Kussmaul's sign (increased jugular venous distension with inspiration) may be evident

A 20-year-old woman complains of recurrent syncope. Each attack has occurred after attending an aerobics

class. On examination, a systolic murmur is heard which worsens with the Valsalva manoeuvre and improves on
squatting.

What could be the diagnosis?

Epilepsy

Hypertrophic obstructive cardiomyopathy


Atrial fibrillation

Aortic stenosis
Vasovagal attack

A 20-year-old woman complains of recurrent syncope. Each attack has occurred after attending an aerobics

class. On examination, a systolic murmur is heard which worsens with the Valsalva manoeuvre and improves on
squatting.

What could be the diagnosis?

Epilepsy
Hypertrophic obstructive cardiomyopathy CORRECT ANSWER
Atrial fibrillation
Aortic stenosis

Vasovagal attack

Hypertrophic obstructive cardiomyopathy


Symptoms
Dyspnoea is usually the most common complaint of patients with hypertrophic obstructive cardiomyopathy
However, angina or syncope may also occur
Most patients with pure or predominant aortic stenosis have gradually increasing obstruction for years but do not
become symptomatic until their sixth to eighth decades

Examination findings
A left ventricular apical impulse, a prominent S4 gallop and a harsh systolic ejection murmur are typical findings in
these cases

Valsalva manoeuvre decreases venous return to the heart, which results in a smaller ventricular size
This leads to an increase in the murmur
An echocardiogram is the diagnostic procedure of choice

A 70-year-old woman is admitted with chest pain and breathlessness of 12 hours' duration. She has

a past history of hypertension controlled with ramipril, but nil else of note. On examination, her
heart rate is 170 beats per minute and her blood pressure is 125/72 mmHg. ECG shows atrial
fibrillation. What is the next step in her management?

O Administration of propranolol
Administration of verapamil

O Asynchronous cardioversion
Administration of warfarin

O immediate heparinisation

A 70-year-old woman is admitted with chest pain and breathlessness of 12 hours' duration. She has
a past history of hypertension controlled with ramipril, but nil else of note. On examination, her

heart rate is 170 beats per minute and her blood pressure is 125/72 mmHg. ECG shows atrial

fibrillation. What is the next step in her management?

Administration of propranolol
Administration of verapamil

Asynchronous cardioversion
Administration of warfarin

Immediate heparinisation CORRECT ANSWER

Atrial fibrillation
High-risk patients include those with
a heart rate greater than 150 bpm
chest pain
an unstable condition
shock

Treatment
The appropriate treatment for atrial fibrillation (AF) is determined by the patient's relative risk from the arrhythmia
These patients require urgent treatment
Immediate heparinisation, to reduce the risk of systemic embolisation, and attempted cardioversion with
synchronised DC shock should be carried out first

Warfarin treatment is indicated in the elderly and those with heart disease, and where the duration of AF is longer
than 48 hours prior to considering cardioversion
Young patients with lone atrial fibrillation in the absence of heart disease may not need anticoagulation

A 54-year-old man suddenly develops weakness of the left side of his face and arm and difficulty in speech. This

episode lasts for 15 minutes. He has a history of hypertension, which is well controlled on a calcium channel
blocking agent. His brother had had a severe, disabling stroke at the age of 50.

His cholesterol level is 5.8 mmol/l. A CT scan performed the same day showed the presence of two old lacunar strokes in
the right middle cerebral artery territory. CT angiogram of the carotid system shows a 60% stenosis of the right internal
carotid artery.

Which one of the following factors is the strongest predictor of his being at a high risk of early recurrent
stroke?

O Positive family history


History of hypertension
Q Hyperlipidaemia

Presence of moderate carotid stenosis

O Presence of previous strokes on CT scan

A 70-year-old woman is admitted with chest pain and breathlessness of 12 hours' duration. She has
a past history of hypertension controlled with ramipril, but nil else of note. On examination, her

heart rate is 170 beats per minute and her blood pressure is 125/72 mmHg. ECG shows atrial

fibrillation. What is the next step in her management?

Administration of propranolol
Administration of verapamil
Asynchronous cardioversion
Administration of warfarin

Immediate heparinisation CORRECT ANSWER

Atrial fibrillation
High-risk patients include those with
a heart rate greater than 150 bpm
chest pain
an unstable condition
shock

Treatment
The appropriate treatment for atrial fibrillation (AF) is determined by the patient's relative risk from the arrhythmia
These patients require urgent treatment
Immediate heparinisation, to reduce the risk of systemic embolisation, and attempted cardioversion with
synchronised DC shock should be carried out first

Warfarin treatment is indicated in the elderly and those with heart disease, and where the duration of AF is longer
than 48 hours prior to considering cardioversion
Young patients with lone atrial fibrillation in the absence of heart disease may not need anticoagulation

A 75-year-old man with stable congestive cardiac failure presents with atrial fibrillation. He is haemodynamically

stable with a ventricular rate of 72. He has a good functional state, although echocardiography revealed a dilated
left atrium and mild mitral regurgitation.

Which drug option would be most beneficial for this patient?


Aspirin

O Digoxin
O Frusemide
Lidocaine
O Warfarin

A 75-year-old man with stable congestive cardiac failure presents with atrial fibrillation. He is haemodynamically
stable with a ventricular rate of 72. He has a good functional state, although echocardiography revealed a dilated

left atrium and mild mitral regurgitation.

Which drug option would be most beneficial for this patient?


Aspirin

Digoxin
Frusemide

Lidocaine
Warfarin CORRECT ANSWER

Atrial fibrillation

Patients with atrial fibrillation who are stable pose an intermediate risk
The initial treatment in this case is anticoagulation with warfarin

This is indicated in valvular heart disease and in the elderly


Digoxin is effective in controlling the heart rate at rest and may improve his symptoms but would not affect stroke
risk
Given that his ventricular rate is relatively low at 72 beats per minute, and that he is in a relatively good functional
state, anticoagulation is indicated above rate control

What is the commonest cause of restrictive cardiomyopathy in the UK?

O Pompe disease
O Amyloidosis
O Endocardial fibroelastosis

O Carnitine deficiency
O Acute coxsackievirus infection

What is the commonest cause of restrictive cardiomyopathy in the UK?

Pompe disease

Amyloidosis CORRECT ANSWER


Endocardial fibroelastosis
Carnitine deficiency
Acute coxsackievirus infection

Restrictive cardiomyopathy
Development
A restrictive cardiomyopathy develops from endocardial fibroelastosis, which is typified by a collagen layer on the
endocardium especially the left ventricle
Most infants with isolated disease present by age 3 months with heart failure
A metabolic cardiomyopathy develops with carnitine deficiency

Causes

Amyloidosis is the commonest cause of restrictive cardiomyopathy in the UK


Pompe disease is characterised by
cardiomyopathy
a rapid onset of muscle hypotonia
weakness
glossomegaly
normal cerebral development
Death is common in the first year of life
A viral myocarditis is possible after coxsackievirus infection, but a cardiomyopathy is usually uncommon

A 54-year-old man suddenly develops weakness of the left side of his face and arm and difficulty in speech. This

episode lasts for 15 minutes. He has a history of hypertension, which is well controlled on a calcium channel
blocking agent. His brother had had a severe, disabling stroke at the age of 50.

His cholesterol level is 5.8 mmol/l. A CT scan performed the same day showed the presence of two old lacunar strokes in
the right middle cerebral artery territory. CT angiogram of the carotid system shows a 60% stenosis of the right internal
carotid artery.

Which one of the following factors is the strongest predictor of his being at a high risk of early recurrent
stroke?
Positive family history
Q History of hypertension

Hyperlipidaemia

O Presence of moderate carotid stenosis


O Presence of previous strokes on CT scan

A 54-year-old man suddenly develops weakness of the left side of his face and arm and difficulty in speech. This

episode lasts for 15 minutes. He has a history of hypertension, which is well controlled on a calcium channel
blocking agent. His brother had had a severe, disabling stroke at the age of 50.

His cholesterol level is 5.8 mmol/l. A CT scan performed the same day showed the presence of two old lacunar strokes in
the right middle cerebral artery territory. CT angiogram of the carotid system shows a 60% stenosis of the right internal
carotid artery.

Which one of the following factors is the strongest predictor of his being at a high risk of early recurrent
stroke?
Positive family history

History of hypertension
Hyperlipidaemia
Presence of moderate carotid stenosis CORRECT ANSWER
Presence of previous strokes on CT scan

Transient ischaemic attack

This is a transient ischaemic attack (TIA)


About 15-20% of patients with stroke have a preceding TIA
The issue of subsequent stroke prevention is therefore paramount when managing such a 'warning event'
The urgency of treatment of minor stroke or transient ischaemic attack should depend on the early risk of major
stroke

The risk of recurrent stroke during the first few days after a TIA or minor stroke is much higher than previously
estimated
Recent studies have identified potential risk factors for those at highest risk of subsequent stroke:
age>60 years

hypertension
duration of symptoms >60 min
certain clinical features (unilateral weakness, speech impairment)
presence of diabetes mellitus
Brain imaging also seems to be of prognostic value:
the presence of infarction on CT brain scans in patients with TIA or minor stroke is associated with an
increased risk of stroke recurrence

More research is needed to determine the optimal medical management according to individual risk factors
There are several treatments that are likely to be effective in preventing stroke in the acute phase after a TIA or

ischaemic stroke including aspirin, possibly in combination with clopidogrel and anticoagulation in patients with atrial
fibrillation, and possibly statins
The subgroup of patients with large-artery atherosclerosis (usually carotid bifurcation stenosis) accounts for the largest
proportion of early recurrent strokes
A recent population-based study of prognosis of patients with TIA and 50% symptomatic carotid-artery stenosis reported
risks of stroke of about 20% during the 2 weeks before endarterectomy
Other studies have highlighted the high risk of stroke if endarterectomy is delayed, and hence the rapid decrease in
benefit from surgery with increasing time since event
For neurologically stable patients with TIA and minor stroke, benefit from endarterectomy is greatest if done within 2
weeks of the event
Note: The risk benefit ratio of treating symptomatic carotid stenosis (secondary stroke prevention) differs from that of
treating asymptomatic stenosis as part of primary prevention, where stenosis must be severe in order to justify the risk of
surgery

From the American Heart Association guidelines (2006)


For patients with recent TIA or ischemic stroke within the last 6 months and ipsilateral severe (70-99%) carotid
artery stenosis, CEA is recommended by a surgeon with a perioperative morbidity and mortality of <6%

Class

1, Level
A

For patients with recent TIA or ischemic stroke and ipsilateral moderate (50-69%) carotid stenosis, CEA is
recommended, depending on patient-specific factors such as age, gender, comorbidities and severity of initial
symptoms.

When degree of stenosis is <50%, there is no indication for CEA

Class
1, Level
A

Class
III, Level
A

A patient has broad-complex tachycardia with features resembling ventricular tachycardia rather than
supraventricular tachycardia with a bundle-branch conduction defect.

Which one of the following ECG features is most consistent with a diagnosis of Wolff-Parkinson-White
syndrome?
O Absence of capture or fusion beat

ECG in sinus rhythm reveals right bundle-branch block

O QRS duration less than 140 ms

G P wave preceding wide QRS complex


O V-lead polarity is discordant

A patient has broad-complex tachycardia with features resembling ventricular tachycardia rather than

supraventricular tachycardia with a bundle-branch conduction defect.

Which one of the following ECG features is most consistent with a diagnosis of Wolff-Parkinson-White
syndrome?
Absence of capture or fusion beat

ECG in sinus rhythm reveals right bundle-branch block CORRECT ANSWER


QRS duration less than 140 ms

P wave preceding wide QRS complex


V-lead polarity is discordant

Diagnosis of ventricular tachycardia


ECG in sinus rhythm with a right bundle-branch block like appearance the index of suspicion that Wolff-ParkinsonWhite is the underlying diagnosis
Supraventricular tachycardia with bundle-branch block may resemble ventricular tachycardia on the ECG
Eighty percent of all broad complex tachycardias are owing to ventricular tachycardia and the proportion is even
higher in patients with structural heart disease
Therefore in all cases of doubt, ventricular tachycardia should be diagnosed

ECG diagnosis
The ECG shows a rapid ventricular rhythm with broad (often 0.14 s or more) abnormal QRS complexes
AV dissociation may result in visible P waves
Capture beats (intermittent narrow QRS complex owing to normal ventricular activation via the AV node and
conducting system) and fusion beats (intermediate between ventricular tachycardia beat and capture beat) are
seen

Ventricular tachycardia is more likely than supraventricular tachycardia with bundle-branch block when there is
a very broad QRS (> 0.14 seconds)
atrioventricular dissociation
a bifid upright QRS with a taller first peak in V1
a deep S wave in V6
a concordant (same polarity) QRS direction in all chest leads (V1 - V6)

An elderly, normotensive man with poor left ventricular function presents with a broad-complex tachycardia. His

blood pressure is stable at 125/70 mmHg; his pulse is 145 beats per minute. A previous ECG in his records
shows that he was in left bundle-branch block 2 years earlier.

Which one of the following drugs would be the first choice in treatment of his underlying rhythm?

O Sotalol
O Amiodarone
O Verapamil
O Lidocaine
Q Flecainide

An elderly, normotensive man with poor left ventricular function presents with a broad-complex tachycardia. His
blood pressure is stable at 125/70 mmHg; his pulse is 145 beats per minute. A previous ECG in his records

shows that he was in left bundle-branch block 2 years earlier.

Which one of the following drugs would be the first choice in treatment of his underlying rhythm?
Sotalol
Amiodarone CORRECT ANSWER
Verapamil

Lidocaine
Flecainide

Treatment of ventricular tachycardia


The patient may have either paroxysmal SVT with bundle-branch block, or less likely, given his ventricular rate,
ventricular tachycardia

Verapamil may precipitate a circulatory collapse in VT and is therefore contraindicated here


In the presence of poor left ventricular function, lidocaine and ft-blockers should not be used
Flecainide may cause ventricular fibrillation in stable tachycardias
Amiodarone would therefore be the first choice agent here
Anticoagulation should be considered here, based on the lack of information around duration of his rhythm
disturbance

ich one of the following is a feature of coarctation of the aorta?

If it occurs above the left subclavian artery, blood pressure elevation may be evident only in the left
arm

It is always associated with a continuous murmur


It is accompanied by a bicuspid aortic valve in around 20% of cases
It presents with the inability to augment cardiac output with exercise
Surgical correction usually resolves the hypertension

Which one of the following is a feature of coarctation of the aorta?


If it occurs above the left subclavian artery, blood pressure elevation may be evident only in the left
arm

It is always associated with a continuous murmur

It is accompanied by a bicuspid aortic valve in around 20% of cases CORRECT ANSWER


It presents with the inability to augment cardiac output with exercise

Surgical correction usually resolves the hypertension

Coarctation of the aorta

Coarctation of the aorta usually occurs just distal to the origin of the left subclavian artery

If it arises above the left subclavian, blood pressure may be elevated only in the right arm
A continuous murmur is heard only if the obstruction is severe
A diastolic murmur of aortic regurgitation may be heard, as a bicuspid aortic valve may accompany this condition in
around 20-30%

Cardiac-output response to exercise is not affected unless there is cardiac failure


Hypertension is the major problem and may persist even after complete surgical correction

A young man comes to the Emergency Department complaining of feeling unwell and palpitations.
Supraventricular tachycardia is confirmed on ECG and he responds to carotid sinus massage. Subsequently, the

ECG shows a PR interval of 0.09 s, widened QRS complex in all leads with a slurred upstroke, dominant R wave in
V1 and left axis deviation.

What is the most likely diagnosis?

O Rheumatic fever

Wolff-Parkinson-White syndrome

O Atrial fibrillation
ASD
O Right bundle-branch block

A young man comes to the Emergency Department complaining of feeling unwell and palpitations.

Supraventricular tachycardia is confirmed on ECG and he responds to carotid sinus massage. Subsequently, the
ECG shows a PR interval of 0.09 s, widened QRS complex in all leads with a slurred upstroke, dominant R wave in
V1 and left axis deviation.

What is the most likely diagnosis?


Rheumatic fever
Wolff-Parkinson-White syndrome CORRECT ANSWER
Atrial fibrillation

ASD

Right bundle-branch block

Diagnosis of Wolff-Parkinson-White syndrome

The ECG features are typical of Wolff-Parkinson-White (WPW) syndrome


One of the features of myocarditis due to rheumatic fever is a prolonged PR interval
This occurs owing to a first- or second-degree block

In atrial fibrillation, the ECG shows normal but irregular QRS complexes; there are no P waves but the baseline
may show irregular fibrillation waves
A right bundle-branch block presents with wide QRS complexes, dominant R in lead V-j, inverted T waves in V-j - V4
and a deep wide S wave in lead Vg

atrial septal defects are usually associated with a right bundle-branch block
this leads to an rSR pattern

Diagnosis and treatment


Wolff-Parkinson-White syndrom is classically associated with a short PR interval (<0.12 s)
Slurring of the QRS complex is owing to an extra wave called a delta wave
As the AV node and bypass tract have different conduction speeds and refractory periods, a re-entry circuit can
develop, causing paroxysms of tachycardia
Carotid sinus massage or intravenous adenosine will often terminate an episode of this form of tachycardia

A patient with angina is admitted for cardiac catheterisation. There is a suspicion that she may be suffering from

hyperthyroidism, and this is confirmed by a suppressed TSH.

Which investigation is most predictive as to whether the use of contrast media may worsen any underlying
thyroid condition?

O Thyroid radionuclide isotope scan


O TSH levels

T4 levels
Measurement of TPO antibodies

O Ultrasound scan

A patient with angina is admitted for cardiac catheterisation. There is a suspicion that she may be suffering from
hyperthyroidism, and this is confirmed by a suppressed TSH.

Which investigation is most predictive as to whether the use of contrast media may worsen any underlying
thyroid condition?

Thyroid radionuclide isotope scan CORRECT ANSWER


TSH levels

T4 levels
Measurement of TPO antibodies

Ultrasound scan

Hyperthyroidism and cardiac catheterisation


Cardiac catheterisation requires the use of an iodine-containing contrast
This may worsen hyperthyroidism caused by toxic multinodular goitre, whereas it may improve the symptoms in
patients with Grave's disease

Diagnosis
Thyroid-stimulating hormone (TSH) and T4 levels do not differentiate the two conditions
Thyroid peroxidase (TPO) antibodies occur in autoimmune hypothyroidism and Grave's disease and thus do not
necessarily indicate the presence of hyperthyroidism
About 5-15% of euthyroid women and up to 2% of euthyroid men have thyroid antibodies
Ultrasound can be used to show the presence of a solitary lesion or a multinodular goitre, but will not provide a
definitive diagnosis
#

The most reliable diagnostic method is a radionuclide (99Tcm, 1231 or 1311) scan of the thyroid, which will distinguish
the diffuse, high uptake of Grave's disease from nodular thyroid disease

Medication
If a toxic multinodular goitre or toxic adenoma is detected, the patient should receive an antithyroid drug before
undergoing catheterisation

The antithyroid medication must be continued for at least 2 weeks after the procedure

A 32-year-old man with Wolff-Parkinson-White syndrome presents with a 2-hour history of palpitations and

breathlessness. On examination, his heart rate is 190 beats per minute with blood pressure of 90/60 mmHg. ECG
shows broad-complex tachycardia.

What would be your first line of treatment?

O Intravenous amiodarone
O Intravenous flecainide
Intravenous adenosine
Q DC cardioversion

Intravenous verapamil

A 32-year-old man with Wolff-Parkinson-White syndrome presents with a 2-hour history of palpitations and
breathlessness. On examination, his heart rate is 190 beats per minute with blood pressure of 90/60 mmHg. ECG
shows broad-complex tachycardia.

What would be your first line of treatment?


Intravenous amiodarone

Intravenous flecainide
Intravenous adenosine
DC cardioversion CORRECT ANSWER

Intravenous verapamil

Broad-complex tachycardia in Wolff-Parkinson-White syndrome


Wolff-Parkinson-White (WPW) syndrome presents with atrioventricular re-entrant tachycardia caused by an
aberrant re-entry circuit via the atrium, atrioventricular (AV) node and ventricle
Orthodromic tachycardia occurs most commonly and is associated with narrow complex SVT; this occurs when
conduction goes via the AV node and then back via the accessory pathway

Antidromic tachycardia due to conduction from the atria directly to the ventricle via the
accessory pathway is associated with broad complex tachycardiaTreatment
DC cardioversion is recommended for antidromic tachycardias
Treatment of orthodromic tachycardia aims to block the AV node and cut the re-entry circuit, thereby restoring
sinus rhythm
The drug of choice here is adenosine
Flecainide is a potential option for atrial fibrillation in WPW syndrome although it is best used by a specialist
cardiologist in this situation
Cardioversion is only required when drug therapy fails or the patient's condition is compromised

A 40-year-old healthy man attends a health check-up clinic prior to procuring a health insurance policy. He is
found to have a faint systolic murmur. An echocardiogram reveals a bicuspid aortic valve.

What should he be told?


O His family members have a high chance of also having a bicuspud aortic valve

O Statins do not delay progression of aortic valve disease

q He should undergo further tests to check for any autoimmune disorder


O He may require heart surgery at a later date
O He should start treatment with low-dose aspirin

A 40-year-old healthy man attends a health check-up clinic prior to procuring a health insurance policy. He is

found to have a faint systolic murmur. An echocardiogram reveals a bicuspid aortic valve.

What should he be told?


His family members have a high chance of also having a bicuspud aortic valve
Statins do not delay progression of aortic valve disease

He should undergo further tests to check for any autoimmune disorder

He may require heart surgery at a later date CORRECT ANSWER


He should start treatment with low-dose aspirin

Bicuspid aortic valve


Bicuspid aortic valve occurs in 1-2% of adults and is one of the commonest forms of congenital heart disease in
adults

There is a familial incidence of around 9% but the occurrence tends to be sporadic. As such family members do not
have a high chance of also having a bicuspid aortic valve
Around 1-2% of affected individuals require surgery in their fifth or sixth decade for worn-out or calcified valves
Studies have shown that statins, but not ezetimibe may impact on progression of aortic valve disease although the
potential NNT to prevent one case of aortic stenosis progression is very large

A patient with an artificial aortic valve replacement develops right hemiparesis. CT scan shows cerebral
infarction. There is no evidence of cerebral haemorrhage. The INR is 2.0.

How would you manage this case?


Reverse the anticoagulation with vitamin K
Q Stop warfarin and start intravenous heparin

Increase the dose of warfarin

O Continue warfarin and add intravenous heparin


O Decrease the dose of warfarin until the INR is 1.5

A patient with an artificial aortic valve replacement develops right hemiparesis. CT scan shows cerebral

infarction. There is no evidence of cerebral haemorrhage. The INR is 2.0.

How would you manage this case?


Reverse the anticoagulation with vitamin K
Stop warfarin and start intravenous heparin CORRECT ANSWER
Increase the dose of warfarin

Continue warfarin and add intravenous heparin

Decrease the dose of warfarin until the INR is 1.5

Treatment of cerebral infarction

Use of oral anticoagulation may result in haemorrhage in the infarcted area


As the patient has a mechanical valve, anticoagulation must be continued

The best option therefore would be to stop the warfarin and start intravenous heparin
The reason for this is that heparin can be more easily controlled than oral anticoagulation during the acute period
of stroke recovery

A 45-year-old asthmatic patient presents with palpitations. He is on no other medication apart from his asthma
therapies. An ECG shows supraventricular tachycardia, with narrow QRS complexes. Carotid sinus massage is
not successful. His blood pressure is maintained at 128/72 mmHg.

What would you do next?

O Administer intravenous adenosine


Administer intravenous verapamil
O Administer intravenous digoxin

Administer intravenous sotolol

O DC cardioversion

A 45-year-old asthmatic patient presents with palpitations. He is on no other medication apart from his asthma

therapies. An ECG shows supraventricular tachycardia, with narrow QRS complexes. Carotid sinus massage is
not successful. His blood pressure is maintained at 128/72 mmHg.

What would you do next?


Administer intravenous adenosine
Administer intravenous verapamil CORRECT ANSWER
Administer intravenous digoxin
Administer intravenous sotolol
DC cardioversion

Supraventricular tachycardia and asthma


Treatment
Although adenosine is the drug of choice for terminating paroxysmal supraventricular tachycardia, it can cause
bronchospasm and is thus contraindicated in patients with asthma; sotalol should be avoided for the same reason
Verapamil would therefore be the drug of choice in this case

Contraindications
However, verapamil should not be used for tachyarrhythmias where the QRS complex is wide
It is also contraindicated in patients with the Wolff-Parkinson-White syndrome

A previously fit young man with a history of heavy smoking comes to the Emergency Department complaining of

breathlessness and pleuritic pain occurring suddenly in the middle of a pub team football match. On examination
a systolic click is heard over the precordium, and there are decreased breath sounds on the left.

What is the possible diagnosis?

O Pulmonary embolism
O Mitral valve prolapse
Unstable angina
Q Myocardial infarction

Pneumothorax

Pneumothorax
Sudden onset of exertional breathlessness with chest pain should suggest pneumothorax
Primary spontaneous pneumothorax is usually due to rupture of apical pleural blebs and occurs almost exclusively
in smokers
A systolic click may be heard in these cases.

Other notes
Such features as tachycardia, dyspnoea or tachypnoea may be absent in patients with even a moderate to large
pulmonary embolism

There may be an accentuated pulmonic component of the second heart sound


Breathlessness in those with mitral valve prolapse is insidious and progresses gradually
Severe pain is the most common presenting complaint in acute myocardial infarction

It is often described as heavy, crushing or squeezing and may radiate to the arms, abdomen, back, lower jaw and
neck
Ischaemic pain is also common in unstable angina pectoris.

A previously fit young man with a history of heavy smoking comes to the Emergency Department complaining of
breathlessness and pleuritic pain occurring suddenly in the middle of a pub team football match. On examination
a systolic click is heard over the precordium, and there are decreased breath sounds on the left.

What is the possible diagnosis?

Pulmonary embolism
Mitral valve prolapse
Unstable angina
Myocardial infarction

Pneumothorax CORRECT ANSWER

An obese 50-year-old woman suddenly develops dyspnoea and hypotension 3.5 days after

undergoing a total abdominal hysterectomy. There is mild jugular venous distension. The lung

fields are clear. ECG shows tachycardia with a right bundle-branch block and minor ST-segment
changes. What is the most likely diagnosis?
Acute myocardial infarction

O Pulmonary embolism
O Aspiration pneumonia
Aortic dissection

O Pneumothorax

An obese 50-year-old woman suddenly develops dyspnoea and hypotension 3.5 days after
undergoing a total abdominal hysterectomy. There is mild jugular venous distension. The lung
fields are clear. ECG shows tachycardia with a right bundle-branch block and minor ST-segment

changes. What is the most likely diagnosis?

Acute myocardial infarction


Pulmonary embolism CORRECT ANSWER

Aspiration pneumonia
Aortic dissection

Pneumothorax

Diagnosing pulmonary embolism


Pulmonary embolism presents with a raised jugular venous pressure (JVP) and right bundle-branch block due to
acute right heart failure

Hypotension in an acute myocardial infarction (Ml) would cause gross ST-segment abnormalities on ECG.
Clear lung fields on auscultation preclude a diagnosis of pneumonia
Aortic dissection would cause a Ml or aortic regurgitation before causing respiratory distress

A 72-year-old man is being reviewed in the Cardiac Unit. He has developed a ventricular tachycardia of 160 beats

per minute, looks unwell and has a blood pressure of 90/62 mmHg.

Which one of the following would be the most immediate treatment of choice?

O Immediate heparinisation
O Intravenous lidocaine
O DC cardioversion
O Intravenous adenosine
O Carotid sinus massage

A 72-year-old man is being reviewed in the Cardiac Unit. He has developed a ventricular tachycardia of 160 beats

per minute, looks unwell and has a blood pressure of 90/62 mmHg.

Which one of the following would be the most immediate treatment of choice?
Immediate heparinisation
Intravenous lidocaine
DC cardioversion CORRECT ANSWER

Intravenous adenosine
Carotid sinus massage

Treating ventricular arrhythmia


DC cardioversion is the treatment of choice owing to the poor state of the patient
Intravenous lidocaine is an excellent alternative if the arrhythmia is well tolerated
Intravenous adenosine is useful in cases of supraventricular tachycardia, as is carotid sinus massage

A patient presents with congestive heart failure.

Which drug may be effective in reducing mortality outside the acute situation?

O Enalapril
O Aspirin
O Digoxin
O Furosemide
Lidocaine

A patient presents with congestive heart failure.

Which drug may be effective in reducing mortality outside the acute situation?
Enalapril CORRECT ANSWER
Aspirin

Digoxin
Furosemide
Lidocaine

Treating congestive heart failure


Standard drugs like digitalis and diuretics have not been shown to improve survival rates
A number of studies have conclusively demonstrated that reduction in left ventricular afterload prolongs survival
rates in congestive heart failure

Vasodilators such as angiotensin-converting enzyme (ACE) inhibitors, eg enalapril, are thus effective by inhibiting
the formation of angiotensin II and thus affecting coronary artery tone and arterial wall hyperplasia
Lidocaine and other antiarrhythmic agents are useful only when there is arrhythmia associated with heart failure
Aspirin is indicated only in cases of coronary occlusion or myocardial infarction

A 25-year-old primigravida who is 26 weeks' pregnant, presents to the Emergency Department with symptoms of
headache, flashing lights and vomiting. Her blood pressure was recorded at 140/100 mmHg and her antenatal
diary showed consistent systolic readings of 110-120 mmHg and consistent diastolic readings of less than 80
mmHg. She has a history of mild asthma but was otherwise in good health prior to pregnancy, and there is no
family history of note.

Which one of the following would be the preferred option?


Valsartan

O Labetalol
O Methyldopa
Nifedipine

O Ramipril

A 25-year-old primigravida who is 26 weeks' pregnant, presents to the Emergency Department with symptoms of
headache, flashing lights and vomiting. Her blood pressure was recorded at 140/100 mmHg and her antenatal
diary showed consistent systolic readings of 110-120 mmHg and consistent diastolic readings of less than 80
mmHg. She has a history of mild asthma but was otherwise in good health prior to pregnancy, and there is no
family history of note.

Which one of the following would be the preferred option?


Valsartan
Labetalol

Methyldopa CORRECT ANSWER


Nifedipine

Ramipril

Pre-eclampsia
Any of labetalol, methyldopa and nifedipine can be used in pre-eclampsia, although the first two are preferred
largely based on evidence from case series

NICE guidance recommends Labetalol as first line therapy for gestational hypertension, but in this case
Methyldopa would be the preferred choice due to her history of asthma
In this patient's case (-blockers would be contraindicated owing to her history of asthma
ACE inhibitors are contraindicated in pregnancy because they can cause oligohydramnios, renal failure and

intrauterine death
Valsartan (an angiotensin receptor blocker) is contraindicated
Specialist care is of course required
Magnesium sulphate is used for seizure prevention and treatment, and aspirin may be indicated in some cases
http://www.nice.org.uk/nicemedia/live/13098/50416/50416.pdf

A 75-year-old man with isolated systolic hypertension, who also has urinary incontinence, gout and asthma,
attends outpatients with a blood pressure reading of 190/86 mmHg.

Which one of the following drugs would be most appropriate for this patient?

O Amlodipine
O Atenolol
Bendrofluazide

0 Doxazosin
Valsartan

A 75-year-old man with isolated systolic hypertension, who also has urinary incontinence, gout and asthma,
attends outpatients with a blood pressure reading of 190/86 mmHg.

Which one of the following drugs would be most appropriate for this patient?
Amlodipine CORRECT ANSWER

Atenolol
Bendrofluazide

Doxazosin
Valsartan

Treating systolic hypertension


Amlodipine, a dihydropyridine calcium-channel blocker, is the drug of choice for the treatment of isolated systolic
hypertension in the elderly
Thiazides are likely only to worsen his incontinence and make his gout worse
Whilst doxazosin may be helpful for benign prostatic hyperplasia symptoms it is not a first choice antihypertensive

Asthma rules out the use of U-blockers


With respect to guidance on choice of anti-hypertensives, either an ACE inhibitor/ARB or calcium antagonist is
recommended, but given the evidence in this population, it is stronger for a calcium antagonist

A 60-year-old Caucasian man with NYHA (New York Heart Association) class II heart failure, is taking an
angiotensin-converting enzyme (ACE) inhibitors and bisoprolol for his heart failure. He is also taking 50mg of
furosemide. He is generally well in himself. On direct questioning at his routine outpatient visit, he has noticed a
severe deterioration in his exercise tolerance over the last year and he now sleeps in a chair downstairs because
he finds it impossible to climb the stairs owing to shortness of breath. Examination reveals a blood pressure of
105/72 mmHg, pulse of 85/min and regular. There are bilateral basal crackles on auscultation of the chest, and
bilateral ankle swelling.

Which one of the following drugs should be added to his list of medications with respect to his overall

prognosis?

O Digoxin
O Bumetanide
O Isosorbide mononitrate
O Spironolactone
O Valsartan

A 60-year-old Caucasian man with NYHA (New York Heart Association) class II heart failure, is taking an
angiotensin-converting enzyme (ACE) inhibitors and bisoprolol for his heart failure. He is also taking 50mg of
furosemide. He is generally well in himself. On direct questioning at his routine outpatient visit, he has noticed a
severe deterioration in his exercise tolerance over the last year and he now sleeps in a chair downstairs because
he finds it impossible to climb the stairs owing to shortness of breath. Examination reveals a blood pressure of
105/72 mmHg, pulse of 85/min and regular. There are bilateral basal crackles on auscultation of the chest, and
bilateral ankle swelling.

Which one of the following drugs should be added to his list of medications with respect to his overall

prognosis?

Digoxin
Bumetanide
Isosorbide mononitrate

Spironolactone CORRECT ANSWER


Valsartan

Treating heart failure


The European Society of Cardiology recommends the addition of spironolactone for improving the survival of

patients who are in the transition from well-controlled class II to class III or IV heart failure
Diuretics are only indicated if there is fluid retention
Digoxin helps to relieve symptoms to some extent, and is more useful if the patient is in atrial fibrillation
Similarly, nitrates and hydralazine help to improve symptoms in patients with class III and IV heart failure
Most recently NICE guidelines have suggested adding an angiotensin receptor blocker in patients with milder (class
ll-lll) heart failure, and spironolactone in patients with severe heart failure, to ACE inhibitor and ft-blocker therapy
In this case with progressive disease, and quite severe symptoms, it is spironolactone which is the next obvious
step

A 50-year-old man suffers an extensive anterior myocardial infarction but recovers well in hospital. His

predischarge echo shows him to have an ejection fraction of 35%. He is otherwise asymptomatic.

His medications on discharge should include which one of the following with respect to proven outcome
benefit?

O Aspirin, bisoprolol, ramipril and a statin


O Aspirin, atenolol, ramipril,frusemide and a statin
Aspirin, isosorbide mononitrate,ramipril and a statin

Q Aspirin, nitrate, losartan and a statin


Aspirin, atenolol, ramipril,losartan and a statin

A 50-year-old man suffers an extensive anterior myocardial infarction but recovers well in hospital. His
predischarge echo shows him to have an ejection fraction of 35%. He is otherwise asymptomatic.

His medications on discharge should include which one of the following with respect to proven outcome
benefit?
Aspirin, bisoprolol, ramipril and a statin CORRECT ANSWER
Aspirin, atenolol, ramipril,frusemide and a statin
Aspirin, isosorbide mononitrate,ramipril and a statin

Aspirin, nitrate, losartan and a statin


Aspirin, atenolol, ramipril,losartan and a statin

Treating myocardial infarction


There is no evidence to suggest any drug apart from ACE (angiotensin-converting enzyme) inhibitors in
asymptomatic left ventricular (LV) dysfunction

Following a myocardial infarction (Ml), patients with LV dysfunction benefit from taking (3-blockers
There is strong evidence supporting the use of aspirin and a statin post-MI

A 55-year-old obese woman presents to the Emergency Department with worsening dyspnoea and

ankle swelling due to end-stage heart failure. Her BP in the department is measured at 135/72
mmHg, her pulse is 79/min and regular. There are crackles up to the mid-zones bilaterally on
auscultation of her chest, and bilateral pitting oedema to the knees. Investigations reveal a
creatinine of 155 pmol/l. Which one of the following combinations of drugs is best suited for her in
terms of relieving symptoms and mortality benefit once her fluid balance has been appropriately
managed?

O Ramipril, amlodipine and bendrofluazide


O Ramipril, amiloride, furosemide and atenolol
Ramipril, furosemide and amlodipine

Q Ramipril, furosemide and atenolol


Ramipril, furosemide, bisoprolol and spironolactone

A 55-year-old obese woman presents to the Emergency Department with worsening dyspnoea and

ankle swelling due to end-stage heart failure. Her BP in the department is measured at 135/72
mmHg, her pulse is 79/min and regular. There are crackles up to the mid-zones bilaterally on

auscultation of her chest, and bilateral pitting oedema to the knees. Investigations reveal a
creatinine of 155 pmol/l. Which one of the following combinations of drugs is best suited for her in
terms of relieving symptoms and mortality benefit once her fluid balance has been appropriately
managed?

Ramipril, amlodipine and bendrofluazide


Ramipril, amiloride, furosemide and atenolol
Ramipril, furosemide and amlodipine
Ramipril, furosemide and atenolol
Ramipril, furosemide, bisoprolol and spironolactone CORRECT ANSWER

End-stage heart failure


Outcome trials exist which have demonstrated a benefit for ACE inhibitors, cardioselective beta blockers such as
bisoprolol or carvedilol, and for spironolactone or eplerenone in the management of end stage cardiac failure
The last option listed here is the one which combines all of these potential therapies
Clearly the loop diuretic is also required because of the evidence of fluid retention
Cardiac resynchronisation therapy may also be of benefit in patients on maximal medical therapy who have QRS
prolongation

A 50-year-old woman, who is already on ramipril, frusemide and bisoprolol for heart failure, decompensates and
presents to the Emergency Department with pulmonary oedema. Her heart rate is 120 bpm and her blood
pressure is 84/62 mmHg. She is given oxygen and diamorphine. There is marked peripheral pitting oedema.

Which one of the following actions is indicated in her further management?

O Increase diuretics and maintain the current dose of

(J-blocker

Increase diuretics, reduce the B-blocker dose


O Increase diuretics, increase the E-blocker dose

Increase diuretics, stop B-blockers permanently

O Increase diuretics, stop fJ-blockers and restart R-blockers when her lungs are dry

A 50-year-old woman, who is already on ramipril, frusemide and bisoprolol for heart failure, decompensates and
presents to the Emergency Department with pulmonary oedema. Her heart rate is 120 bpm and her blood
pressure is 84/62 mmHg. She is given oxygen and diamorphine. There is marked peripheral pitting oedema.

Which one of the following actions is indicated in her further management?


Increase diuretics and maintain the current dose of ft-blocker

Increase diuretics, reduce the R-blocker dose


Increase diuretics, increase the ft-blocker dose
Increase diuretics, stop R-blockers permanently

Increase diuretics, stop ft-blockers and restart ft-blockers when her lungs are dry CORRECT
ANSWER

Pulmonary oedema
Modern recommendations, including guidelines from the ESC, suggest in this case it is prudent to stop ft-blockade
temporarily, until fluid overload is successfully managed and BP is above 85 mmHg systolic
http://eurhartj.oxfordjournals.org/content/33/14/1787.full.pdf

A 50-year-old man had a mechanical aortic-valve replacement for severe aortic stenosis, and was discharged
home 10 days later. Two weeks later, he started feeling unwell and had lethargy, nausea and pyrexia of 38.3 C.
Echocardiography showed vegetations on the aortic valve.

Which one of the following is the most likely causative organism?

O Enterococci
O Group D streptococci
O Haemophilus influenzae
Staphylococcus epidermidis

Streptococcus viridans

A 50-year-old man had a mechanical aortic-valve replacement for severe aortic stenosis, and was discharged

home 10 days later. Two weeks later, he started feeling unwell and had lethargy, nausea and pyrexia of 38.3 C.
Echocardiography showed vegetations on the aortic valve.

Which one of the following is the most likely causative organism?


Enterococci
Group D streptococci

Haemophilus influenzae
Staphylococcus epidermidis CORRECT ANSWER

Streptococcus viridans

Prosthetic valvular endocarditis


Prosthetic valvular endocarditis in the immediate postoperative period (< 2 months) is usually caused by
antimicrobial-resistant contamination at surgery, especially with

Staphylococcus epidermidis
diphtheroids
Candida spp

All the other organisms commonly cause endocarditis following asymptomatic bacteraemias from the mouth, gut
and urinary tract

Other notes
You can be reasonably assured that a question on infective endocarditis will come up for the exam, so study this
topic in depth

Which one of the following patients would be best served by a permanent pacemaker?

40-year-old man with third-degree AV block and a maximum documented period of asystole of 1.5 s
40-year-old man with type II second-degree AV block and an escape rate of 30 bpm when awake and
asymptomatic
40-year-old man with Lyme disease having symptomatic complete AV block

O 40-year-old man with chronic asymptomatic trifascicular block and first-degree AV block
40-year-old man, 3 days after suffering an acute anterior Ml and having a persistent first-degree AV
block and old right bundle-branch block

Which one of the following patients would be best served by a permanent pacemaker?

40-year-old man with third-degree AV block and a maximum documented period of asystole of 1.5 s
40-year-old man with type II second-degree AV block and an escape rate of 30 bpm when awake

and asymptomatic

CORRECT ANSWER
40-year-old man with Lyme disease having symptomatic complete AV block
40-year-old man with chronic asymptomatic trifascicular block and first-degree AV block
40-year-old man, 3 days after suffering an acute anterior Ml and having a persistent first-degree AV
block and old right bundle-branch block

Pacemakers
Third-degree and advanced second-degree atrioventricular (AV) block associated with any of the following
conditions definitely needs a permanent pacemaker
symptomatic bradycardia

documented periods of asystole of 3 s or more


any escape rate less than 40 bpm in awake, asymptomatic patients

Other notes
In Lyme disease, the AV block is usually temporary and so does not need permanent pacing
Chronic asymptomatic bi- and trifascicular block needs pacing if associated with type II second-degree or thirddegree AV block, but not otherwise
Following an acute myocardial infarct, pacing is generally indicated for a second- and third-degree block only at or
below the AV node level

A 20-week pregnant woman with a history of asthma is noted to have consistent blood pressure readings over
170/95 mmHg.

Which one of the following antihypertensives would you initiate for this patient?
Nifedipine

O Diltiazem
O Bendrofluazide
O Enalapril
Losartan

A 20-week pregnant woman with a history of asthma is noted to have consistent blood pressure readings over

170/95 mmHg.

Which one of the following antihypertensives would you initiate for this patient?

Nifedipine CORRECT ANSWER


Diltiazem
Bendrofluazide
Enalapril

Losartan

Use of antihypertensives in pregnancy


Dihydropyridines (eg nifedipine) are safe and effective in pregnancy, whereas the role of diltiazem is less clear
Bendrofluazide is relatively contraindicated, while ACE (angiotensin-converting enzyme) inhibitors and angiotensin
receptor blockers are absolutely contraindicated

An 80-year-old man has isolated systolic hypertension. He also suffers from angina, gout and peripheral vascular

disease.

Which one of the following antihypertensives is best suited for him initially?

O Bendrofluazide
Frusemide

O Atenolol
O Modified release nifedipine
O Ramipril

An 80-year-old man has isolated systolic hypertension. He also suffers from angina, gout and peripheral vascular
disease.

Which one of the following antihypertensives is best suited for him initially?
Bendrofluazide

Frusemide
Atenolol
Modified release nifedipine CORRECT ANSWER
Ramipril

Isolated systolic hypertension


Dihydropyridines are especially suited for elderly patients with isolated systolic hypertension, angina and
peripheral vascular disease
Thiazide diuretics are contraindicated in gout
ft-Blockers are relatively contraindicated in peripheral vascular disease

Other notes
In the given setting, ACE (angiotensin-converting enzyme) inhibitors would not be a first-choice agent, but if the
question was worded in such a way that the patient had congestive cardiac failure or type 1 diabetic nephropathy,
then the choice would have been ramipril

An article in a leading medical journal reads: 'an insertion of five nucleotides in the gene was identified as the

cause of hypertrophic cardiomyopathy in this family'.

Which one of the following type of mutation is the author referring to?

O Frame-shift mutation
O Repeat mutation

0 Missense mutation
O Nonsense mutation
O Point mutation

An article in a leading medical journal reads: 'an insertion of five nucleotides in the gene was identified as the

cause of hypertrophic cardiomyopathy in this family'.

Which one of the following type of mutation is the author referring to?
Frame-shift mutation CORRECT ANSWER
Repeat mutation
Missense mutation

Nonsense mutation
Point mutation

The genetics of hypertrophic cardiomyopathy


Frame-shift mutation
Because the group of nucleotides inserted is not a multiple of three, the mutation changes the frame in which
translation occurs and hence the name 'frame-shift mutation'

Other mutations
A missense mutation is an alteration in a nucleotide sequence that converts a codon for one amino acid into a
codon for a second amino acid
An insertion mutation arises by the insertion of one or more nucleotides into a DNA sequence
A nonsense mutation is an alteration in nucleotide sequence that changes a triplet coding for an amino acid into a
termination codon

A point mutation results from a single nucleotide change in a DNA molecule


Another type of mutation relevant for MRCP Part 1 is a read-through mutation, which changes a termination codon
into a codon specifying an amino acid and hence results in read-through of the termination codon

A 50-year-old man has effort-related angina. His total cholesterol is 5.5 mmol/litre. He has no other cardiac risk
factors and no other relevant medical history. His GP has already started aspirin. His BP in the clinic over the
past few weeks has been in the range of 140-155 mmHg systolic.

Which one of the following is the most appropriate initial treatment?

O ft-Blocker and statin


O Calcium-channel blocker and nitrate
O Diltiazem and statin
Isosorbide dinitrate, B-blocker and calcium-channel blocker

Isosorbide dinitrate and statin

A 50-year-old man has effort-related angina. His total cholesterol is 5.5 mmol/litre. He has no other cardiac risk
factors and no other relevant medical history. His GP has already started aspirin. His BP in the clinic over the
past few weeks has been in the range of 140-155 mmHg systolic.

Which one of the following is the most appropriate initial treatment?


IJ-Blocker and statin CORRECT ANSWER
Calcium-channel blocker and nitrate
Diltiazem and statin

Isosorbide dinitrate, B-blocker and calcium-channel blocker


Isosorbide dinitrate and statin

Effort-related angina
Beta blockers
Although experts may use any of the combinations, the choice of first-line treatment for the prophylaxis of effortrelated angina in someone without contraindications is a ft-blocker
ft-Blockers also remain the choice in someone who has suffered a previous myocardial infarction because there is
outcome data to support their use

Other drugs
Calcium-channel blockers - especially diltiazem and verapamil - can be used, although they should be avoided in
the presence of significant left ventricular dysfunction
Nitrates are of good value, but they are second line to fJ-blockade
Care should be taken before combining a fJ-blocker and a calcium-channel blocker, especially if there is evidence
of conduction disturbance or left ventricular dysfunction owing to the risk of bradycardia/hypotension
The Scandinavian Simvastatin Survival Study showed that a statin given to patients with angina pectoris and a
cholesterol level of 5.5-8.0 mmol/litre (212-308 mg/dl) significantly reduced the risk of myocardial infarction

As such, a statin is the second essential component of the correct answer


Aspirin would also of course be recommended for secondary prevention of ischaemic events and nitrates could be
used for symptom relief

A 30-year-old pilot, who is otherwise fit and well, is found to have a WPW (Wolff-Parkinson-White) pre-excitation
pattern on the ECG.

Which one of the following is the most suitable management?


O Amiodarone

Class la antiarrhythmics

Electrophysiological study for risk stratification

O Leave alone and repeat ECG after 6 months


O Radiofrequency catheter ablation of accessory pathway

A 30-year-old pilot, who is otherwise fit and well, is found to have a WPW (Wolff-Parkinson-White) pre-excitation
pattern on the ECG.

Which one of the following is the most suitable management?


Amiodarone
Class la antiarrhythmics

Electrophysiological study for risk stratification


Leave alone and repeat ECG after 6 months
Radiofrequency catheter ablation of accessory pathway CORRECT ANSWER

Wolff-Parkinson-White syndrome in high-risk professions


Asymptomatic Wolff-Parkinson-White (WPW) syndrome in high-risk professions (eg pilots) is best managed by
catheter ablation of the accessory pathway
WPW usually causes atrial arrhythmias
It can rarely cause rapid activation of the ventricles via an accessory atrioventricular pathway with a short
anterograde refractory period, and then degenerate into ventricular fibrillation and sudden cardiac death
Therefore, asymptomatic WPW in someone with a family history of sudden cardiac death is another indication for
radiofrequency catheter ablation

Other notes
Electrophysiological study for risk stratification can be undertaken
But in people in high-risk professions it is probably adding more delay, since the accessory pathway has to be
ablated anyway if they wish to continue in their profession
Also, the risk stratification is not foolproof
Class la/lc antiarrhythmics and amiodarone are an alternative to ablation, but not in high-risk professionals or
those who are symptomatic

Which one of the following conditions is most likely to produce a wide, relatively fixed split of S2?

O Congestive cardiac failure


O Left bundle-branch block
Moderate ventricular septal defect

0 Right bundle-branch block and heart failure


Wolff-Parkinson-White syndrome

Which one of the following conditions is most likely to produce a wide, relatively fixed split of S2?

Congestive cardiac failure


Left bundle-branch block
Moderate ventricular septal defect

Right bundle-branch block and heart failure CORRECT ANSWER


Wolff-Parkinson-White syndrome

Fixed split of S2
Right bundle-branch block makes the split wide and the heart failure fixes the split
Heart failure does not permit much of a change in ventricular volume with respiration, because breathing with
congested lungs is shallow
Right ventricular failure secondary to pulmonary hypertension is another condition where you can get a wide fixed
split, the wideness of the split being due to a prolonged isovolumetric contraction time in the failing right ventricle

Other notes
In cases of moderate ventral septal defects, the widening is owing to delayed P2, but it is not fixed
In Wolff-Parkinson-White syndrome and other causes of electrical delay of left ventricular conduction, including left
bundle-branch block, there is a narrowly split S2

A 30-year-old man presents complaining of periodic attacks of wheezing and loose motions. He has no past

history of note and is married with two children. On examination he has prominent precordial pulsations.

What is the most likely diagnosis?

O Carcinoid heart disease


Congenital tricuspid regurgitation

O HiV-associated heart disease


O Rheumatic heart disease
O Traumatic heart disease

A 30-year-old man presents complaining of periodic attacks of wheezing and loose motions. He has no past

history of note and is married with two children. On examination he has prominent precordial pulsations.

What is the most likely diagnosis?


Carcinoid heart disease CORRECT ANSWER

Congenital tricuspid regurgitation

HIV-associated heart disease


Rheumatic heart disease
Traumatic heart disease

Carcinoid heart disease


Carcinoid heart disease occurs when there is an accumulation of fibrous tissue in the heart, especially to the
undersurface of the tricuspid valve

It can hold the tricuspid valve in a semiclosed position, so tricuspid stenosis and regurgitation can occur
If it affects the pulmonary valve, pulmonary regurgitation or stenosis occurs

Associations
It is usually associated with carcinoid tumour of the bowel and with liver metastases
Bronchospasm, diarrhoea and flushing are part of the carcinoid syndrome

Other notes
Traumatic tricuspid regurgitation can occur owing to ruptured chordae tendinae, but there is no wheezing or
diarrhoea

HIV can cause cardiomyopathy, pericardial diseases, myocarditis and heart failure

A 30-year-old man presents complaining of periodic attacks of wheezing and loose motions. He has no past
history of note and is married with two children. On examination he has prominent precordial pulsations.

What is the most likely diagnosis?


Carcinoid heart disease

0 Congenital tricuspid regurgitation


HIV-associated heart disease

O Rheumatic heart disease


O Traumatic heart disease

A 30-year-old man presents complaining of periodic attacks of wheezing and loose motions. He has no past
history of note and is married with two children. On examination he has prominent precordial pulsations.

What is the most likely diagnosis?


Carcinoid heart disease CORRECT ANSWER

Congenital tricuspid regurgitation

HIV-associated heart disease


Rheumatic heart disease
Traumatic heart disease

Carcinoid heart disease


Carcinoid heart disease occurs when there is an accumulation of fibrous tissue in the heart, especially to the
undersurface of the tricuspid valve

It can hold the tricuspid valve in a semiclosed position, so tricuspid stenosis and regurgitation can occur
If it affects the pulmonary valve, pulmonary regurgitation or stenosis occurs

Associations
It is usually associated with carcinoid tumour of the bowel and with liver metastases
Bronchospasm, diarrhoea and flushing are part of the carcinoid syndrome

Other notes
Traumatic tricuspid regurgitation can occur owing to ruptured chordae tendinae, but there is no wheezing or
diarrhoea

HIV can cause cardiomyopathy, pericardial diseases, myocarditis and heart failure

What does a prominent left precordium in a 16-year-old young man with an ejection murmur in the

second left intercostal space indicate?

O ASD with aortic regurgitation


ASD with aortic stenosis

O ASD with mitral stenosis

O ASD with pulmonary hypertension


O Uncomplicated ASD

What does a prominent left precordium in a 16-year-old young man with an ejection murmur in the

second left intercostal space indicate?


ASD with aortic regurgitation
ASD with aortic stenosis
ASD with mitral stenosis

ASD with pulmonary hypertension CORRECT ANSWER


Uncomplicated ASD

Atrial septal defect


A prominent left precordium suggests that the right ventricle was dilated during childhood
It also suggests that it was working against a high pressure
Ostium secondum atrial septal defect (ASD) in combination with rheumatic mitral stenosis (Lutembacher syndrome)
can cause the same picture in advanced cases when there is pulmonary hypertension, but option D is a better

choice than C

A 25-year-old man's blood pressure is consistently 30 mmHg higher when measured in his right arm compared
with in his left arm. Routine haematology, biochemistry and ESR findings are normal.

What is the most likely diagnosis?

O Arterial obstructive disease


O Artefact due to lack of simultaneous recording
O Subclavian steal syndrome
O Supravalvular aortic stenosis
Q Takayasu's arteritis

A 25-year-old man's blood pressure is consistently 30 mmHg higher when measured in his right arm compared
with in his left arm. Routine haematology, biochemistry and ESR findings are normal.

What is the most likely diagnosis?


Arterial obstructive disease
Artefact due to lack of simultaneous recording
Subclavian steal syndrome
Supravalvular aortic stenosis CORRECT ANSWER

Takayasu's arteritis

Supravalvular aortic stenosis


In supravalvular aortic stenosis, the jet is preferentially directed up the innominate artery
Other findings might include
a difference in carotid pulsation
and the murmur of aortic stenosis

Other notes
In coarctation of the aorta, the narrowing is usually found just beyond the origin of the left subclavian artery
Hypertension in the arms and weak femoral pulses are the classic features of coarctation
The other choices are possibilities, but not the most likely diagnosis

Which one of the following pharmacological agents is most likely to benefit a patient with angina
due to cardiac syndrome X?
Aspirin

O Bisoprolol

O Diazepam
O Atenolol

O Isosorbide mononitrate

Which one of the following pharmacological agents is most likely to benefit a patient with angina
due to cardiac syndrome X?
Aspirin
Bisoprolol

Diazepam

Atenolol
Isosorbide mononitrate CORRECT ANSWER

Syndrome X
Nitrates are often effective in patients with syndrome X, as are the range of calcium antagonists including
dihydropyridine and non-dihydropyridine agents.

Cardiac syndrome X
Cardiac syndrome X consists of
angina-like chest pain during exertion
characteristic ECG changes during exercise testing
normal coronary arteries on cardiac catheterisation
no inducible coronary artery spasm during catheterisation

Metabolic syndrome X
It should not be confused with the metabolic syndrome X, which comprises
central obesity
glucose intolerance
dyslipidaemia
high blood pressure

The dyslipidaemia in this case is primarily high triglycerides and low high-density lipoprotein cholesterol
People with metabolic syndrome are at increased risk of coronary artery disease

A 25-year-old man with right ventricular cardiomyopathy, previously asymptomatic, has sustained monomorphic

ventricular tachycardia on treadmill exercise. Clinical examination and resting ECG is unremarkable.

Which one of the following treatment options is considered the usual first-line option?

O Flecainide
Q Implantable cardioverter defibrillator
Q Radiofrequency catheter ablation

O Rate-responsive, dual-chamber pacemaker

O Sotalol

A 25-year-old man with right ventricular cardiomyopathy, previously asymptomatic, has sustained monomorphic

ventricular tachycardia on treadmill exercise. Clinical examination and resting ECG is unremarkable.

Which one of the following treatment options is considered the usual first-line option?
Flecainide

Implantable cardioverter defibrillator


Radiofrequency catheter ablation

Rate-responsive, dual-chamber pacemaker


Sotalol CORRECT ANSWER

Treating ventricular tachycardia


Sotalol
Among all antiarrhythmics tested, sotalol has shown the highest efficacy in trials so far
It is therefore recommended as the first-choice drug to prevent a recurrence of ventricular tachycardia (VT)

Other notes
Radiofrequency ablation is a procedure for medically refractory VT in selected patients only, and there is no
information regarding its efficacy in preventing sudden cardiac death

In patients with aborted sudden cardiac death and in high-risk patients with VT with compromised right and left
ventricular function, implantable cardioverter defibrillator (ICD) therapy, might reduce mortality
There are problems with using ICD in right ventricular cardiomyopathy (where the RV muscle mass might be too
thin), such a

low R-wave amplitude


high thresholds
lead perforation

A 16-year-old young man had a cardiac arrest while playing football and was resuscitated. He

recovered fully and was later found to have HOCM (hypertrophic obstructive cardiomyopathy).
Which one of the following is the best treatment option?

O Implantable cardioverter defibrillator


Amiodarone
O (3-Blockers
Verapamil

O Rate-responsive, dual-chamber pacemaker

A 16-year-old young man had a cardiac arrest while playing football and was resuscitated. He
recovered fully and was later found to have HOCM (hypertrophic obstructive cardiomyopathy).

Which one of the following is the best treatment option?

Implantable cardioverter defibrillator CORRECT ANSWER

Amiodarone
(3-Blockers
Verapamil

Rate-responsive, dual-chamber pacemaker

Hypertrophic obstructive cardiomyopathy


For the secondary prevention of sudden cardiac death (SCD) in patients with HOCM, there is evidence and
general agreement that implantable cardioverter defibrillator is the most useful option

Even for the primary prevention of SCD in HOCM, the weight of evidence is currently in favour of its efficacy,
although in selected patients amiodarone has a role

Options C, D and E are not considered effective in preventing SCD in HOCM

A patient has tuboeruptive xanthomas, distributed subcutaneously and mainly on the extensor surface of

extremities.

What is the probable diagnosis?


O Type I hyperlipoproteinaemia

O Type II hyperlipoproteinaemia
O Type III hyperlipoproteinaemia
O Type IV hyperlipoproteinaemia
Q Type V hyperlipoproteinaemia

A patient has tuboeruptive xanthomas, distributed subcutaneously and mainly on the extensor surface of

extremities.

What is the probable diagnosis?

Type I hyperlipoproteinaemia
Type II hyperlipoproteinaemia
Type III hyperlipoproteinaemia CORRECT ANSWER

Type IV hyperlipoproteinaemia
Type V hyperlipoproteinaemia

Xanthomas
Tuboeruptive xanthomas occur in type III hyperlipoproteinaemia
Eruptive xanthomas are associated with hyperchylomicronaemia (type I and type V hyperlipoproteinaemia)
Xanthoma tendinosum, which are nodular swellings of tendons, usually occur in type II hyperlipoproteinaemia

To establish the aetiology of pulmonary hypertension, a cardiac catheter study was performed. The wedge
pressure was normal, and the mean mitral valve diastolic pressure gradient was > 3 mmHg at rest, both of which
increased with exercise.

From these data, what is the probable diagnosis?

O Congenital heart disease

O Left ventricular diastolic dysfunction


Major pulmonary artery occlusion
Q Mitral regurgitation

Mitral stenosis

To establish the aetiology of pulmonary hypertension, a cardiac catheter study was performed. The wedge
pressure was normal, and the mean mitral valve diastolic pressure gradient was > 3 mmHg at rest, both of which

increased with exercise.

From these data, what is the probable diagnosis?


Congenital heart disease
Left ventricular diastolic dysfunction
Major pulmonary artery occlusion
Mitral regurgitation

Mitral stenosis CORRECT ANSWER

Mitral stenosis
The haemodynamic pattern is typical of mitral stenosis and may also be seen in cor triatriatum, in which there is a
left atrial membrane

A large systolic pressure wave in wedge tracing can be seen in patients with mitral regurgitation
In addition, regurgitation of contrast from a left ventricular angiogram to the left atrium can be seen
The left ventricular end-diastolic pressure (LVEDP) is > 15 mmHg in those with LV diastolic dysfunction
There will be
a focal pressure gradient in a lobar or larger pulmonary artery
intravascular filling defect
or narrowing in patients with major pulmonary artery occlusion by clot or tumour

In congenital heart disease, there will be a step up in oxygen saturation in the right heart and a step down in the left
heart

A 17-year-old young man presents with palpitations. His physical examination is normal except for a systolic
murmur in the second left intercostal space and prominent precordial motion with a late systolic impulse.

Which one of the following conditions is he most likely to have?

O Aortic stenosis
Atrial septal defect

O Hypertrophic cardiomyopathy
Mitral valve prolapse

O Mixed aortic valve disease

A 17-year-old young man presents with palpitations. His physical examination is normal except for a systolic

murmur in the second left intercostal space and prominent precordial motion with a late systolic impulse.

Which one of the following conditions is he most likely to have?


Aortic stenosis
Atrial septal defect

Hypertrophic cardiomyopathy CORRECT ANSWER


Mitral valve prolapse
Mixed aortic valve disease

Diagnosing hypertrophic cardiomyopathy


A mid to late systolic impulse in the precordial motion (triple ripple) is seen in patients with hypertrophic
cardiomyopathy

A hyperdynamic impulse is seen in mild to moderate AR


Aortic stenosis, atrial septal defect, mitral valve proplapse and mixed aortic valve disease can produce the murmur
but not this characteristic finding

A 25-year-old woman is seen in outpatients and is found to have loud first heart sound, an early diastole sound,

followed by a mid-diastolic murmur.

What is the likely diagnosis?


Mitral stenosis with a fourth heart sound

O Mitral stenosis with atrial fibrillation


O Mitral stenosis with mobile leaflets
Mitral stenosis with pulmonary hypertension

O Mitral valve prolapse

A 25-year-old woman is seen in outpatients and is found to have loud first heart sound, an early diastole sound,

followed by a mid-diastolic murmur.

What is the likely diagnosis?


Mitral stenosis with a fourth heart sound
Mitral stenosis with atrial fibrillation

Mitral stenosis with mobile leaflets CORRECT ANSWER


Mitral stenosis with pulmonary hypertension
Mitral valve prolapse

Mitral stenosis with mobile leaflets

From the information given, the diagnosis is mitral stenosis with mobile leaflets
There is no information regarding atrial fibrillation or pulmonary hypertension
There is a loud first heart sound and when an opening snap is heard this indicates that the mitral valve is mobile
The snap occurs when the superior systolic bowing of the anterior mitral valve leaflet is rapidly reversed towards
the left ventricle in early diastole, owing to the high left atrial pressure

Other notes
S4 is classically late diastolic or presystolic
It occurs when augmented atrial contraction causes presystolic ventricular distension so that the ventricle then
contracts with greater force

Atrial myxoma can produce an early diastolic sound (tumour plop), which is owing to the abrupt diastolic seating of
the tumour within the right or left atrioventricular orifice

Which one of the following microanatomical structures within the heart interacts with conventional
calcium-channel blockers?

O L-type calcium channels


O Calcium-channel T type
O T tubules
O Titin
O Tropomyosin

Which one of the following microanatomical structures within the heart interacts with conventional
calcium-channel blockers?

L-type calcium channels CORRECT ANSWER


Calcium-channel T type

T tubules
Titin
Tropomyosin

Microanatomical structures
T tubules and calcium channels
The T tubules are a tubular network formed by the invagination of the sarcolemma of the myocyte
Sarcolemmal calcium channels are located on the T tubules
There are two main types of channels
T (transient) channels do not interact with conventional calcium-channel blockers
L-type calcium channels do interact with calcium-channel blockers

Titin
Titin tethers the myosin molecule to the Z line, and its elasticity explains the stress-strain elastic relation of
striated muscle
It is the largest protein molecule yet described

Tropomyosin
The thin actin filaments intertwine and are carried on a heavier tropomyosin molecule that functions as a backbone
At regular intervals along this structure is a group of three regulatory proteins called the 'troponin complex', which is

composed of troponin C, I and M

Which one of the following antiarrhythmic agents works primarily by its action on SA and AV

nodes?

O Amiodarone
O Atenolol
O Flecainide
O Sotalol
Verapamil

Which one of the following antiarrhythmic agents works primarily by its action on SA and AV

nodes?

Amiodarone
Atenolol
Flecainide
Sotalol
Verapamil CORRECT ANSWER

Antiarrhythmic agents
Calcium-channel blockers act mainly on the sinoatrial (SA) and atrioventricular (AV) nodes (direct membrane
effect), as these structures are almost exclusively depolarised by the slow calcium channels

Flecainide binds to the sodium channel and decreases the speed of depolarisation (in other words, decreases
conduction velocity)

Atenolol decreases sympathetic tone


Amiodarone and sotalol increase the action-potential duration and therefore the refractory periods
they have little effect on conduction velocity

Which one of the following best describes the mechanism of action of flecainide as an

antiarrhythmic agent?

O Slows the upstroke of the action potential


O Increases the action-potential duration

q Has a direct membrane effect


O Increases vagal tone
O Affects SA and AV nodes

Which one of the following best describes the mechanism of action of flecainide as an
antiarrhythmic agent?
Slows the upstroke of the action potential CORRECT ANSWER

Increases the action-potential duration


Has a direct membrane effect
Increases vagal tone
Affects SA and AV nodes

Mechanisms of action of antiarrhythmic agents


Flecainide
Flecainide, a class Ic agent slows the upstroke of the action potential and is its main mechanism of action
It has minimal effects on action-potential duration
In other words, it causes a marked decrease in conductivity, with little effect on refractoriness

Other agents
The antiarrhythmic group that mainly affects sinoatrial and atrioventricular nodes, and thus has a direct membrane
effect, is the calcium-channel blockers
Class V agents (digitalis agents) affect SA and AV nodes by increasing vagal tone

Which one of the following antiarrhythmics has the highest risk of producing torsades de pointes?

O Flecainide
Lidocaine

O Phenytoin
O Propafenone
O Sotalol

Which one of the following antiarrhythmics has the highest risk of producing torsades de pointes?

Flecainide
Lidocaine
Phenytoin
Propafenone

Sotalol CORRECT ANSWER

Risk of torsades de pointes


Among these agents sotalol, which is a class III agent, has the highest risk of producing torsades, which is a
polymorphic, pause-dependent ventricular tachycardia causing syncope and sudden cardiac death

Lidocaine and phenytoin (which are class lb agents) and flecainide and propafenone (class Ic) have an almost zero
risk of producing TdP

Urinary retention as a sign of drug-induced toxicity is characteristic of which one of the following

antiarrhythmics?

O Amiodarone
Sotalol

O Disopyramide
Flecainide

O Verapamil

Urinary retention as a sign of drug-induced toxicity is characteristic of which one of the following

antiarrhythmics?

Amiodarone
Sotalol
Disopyramide CORRECT ANSWER

Flecainide
Verapamil

Side-effects of antiarrhythmics
Amiodarone causes
hepatic effects
peripheral neuropathy
proximal myopathy
thyroid dysfunction
skin discoloration
pneumonitis
others
Sotalol (ft-blockers) and flecainide have negative inotropy and CNS effects
Verapamil causes bradycardia

A 25-year-old medical student noticed that he had a murmur when he tested his new stethoscope. On

assessment in the Cardiology Clinic, he was found to have a harsh systolic murmur over his precordium, which

did not change with inspiration. ECG showed features of biventricular hypertrophy.

What is the most likely diagnosis?

O Aortic stenosis
O Hypertrophic cardiomyopathy
O Mitral regurgitation
O Tricuspid regurgitation

q Ventricular septal defect

A 25-year-old medical student noticed that he had a murmur when he tested his new stethoscope. On
assessment in the Cardiology Clinic, he was found to have a harsh systolic murmur over his precordium, which

did not change with inspiration. ECG showed features of biventricular hypertrophy.

What is the most likely diagnosis?


Aortic stenosis

Hypertrophic cardiomyopathy
Mitral regurgitation

Tricuspid regurgitation

Ventricular septal defect CORRECT ANSWER

Diagnosing ventricular septal defect


Right-sided murmurs increase with inspiration (eg tricuspid regurgitation, TR), whereas left-sided murmurs do not
increase with inspiration
The clue to diagnosis is in the ECG finding
Aortic stenosis and mitral regurgitation produce left ventricular hypertrophy, TR produces right ventricular
hypertrophy and a ventricular septal defect produces biventricular hypertrophy

In the Cardiology Outpatients' Department, a 50-year-old obese woman was noticed to have a systolic murmur
loudest at the apex. Isometric exercise by handgrip intensified the murmur.

What is the most likely cause of her murmur?

O Aortic stenosis
Hypertrophic obstructive cardiomyopathy

O Mitral regurgitation
Mitral valve prolapse

O Tricuspid regurgitation

In the Cardiology Outpatients' Department, a 50-year-old obese woman was noticed to have a systolic murmur
loudest at the apex. Isometric exercise by handgrip intensified the murmur.

What is the most likely cause of her murmur?


Aortic stenosis

Hypertrophic obstructive cardiomyopathy


Mitral regurgitation CORRECT ANSWER
Mitral valve prolapse
Tricuspid regurgitation

Mitral regurgitation
The murmur of mitral regurgitation is intensified by isometric exercise and thus helps to differentiate it from other
systolic murmurs

Sudden standing and amylnitrite decrease the murmur

Other notes
The murmurs of aortic stenosis, mitral prolapse and HOCM are reduced by handgrip exercises

A 25-year-old man presents to the Emergency Department with a 1-week history of fever and myalgia. He had

travelled to Chile 8 weeks ago. On examination there are no positive findings, although the patient recollects that
his right eyelid was swollen for a few weeks after he left Chile. ECG reveals non-specific, T-wave changes in all
leads.

What is the most likely diagnosis?

O Echinococcosis
O Falciparum malaria

Schistosomiasis
Toxoplasmosis

Trypanosomiasis

A 25-year-old man presents to the Emergency Department with a 1-week history of fever and myalgia. He had
travelled to Chile 8 weeks ago. On examination there are no positive findings, although the patient recollects that
his right eyelid was swollen for a few weeks after he left Chile. ECG reveals non-specific, T-wave changes in all

leads.

What is the most likely diagnosis?


Echinococcosis
Falciparum malaria

Schistosomiasis
Toxoplasmosis
Trypanosomiasis CORRECT ANSWER

Trypanosomiasis
Trypanosoma cruzi causes American trypanosomiasis or Chagas disease and is quite common in South America
the vectors are reduvid bugs
The trypanosomes are transmitted by scratching infected faeces of the bug into skin abrasions caused by the bug
during blood sucking

The other conditions listed can cause myocarditis, but the best choice is trypanosomiasis

Signs and symptoms


In acute trypanosomiasis, the patient presents with fever, myalgia, hepatosplenomegaly and myocarditis
Unilateral periorbital oedema and swelling of the eyelid can result from a bug bite around the eyes
this is called Romana's sign

A 30-year-old woman presents with pleuritic chest pain and haemoptysis. Her blood pressure is stable at 130/80
mmHg. A CTPA shows evidence of bilateral basal pulmonary emboli. There is no evidence of RV dysfunction,
clinically and on echocardiography.

In addition to oxygen, which one of the following is the appropriate management for this patient?

O Heparin and consideration for surgery


Heparin and paracetamol

O Heparin plus mechanical intervention


O Heparin plus thrombolytic therapy
O Supportive

A 30-year-old woman presents with pleuritic chest pain and haemoptysis. Her blood pressure is stable at 130/80
mmHg. A CTPA shows evidence of bilateral basal pulmonary emboli. There is no evidence of RV dysfunction,

clinically and on echocardiography.

In addition to oxygen, which one of the following is the appropriate management for this patient?
Heparin and consideration for surgery

Heparin and paracetamol CORRECT ANSWER


Heparin plus mechanical intervention
Heparin plus thrombolytic therapy

Supportive

Pulmonary embolism
This patient has potentially had up to two small to moderate pulmonary embolisms (PE), probably associated with
pulmonary infarction

Management
The management in this case would be heparin and simple analgesics to control her chest pain
If there were signs of a large/massive PE (hypotension, right ventricular dysfunction), the ideal management would
include thrombolytic therapy or mechanical intervention

A 50-year-old man underwent coronary artery bypass grafting 2 days ago. A routine liver function test result now
shows that both the direct and indirect bilirubin are elevated. All the other liver function tests are normal.

Which one of the following is the most likely cause?


O 'Shock liver syndrome

O Anaesthetic-induced
O Haemolysis on bypass
O Narcotic-induced

q Right heart failure

A 50-year-old man underwent coronary artery bypass grafting 2 days ago. A routine liver function test result now

shows that both the direct and indirect bilirubin are elevated. All the other liver function tests are normal.

Which one of the following is the most likely cause?


'Shock liver' syndrome

Anaesthetic-induced

Haemolysis on bypass CORRECT ANSWER


Narcotic-induced

Right heart failure

Haemolysis on cardiopulmonary bypass


Isolated elevation of direct and indirect bilirubin, indicates haemolysis on the cardiopulmonary bypass and can be
confirmed by increased plasma free-haemoglobin levels

There is no specific treatment

Other notes
Markedly raised enzyme levels are seen in patients with the 'shock liver' syndrome, and the treatment is aimed at
maximising cardiac output and oxygenation

Right heart failure is another cause of hyperbilirubinaemia in the immediate post-bypass period, and, in this case,
the direct bilirubin and alkaline phosphatase are increased without enzyme elevation
Treatment is as for right heart failure

In an asymptomatic patient, a permanent pacemaker is indicated in which one of the following

conditions?

O First-degree block at the AV node


First-degree block in the distal conduction system with an HV interval < 100 ms

Second-degree block at the AV node

O Second-degree block at the distal conduction system


O Right bundle-branch block

In an asymptomatic patient, a permanent pacemaker is indicated in which one of the following


conditions?
First-degree block at the AV node
First-degree block in the distal conduction system with an HV interval < 100 ms

Second-degree block at the AV node


Second-degree block at the distal conduction system CORRECT ANSWER

Right bundle-branch block

Permanent pacemaker in asymptomatic patient


In an asymptomatic patient, a permanent pacemaker (PPM) is indicated in second- and third-degree heart block at
the distal conduction system
If the third-degree block at the atrioventricular (AV) node had been associated with symptoms, it would have been
an indication for PPM
Also, a PPM is indicated for cases of first-degree AV block in the distal conduction system with an HV (Bundle of
His to ventricular depolarisation) interval of greater than 100 ms associated with symptoms

An 80-year-old man with a history of intermittent atrial fibrillation presents with syncope. ECG documents a type II,

second-degree AV block.

Which one of the following types of pacemaker is best indicated for him?

DDD
DDDR
O VOO

Wl
WIR

An 80-year-old man with a history of intermittent atrial fibrillation presents with syncope. ECG documents a type II,

second-degree AV block.

Which one of the following types of pacemaker is best indicated for him?

DDD
DDDR
VOO

Wl

WIR CORRECT ANSWER

Types of pacemaker
There is a suspicion here that this patient probably has sinus node disease with intermittent atrial fibrillation (AF)
and periods of heart block
In a patient with long periods of AF, a single-chamber device (Wl) is indicated
The addition of rate modulation (WIR) makes it even better
A DDD (atrial sensing) device is not indicated in this case, since it will inappropriately sense all the fibrillating P
waves and can defeat the objective

A VOO (ventricular pacing only) device will simply pace the ventricle, and there is a risk that if the pacing falls in
the relative refractory period of the ventricle, it can induce the R on T phenomenon, which can lead on to
ventricular tachyarrhythmia (VT) and ventricular fibrillation (VF)

A 60-year-old man suffered an anterior myocardial infarction. He had all the risk factors for coronary artery

disease.

Which one of the following non-pharmacological interventions will be most helpful in reducing his risk of a
future ischaemic event?

O Alcohol reduction
O Diet control
Regular exercise

Stopping smoking
Weight reduction

A 60-year-old man suffered an anterior myocardial infarction. He had all the risk factors for coronary artery

disease.

Which one of the following non-pharmacological interventions will be most helpful in reducing his risk of a
future ischaemic event?
Alcohol reduction
Diet control

Regular exercise
Stopping smoking CORRECT ANSWER

Weight reduction

Non-pharmacological interventions after myocardial infarction


All these interventions are effective but stopping smoking is the single most effective, non-pharmacological
intervention that will help to reduce the risk of a future event
There are trials showing that smoking cessation is as effective as having a coronary artery bypass graft

A 70-year-old man, previously fit and well, is referred to outpatients with exertional chest pain that comes on at

around 0.8 km (0.5 mile) on the flat. Examination reveals him to be in sinus rhythm at 80 bpm, blood pressure
100/70 mmHg, a diminished carotid upstroke and a loud ejection systolic murmur over the aortic area. Echo
confirms left ventricular hypertrophy, preserved systolic function with peak aortic valve gradients of 80 mmHg.
Subsequent coronary angiography reveals significant disease in the left anterior descending artery.

What is the optimal treatment strategy?


Angioplasty and stenting to the left anterior descending artery

Q Aortic valve replacement and bypass graft


Aspirin and review in 3 months' time with a repeat echo

O Aspirin and a p-blocker, and review in 3 months' time with a repeat echo
O Percutaneous aortic valvotomy

A 70-year-old man, previously fit and well, is referred to outpatients with exertional chest pain that comes on at

around 0.8 km (0.5 mile) on the flat. Examination reveals him to be in sinus rhythm at 80 bpm, blood pressure
100/70 mmHg, a diminished carotid upstroke and a loud ejection systolic murmur over the aortic area. Echo
confirms left ventricular hypertrophy, preserved systolic function with peak aortic valve gradients of 80 mmHg.
Subsequent coronary angiography reveals significant disease in the left anterior descending artery.

What is the optimal treatment strategy?

Angioplasty and stenting to the left anterior descending artery


Aortic valve replacement and bypass graft CORRECT ANSWER

Aspirin and review in 3 months' time with a repeat echo


Aspirin and a p-blocker, and review in 3 months' time with a repeat echo

Percutaneous aortic valvotomy

Aortic stenosis
This patient has severe (aortic valve gradient > 70 mmHg), symptomatic aortic stenosis and as such valve
replacement is indicated, unless precluded by co-morbidity
Coronary angiography is performed to assess the need for concomitant coronary artery bypass grafting

Development of symptoms
With acquired aortic stenosis, patients most commonly present in their sixth decade with symptoms of angina,
syncope or heart failure

The development of symptoms is associated with a poor outcome if left untreated


The average life expectancy from the onset of symptoms to death is 2 years in patients with heart failure, 3 years
in those with syncope and 5 years in those with angina

A 52-year-old woman, with a prior history of rheumatic fever, presents with shortness of breath on strenuous
exertion while working as a landscape gardener. She is in permanent atrial fibrillation and is on long-term warfarin
and digoxin (125 pg once daily). Clinical examination reveals her to be in atrial fibrillation at a rate of around 150
bpm. Echo demonstrates preserved left ventricular function, a heavily calcified mitral valve with moderate mitral

stenosis (mitral valve area 1.5 cm2) and moderate mitral regurgitation. Her left atrium is dilated.

What is the most appropriate initial treatment option?


Amiodarone

O Atenolol
O DC shock
Mitral valve replacement

O Percutaneous mitral valvotomy

A 52-year-old woman, with a prior history of rheumatic fever, presents with shortness of breath on strenuous
exertion while working as a landscape gardener. She is in permanent atrial fibrillation and is on long-term warfarin
and digoxin (125 pg once daily). Clinical examination reveals her to be in atrial fibrillation at a rate of around 150
bpm. Echo demonstrates preserved left ventricular function, a heavily calcified mitral valve with moderate mitral

stenosis (mitral valve area 1.5 cm2) and moderate mitral regurgitation. Her left atrium is dilated.

What is the most appropriate initial treatment option?


Amiodarone

Atenolol CORRECT ANSWER


DC shock

Mitral valve replacement


Percutaneous mitral valvotomy

Mitral valve disease


This woman has moderate, mixed mitral valve disease and therefore surgery is not currently indicated; it may,

however, be required later if features of left heart failure progress


Even if the mitral stenosis were to be severe, the presence of heavy calcification of the valve and concomitant
mitral regurgitation would preclude percutaneous valvotomy
She is in permanent atrial fibrillation and as such, by definition, sinus rhythm cannot be restored (as opposed to
persistent or paroxysmal)
Better rate control is required
Digoxin on its own may not control catecholamine-driven tachycardia (eg during exertion)
Amiodarone, while effective, would not be the first choice for this young patient working outdoors because of its
side-effect profile

A 63-year-old man with known chronic heart failure is admitted with symptoms at rest. Examination

reveals pitting oedema to his knees, elevated jugular venous pressure and basal crepitations. He is
in sinus rhythm at a rate of 80 bpm and his blood pressure is 100/60 mmHg. Current medication

includes bisoprolol 10 mg once daily, frusemide 80 mg once daily and ramipril 2.5 mg twice daily.
Blood tests reveal a sodium concentration of 133 mmol/litre, potassium 4.9 mmol/litre and
creatinine of 169 pmol/litre. The admitting doctor commences him on iv frusemide 80 mg twice
daily and increases his ramipril to 5 mg twice daily. When you review him the following day what
other drug would be most appropriate to include?

O Amiloride 5 mg od
O Bendrofluazide 2.5 mg od

O Bumetanide 2 mg bd
O Metolazone 5 mg od
O Spironolactone 25 mg od

A 63-year-old man with known chronic heart failure is admitted with symptoms at rest. Examination

reveals pitting oedema to his knees, elevated jugular venous pressure and basal crepitations. He is

in sinus rhythm at a rate of 80 bpm and his blood pressure is 100/60 mmHg. Current medication
includes bisoprolol 10 mg once daily, frusemide 80 mg once daily and ramipril 2.5 mg twice daily.
Blood tests reveal a sodium concentration of 133 mmol/litre, potassium 4.9 mmol/litre and
creatinine of 169 pmol/litre. The admitting doctor commences him on iv frusemide 80 mg twice
daily and increases his ramipril to 5 mg twice daily. When you review him the following day what
other drug would be most appropriate to include?
Amiloride 5 mg od
Bendrofluazide 2.5 mg od

Bumetanide 2 mg bd

Metolazone 5 mg od
Spironolactone 25 mg od CORRECT ANSWER

Management of decompensated chronic heart failure


This man has decompensated CHF with symptoms at rest (New York Heart Association class IV)
Examination has revealed significant fluid retention
The initial management plan of changing to iv frusemide is sensible since coexistent gut wall oedema is likely to
impinge on oral absorption, and increasing ramipril, a vasodilator, is also a sensible approach

The addition of a thiazide (inhibiting sodium reabsorption in distal tubule) may work in synergy with a loop diuretic;
the same is true for metolazone

Spironolactone, an aldosterone antagonist, has been shown to improve the mortality rate and symptoms and
reduce hospitalisation in patients with severe CHF already on conventional treatment
Benefits are in addition to its diuretic effect since aldosterone itself has adverse effects on myocardial structure
and function
Clearly, careful monitoring of renal function and biochemistry is important in such patients

A 72-year-old white man is referred to outpatients with a 6-month history of progressive exertional dyspnoea. His
ankles swell as the day progresses. There is no associated chest discomfort. He is an ex-smoker of 3 years and
drinks 12 pints of beer per week. He has not seen his GP in the previous 15 years. The only past history is that of
mild asthma as a child. His father died of a myocardial infarct aged 65 years. Blood pressure is 150/86 mmHg.
Results of investigations are as follows: renal function, normal; cholesterol, 6.8 mmol/l; ECG, sinus rhythm LBBB;
echo, dilated and impaired left ventricular function with ejection fraction of 30%, mild to moderate mitral

regurgitation, no LVH.

What is the most likely underlying aetiology?


Alcohol

O Coronary artery disease


O Hypertension

O Valvular heart disease


O Viral myocarditis

A 72-year-old white man is referred to outpatients with a 6-month history of progressive exertional dyspnoea. His
ankles swell as the day progresses. There is no associated chest discomfort. He is an ex-smoker of 3 years and

drinks 12 pints of beer per week. He has not seen his GP in the previous 15 years. The only past history is that of
mild asthma as a child. His father died of a myocardial infarct aged 65 years. Blood pressure is 150/86 mmHg.
Results of investigations are as follows: renal function, normal; cholesterol, 6.8 mmol/l; ECG, sinus rhythm LBBB;
echo, dilated and impaired left ventricular function with ejection fraction of 30%, mild to moderate mitral
regurgitation, no LVH.

What is the most likely underlying aetiology?


Alcohol

Coronary artery disease CORRECT ANSWER


Hypertension

Valvular heart disease


Viral myocarditis

Coronary artery disease


It is important to remember that chronic heart failure is not an absolute diagnosis; there is a cause to be found
In the UK the commonest aetiology is underlying ischaemic heart disease (around 50-66% of all cases)
This patient has several risk factors for ischaemic heart disease, which adds weight to the likelihood of this being
the diagnosis

LBBB is also more likely to be ischaemic in origin


It is of course important to exclude other potentially reversible causes such as
thyroid disease
ethanol
nutritional deficiencies
primary valvular pathology
uncontrolled prolonged tachycardia

The mitral regurgitation in this patient is most likely to be secondary to left ventricular dilatation and subsequent
annular dilatation of the mitral valve ring

This will contribute to the development of pulmonary hypertension

A 40-year-old woman presents with a 3-month history of fatigue, weight loss, night sweats and a degree of
exertional dyspnoea. Her past history includes a prosthetic mitral valve replacement 2.5 years ago. She is
pyrexia! with evidence of mitral regurgitation and splinter haemorrhages. Echo confirms moderate paravalvular
mitral regurgitation. Blood cultures are taken and a diagnosis of infective endocarditis made.

What is the most likely infecting organism in this case?

Coxiella burnetii

O Enterococcus spp
O Staphylococcus aureus
Staphylococcus epidermidls
Streptococcus viridans

A 40-year-old woman presents with a 3-month history of fatigue, weight loss, night sweats and a degree of

exertional dyspnoea. Her past history includes a prosthetic mitral valve replacement 2.5 years ago. She is
pyrexial with evidence of mitral regurgitation and splinter haemorrhages. Echo confirms moderate paravalvular
mitral regurgitation. Blood cultures are taken and a diagnosis of infective endocarditis made.

What is the most likely infecting organism in this case?


Coxiella burnetii

Enterococcus spp
Staphylococcus aureus
Staphylococcus epidermidis
Streptococcus viridans CORRECT ANSWER

Infections causing endocarditis


The commonest infective cause of native valve, sub-acute endocarditis in the UK is still Streptococcus viridans ,
accounting for around 40% of all cases

Enterococcus spp accounts for approximately 10% of cases and is more prevalent in the elderly
Staphylococcus spp account for around 25% of cases of endocarditis
In the first year following prosthetic valve replacement the spectrum of infecting organisms is somewhat different,
with coagulase-negative staphylococci being the most common (around 50%)
The majority of these are Staphylococcus epidermidis
After the first year following valvular surgery the spectrum of infecting organisms is very similar to that for native
valve endocarditis

A 50-year-old man with long-standing hypertension presents acutely with severe chest pain

radiating through to his back. He looks unwell, with a resting tachycardia (110 bpm) and blood

pressure of 150/96 mmHg. There are no murmurs and neurological examination is normal. An
urgent CT scan of his chest confirms type-A aortic dissection. The local cardiothoracic centre is
contacted and urgent transfer arranged. He has received appropriate opiate analgesia. What
additional drug treatment should be instigated as part of his immediate treatment plan?

O Intravenous GTN
Intravenous labetalol

O Intravenous nitroprusside
Oral amlodipine

O Oral enalapril

A 50-year-old man with long-standing hypertension presents acutely with severe chest pain

radiating through to his back. He looks unwell, with a resting tachycardia (110 bpm) and blood
pressure of 150/96 mmHg. There are no murmurs and neurological examination is normal. An
urgent CT scan of his chest confirms type-A aortic dissection. The local cardiothoracic centre is
contacted and urgent transfer arranged. He has received appropriate opiate analgesia. What

additional drug treatment should be instigated as part of his immediate treatment plan?

Intravenous GTN
Intravenous labetalol CORRECT ANSWER

Intravenous nitroprusside
Oral amlodipine

Oral enalapril

Type-A aortic dissection


Management
Type-A dissection involves the ascending aorta, and treatment with urgent surgical intervention is recommended
unless severe co-morbidity would preclude the patient from surgery
Immediate management includes the liberal use of opiates for complete pain relief, thereby decreasing the
sympathetic drive

Meticulous blood pressure control is vital while awaiting surgery/transfer, in an attempt to reduce the chances of
extension or rupture

Drug treatment
Short-acting intravenous (3-blockers (eg labetalol) are the first choice drugs as they reduce both blood pressure
and force of ejection

A short half-life is important since haemodynamics can change rapidly


If (3-blockers are contraindicated then sodium nitroprusside or calcium-channel blockers may be appropriate
alternatives

A 40-year-old woman presents with a 1-year history of increasing exertional dyspnoea and swollen ankles. Her GP
has performed an ECG, which shows evidence of right ventricular hypertrophy and right axis deviation.
Examination in clinic reveals the following: body mass index of 30, sinus rhythm 90 bpm, blood pressure 110/60
mmHg, elevated jugular venous pressure, left parasternal heave, peripheral oedema and a loud P2. There is no
prior medical or family history of note. She is a non-smoker and drinks little alcohol. Whilst she denies current
drug use, she has in the past used stimulant drugs purchased over the Internet to help her lose weight (previous
body mass index of 34).

What is the likely diagnosis?

O Isolated cor pulmonale


O Chronic pulmonary emboli
O Pulmonary hypertension in the absence of thromboemboli
O Mitral stenosis
O Tricuspid valve endocarditis

A 40-year-old woman presents with a 1-year history of increasing exertional dyspnoea and swollen ankles. Her GP
has performed an ECG, which shows evidence of right ventricular hypertrophy and right axis deviation.
Examination in clinic reveals the following: body mass index of 30, sinus rhythm 90 bpm, blood pressure 110/60
mmHg, elevated jugular venous pressure, left parasternal heave, peripheral oedema and a loud P2. There is no
prior medical or family history of note. She is a non-smoker and drinks little alcohol. Whilst she denies current
drug use, she has in the past used stimulant drugs purchased over the Internet to help her lose weight (previous
body mass index of 34).

What is the likely diagnosis?


Isolated cor pulmonale
Chronic pulmonary emboli

Pulmonary hypertension in the absence of thromboemboli CORRECT ANSWER


Mitral stenosis

Tricuspid valve endocarditis

Pulmonary hypertension
This woman has clinical features of pulmonary hypertension
The onset of symptoms (eg dyspnoea, syncope, chest pain) is usually insidious
Other clinical findings may include
murmurs of tricuspid or pulmonary regurgitation
ascites
hepatomegaly
It is important to exclude a secondary cause such as
chronic pulmonary emboli
underlying collagen vascular disease
left-sided heart disease
chronic obstructive pulmonary disease
Primary pulmonary hypertension is diagnosed by exclusion and is around three times more common in females
The aetiology is uncertain but a genetic component with additional triggering factors seems likely
There appears to be a clear-cut relationship between the use of appetite suppressants (fenfluramine) and the
development of primary pulmonary hypertension, which would fit with this scenario

Cardiac catheterisation is performed on a 25-year-old man with a systolic murmur but no symptoms. ECG and
chest X-ray are normal. The findings are as follows (pressures mmHg): aorta, 125/70; left ventricle, 120/12; right
atrium, mean 8; right ventricle, 40/8; pulmonary artery, 44/14; pulmonary capillary wedge, mean 13. Saturations
(%): aorta, 97; superior vena cava, 70; right atrium, 70; right ventricle, 82; pulmonary artery, 85.

What is the most likely cardiac diagnosis?

- primum
ASD - secundum
ASD

0 Mitral stenosis
O Primary pulmonary hypertension
O Ventricular septal defect

Cardiac catheterisation is performed on a 25-year-old man with a systolic murmur but no symptoms. ECG and
chest X-ray are normal. The findings are as follows (pressures mmHg): aorta, 125/70; left ventricle, 120/12; right
atrium, mean 8; right ventricle, 40/8; pulmonary artery, 44/14; pulmonary capillary wedge, mean 13. Saturations
(%): aorta, 97; superior vena cava, 70; right atrium, 70; right ventricle, 82; pulmonary artery, 85.

What is the most likely cardiac diagnosis?

- primum
ASD - secundum
ASD

Mitral stenosis

Primary pulmonary hypertension


Ventricular septal defect CORRECT ANSWER

Ventricular septal defect


The catheter data demonstrate a step-up in saturations from the right atrium to the right ventricle, in keeping with a
left to right shunt at the level of the ventricle

There is a mild elevation in pulmonary artery pressure in keeping with the shunt
Ventricular septal defect (VSD) is the commonest form of congenital heart disease
In adults a small defect may present as an asymptomatic murmur; or in the extreme as Eisenmenger syndrome,
where reversal of a left to right shunt has occurred as a consequence of advanced pulmonary hypertension
VSD is associated with an increased risk of endocarditis although recent NICE guidelines do not say that routine
antibiotic prophylaxis is recommended

Management depends upon the actual size of the shunt

A 65-year-old man presents to the Emergency Department with severe chest pain. ECG shows anterior STsegment elevation and he receives prompt thrombolysis with reteplase with good resolution of changes. He is
commenced on aspirin, a ft-blocker, an ACE inhibitor and a statin. His initial progress is complicated by further
pain, worse with inspiration and movement and relieved by non-steroidal drugs. You are called to see him on day
5 postinfarct when he complains of shortness of breath on walking to the bathroom. He looks unwell with a cool
periphery and resting tachycardia. Blood pressure is reduced at 90/50 mmHg. Jugular venous pressure is
elevated to around 8 cm and rises with inspiration. His ECG shows preserved R wave progression, although
complexes appear diminished in size, and anterolateral T-wave inversion together with sinus tachycardia. Chest
X-ray shows an increase in the cardiothoracic ratio but clear lung fields.

What is the most likely complication to have developed to account for this deterioration?

O Cardiogenic shock
Q Mitral regurgitation

O Pericardial tamponade
Pulmonary embolism

O Ventricular septal defect

A 65-year-old man presents to the Emergency Department with severe chest pain. ECG shows anterior STsegment elevation and he receives prompt thrombolysis with reteplase with good resolution of changes. He is
commenced on aspirin, a ft-blocker, an ACE inhibitor and a statin. His initial progress is complicated by further
pain, worse with inspiration and movement and relieved by non-steroidal drugs. You are called to see him on day
5 postinfarct when he complains of shortness of breath on walking to the bathroom. He looks unwell with a cool
periphery and resting tachycardia. Blood pressure is reduced at 90/50 mmHg. Jugular venous pressure is

elevated to around 8 cm and rises with inspiration. His ECG shows preserved R wave progression, although
complexes appear diminished in size, and anterolateral T-wave inversion together with sinus tachycardia. Chest
X-ray shows an increase in the cardiothoracic ratio but clear lung fields.

What is the most likely complication to have developed to account for this deterioration?

Cardiogenic shock
Mitral regurgitation

Pericardial tamponade CORRECT ANSWER

Pulmonary embolism
Ventricular septal defect

Pericardial tamponade
Devastating complications still do occur following acute myocardial infarction
Cardiogenic shock tends to occur early following a large infarct (or in the presence of already impaired left
ventricular function), typically in the first 24-48 hours
The development of acute mitral regurgitation or ventricular septal defect is associated with severe pulmonary
oedema
While they would give rise to an elevation of jugular venous pressure, Kussmaul's sign would not be present
(increase in jugular venous pressure with inspiration)
Although pericarditis is common following a myocardial infarction (Ml), and in particular an anterior Ml, tamponade
occurs relatively infrequently

In tamponade the chest X-ray shows a large heart with normal pulmonary vasculature
Echocardiography is vital to assist in the management of such a patient and permits easy differentiation of the
possible causes of haemodynamic collapse

The markedly elevated JVP which rises with inspiration fits best with tamponade

A 54-year-old man is referred with increased swelling of his ankles and abdomen, and a degree of shortness of
breath on exertion. His jugular venous pressure is elevated with prominent x- and y-descents. Apex beat is
normal. ECG shows atrial fibrillation with widespread non-specific ST-segment abnormalities. Echo reveals
preserved left ventricular systolic function with biatrial enlargement and an estimated pulmonary artery systolic
pressure of around 60 mmHg. Chest X-ray shows atrial enlargement but no other abnormalities.

What is the most likely cardiac diagnosis?

O Chronic pulmonary emboli


Dilated cardiomyopathy

O Restrictive cardiomyopathy
O Secundum ASD
O Tricuspid regurgitation

A 54-year-old man is referred with increased swelling of his ankles and abdomen, and a degree of shortness of

breath on exertion. His jugular venous pressure is elevated with prominent x- and y-descents. Apex beat is
normal. ECG shows atrial fibrillation with widespread non-specific ST-segment abnormalities. Echo reveals
preserved left ventricular systolic function with biatrial enlargement and an estimated pulmonary artery systolic
pressure of around 60 mmHg. Chest X-ray shows atrial enlargement but no other abnormalities.

What is the most likely cardiac diagnosis?


Chronic pulmonary emboli

Dilated cardiomyopathy

Restrictive cardiomyopathy CORRECT ANSWER


Secundum ASD
Tricuspid regurgitation

Restrictive cardiomyopathy
Restrictive cardiomyopathy results from fibrosis or infiltration of the endo- or myocardium
The result is failure of the ventricles to relax, with a subsequent increase in ventricular end-diastolic pressures
leading on to biatrial enlargement

Systolic function is normal

Causes
Underlying causes include
amyloidosis
storage disorders
sarcoidosis
haemochromatosis
endomyocardial fibrosis

Differentiation
It can be very difficult to differentiate restrictive cardiomyopathy from constrictive pericarditis
In restriction the pulmonary artery systolic pressure is usually elevated to > 45 mmHg, while it is lower in
constriction
Right and left heart catheter may aid differentiation

Other notes
Symptoms are usually those of predominant right heart failure and atrial fibrillation is common
The ECG may be normal but diffuse ST-segment and T-wave changes are commonly seen
Diuretics are the mainstay of symptomatic treatment

A 70-year-old woman is admitted to hospital with a swollen left leg 4 weeks after undergoing an elective total hip
replacement. An above-knee DVT is diagnosed by ultrasound. She is in sinus rhythm at 60 bpm and her blood pressure is
160/80 mmHg. She is commenced on the appropriate dose of low molecular weight heparin and warfarin loading. The
following day she becomes acutely short of breath. Examination reveals a resting tachycardia (110 bpm) with blood
pressure of 100/60 mmHg. Her jugular venous pressure is elevated at 7 cm above the sternal notch. Arterial blood gas
measurement reveals her to be hypoxaemic with a pa(O2) of 7 kPa.

What would be the first-line therapy after administering high-flow oxygen?

O Aspirin
O Intravenous heparin
O Surgical embolectomy
Thrombolysis with reteplase

O Vena caval filter

A 70-year-old woman is admitted to hospital with a swollen left leg 4 weeks after undergoing an elective total hip
replacement. An above-knee DVT is diagnosed by ultrasound. She is in sinus rhythm at 60 bpm and her blood pressure is
160/80 mmHg. She is commenced on the appropriate dose of low molecular weight heparin and warfarin loading. The
following day she becomes acutely short of breath. Examination reveals a resting tachycardia (110 bpm) with blood
pressure of 100/60 mmHg. Her jugular venous pressure is elevated at 7 cm above the sternal notch. Arterial blood gas
measurement reveals her to be hypoxaemic with a pa(O2) of 7 kPa.

What would be the first-line therapy after administering high-flow oxygen?

Aspirin
Intravenous heparin

Surgical embolectomy

Thrombolysis with reteplase CORRECT ANSWER


Vena caval filter

Pulmonary embolism
This patient has clinical features of a massive pulmonary embolus
This results from significant obstruction of the pulmonary arteries causing haemodynamic compromise - namely
shock or systemic hypotension (systolic blood pressure < 90 mmHg or a drop of > 40 mmHg for > 15 minutes)

Treatment
The initial treatment of choice is thrombolysis using a recognised protocol
Whilst she is only 4 weeks out from her hip replacement, the benefits of thrombolysis would outweigh the risks in
this case

Inotropic support and the judicious use of fluids may also be required in the interim
Subsequent intravenous unfractionated heparin should then be commenced.

A 54-year-old man is referred with increased swelling of his ankles and abdomen, and a degree of shortness of
breath on exertion. His jugular venous pressure is elevated with prominent x- and y-descents. Apex beat is
normal. ECG shows atrial fibrillation with widespread non-specific ST-segment abnormalities. Echo reveals
preserved left ventricular systolic function with biatrial enlargement and an estimated pulmonary artery systolic
pressure of around 60 mmHg. Chest X-ray shows atrial enlargement but no other abnormalities.

What is the most likely cardiac diagnosis?

O Chronic pulmonary emboli


O Dilated cardiomyopathy
O Restrictive cardiomyopathy
O Secundum ASD

0 Tricuspid regurgitation

A 54-year-old man is referred with increased swelling of his ankles and abdomen, and a degree of shortness of

breath on exertion. His jugular venous pressure is elevated with prominent x- and y-descents. Apex beat is
normal. ECG shows atrial fibrillation with widespread non-specific ST-segment abnormalities. Echo reveals
preserved left ventricular systolic function with biatrial enlargement and an estimated pulmonary artery systolic
pressure of around 60 mmHg. Chest X-ray shows atrial enlargement but no other abnormalities.

What is the most likely cardiac diagnosis?


Chronic pulmonary emboli

Dilated cardiomyopathy

Restrictive cardiomyopathy CORRECT ANSWER


Secundum ASD
Tricuspid regurgitation

Restrictive cardiomyopathy
Restrictive cardiomyopathy results from fibrosis or infiltration of the endo- or myocardium
The result is failure of the ventricles to relax, with a subsequent increase in ventricular end-diastolic pressures
leading on to biatrial enlargement

Systolic function is normal

Causes
Underlying causes include
amyloidosis
storage disorders
sarcoidosis
haemochromatosis
endomyocardial fibrosis

Differentiation
It can be very difficult to differentiate restrictive cardiomyopathy from constrictive pericarditis
In restriction the pulmonary artery systolic pressure is usually elevated to > 45 mmHg, while it is lower in

constriction
Right and left heart catheter may aid differentiation

Differentiation
It can be very difficult to differentiate restrictive cardiomyopathy from constrictive pericarditis
In restriction the pulmonary artery systolic pressure is usually elevated to > 45 mmHg, while it is lower in
constriction
Right and left heart catheter may aid differentiation

Other notes
Symptoms are usually those of predominant right heart failure and atrial fibrillation is common
The ECG may be normal but diffuse ST-segment and T-wave changes are commonly seen
Diuretics are the mainstay of symptomatic treatment

A 70-year-old woman is admitted to hospital with a swollen left leg 4 weeks after undergoing an elective total hip
replacement. An above-knee DVT is diagnosed by ultrasound. She is in sinus rhythm at 60 bpm and her blood pressure is
160/80 mmHg. She is commenced on the appropriate dose of low molecular weight heparin and warfarin loading. The
following day she becomes acutely short of breath. Examination reveals a resting tachycardia (110 bpm) with blood
pressure of 100/60 mmHg. Her jugular venous pressure is elevated at 7 cm above the sternal notch. Arterial blood gas
measurement reveals her to be hypoxaemic with a pa(O2) of 7 kPa.

What would be the first-line therapy after administering high-flow oxygen?

Aspirin

O Intravenous heparin
O Surgical embolectomy
O Thrombolysis with reteplase
O Vena caval filter

A 70-year-old woman is admitted to hospital with a swollen left leg 4 weeks after undergoing an elective total hip
replacement. An above-knee DVT is diagnosed by ultrasound. She is in sinus rhythm at 60 bpm and her blood pressure is

160/80 mmHg. She is commenced on the appropriate dose of low molecular weight heparin and warfarin loading. The
following day she becomes acutely short of breath. Examination reveals a resting tachycardia (110 bpm) with blood

pressure of 100/60 mmHg. Her jugular venous pressure is elevated at 7 cm above the sternal notch. Arterial blood gas
measurement reveals her to be hypoxaemic with a pa(O2) of 7 kPa.

What would be the first-line therapy after administering high-flow oxygen?


Aspirin

Intravenous heparin

Surgical embolectomy

Thrombolysis with reteplase CORRECT ANSWER


Vena caval filter

Pulmonary embolism
This patient has clinical features of a massive pulmonary embolus
This results from significant obstruction of the pulmonary arteries causing haemodynamic compromise - namely
shock or systemic hypotension (systolic blood pressure < 90 mmHg or a drop of > 40 mmHg for > 15 minutes)

Treatment
The initial treatment of choice is thrombolysis using a recognised protocol
Whilst she is only 4 weeks out from her hip replacement, the benefits of thrombolysis would outweigh the risks in
this case

Inotropic support and the judicious use of fluids may also be required in the interim
Subsequent intravenous unfractionated heparin should then be commenced.

A 70-year-old woman with long-standing hypertension is referred to outpatients with a diagnosis of asymptomatic

atrial fibrillation. Echocardiography demonstrates normal left ventricular function, mild LVH and normal mitral
valve structure. The left atrium is slightly enlarged (4.2 cm). She is not keen on cardioversion and her rate is well
controlled at 70 bpm.

What would be the optimal strategy for long-term anticoagulation?


O Aspirin
Clopidogrel

O Dipyridamole
O Low molecular-weight heparin
O Warfarin

A 70-year-old woman with long-standing hypertension is referred to outpatients with a diagnosis of asymptomatic
atrial fibrillation. Echocardiography demonstrates normal left ventricular function, mild LVH and normal mitral
valve structure. The left atrium is slightly enlarged (4.2 cm). She is not keen on cardioversion and her rate is well
controlled at 70 bpm.

What would be the optimal strategy for long-term anticoagulation?


Aspirin

Clopidogrel

Dipyridamole
Low molecular-weight heparin
Warfarin CORRECT ANSWER

Atrial fibrillation
Atrial fibrillation (AF) is common and affects around 2-5% of the population who are over 60 years old

Risk factors
It confers an approximate fivefold increased risk of stroke
The absolute risk of stroke is related to the coexistence of other cardiovascular disease

Treatment
In patients with AF and additional risk factors for stroke, such as hypertension, warfarin has been shown to be
superior to antiplatelet therapy (primarily aspirin)
A Cochrane systemic review has concluded clear superiority of warfarin over anti-platelet agents in non-valvular
AF
warfarin could be expected to prevent 30 strokes at the expense of 6 major bleeding events
aspirin had some benefit in stroke prevention but was clearly not as efficacious in patients at high risk of stroke
the use of low-dose warfarin or low-dose warfarin combined with aspirin was found to be of little benefit for
stroke prevention
newer agents such as low molecular weight heparin appear to be beneficial and require further study

Other notes
This patient has evidence of structural cardiac disease with left ventricular hypertrophy and an enlarged left atrium,
thereby reflecting a higher risk of developing a thromboembolic complication

A 30-year-old woman is routinely seen by her GP 24 weeks into her first pregnancy. She is well without adverse
symptoms. Her blood pressure is 150/96 mmHg. Her baseline blood pressure at booking was 136/84 mmHg. No

other abnormalities are found.

What drug therapy would you prescribe?

Bendrofluazide
Moxonidirie
Labetalol
O Losartan
Ramipril

A 30-year-old woman is routinely seen by her GP 24 weeks into her first pregnancy. She is well without adverse

symptoms. Her blood pressure is 150/96 mmHg. Her baseline blood pressure at booking was 136/84 mmHg. No

other abnormalities are found.

What drug therapy would you prescribe?


Bendrofluazide

Moxonidine
Labetalol CORRECT ANSWER

Losartan
Ramipril

Hypertension in pregnancy
Hypertension in pregnancy is defined as a blood pressure > 140/90 mmHg or a rise of 25 mmHg of systolic and/or
15 mmHg of diastolic pressure above baseline
It is seen in around 10% of all pregnancies
Gestational hypertension is more common than pre-eclampsia, which is associated with maternal organ
dysfunction

Treatment
While the latter is treated by delivery, drug therapy is often required to treat gestational hypertension
First-line agents, proven to be safe in pregnancy, include
labetalol
methyldopa
Second-line agents include
nifedipine
hydralazine
prazosin
Thiazides, angiotensin-converting enzyme (ACE) inhibitors and angiotensin receptor blockers are not
recommended for use in pregnancy owing to the risk of adverse effects on the developing fetus

A 65-year-old man is referred to out-patients with resistant hypertension. He is already taking

bendrofluazide 2.5 mg once daily, lisinopril 20 mg once daily and amlodipine 10 mg once daily. He
is an ex-smoker with a past history of uncomplicated myocardial infarction. Blood pressure is
170/100 mmHg in both arms. The only other abnormality on examination is a left femoral bruit.
Results of investigations are as follows: LVH on ECG; creatinine, 165 pmol/l; sodium, 138 mmol/l;

potassium, 3.9 mmol/l; chest X-ray, normal; 24-hour blood pressure, sustained systolic and
diastolic hypertension with no evidence of nocturnal dip. What is the most likely underlying
aetiology for his hypertension?

Coarctation

0 Conn syndrome
Cushing syndrome

O Polycystic kidney disease


O Renal artery stenosis

A 65-year-old man is referred to out-patients with resistant hypertension. He is already taking

bendrofluazide 2.5 mg once daily, lisinopril 20 mg once daily and amlodipine 10 mg once daily. He
is an ex-smoker with a past history of uncomplicated myocardial infarction. Blood pressure is
170/100 mmHg in both arms. The only other abnormality on examination is a left femoral bruit.
Results of investigations are as follows: LVH on ECG; creatinine, 165 pmol/l; sodium, 138 mmol/l;
potassium, 3.9 mmol/l; chest X-ray, normal; 24-hour blood pressure, sustained systolic and
diastolic hypertension with no evidence of nocturnal dip. What is the most likely underlying
aetiology for his hypertension?

Coarctation
Conn syndrome

Cushing syndrome

Polycystic kidney disease


Renal artery stenosis CORRECT ANSWER

Renal artery stenosis


A secondary cause for hypertension is more likely in patients with resistant hypertension and in those who fail to
show a nocturnal dip (usually an approximately 20% drop)
Other clinical and investigative findings may raise the index of suspicion
In this case, renovascular disease should be suspected since he has documented evidence of co-morbid vascular
disease and arterial bruit
A discrepancy in renal size on ultrasound would add further weight to the diagnosis
Further imaging, such as angiography or magnetic resonance angiography, should be considered in patients with a
high index of suspicion for renovascular disease

A 70-year-old man is referred by his GP for advice regarding optimisation of secondary prevention. He has a

history of angina, with excellent control of symptoms on a combination of aspirin, dipyridamole MR, atenolol 50
mg od, simvastatin 40 mg od and isosorbide mononitrate 20 mg bd. His pulse rate is 70 bpm and blood pressure

is 144/86 mmHg. The only other relevant past history includes an ischaemic stroke 2 years ago from which he
made a complete recovery.

What additional therapy would you consider adding?

O Bendroflumethiazide

O Diltiazem
O Doxazosin
O Nicorandil

0 Perindopril

A 70-year-old man is referred by his GP for advice regarding optimisation of secondary prevention. He has a

history of angina, with excellent control of symptoms on a combination of aspirin, dipyridamole MR, atenolol 50
mg od, simvastatin 40 mg od and isosorbide mononitrate 20 mg bd. His pulse rate is 70 bpm and blood pressure

is 144/86 mmHg. The only other relevant past history includes an ischaemic stroke 2 years ago from which he
made a complete recovery.

What additional therapy would you consider adding?


Bendroflumethiazide

Diltiazem
Doxazosin
Nicorandil

Perindopril CORRECT ANSWER

Treating vascular disease


Most clinicians would now recommend the addition of angiotensin-converting enzyme (ACE) inhibitors for patients

with vascular disease, irrespective of left ventricular function


This is based on evidence from large trials, such as PROGRESS (perindopril) and HOPE (ramipril)
Benefits of ACE inhibition seem to not be purely related to a reduction in blood pressure; beneficial local vascular
and myocardial effects are also seen
Blood pressure is not yet optimised in this patient and further antihypertensive therapy is warranted
The addition of an ACE inhibitor should bring this to the desired level (< 140/85 mmHg)

An 81-year-old woman is referred to cardiology out-patients with a history of dizzy episodes and one episode of

syncope. She is known to have long-standing atrial fibrillation. A 24-hour tape confirms atrial fibrillation, with
rates varying from 30 to 140 bpm. There are several daytime pauses of over 3 seconds. Sinus node disease

appears to be the most likely diagnosis. She is listed for a permanent pacemaker.

Which one of the following would be the most appropriate device?

O DDD

DDI
O VOO

Wl
VDD

An 81-year-old woman is referred to cardiology out-patients with a history of dizzy episodes and one episode of
syncope. She is known to have long-standing atrial fibrillation. A 24-hour tape confirms atrial fibrillation, with
rates varying from 30 to 140 bpm. There are several daytime pauses of over 3 seconds. Sinus node disease
appears to be the most likely diagnosis. She is listed for a permanent pacemaker.

Which one of the following would be the most appropriate device?

DDD
DDI
VOO

Wl CORRECT ANSWER
VDD

Use of pacemakers in atrial fibrillation


Definition
Pacemaker types are defined by an international code, which uses three or more letters in a set sequence
The first letter is related to the chamber that is to be paced: A = atrium, V = ventricle, D = both
The second letter refers to the chamber that is sensed (A, V or D)
The third letter refers to the response to a sensed beat by the pacemaker: I = inhibits, T = trigger or D = both (ie
either inhibits or triggers)
VOO is a fixed output setting (eg pacing at 60 beats per minute irrespective of intrinsic activity); the 'O' as
second and third symbol implies that the chamber is not sensed and therefore there is no response to a sensed
beat

A fourth letter refers to whether or not the pacemaker has rate-adaptive properties (R)

Case study

This woman has atrial fibrillation and as such there is no need for an atrial lead
Wl means there is one lead in the ventricle (pacing and sensing the ventricle)
If the pacemaker senses an intrinsic QRS complex then pacing is inhibited (I)
This means the pacemaker will only deliver a stimulus when there is a significant pause

An 18-year-old young man presents to the Emergency Department having developed palpitations while playing

football. ECG shows rapid atrial fibrillation with a ventricular rate of around 250 bpm. QRS duration is prolonged
at around 130 ms. DC cardioversion is performed. Subsequent ECG in sinus rhythm demonstrates a PR interval
of 100 ms, positive R wave in V1 and the presence of a delta wave.

What further treatment would you recommend?

O Atrial defibrillator implantation


Intravenous and then oral loading with amiodarone

O Radiofrequency ablation of the accessory pathway


O Radiofrequency ablation of the AV node
O Surgical ablation of the accessory pathway

An 18-year-old young man presents to the Emergency Department having developed palpitations while playing
football. ECG shows rapid atrial fibrillation with a ventricular rate of around 250 bpm. QRS duration is prolonged
at around 130 ms. DC cardioversion is performed. Subsequent ECG in sinus rhythm demonstrates a PR interval
of 100 ms, positive R wave in V1 and the presence of a delta wave.

What further treatment would you recommend?


Atrial defibrillator implantation

Intravenous and then oral loading with amiodarone


Radiofrequency ablation of the accessory pathway CORRECT ANSWER

Radiofrequency ablation of the AV node


Surgical ablation of the accessory pathway

Arrhythmias associated with Wolff-Parkinson-White syndrome


This young man has Wolff-Parkinson-White (WPW) syndrome
The most common arrhythmia is an atrioventricular re-entry tachycardia (AVRT)
This is a narrow complex with anterograde conduction through the AV node and retrograde conduction via the
accessory pathway

Patients who develop atrial fibrillation are at risk of rapid anterograde conduction to the ventricles via the
accessory pathway, and this may subsequently degenerate to ventricular fibrillation

The extremely rapid conduction with broad QRS duration is typical of this complication
Radiofrequency ablation of the accessory pathway is recommended in this setting and is potentially curative

A 70-year-old man is brought into the Emergency Department. He is unwell with a cool periphery and blood
pressure of 70/40 mmHg. ECG shows a regular broad-complex tachycardia with a rate of 190 bpm. He is unable to
provide a clear history, but a recent prescription in his wallet shows that he is taking aspirin, ramipril, frusemide
and spironolactone. An arterial blood gas is performed in the Emergency Department shows a raised urea,
acidosis and raised potassium.

What is the likely arrhythmia?

O Atrial fibrillation
O Atrial flutter with a 2:1 block
SVT with aberrant conduction

VT
Wolff-Parkinson-White syndrome

A 70-year-old man is brought into the Emergency Department. He is unwell with a cool periphery and blood

pressure of 70/40 mmHg. ECG shows a regular broad-complex tachycardia with a rate of 190 bpm. He is unable to
provide a clear history, but a recent prescription in his wallet shows that he is taking aspirin, ramipril, frusemide
and spironolactone. An arterial blood gas is performed in the Emergency Department shows a raised urea,

acidosis and raised potassium.

What is the likely arrhythmia?


Atrial fibrillation
Atrial flutter with a 2:1 block
SVT with aberrant conduction

VT CORRECT ANSWER
Wolff-Parkinson-White syndrome

Ventricular tachycardia
This patient is haemodynamically compromised as a consequence of the arrhythmia
The prescription would suggest the presence of underlying cardiac disease with probable cardiac dysfunction
In the context of known ischaemic heart disease or left ventricular dysfunction, a broad-complex tachycardia
should be assumed to be ventricular tachycardia (VT) until proved otherwise

ECG criteria
Many ECG criteria exist to aid the differentiation of VT and supraventricular tachycardia (SVT) with aberrancy and
include

A-V dissociation
capture beats
fusion beats
extreme QRS axis
concordance across ventricular leads

A previous ECG can be exceedingly helpful

Other clinical signs


Subtle clinical signs, such as intermittent cannon waves in the jugular venous pressure and a variable first heart
sound may be present, in patients with A-V dissociation

A 68-year-old man is admitted with syncope. He is known to have ischaemic cardiomyopathy. His

medications include: aspirin 75 mg od, frusemide 80 mg bd and lisinopril 10 mg od. An initial ECG

shows sinus bradycardia (50 bpm) and RBBB. Results of blood tests are as follows: sodium, 134
mmol/litre; potassium, 3.5 mmol/litre; creatinine 124 pmol/litre. He has recurrent syncopal episodes
on the CCU, where monitoring shows episodes of non-sustained torsades de pointes (polymorphic
VT). Which one of the following would be your initial line of treatment?
DC cardioversion

O Intravenous amiodarone
O Intravenous magnesium
Oral metoprolol

O Temporary pacing

A 68-year-old man is admitted with syncope. He is known to have ischaemic cardiomyopathy. His
medications include: aspirin 75 mg od, frusemide 80 mg bd and lisinopril 10 mg od. An initial ECG

shows sinus bradycardia (50 bpm) and RBBB. Results of blood tests are as follows: sodium, 134

mmol/litre; potassium, 3.5 mmol/litre; creatinine 124 pmol/litre. He has recurrent syncopal episodes
on the CCU, where monitoring shows episodes of non-sustained torsades de pointes (polymorphic
VT). Which one of the following would be your initial line of treatment?

DC cardioversion
Intravenous amiodarone

Intravenous magnesium CORRECT ANSWER


Oral metoprolol

Temporary pacing

QT prolongation

Torsades de pointes (polymorphic VT with QRS complexes of different amplitude twisting around the isoelectric

line) occurs in patients with a prolonged QT interval

Predisposition
Any cause of QT prolongation can predispose to the arrhythmia
These include
congenital (the Jervell-Lange-Neilsen or Romano-Ward syndromes)
with QRS complexes of different amplitude
metabolic (hypocalcaemia, hypomagnesaemia or hypokalaemia)

drugs (eg amiodarone, tricyclic antidepressants, phenothiazines)


ischaemic heart disease
mitral valve prolapsed
The arrhythmia often occurs in the context of bradycardia
Treatment

The key here is that amiodarone may exacerbate the situation


Intravenous magnesium (even if the serum magnesium concentration is normal) is the first-line therapy
Temporary pacing at higher rates with or without f3>-blockers is the next line of therapy
DC shock would not be helpful since episodes are non-sustained

A 65-year-old man is admitted via the Emergency Department with acute shortness of breath. His past medical
history includes an anterior Ml 5 years ago. He is usually short of breath after walking approximately 400 m (0.25
mile), but is not on regular treatment. Clinically he is distressed: respiratory rate 30 bpm; basal crepitations to
mid-zones; saturations 90%; pulse rate 110 sinus; blood pressure 180/100 mm Hg. The patient has already been
given iv diamorphine 5 mg, iv metoclopramide and iv frusemide 40 mg (twice) but the patient remains short of
breath, although saturations have increased to 94% with high-flow oxygen. CXR confirms pulmonary oedema.

What further intravenous therapy would you commence?

O Atenolol iv
Dobutamine iv

O Dopamine iv
O GTN iv
O Milrinone iv

A 65-year-old man is admitted via the Emergency Department with acute shortness of breath. His past medical
history includes an anterior Ml 5 years ago. He is usually short of breath after walking approximately 400 m (0.25
mile), but is not on regular treatment. Clinically he is distressed: respiratory rate 30 bpm; basal crepitations to
mid-zones; saturations 90%; pulse rate 110 sinus; blood pressure 180/100 mm Hg. The patient has already been
given iv diamorphine 5 mg, iv metoclopramide and iv frusemide 40 mg (twice) but the patient remains short of
breath, although saturations have increased to 94% with high-flow oxygen. CXR confirms pulmonary oedema.

What further intravenous therapy would you commence?


Atenolol iv
Dobutamine iv

Dopamine iv
GTN iv CORRECT ANSWER

Milrinone iv

Pulmonary oedema
This patient has severe pulmonary oedema.

Treatment
Initial treatment includes sitting the patient up and administering high-flow oxygen
High catecholamine levels and activation of the renin-angiotensin-aldosterone systems drive the peripheral

vasoconstriction, which increases myocardial oxygen demand


Traditionally, treatment has been with diuretics, but the benefits probably relate to their vasodilatator actions
Early use of more powerful vasodilators represents a more attractive strategy and includes iv diamorphine and iv
GTN (short half-life)
Early administration of an oral ACE inhibitor would also be recommended
Inotropic agents should be considered in the context of cardiogenic shock (low blood pressure with impaired tissue
perfusion)

A 78-year-old woman presents to the Emergency Department with three episodes of syncope in the last 24 hours.
There is no history of chest pain. She is taking frusemide 80 mg od and ramipril 10 mg od for known
hypertension. She is conscious with a blood pressure of 100/40 mmHg. Potassium is 5.3 mmol/litre. Her ECG
shows complete heart block with rate of 40 bpm. QRS duration is 150 ms with a right bundle-branch block
configuration.

What is the optimum initial management?

O Dobutamine
O Isoprenaline
Intravenous calcium chloride
Q Temporary transvenous pacing

Withhold medication and observe

A 78-year-old woman presents to the Emergency Department with three episodes of syncope in the last 24 hours.
There is no history of chest pain. She is taking frusemide 80 mg od and ramipril 10 mg od for known

hypertension. She is conscious with a blood pressure of 100/40 mmHg. Potassium is 5.3 mmol/litre. Her ECG
shows complete heart block with rate of 40 bpm. QRS duration is 150 ms with a right bundle-branch block

configuration.

What is the optimum initial management?


Dobutamine
Isoprenaline

Intravenous calcium chloride

Temporary transvenous pacing CORRECT ANSWER


Withhold medication and observe

Complete heart block


Presenting complaint
This woman has complete heart block with an unstable escape rhythm
The latter is exemplified by the fact that she has already had three syncopal episodes
Her QRS duration is prolonged (normal up to 120 ms), and this is generally more unstable than an escape rhythm
of normal duration (ie <120 ms) since this originates from around the His bundle

Her blood pressure is low, particularly with a background of hypertension


In addition, it is important to remember that cardiac output will be influenced by heart rate

Treatment
In the elderly, cerebral vascular dysregulation may compound the effect thereby contributing to cerebral
hypoperfusion

Ideally she should receive a transvenous temporary pacemaker


If further acute problems occur while waiting for a transvenous temporary pacemaker (eg awaiting transfer to a
room with fluoroscopy), then external pacing can be instituted in the short term with appropriate sedation

A 50-year-old man presents with a 1-hour history of severe central chest pain. There is no significant past
medical history. He is haemodynamically stable with a pulse rate of 90 bpm and blood pressure of 120/70 mm Hg.

ECG shows 5 mm of ST-segment elevation in the anterior leads (V2-V4). He received aspirin 300 mg in the

ambulance and diamorphine 5 mg.

What would be the definitive therapy?

0 Clopidogrel 75 mg
O Enoxaparin
O Gilb/ila blocker
O Percutaneous coronary intervention
Tissue plasminogen activator

A 50-year-old man presents with a 1-hour history of severe central chest pain. There is no significant past

medical history. He is haemodynamically stable with a pulse rate of 90 bpm and blood pressure of 120/70 mm Hg.
ECG shows 5 mm of ST-segment elevation in the anterior leads (V2-V4). He received aspirin 300 mg in the
ambulance and diamorphine 5 mg.

What would be the definitive therapy?


Clopidogrel 75 mg
Enoxaparin

Gllb/lla blocker
Percutaneous coronary intervention CORRECT ANSWER

Tissue plasminogen activator

Anterior myocardial infarction


This relatively young man has presented early with acute anterior myocardial infarction (Ml)
The key therapeutic aim is early reperfusion in an attempt to save the myocardium

Treatment
In centres with rapid access to primary angioplasty this would be the optimum strategy and guidelines now suggest
this should be the norm in the UK
If angioplasty is not available then thrombolysis with tissue plasminogen activator is the next best alternative

A 52-year-old man undergoes Bruce-protocol exercise testing 6 weeks following an uncomplicated inferior

myocardial infarction. He had echocardiography prior to his exercise test, which demonstrated abnormal inferior
wall motion. He is currently on aspirin 75 mg od, simvastatin 40 mg od, lisinopril 20 mg od and atenolol 25 mg od.
Resting heart rate is 72 bpm and blood pressure is 130/70 mmHg. He achieves 4 minutes 15 seconds, stopping
secondary to chest pain and associated ST-segment depression in the inferolateral leads.

What would be the next stage in his management?

O Add diltiazem and review in clinic


Arrange a stress echocardiogram

O Increase atenolol 50 mg od and repeat the exercise test


Refer for coronary angiography

O Refer for a myocardial perfusion scan

A 52-year-old man undergoes Bruce-protocol exercise testing 6 weeks following an uncomplicated inferior
myocardial infarction. He had echocardiography prior to his exercise test, which demonstrated abnormal inferior

wall motion. He is currently on aspirin 75 mg od, simvastatin 40 mg od, lisinopril 20 mg od and atenolol 25 mg od.
Resting heart rate is 72 bpm and blood pressure is 130/70 mmHg. He achieves 4 minutes 15 seconds, stopping
secondary to chest pain and associated ST-segment depression in the inferolateral leads.

What would be the next stage in his management?


Add diltiazem and review in clinic
Arrange a stress echocardiogram

Increase atenolol 50 mg od and repeat the exercise test


Refer for coronary angiography CORRECT ANSWER

Refer for a myocardial perfusion scan

The Bruce protocol


The purpose of the exercise test postmyocardial infarction is twofold: risk stratification and patient self-confidence
Patients who are unable to perform two stages of the Bruce protocol (< 6 minutes) are at higher risk of adverse
cardiovascular events
This youngish man has an early positive exercise test, as exemplified by symptoms and associated ECG changes

Investigation and treatment


He should be referred for coronary angiography to accurately determine whether he has prognostic disease
warranting coronary artery bypass grafting eg

left main stem disease


proximal three-vessel disease
proximal two-vessel disease including the left anterior descending artery
Percutaneous intervention can also be considered if he has ongoing symptoms (likely in view of his symptoms on
exercise testing)
Increasing his atenolol dose would be a sensible amendment to his current medical therapy

As medical registrar on call you are summoned to assist with a cardiac arrest on CCU. A 60-year-old man is being
resuscitated having presented with unstable angina 3 days before. He has had three unsuccessful shocks for
ventricular fibrillation. An anaesthetist is looking after his airway. He has a large-bore iv access in his antecubital
fossa.

What additional therapy would you consider at this point?


O Amiodarone

Bretylium

O Calcium chloride
O Lidocaine
O Sodium bicarbonate

As medical registrar on call you are summoned to assist with a cardiac arrest on CCU. A 60-year-old man is being
resuscitated having presented with unstable angina 3 days before. He has had three unsuccessful shocks for
ventricular fibrillation. An anaesthetist is looking after his airway. He has a large-bore iv access in his antecubital
fossa.

What additional therapy would you consider at this point?


Amiodarone CORRECT ANSWER

Bretylium
Calcium chloride

Lidocaine
Sodium bicarbonate

Therapy during cardiac arrest


Current recommendations for advanced life support in the UK are provided by the European Resuscitation Council
and the Resuscitation Council UK

In patients with refractory ventricular fibrillation or pulseless ventricular tachycardia (ie after three initial shocks),
intravenous amiodarone should be considered

The standard dose used is a 300-mg bolus


If central access is available this is the desired route
However, if this is not the case then a large-bore peripheral access can be used
Lidocaine (lignocaine) is used when amiodarone is unavailable

A 72-year-old white woman is referred to out-patients for advice regarding her hypertension management. She

has been on treatment in the form of perindopril 4 mg od for the past 3 years. However, on repeated
measurements, her readings have been >160 mmHg systolic, with diastolic readings being in the order of 80-85
mmHg. Renal function is normal as is urine dipstick testing. There is no evidence of left ventricular hypertrophy
on ECG. She is obese with a body mass index of 33

kg/m2.

What would you consider adding as your next drug?

O Atenolol
Bendrofluazide
O Doxazosin
Amlodipine

O Spironolactone

A 72-year-old white woman is referred to out-patients for advice regarding her hypertension management. She

has been on treatment in the form of perindopril 4 mg od for the past 3 years. However, on repeated
measurements, her readings have been >160 mmHg systolic, with diastolic readings being in the order of 80-85
mmHg. Renal function is normal as is urine dipstick testing. There is no evidence of left ventricular hypertrophy
on ECG. She is obese with a body mass index of 33

kg/m2.

What would you consider adding as your next drug?


Atenolol
Bendrofluazide

Doxazosin
Amlodipine CORRECT ANSWER
Spironolactone

Resistant hypertension
This woman has hypertension resistant to a single agent
It is increasingly recognised that more than one agent is required to adequately control blood pressure
Whilst guidelines suggest either a thiazide or calcium channel antagonist could be added next, in view of the fact
that she is obese, a calcium channel antagonist may be the better choice

Meta-analyses have shown that both thiazides and (B-blockers are associated with an increased risk of the
development of type 2 diabetes in at-risk patients
Indeed, the ASCOT study did suggest that the combination of ACE inhibitor and calcium antagonist was associated
with the development of less type 2 diabetes than a (3-blocker/thiazide alternative

A 53-year-old bus driver presents with a history of chest pain at rest. Initial ECG shows minor ST-segment
depression in the lateral leads. Cardiac enzymes, including troponin, are normal. He has known angina, with
angiography 3 years previously demonstrating minor right coronary artery disease for which he was using a GTN
spray only 1-2 times per year, on extreme exercise. He is commenced on aspirin, a R-blocker and a statin. His
symptoms settle over 24 hours, 12-hour troponin is normal and, following mobilisation, he is discharged home.
He needs to know how this episode might affect his future employment.

Assuming his condition remains stable with no further symptoms, what would be the next stage of
investigation with respect to his regaining his bus driving licence?

q Angiography
O Echocardiogram
O Exercise testing
Myocardial perfusion imaging

Review in outpatients after 6 weeks

A 53-year-old bus driver presents with a history of chest pain at rest. Initial ECG shows minor ST-segment
depression in the lateral leads. Cardiac enzymes, including troponin, are normal. He has known angina, with
angiography 3 years previously demonstrating minor right coronary artery disease for which he was using a GTN
spray only 1-2 times per year, on extreme exercise. He is commenced on aspirin, a R-blockerand a statin. His
symptoms settle over 24 hours, 12-hour troponin is normal and, following mobilisation, he is discharged home.
He needs to know how this episode might affect his future employment.

Assuming his condition remains stable with no further symptoms, what would be the next stage of
investigation with respect to his regaining his bus driving licence?

Angiography
Echocardiogram
Exercise testing CORRECT ANSWER
Myocardial perfusion imaging

Review in outpatients after 6 weeks

Coronary artery disease and driving


This man holds a group 2 licence, and hence he should be advised to inform the DVLA of his recent presentation
with unstable angina and stop work until re-licensing can be performed
For group 2 licence holders, all acute coronary syndromes are considered relevant and this disqualifies the
individual from driving for at least 6 weeks

Re-licensing may be permitted if a suitable exercise test is achieved (need to complete three stages of the Bruce
protocol or equivalent safely, without antianginal medication for 48 hours and without significant symptoms, ECG or
haemodynamic abnormalities) and there are no other disqualifying conditions
Although angiography is not required for re-licensing, if it is performed then specific guidelines are available
In this case he would be exercised at the 6 weeks stage
All up-to-date information can be obtained from the DVLA (www.dvla.gov.uk)

A 50-year-old woman is referred to out-patients for a previously asymptomatic atrial septal defect (ASD). She is
new to the area and was last seen around 6 years ago in her previous local hospital. She is a smoker but without

other significant medical history. She now complains of shortness of breath on exertion, together with peripheral
oedema. Clinical examination reveals her to be clubbed and cyanosed. Her pulse rate is 90 bpm and blood
pressure 98/60 mmHg. Echo demonstrates a dilated right heart with an estimated right ventricular pressure of 90
mmHg and significant tricuspid and pulmonary regurgitation.

What is the likely diagnosis?


O Cor pulmonale

O Eisenmenger syndrome
O Infective endocarditis
O Primary pulmonary hypertension
O Pulmonary emboli disease

A 50-year-old woman is referred to out-patients for a previously asymptomatic atrial septal defect (ASD). She is
new to the area and was last seen around 6 years ago in her previous local hospital. She is a smoker but without
other significant medical history. She now complains of shortness of breath on exertion, together with peripheral
oedema. Clinical examination reveals her to be clubbed and cyanosed. Her pulse rate is 90 bpm and blood
pressure 98/60 mmHg. Echo demonstrates a dilated right heart with an estimated right ventricular pressure of 90
mmHg and significant tricuspid and pulmonary regurgitation.

What is the likely diagnosis?


Cor pulmonale

Eisenmenger syndrome CORRECT ANSWER


Infective endocarditis

Primary pulmonary hypertension


Pulmonary emboli disease

Eisenmenger syndrome
This woman has developed massive irreversible pulmonary hypertension as a consequence of a previous left to
right shunt

Pulmonary pressures have now reached systemic level


The reversal of a left to right shunt as a consequence of pulmonary hypertension is known as Eisenmenger
syndrome, and is generally the result of a previously undiagnosed ASD, ventricular septal defect (VSD) or patent
ductus arteriosus

It may also result from an incompletely corrected Fallot's tetralogy or Ebstein's anomaly
Prognosis is poor, although a few patients may be candidates for heart-lung transplantation
Symptomatic treatment is directed towards right heart failure

Complications
Complications include polycythaemia, bleeding disorders and cerebral embolism or abscess
Since there is a particularly high risk in those of childbearing age, patients should be given appropriate advice and
information to avoid pregnancy

A 40-year-old man is referred by his GP for advice with regard to primary prevention of cardiovascular disease.
He is a smoker with a strong family history of premature death from ischaemic heart disease. Following a period
of lifestyle modification, his fasting cholesterol concentration is 7.2 mmol/litre. On consultation of the local
guidelines you find that his estimated 10-year risk of a coronary heart disease event is > 30%.

What would you advise?


Cholestyramine
Q Dietician advice

Fibrate

O Nicotinic acid
O Statin

A 40-year-old man is referred by his GP for advice with regard to primary prevention of cardiovascular disease.
He is a smoker with a strong family history of premature death from ischaemic heart disease. Following a period

of lifestyle modification, his fasting cholesterol concentration is 7.2 mmol/litre. On consultation of the local
guidelines you find that his estimated 10-year risk of a coronary heart disease event is > 30%.

What would you advise?

Cholestyramine

Dietician advice
Fibrate
Nicotinic acid
Statin CORRECT ANSWER

Prevention of cardiovascular disease


The National Service Framework for Coronary Heart Disease (2000) and the Joint British Society Guidelines
recommend targeting individuals with a 10-year risk of a coronary heart disease event of > 30%
Individuals should be offered interventions to address all modifiable risk factors, including
dietary advice
smoking cessation advice and support

moderation of alcohol consumption


weight reduction where appropriate
In respect of lipid management, non-pharmacological and pharmacological interventions should be utilised to
achieve a total cholesterol concentration < 5.0 mmol/litre and an low-density lipoprotein (LDL) cholesterol
concentration of < 3.0 mmol/litre

The results of several important trials support the use of statins in primary prevention, these include
WOSCOPS (pravastatin)
AFCAPS/TEXCAPS (levastatin)

Which one of the following statements best describes primary pulmonary hypertension?

O The familial form is inherited as sex-linked recessive


O Chronic thromboembolic disease can be identified in 30% of primary cases
q Spontaneous remission is the rule in more than half the cases

O Cannabis inhalation may induce similar disease


The risk for subacute bacterial endocarditis is low and antibiotic prophylaxis is seldom required

Which one of the following statements best describes primary pulmonary hypertension?

The familial form is inherited as sex-linked recessive


Chronic thromboembolic disease can be identified in 30% of primary cases
Spontaneous remission is the rule in more than half the cases

Cannabis inhalation may induce similar disease

The risk for subacute bacterial endocarditis is low and antibiotic prophylaxis is seldom
required
CORRECT ANSWER

Primary pulmonary hypertension


One of the diagnostic criteria includes a mean pulmonary artery pressure of more than 25 mmHg at rest or more
than 30 mmHg with exercise

Recurrent thromboembolism is one cause of secondary pulmonary hypertension (not primary)


Fenfluramine, cocaine inhalation and HIV infection can cause pulmonary vascular disease with clinical and
pathological features similar to those of primary pulmonary hypertension

The familial form is inherited as autosomal dominant


Prognosis

The medium period of survival is two to three years after the diagnosis
Recent improvement in diagnosis and newer forms of treatment have improved survival, but the prognosis is
generally very poor and most patients gradually succumb to progressive right-sided heart failure

Which one of the following statements is most accurate regarding coarctation of the aorta?

The coarctation is proximal to the left subclavian artery origin if the right arm blood pressure is
significantly higher than in the left arm
Continuous murmur over the thoracic spine usually originates from extensive collaterals

Rib notching on plain chest X-ray can be identified as early as three months after birth
Atrial septal defect (ASD) is the commonest associated congenital abnormality
The risk of bacterial endocarditis means that antibiotic prophylaxis is required prior to all dental
procedures

Which one of the following statements is most accurate regarding coarctation of the aorta?
The coarctation is proximal to the left subclavian artery origin if the right arm blood pressure is
significantly higher than in the left arm
CORRECT ANSWER
Continuous murmur over the thoracic spine usually originates from extensive collaterals

Rib notching on plain chest X-ray can be identified as early as three months after birth
Atrial septal defect (ASD) is the commonest associated congenital abnormality

The risk of bacterial endocarditis means that antibiotic prophylaxis is required prior to all dental
procedures

Features of coarctation of the aorta


The commonest site of discrete obstruction of the aortic lumen is just distal to the origin of the left subclavian
artery

The systolic arterial pressure in the arms exceeds that in the leg
However, if the systolic arterial pressure in the right arm is higher than that of the left arm by more than 30 mmHg,
the left subclavian is involved in the coarctation (i.e. coarctation is proximal to the origin of the subclavian as in
this case)
A continuous murmur over the thoracic spine usually originates from small, tight coarctation (< 2 mm)
Other cardiac malformations are frequent, the commonest being a bicuspid aortic valve
Notching of the inferior border of the ribs from collateral vessels is common and usually manifest in adults and older
children

Patients with unrepaired coarctation are at risk of bacterial endocarditis and should be advised about this
possibility

Antibiotic prophylaxis is no longer advised given that this is effective in reducing the risks of infection following
dental or other procedures

Which one of the following is characteristic of atrial myxoma?

O Usually originates in the right atrium

Fragments of tumour easily break off and grow in its peripheral sites

O Echocardiogram is diagnostic in most cases

O The clinical signs can mimic severe mitral regurgitation


Recurrence is frequent even after successful surgical removal of the tumour

Which one of the following is characteristic of atrial myxoma?

Usually originates in the right atrium


Fragments of tumour easily break off and grow in its peripheral sites
Echocardiogram is diagnostic in most cases CORRECT ANSWER

The clinical signs can mimic severe mitral regurgitation


Recurrence is frequent even after successful surgical removal of the tumour

Atrial myxoma
Atrial myxoma is a benign tumour of the heart
Approximately 75% originate in the left atrium
The clinical features are characterised by a triad of
embolism
intracardiac obstruction
constitutional symptoms

The clinical signs can mimic mitral stenosis and the murmur may vary with body position
Fragments of tumour easily break off but do not grow in its peripheral sites
After complete and careful removal of the tumour recurrence is very rare

A 57-year-old female is admitted with Gram-negative septicaemia. She is given intravenous

antibiotics and normal saline. Two days later she becomes anxious, tachypnoeic and short of
breath. An emergency chest X-ray demonstrates diffuse, bilateral interstitial and alveolar infiltrates.
Her past medical history revealed hypertension and that she has been on regular antihypertensive
treatment for seven years. She has never had any evidence of congestive heart failure. In this case,
adult respiratory distress syndrome can be distinguished from cardiogenic pulmonary oedema by
which one of the following?

Asymmetrical hypertrophy of the interventricular septum is revealed on echocardiography


Q Calculation of the alveolar-arterial p O2 difference
Measurement of pulmonary artery wedge pressure
Measurement of lung compliance
Measurement of ejection fraction

A 57-year-old female is admitted with Gram-negative septicaemia. She is given intravenous

antibiotics and normal saline. Two days later she becomes anxious, tachypnoeic and short of
breath. An emergency chest X-ray demonstrates diffuse, bilateral interstitial and alveolar infiltrates.
Her past medical history revealed hypertension and that she has been on regular antihypertensive
treatment for seven years. She has never had any evidence of congestive heart failure. In this case,
adult respiratory distress syndrome can be distinguished from cardiogenic pulmonary oedema by
which one of the following?

Asymmetrical hypertrophy of the interventricular septum is revealed on echocardiography


Calculation of the alveolar-arterial p O2 difference

Measurement of pulmonary artery wedge pressure CORRECT ANSWER


Measurement of lung compliance
Measurement of ejection fraction

Adult respiratory distress syndrome


The adult respiratory distress syndrome (ARDS) is a clinical triad of
hypoxaemia
diffuse lung infiltrates
reduced lung compliance not attributable to congestive cardiac failure
This has been reported as a complication of apparently unrelated conditions, examples include
sepsis
lung contusion
drug overdose
Increase in lung water in ARDS occurs as a result of an increase in alveolar capillary permeability and is not due to
an increase in hydrostatic forces

Distinction from pulmondary oedema


Clinically and radiographically, ARDS closely resembles severe haemodynamic pulmonary oedema due to heart
failure

The distinction between these disorders is often apparent from the clinical circumstances associated with the
onset of respiratory distress, whereas differentiation by radiographic means alone is often extremely difficult

As in cardiac pulmonary oedema, the increase in lung water associated with ARDS produces interstitial oedema
and alveolar collapse, and so the affected lung becomes stiff and the alveolar-arterial oxygen tension difference
widens

The central venous pressure and ejection fraction may alter but would not reflect the underlying pathophysiological
mechanism

A Swan-Ganz catheter should be placed if the mechanism of oedema formation cannot be discerned with
confidence

A pulmonary capillary wedge pressure of < 18 mmHg favours acute lung injury over haemodynamic pulmonary
oedema

In clinical practice, determination of pulmonary artery wedge pressure is the most helpful discriminate between
ARDS and cardiac failure

A 30-year-old woman presents with a three month history of chest pain. On auscultation, there is a midsystolic
click and a late systolic murmur. Her ECG shows T-wave inversions in leads II, III, and aVF.

Which one of the following statements concerning her condition is true?


The woman's chest pain is almost certainly due to associated coronary artery disease
Q The click and murmur is likely to occur earlier in systole when the patient stands

An exercise stress test would most likely be positive


Asymmetrical hypertrophy of the interventricular septum is revealed on echocardiography

O Prophylactic measures to prevent subacute bacterial endocarditis are warranted

A 30-year-old woman presents with a three month history of chest pain. On auscultation, there is a midsystolic
click and a late systolic murmur. Her ECG shows T-wave inversions in leads II, III, and aVF.

Which one of the following statements concerning her condition is true?


The woman's chest pain is almost certainly due to associated coronary artery disease
The click and murmur is likely to occur earlier in systole when the patient stands CORRECT
ANSWER

An exercise stress test would most likely be positive


Asymmetrical hypertrophy of the interventricular septum is revealed on echocardiography
Prophylactic measures to prevent subacute bacterial endocarditis are warranted

Systolic click-murmur syndrome

The systolic click-murmur syndrome is associated with mitral valve prolapse


It occurs in approximately 4% of the normal asymptomatic population

It can place excessive stress on the papillary muscles and lead to ischaemia and chest pain

Examination findings

Although often associated with inferior T-wave changes, the systolic click-murmur syndrome only occasionally
results in an ischaemic response to exercise
On standing or during the Valsalva manoeuvre, as ventricular volume gets smaller, the click and murmur move
earlier in systole
Echocardiogram reveals mid-systolic prolapse of the posterior mitral leaflet or, on occasion, both mitral leaflets
into the left atrium

Other notes

Asymmetrical hypertrophy of the interventricular septum is a feature of hypertrophic obstructive cardiomyopathy


(HOCM)
According to recent guidelines (American Heart Association, 2007), prophylaxis against infective endocarditis
(IE) is no longer recommended for patients with a mitral valve prolapse (MVP)

Reference
American Heart Association (2007). Available online: http://circ.ahajournals.org/cgi/reprint/116/15/1736

Aortic stenosis in adults is commonly the result of which one of the following?

Bicuspid aortic valve disease

Q Left ventricular membrane

Hypertrophic obstructive cardiomyopathy (HOCM)

O Rheumatic fever
O Cystic medial necrosis

Aortic stenosis in adults is commonly the result of which one of the following?
Bicuspid aortic valve disease CORRECT ANSWER

Left ventricular membrane


Hypertrophic obstructive cardiomyopathy (HOCM)

Rheumatic fever
Cystic medial necrosis

Bicuspid aortic valve disease


Approximately 1% of the general population has a bicuspid aortic valve defect
The bicuspid aortic valve may function normally throughout life, with late stenosis resulting from fibrocalcific
thickening

Aortic stenosis resulting from bicuspid valve disease occurs from increasing rigidity of the abnormal aortic valve
and increasing calcification
The congenital form of bicuspid valve disease is conjoined anteriorly

Normal pregnancy is associated with which one of the following haemodynamic changes?

O A 20% reduction in blood volume and cardiac output


A 10 mmHg drop in diastolic blood pressure during the second trimester

O Bradycardia with a radial pulse rate between 45 and 55 beats per minute
Grade 2/6 diastolic murmur at the mitral area

O Pulsus alternans

Normal pregnancy is associated with which one of the following haemodynamic changes?

A 20% reduction in blood volume and cardiac output

A 10 mmHg drop in diastolic blood pressure during the second trimester CORRECT ANSWER

Bradycardia with a radial pulse rate between 45 and 55 beats per minute
Grade 2/6 diastolic murmur at the mitral area

Pulsus alternans

Haemodynamic changes in pregnancy


Despite an expansion of the plasma volume and cardiac output of 50%, mean and diastolic blood pressures fall by
approximately 15% owing to a reduction in peripheral vascular resistance
There is little change in systolic blood pressure, but diastolic pressure is reduced (5-10 mmHg) from about 12-16
weeks

Diastolic pressure usually increases thereafter to pre-pregnancy levels by about 36 weeks


Tachycardia rather than bradycardia is a recognised physiological change during pregnancy as a consequence of
reduced peripheral vascular resistance and fall in blood pressure levels
The heart may be slightly enlarged and may be displaced outward because of the high diaphragm
A pulmonary systolic murmur from a high blood flow is common and there may be a physiological third heart sound
Diastolic murmurs are generally pathological and at the mitral area may signify mitral stenosis

The presence of pulsus alternans usually signifies advanced heart failure

A 60-year-old man underwent a coronary angiogram for unstable angina. The next day while recovering in

hospital he complains of severe pain in his right foot and partial loss of sight in the left eye. On examination the
lower limb peripheral pulses are present and of good volume. There is gangrene of the lateral two toes on the
right foot. Fundoscopy reveals cholesterol emboli in a branch of the central retinal artery in the left eye.

Which one of the following is the most probable diagnosis in this case?
Atheroembolic disease
Q Polyarteritis nodosa

Buerger's disease

O Arterial thromboembolism
O Disseminated intravascular coagulopathy

A 60-year-old man underwent a coronary angiogram for unstable angina. The next day while recovering in

hospital he complains of severe pain in his right foot and partial loss of sight in the left eye. On examination the
lower limb peripheral pulses are present and of good volume. There is gangrene of the lateral two toes on the
right foot. Fundoscopy reveals cholesterol emboli in a branch of the central retinal artery in the left eye.

Which one of the following is the most probable diagnosis in this case?
Atheroembolic disease CORRECT ANSWER

Polyarteritis nodosa
Buerger's disease
Arterial thromboembolism

Disseminated intravascular coagulopathy

Artheroembolic disease
Although each of the mentioned options is a valid possible underlying cause behind this presentation, it is clear that
the picture is more typical of atheroembolic disease
It is due to cholesterol emboli lodged in peripheral arteries, commonly as a result of angiographic or other surgical
vascular procedures

Clinical features

Clearly the clinical features will depend on the site of embolisation

The most common clinical findings are cutaneous features, renal failure and worsening hypertension
The presence of foot pulses with gangrenous toes should suggest cholesterol embolisation
The retina provides a unique opportunity to visualise the cholesterol emboli
Renal failure may manifest as gradual deterioration of renal function following angiography or may be acute (this
may mimic acute dissection of the renal artery during renal angiography)

Eosinophilia, eosinophiluria, a raised ESR and hypocomplementinaemia have been found in atheroembolic disease
Arterial thromboembolism is related to distal embolisation of proximal, pre-existing atheroma

Which one of the following features is more common in constrictive pericarditis than in cardiac

tamponade?

O Pulsus paradoxus
O Kussmaul's sign
O Prominent x trough
O 4-chamber diastolic equilibrium
Hypotension

Which one of the following features is more common in constrictive pericarditis than in cardiac

tamponade?

Pulsus paradoxus
Kussmaul's sign CORRECT ANSWER

Prominent x trough
4-chamber diastolic equilibrium
Hypotension

Features of constrictive pericarditis


An inspiratory increase in venous pressure (Kussmaul's sign) and a steep y descent in the jugular pulse are
features of constrictive pericarditis

Pericardial knock in early diastole is often seen in constrictive pericarditis


Both conditions cause failure of either side of the heart and the diastolic pressure in all cardiac chambers are
equal
A paradoxical pulse and prominent x trough in the jugular pulse are more common in tamponade than in constrictive
pericarditis

A 30-year-old man with known hypertrophic obstructive cardiomyopathy (HOCM) presents to casualty with an

episode of witnessed syncope: a passer-by provided initial resuscitation. On admission he is unwell with pulse
rate of 160 bpm, blood pressure 70/40 mmHg and decreased conscious level. ECG confirms ventricular
tachycardia. Sinus rhythm is restored with a DC shock.

What would be the most appropriate strategy for the long term?

O Amiodarone
Automatic implantable cardioverter defibrillator

O Dual-chamber pacemaker

Sotalol
O Verapamil

A 30-year-old man with known hypertrophic obstructive cardiomyopathy (HOCM) presents to casualty with an

episode of witnessed syncope: a passer-by provided initial resuscitation. On admission he is unwell with pulse
rate of 160 bpm, blood pressure 70/40 mmHg and decreased conscious level. ECG confirms ventricular
tachycardia. Sinus rhythm is restored with a DC shock.

What would be the most appropriate strategy for the long term?

Amiodarone
Automatic implantable cardioverter defibrillator CORRECT ANSWER

Dual-chamber pacemaker
Sotalol
Verapamil

This man has survived an out-of-hospital cardiac arrest and therefore an automatic implantable cardioverter
defibrillator (AICD) is warranted. Overall, patients with HOCM have an annual mortality rate of around 1%.
Identifying those at greatest risk of sudden cardiac death (SCD) is challenging. However, several factors have
been identified that are associated with an increased risk:

maximum wall thickness > 30 mm

non-sustained ventricular tachycardia on a 48-hour tape


a history of SCD in a relative under 45 years of age and a history of syncope
resting, left ventricular outflow-tract gradient > 30 mmHg
abnormal blood-pressure response to exercise.

Although a single risk factor does not, on its own, have a particularly high positive-predictive accuracy, the
presence of two or more risk factors does identify a much higher risk population. Dual-chamber pacing, pblockers or verapamil may be used to reduce symptoms in patients with a left ventricular outflow-tract
obstruction.

A 45-year-old woman is being investigated for heart disease. It is found that the pressure-volume curve of the

left ventricle is shifted to the right.

What is the most likely diagnosis in this case?


Aortic regurgitation

O Mitral stenosis
O Aortic stenosis
Mitral regurgitation

O Tricuspid stenosis

A 45-year-old woman is being investigated for heart disease. It is found that the pressure-volume curve of the

left ventricle is shifted to the right.

What is the most likely diagnosis in this case?


Aortic regurgitation

Mitral stenosis
Aortic stenosis CORRECT ANSWER
Mitral regurgitation

Tricuspid stenosis

Diagnosis using pressure-volume curves


Aortic stenosis

The pressure-volume curve denotes the pumping mechanics of the heart chambers
Most studies refer to the left ventricle, as this is the main chamber
In aortic stenosis, there is pressure overload leading to concentric hypertrophy of the left ventricle
This causes increased contractility and decreased compliance of the chamber
More pressure is thus exerted to eject the same volume of blood
The pressure-volume curve therefore shifts to the right

Other heart disease diagnoses


In mitral and tricuspid stenosis, there is pressure overload in the left atrium and not the left ventricle
There would thus be no change in the pressure-volume curve of the left ventricle

The pressure-volume curve in a patient with heart failure is shifted to the right.

What is the most important feature in cardiovascular dynamics responsible for this right shift?

q Increased contractility of the chamber


O Increased sympathetic activity
O Concentric hypertrophy of the chamber
O Increased compliance of the chamber
Pressure overload in the chamber

The pressure-volume curve in a patient with heart failure is shifted to the right.

What is the most important feature in cardiovascular dynamics responsible for this right shift?
Increased contractility of the chamber
Increased sympathetic activity

Concentric hypertrophy of the chamber

Increased compliance of the chamber CORRECT ANSWER


Pressure overload in the chamber

Right shift of the pressure-volume curve


Shifting of the pressure-volume curve to the right occurs when there is volume overload, as in aortic and mitral
regurgitation

There is decreased contractility and increased compliance


The ventricle dilates to accommodate the increased volume
Hypertrophy of the cardiac muscle does not occur
Increased sympathetic activity, catecholamine administration or exercise would shift the pressure-volume curve to
the left

A 65-year-old man with angina pectoris undergoes serum lipid testing. Which one of the following

abnormalities is most likely to be found?

O Increased triglyceride levels


O Increased low-density lipoprotein cholesterol levels

O Increased high-density lipoprotein cholesterol levels


O Increased chylomicrons

q Increased intermediate-density lipoprotein cholesterol levels

A 65-year-old man with angina pectoris undergoes serum lipid testing. Which one of the following

abnormalities is most likely to be found?


Increased triglyceride levels
Increased low-density lipoprotein cholesterol levels CORRECT ANSWER

Increased high-density lipoprotein cholesterol levels


Increased chylomicrons
Increased intermediate-density lipoprotein cholesterol levels

Serum lipid testing


Low-density lipoprotein

Low-density lipiprotein (LDL) particles are the main carriers of cholesterol


These particles can deposit lipid into the walls of the peripheral vasculature
There is a strong association between both total- and LDL-cholesterol concentration and coronary heart risk
There is a relatively weak independent link between raised concentrations of (triglyceride-rich) VLDL (very lowdensity lipoprotein) particles and cardiovascular risk

Triglycerides

Very highly raised triglyceride levels (> 6 mmol/litre) cause a greatly increased risk of acute pancreatitis and
retinal vein thrombosis

High-density lipoprotein

Higher high-density lipoprotein (HDL) concentrations protect against cardiovascular disease


HDL also has effects on the function of platelets and of the haemostatic cascade
These properties may favourably influence thrombogenesis

Chylomicrons

Excess chylomicrons do not confer an excess cardiovascular risk but do raise the total plasma triglyceride
concentration

A 70-year-old obese man is admitted with a 6-hour history of chest pain. An ECG reveals an inferior

wall myocardial infarction. Measurement of which one of the following would be most sensitive
and specific in confirming myocardial damage?

O Creatine kinase
Creatine kinase MB

O Cardiac-specific troponin T
Aspartate aminotransferase

O Lactate dehydrogenase

A 70-year-old obese man is admitted with a 6-hour history of chest pain. An ECG reveals an inferior

wall myocardial infarction. Measurement of which one of the following would be most sensitive
and specific in confirming myocardial damage?

Creatine kinase
Creatine kinase MB

Cardiac-specific troponin T CORRECT ANSWER


Aspartate aminotransferase

Lactate dehydrogenase

Diagnosing myocardial infarction


Troponin T and troponin I are regulatory proteins with a very high specificity for cardiac injury
They are released early (2-4 h) and can persist for up to 7 days
Most hospitals check levels at 6 and 12 hours after admission
They are more sensitive and cardiospecific than creatinine kinase MB, a cardiac-specific isoform of creatine
kinase allowing greater diagnostic accuracy than creatine kinase
Both aspartate aminotransferase and lactate dehydrogenase (LDH) are non-specific enzymes that are rarely used
nowadays for the diagnosis of myocardial infarction
LDH peaks at 3-4 days and remains elevated for up to 10 days following a cardiac event, and can thus be useful
in confirming myocardial infarction in patients presenting several days after an episode of chest pain

A 40-year-old salesman presents with frequent flushing of his face and neck, abdominal pain and

watery diarrhoea, fatigue, breathlessness, anorexia and nausea. On examination, there is jugular
venous distension with prominent v waves, hepatomegaly and dependent oedema. On
auscultation, a blowing pansystolic murmur is heard on inspiration at the lower left sternal edge.
What is the most likely cardiac abnormality in this case?

O Mitral regurgitation
Tricuspid incompetence

O Tricuspid stenosis
O Pulmonary stenosis
O Prolapsing mitral valve

A 40-year-old salesman presents with frequent flushing of his face and neck, abdominal pain and

watery diarrhoea, fatigue, breathlessness, anorexia and nausea. On examination, there is jugular

venous distension with prominent v waves, hepatomegaly and dependent oedema. On


auscultation, a blowing pansystolic murmur is heard on inspiration at the lower left sternal edge.
What is the most likely cardiac abnormality in this case?
Mitral regurgitation

Tricuspid incompetence CORRECT ANSWER


Tricuspid stenosis

Pulmonary stenosis
Prolapsing mitral valve

Auscultation in cardiac abnormalities


Tricuspid incompetence
This patient most probably has carcinoid syndrome, which can occur in 5% of patients with carcinoid tumours when
there are liver metastases

Cardiac abnormalities are found in 50% of patients, and consist of pulmonary stenosis or tricuspid incompetence
The auscultatory findings in this case are suggestive of tricuspid incompetence.

Pulmonary stenosis
In pulmonary stenosis, the characteristic auscultatory finding is a harsh mid-systolic ejection murmur best heard on
inspiration to the left of the sternum in the second intercostal space
A right ventricular fourth sound and a prominent jugular venous a wave are both present when the stenosis is
moderately severe
A rumbling mid-diastolic murmur is characteristic of tricuspid stenosis along with a prominent jugular venous a wave

Mitral regurgitation
Mitral regurgitation does not usually occur due to carcinoid syndrome
In mitral regurgitation, a pansystolic murmur is heard loudest at the apex and radiating widely over the precordium
and into the axilla

A prominent third heart sound may be present

Mitral valve prolapse


Prolapsing (floppy) mitral valve is commonly seen in young women and has a familial incidence
It may be associated with rheumatic or ischaemic heart disease, Marfan syndrome and thyrotoxicosis
The most common sign is a mid-systolic click followed by a late systolic murmur due to some regurgitation

A patient who has been inadvertently given an intravenous injection of potassium chloride,

develops ventricular tachycardia. His pulse is 150 bpm and blood pressure 60/40 mmHg. What

would be the best line of treatment in this case?

O Lidocaine
Insulin 10 units and 50 ml of 50% glucose
O Amiodarone

DC cardioversion

O 10 ml of 10% calcium gluconate

A patient who has been inadvertently given an intravenous injection of potassium chloride,

develops ventricular tachycardia. His pulse is 150 bpm and blood pressure 60/40 mmHg. What

would be the best line of treatment in this case?

Lidocaine
Insulin 10 units and 50 ml of 50% glucose
Amiodarone
DC cardioversion CORRECT ANSWER
10 ml of 10% calcium gluconate

Inadvertent potassium chloride injection


Since the patient is haemodynamically compromised, the emergency step would be to carry out DC cardioversion
as otherwise there is a risk of death

Injection of 10 ml of 10% calcium gluconate would help to protect the myocardium against hyperkalaemia
Calcium ions protect the cell membranes from the effects of hyperkalaemia but do not alter the potassium
concentration

Insulin drives potassium into the cell and must be accompanied by glucose to avoid hypoglycaemia
Lidocaine and amiodarone are only useful in stable cases

A 65-year-old man with chronic renal failure has a serum potassium level of 7.1 mmol/litre (normal

3.5-5.5 mmol/litre). What would be the most characteristic finding on ECG?

O Reduced P waves
O Prolonged QT intervals
O Prominent U waves
O Narrow QRS complexes
O T-wave inversion

A 65-year-old man with chronic renal failure has a serum potassium level of 7.1 mmol/litre (normal

3.5-5.5 mmol/litre). What would be the most characteristic finding on ECG?

Reduced P waves CORRECT ANSWER

Prolonged QT intervals
Prominent U waves

Narrow QRS complexes


T-wave inversion

ECG findings during hyperkalaemia


Hyperkalaemia causes hyperpolarisation of cell membranes, leading to
decreased cardiac excitability
hypotension
bradycardia
eventual asystole

The ECG shows characteristic tall, peaked T waves with widened QRS complexes
There is a progressive diminution in the amplitude of the P wave, which eventually disappears
Prominent U waves are seen in hypokalaemia while T-wave inversion occurs in ischaemic heart disease
Prolonged QT intervals are seen in
acute myocardial infarction
hypocalcaemia

hypothermia
procainamide administration

A patient with acute inferior wall myocardial infarction develops shock. Auscultation does not reveal any

murmurs.

Which one of the following complications of his Ml is most likely to be the cause?

O Cardiac rupture
Interventricular septal perforation

O Papillary muscle rupture


Right ventricular infarction

O Atrial fibrillation

A patient with acute inferior wall myocardial infarction develops shock. Auscultation does not reveal any

murmurs.

Which one of the following complications of his Ml is most likely to be the cause?
Cardiac rupture

Interventricular septal perforation

Papillary muscle rupture


Right ventricular infarction CORRECT ANSWER
Atrial fibrillation

Hypovolaemic shock
Right ventricular infarction occurs in one-third of cases of inferior wall myocardial infarction, which leads to pooling
of blood in the right ventricle and a consequent decreased preload in the left ventricle

Hypovolaemic shock results


Papillary muscle rupture and atrial fibrillation are other complications which are also more common in inferior
myocardial infarction; interventricular septal rupture is commoner in anterior myocardial infarction
Atrial fibrillation occurs as a complication in 10% of patients with myocardial infarction, and does not give rise to
shock

A 12-year-old boy with known heart disease is being advised regarding the risks of infective endocarditis.

Which cardiac lesion is most likely to be prone to infection?

O Atrial septal defect


Aortic regurgitation

Mitral stenosis
Q Mitral valve prolapse without regurgitation

Mitral regurgitation

A 12-year-old boy with known heart disease is being advised regarding the risks of infective endocarditis.

Which cardiac lesion is most likely to be prone to infection?


Atrial septal defect

Aortic regurgitation CORRECT ANSWER

Mitral stenosis
Mitral valve prolapse without regurgitation
Mitral regurgitation

Cardiac infection
Patients at high risk of infective (bacterial) endocarditis include those with prosthetic materials (valves, patches,
conduits or shunts), particularly in the first 6 months after placement, i.e. before these materials become
dothelialised
Other risk factors include complex cyanotic heart disease and previous episode(s) of endocarditis
Endocarditis more commonly affects the left (high pressure) side of the heart than the right

Infection of previously abnormal valves most commonly involves the aortic valves
Mitral regurgitation and mitral valve prolapse with regurgitation present a moderate risk, while mitral valve prolapse
without regurgitation is a low risk

Infective endocarditis in pure mitral stenosis and atrial septal defect is uncommon
Patients at risk of infective endocarditis should be made aware of presenting symptoms and counseled to seek
medical advice if they become unwell
If the diagnosis is considered it is essential that repeated sets of blood cultures are obtained before antibiotics are
started
Recent UK guidelines no longer support the use of antibiotic prophylaxis to prevent endocarditis after dental
or other procedures

Reference
See www.nice.org.uk/Guidance/CG64/Guidance/pdf/English

A 72-year-old diabetic man is admitted to the Emergency Department with a 40-minute history of
central, crushing chest pain. The pain eases after an hour with bedrest, oxygen and morphine. ECG

shows mild anterior ST flattening. The troponin T level is slightly raised. What would be the
optimal management of the underlying cause of his chest pain besides usual medical measures?

O Discharge home with referral to the outpatients department


O Low molecular weight heparin
O Abciximab
O Thrombolysis with tissue plasminogen activator
O Inpatient coronary angiography

A 72-year-old diabetic man is admitted to the Emergency Department with a 40-minute history of

central, crushing chest pain. The pain eases after an hour with bedrest, oxygen and morphine. ECG
shows mild anterior ST flattening. The troponin T level is slightly raised. What would be the
optimal management of the underlying cause of his chest pain besides usual medical measures?

Discharge home with referral to the outpatients department


Low molecular weight heparin
Abciximab

Thrombolysis with tissue plasminogen activator


Inpatient coronary angiography CORRECT ANSWER

Acute coronary syndrome


This patient falls into the high-risk category of acute coronary syndrome, an NSTEMI
Acute coronary syndrome is a medical emergency, which, if untreated, will progress to myocardial infarction in over
10% of cases

High-risk patients may have one or more of the following features


prolonged, ongoing (> 20 min) rest pain
pulmonary oedema or angina with hypotension
rest angina with dynamic ST changes > 1 mm
raised troponin (I and T)
age > 70 years
diabetes mellitus
Those at high-risk should proceed to inpatient angiography with a view to proceeding to revascularisation, where
appropriate, during that admission

A 20-year-old woman presents with a history of dyspnoea on exertion. On examination she has a

wide, fixed, split-second sound with an ejection systolic murmur in the left second intercostal

space. Her ECG shows left-axis deviation. What is the most probable diagnosis?

O Ostium primum septal defect


O Tricuspid incompetence
O Ostium secondum septal defect
O Pulmonary stenosis
O Aortic stenosis

A 20-year-old woman presents with a history of dyspnoea on exertion. On examination she has a

wide, fixed, split-second sound with an ejection systolic murmur in the left second intercostal

space. Her ECG shows left-axis deviation. What is the most probable diagnosis?
Ostium primum septal defect CORRECT ANSWER
Tricuspid incompetence

Ostium secondum septal defect

Pulmonary stenosis
Aortic stenosis

Systolic murmurs
Septal defects
Wide, fixed splitting of S2 with an ejection systolic murmur in the left second intercostal space points to a diagnosis
of atrial septal defect

Left axis deviation occurs in ostium primum atrial septal defect, whereas right axis deviation is seen in ostium
secundum septal defect
The ejection systolic murmur is due to a large volume of blood passing through the pulmonary valves into the
pulmonary artery

Other defects
Aortic stenosis is associated with an ejection systolic murmur that is usually diamond-shaped (crescendodecrescendo)

There may be a systolic ejection click


Tricuspid incompetence presents with a blowing pansystolic murmur, best heard on inspiration at the lower left
sternal edge
In pulmonary stenosis, there is a harsh mid-systolic ejection murmur, best heard on inspiration to the left of the
sternum in the second intercostal space

A 67-year-old man is admitted with chronic congestive heart failure. Based on this history, what is

the most important factor to be kept in mind when prescribing drugs for this patient?

O Loop diuretic administration would result in a decrease in mortality


Digoxin is more effective than ACE inhibitors in reducing cardiovascular events

O Administration of a (3-blocker reduces the time spent in hospital


Administration of spironolactone has no effect on the incidence of sudden cardiac death

Angiotensin ll-receptor antagonists have a better response rate than ACE inhibitors

A 67-year-old man is admitted with chronic congestive heart failure. Based on this history, what is

the most important factor to be kept in mind when prescribing drugs for this patient?
Loop diuretic administration would result in a decrease in mortality

Digoxin is more effective than ACE inhibitors in reducing cardiovascular events


Administration of a p-blocker reduces the time spent in hospital CORRECT ANSWER

Administration of spironolactone has no effect on the incidence of sudden cardiac death


Angiotensin ll-receptor antagonists have a better response rate than ACE inhibitors

Prescribing drugs for heart failure


Beta blockers
Beta-adrenoceptor blocking agents (metoprolol, bisoprolol and carvedilol) have been found to be useful in patients
with chronic stable heart failure
The studies MERIT and CIBIS 2, using the (3-blockers metoprolol and bisoprolol, respectively, have shown
improved symptomatic class, exercise tolerance, left ventricular function and reduced mortality in heart failure of
any cause
The rapid decrease in symptoms reduces the time spent in hospital

Other drugs
Diuretic administration is associated with a rapid decrease in symptoms, but mortality rates are unchanged
Angiotensin-converting enzyme (ACE) inhibitors and diuretics are recommended in all patients with clinical heart
failure as ACE inhibitors reduce mortality rates by 20%

Spironolactone greatly reduces the mortality and sudden cardiac death rates and should be added to the treatment
A recent trial comparing an angiotensin ll-receptor antagonist (losartan) with an ACE inhibitor (enalapril) has shown
no benefit of the former over the latter

Angiotensin ll-receptor antagonists should be used when ACE inhibitors are contraindicated or cause side-effects
(eg persistent cough)

During a routine medical check-up, a 2-year-old boy has been found to have a continuous

machinery murmur on auscultation just below the left clavicle. Given the likely diagnosis, what
would be the most characteristic investigative finding in this patient?

O Dilated left ventricle on echocardiogram


O Right ventricular hypertrophy on ECG

O Hilar haziness on chest X-ray


O Prominent pulmonary artery and pulmonary plethora on chest X-ray
O Polycythaemia

During a routine medical check-up, a 2-year-old boy has been found to have a continuous

machinery murmur on auscultation just below the left clavicle. Given the likely diagnosis, what
would be the most characteristic investigative finding in this patient?

Dilated left ventricle on echocardiogram CORRECT ANSWER

Right ventricular hypertrophy on ECG


Hilar haziness on chest X-ray
Prominent pulmonary artery and pulmonary plethora on chest X-ray

Polycythaemia

Diagnosing ductus arteriosus


This boy has a persistent ductus arteriosus
Because the aortic pressure exceeds the pulmonary artery pressure throughout a cardiac cycle, a persistent
ductus produces a continuous left to right shunting
This leads to increased pulmonary venous return to the left heart and an increased left ventricular volume load
The ECG shows a dilated left atrium and left ventricle
Right heart changes are apparent in late disease

Other notes
Hilar haziness occurs in pulmonary oedema owing to congestive cardiac failure
A prominent pulmonary artery may be seen on chest X-ray in persistent ductus, but the presence of pulmonary
plethora is more suggestive of atrial septal defect
Polycythaemia may occur if the shunt is reversed (Eisenmenger syndrome)

A 3-month-old boy with a cyanotic heart lesion is found to have a patent ductus arteriosus (PDA).

What is the best treatment for maintaining patency of the PDA prior to surgery?

O Indometacin
O Surgical ligation
O Angiographic ligation of the pulmonary artery
Prostaglandin Ei administration

O No treatment

A 3-month-old boy with a cyanotic heart lesion is found to have a patent ductus arteriosus (PDA)

What is the best treatment for maintaining patency of the PDA prior to surgery?
Indometacin
Surgical ligation

Angiographic ligation of the pulmonary artery


Prostaglandin Ei administration CORRECT ANSWER
No treatment

Treating ductus arteriosus


The ductus arteriosus in neonates and infants is highly sensitive to vasodilatation by prostaglandin E-j (PGE-j)
Patency of the ductus is necessary in patients with cyanotic heart disease until surgical correction of the heart
problem is undertaken
This will ensure additional oxygenation of the blood
Administration of PGE-j has been found to be highly effective in such cases

An Asian boy with a known history of rheumatic heart disease presents with low-grade fever for the
past month. He received a course of antibiotics from his GP a week ago. Which one of the
following investigations would be most useful in the diagnosis?

Blood culture
Q Serological testing

Echocardiogram

O C-reactive protein
O Full blood count

An Asian boy with a known history of rheumatic heart disease presents with low-grade fever for the
past month. He received a course of antibiotics from his GP a week ago. Which one of the
following investigations would be most useful in the diagnosis?

Blood culture
Serological testing
Echocardiogram CORRECT ANSWER
C-reactive protein

Full blood count

Use of echocardiography for visualising vegetations in infective


endocarditis
Echocardiography is extremely useful in allowing vegetations in infective endocarditis to be seen
Although blood cultures are a key diagnostic test in this condition, they may be negative if patients have recently
received antibiotic therapy
The same reasoning applies to serological tests for Coxiella , Bartonella, Legionella, Chlamydia and Brucella spp
that may also cause infective endocarditis
Both C-reactive protein and polymorphonuclear leucocytosis are non-specific tests

A 64-year-old man with Wolff-Parkinson-White syndrome presents with uneasiness and palpitations. The ECG
shows fine oscillations of the baseline and no clear P waves. The QRS rhythm is rapid and irregular. The
ventricular rate is 120 bpm. His blood pressure is 90/60 mmHg.

Which one of the following interventions would be most appropriate in this case?

O Digoxin
Verapamil

O DC cardioversion
O Metoprolol
O Procainamide

A 64-year-old man with Wolff-Parkinson-White syndrome presents with uneasiness and palpitations. The ECG
shows fine oscillations of the baseline and no clear P waves. The QRS rhythm is rapid and irregular. The
ventricular rate is 120 bpm. His blood pressure is 90/60 mmHg.

Which one of the following interventions would be most appropriate in this case?

Digoxin
Verapamil

DC cardioversion CORRECT ANSWER

Metoprolol

Procainamide

Atrial fibrillation and Wolff-Parkinson-White syndrome


This patient most probably has atrial fibrillation superimposed on Wolff-Parkinson-White (WPW) syndrome
The aim of treatment is to suppress the conduction ability of the abnormal pathway
This is achieved by using class-l and -III antiarrhythmic drugs but not by verapamil and digoxin, which may allow a
higher rate of conduction over the abnormal pathway and precipitate ventricular fibrillation

Thus neither verapamil nor digoxin should be used to treat atrial fibrillation associated with WPW syndrome
Previous guidelines suggested that use of adenosine was an acceptable option in these patients, but now DC
cardioversion in unstable situations is seen as the intervention of choice, with procainamide an alternative
Case reports suggest that the risk of VF is increased in patients with WPW who present with AF when they are
treated with adenosine

A 60-year-old man complains of dizziness and palpitations. An ECG shows tachycardia, broad QRS

complexes, AV dissociation and the presence of capture beats. What is the most probable

diagnosis?

Sustained ventricular tachycardia

O Ventricular fibrillation
O Torsades de pointes
Ventricular premature beats

O Atrial tachycardia

A 60-year-old man complains of dizziness and palpitations. An ECG shows tachycardia, broad QRS

complexes, AV dissociation and the presence of capture beats. What is the most probable

diagnosis?

Sustained ventricular tachycardia CORRECT ANSWER

Ventricular fibrillation
Torsades de pointes
Ventricular premature beats
Atrial tachycardia

Interpreting ECG
Ventricular tachychardia

The features described in the question are highly suggestive of sustained ventricular tachycardia

Ventricular fibrillation

In ventricular fibrillation, there is very rapid and irregular ventricular activation with no mechanical effect
The patient is pulseless and rapidly becomes unconscious
The ECG shows shapeless rapid oscillations with no hint of organised complexes

Torsades de pointes

In torsades de pointes, ventricular repolarisation is greatly prolonged (long QT syndrome)


It is characterised on ECG by rapid, irregular, sharp complexes that continuously change from an upright to an
inverted position
Prolonged QT intervals are also seen between spells of tachycardia or immediately preceding the onset of
tachycardia

Ventricular premature beats


Broad QRS complexes may be seen in ventricular premature beats, but, following a premature beat, there is
usually a compensatory pause

This condition is usually asymptomatic

Atrial tachycardia

In atrial tachycardia, the P waves are abnormally shaped and occur in front of the QRS complexes

A 65-year-old man is admitted with a broad complex tachycardia. Which one of the following

features would suggest a diagnosis of supraventricular tachycardia with aberrancy and help to
exclude ventricular tachycardia?
Q Capture beats on the electrocardiogram (ECG)

O Past history of ischaemic heart disease


O Right bundle-branch block morphology with left axis deviation on the ECG
O Temporary alleviation by carotid sinus massage
Variable intensity of the first heart sound

A 65-year-old man is admitted with a broad complex tachycardia. Which one of the following

features would suggest a diagnosis of supraventricular tachycardia with aberrancy and help to
exclude ventricular tachycardia?
Capture beats on the electrocardiogram (ECG)

Past history of ischaemic heart disease

Right bundle-branch block morphology with left axis deviation on the ECG
Temporary alleviation by carotid sinus massage CORRECT ANSWER
Variable intensity of the first heart sound

Distinguishing ventricular from supraventricular tachycardia


Ventricular tachycardia (VT) may be distinguished from supraventricular tachycardia (SVT) by ECG features that
indicate atrioventricular (AV) dissociation (ie that the atria and ventricles are no longer linked in rate and rhythm)
The three characteristic features of AV dissociation are irregular notching of the QRS complex, capture beats and
fusion beats
A variable intensity of the first heart sound in a regular tachycardia suggests AV dissociation causing variable filling
of the ventricles from the atria

Atrial fibrillation is the commonest cause of variable intensity of the first heart sound
VT does not involve the AV node and cannot therefore be affected by adenosine or carotid sinus massage, which
temporarily blocks the AV node

A past history of ischaemic heart disease is associated with a > 95% chance that broad complex tachycardia is
VT

A patient attending the cardiology clinic requires dental treatment.

Which one of the following conditions merits antibiotic prophylaxis?

O Atrial septal defect


O Hypertrophic cardiomyopathy
Patent ductus arteriosus

O All of the above


None of the above

A patient attending the cardiology clinic requires dental treatment.

Which one of the following conditions merits antibiotic prophylaxis?


Atrial septal defect
Hypertrophic cardiomyopathy

Patent ductus arteriosus

All of the above


None of the above CORRECT ANSWER

High-risk lesions

Patent ductus arteriosus carries a high risk of endocarditis, but there is no robust evidence that antibiotic

prophylaxis reduces the risk

The other 'high-risk' lesions are small ventricular septal defects and aortic regurgitation
The risk of endocarditis is highest where there are high-velocity jets of blood that damage the endothelium
Hypertrophic cardiomyopathy may be associated with high-velocity flow in the left ventricular outflow tract (LVOT)
when there is marked LVOT obstruction, although, in practice, the risk of endocarditis is small
Atrial septal defects (ASDs) are large holes in a 'low-pressure' system and therefore carry a low risk of
endocarditis and do not normally require prophylaxis
Mitral valve prolapse only carries appreciable risk where there is associated mitral regurgitation

Previous recommendations for antibiotic prophylaxis prior to dental procedures or instrumentation of the
gastrointestinal/genitourinary tracts have been withdrawn as there is little evidence that these have been effective
in preventing infection

Endocarditis may follow transient bacteraemia with organisms from oral or other mucosal flora, but this is much
more likely to result from normal daily activity (chewing, brushing teeth) than from a visit to the dentist and antibiotic

prophylaxis for such events is impractical and unnecessary

Reference
For the 2008 NICE guidelines on the prevention of antibiotic prophylaxis see
http:www.nice.org.uk/Guidance/CG64/Guidance/pdf/English

A 67-year-old lady, post-myocardial infarction, is suspected to have a left ventricular apical thrombus. Her
neurological status has deteriorated and you want to exclude the possibility that a cardiac embolus has led to her
neurological deterioration.

What is the most suitable imaging technique for confirming this diagnosis?

O Cardiac MR
Left ventricular angiography

O Multiple uptake gated acquisition scanning


Transoesophageal echocardiography

O Transthoracic echocardiography

A 67-year-old lady, post-myocardial infarction, is suspected to have a left ventricular apical thrombus. Her
neurological status has deteriorated and you want to exclude the possibility that a cardiac embolus has led to her
neurological deterioration.

What is the most suitable imaging technique for confirming this diagnosis?
Cardiac MR
Left ventricular angiography
Multiple uptake gated acquisition scanning

T ransoesophageal echocardiography
Transthoracic echocardiography CORRECT ANSWER

Imaging techniques
Although an excellent technique for imaging the posterior cardiac structures (atria, left atrial appendage, valves and
pulmonary veins), transoesophageal echo is less useful for imaging the structure and function of the ventricles,
especially the left ventricular apex, which is better imaged by conventional two-dimensional echo

Atypical thrombus may be apparent on contrast left ventriculography, but this technique carries a risk of dislodging
and embolising interventricular thrombus
Cardiac MR is effective in detecting mural thrombus but may not be easily available

A 30-year-old postman with hypertension but normally in good health presents to the Emergency Department
with sudden severe breathlessness and sweating. Chest examination reveals bilateral basal crackles. He
improves with diamorphine and frusemide (furosemide). ECG and cardiac enzymes are normal. He develops two
further episodes of pulmonary oedema which respond well to diuretics. Investigations in the follow-up clinic
reveal evidence of left ventricular hypertrophy on chest x-ray but with a preserved ejection fraction on
echocardiogram, and an elevated creatinine of 145 pmol/l.

What is the most likely cause of pulmonary oedema?

O Dilated cardiomyopathy
O Myocarditis
O Ischaemic heart disease
Q Phaeochromocytoma

O Renal artery stenosis

A 30-year-old postman with hypertension but normally in good health presents to the Emergency Department
with sudden severe breathlessness and sweating. Chest examination reveals bilateral basal crackles. He

improves with diamorphine and frusemide (furosemide). ECG and cardiac enzymes are normal. He develops two
further episodes of pulmonary oedema which respond well to diuretics. Investigations in the follow-up clinic
reveal evidence of left ventricular hypertrophy on chest x-ray but with a preserved ejection fraction on
echocardiogram, and an elevated creatinine of 145 pmol/l.

What is the most likely cause of pulmonary oedema?


Dilated cardiomyopathy

Myocarditis
Ischaemic heart disease

Phaeochromocytoma
Renal artery stenosis CORRECT ANSWER

Renal artery stenosis


The differential diagnosis of acute pulmonary oedema includes
severe left ventricular dysfunction
paroxysmal arrhythmias
three-vessel or left main stem coronary disease
In the context of hypertension it also includes
renal artery stenosis
phaeochromocytoma
Renal artery stenosis is more common
This man is a postman and therfore would be expected to have reasonable exercise tolerance, given his age this
makes cardiomyopathy, myocarditis and ischaemic heart disease extremely unlikely
Renal artery stenosis in a 30-year-old is much more common than phaeochromocytoma

A 46-year-old Asian man with a past history of coronary artery bypass grafting presents with breathlessness. The
jugular venous pressure (JVP) shows prominent x and y descents.

What is the most likely cause?

O Constrictive pericarditis
O Dilated cardiomyopathy
Pericardial effusion
Q Restrictive cardiomyopathy

Severe mitral regurgitation

A 46-year-old Asian man with a past history of coronary artery bypass grafting presents with breathlessness. The

jugular venous pressure (JVP) shows prominent x and y descents.

What is the most likely cause?


Constrictive pericarditis CORRECT ANSWER

Dilated cardiomyopathy
Pericardial effusion
Restrictive cardiomyopathy
Severe mitral regurgitation

Constrictive pericarditis
A prominent x descent in the jugular venous pressure (JVP) may occur in constrictive pericarditis or pericardial
effusion

The y descent is lost in tamponade but prominent in constrictive pericarditis


Constrictive pericarditis was classically caused by tuberculosis, but today is more commonly associated with
cardiac surgery, renal failure or following infective pericarditis
Restrictive cardiomyopathy may produce clinical features similar to constriction, but is less common

A 57-year-old man with ischaemic heart disease, and a recent transient ischaemic attack, is prescribed

clopidogrel.

How would the mechanism of action of this drug be best described?


Blocks glycoprotein llb/llla receptors

O Blocks thrombin receptors


Blocks thromboxane production

O Blocks platelet ADP receptors

q Potentiates antithrombin-ill action

A 57-year-old man with ischaemic heart disease, and a recent transient ischaemic attack, is prescribed

clopidogrel.

How would the mechanism of action of this drug be best described?


Blocks glycoprotein iib/llla receptors
Blocks thrombin receptors
Blocks thromboxane production
Blocks platelet ADP receptors CORRECT ANSWER

Potentiates antithrombin-lll action

Action of clopidogrel
Clopidogrel blocks platelet adenosine diphosphate receptors, while aspirin blocks thromboxane production, hence
the complementary actions of the two drugs when given together following coronary stenting
The final common pathway for platelet aggregation is through the glycoprotein Ilb/11 la receptor
Hence, the most powerful antiplatelet drugs are the glycoprotein Ilb/11 la blockers such as abciximab and tirofiban

Hirudins act by blocking thrombin receptors but have no current indication in cardiac disease

A 47-year-old female patient attends the cardiology clinic for her symptoms of fatigue and ankle oedema. Two-

dimensional echocardiography shows diffuse ventricular wall thickening and marked dilatation of both atria, with
granular sparkling of the left ventricular myocardium. She has been advised to avoid taking digoxin.

Which one of the following conditions is she most likely to have?

0 Hypertrophic cardiomyopathy
O Dilated cardiomyopathy
O ischaemic cardiomyopathy
O Amyloid heart disease

O Constrictive pericarditis

A 47 -year-old female patient attends the cardiology clinic for her symptoms of fatigue and ankle oedema. Twodimensional echocardiography shows diffuse ventricular wall thickening and marked dilatation of both atria, with
granular sparkling of the left ventricular myocardium. She has been advised to avoid taking digoxin.

Which one of the following conditions is she most likely to have?

Hypertrophic cardiomyopathy
Dilated cardiomyopathy
Ischaemic cardiomyopathy

Amyloid heart disease CORRECT ANSWER


Constrictive pericarditis

Amyloid heart disease


This lady has cardiac amyloid, characterised by heart failure with a restrictive infiltrative pattern
Her echocardiogram appearance is typical of cardiac amyloid and excludes other conditions such as hypertrophic
cardiomyopathy (in which digoxin is usually also contraindicated)

Bradycardia with 2:1 or complete heart block is common in amyloidosis and is much more likely to occur with
digoxin

In addition, the risk of digoxin toxicity is high in amyloidosis


Digoxin increases the risk of ventricular tachyarrhythmias in patients with impaired left ventricular function but this
is not a definite contraindication

A neonate is noted to be cyanosed within the 24 h following delivery.

Which cardiac abnormality would be the most likely cause?

O Ebstein's anomaly
O Eisenmenger ventricular septal defect
Hypoplastic left heart
Q Tetralogy of Fallot
Transposition of the great vessels

A neonate is noted to be cyanosed within the 24 h following delivery.

Which cardiac abnormality would be the most likely cause?


Ebstein's anomaly

Eisenmenger ventricular septal defect


Hypoplastic left heart
Tetralogy of Fallot
Transposition of the great vessels CORRECT ANSWER

Cyanosis in neonates
Although tetralogy of Fallot is much more common than transposition, the right ventricular outflow tract gradient,
which is the major determinant of cyanosis in Fallot's, does not become maximal until 6-9 months after birth
Many babies with Fallot's are, therefore, pink at birth but gradually become cyanosed over the first few months of
life
Ebstein's anomaly is a congenital abnormality of the tricuspid valve associated with in utero exposure to lithium,
which does not normally cause cyanosis

A 49-year-old man is noted to have shortening of the QT interval on the ECG. Which drug is most

likely to be responsible?

O Amiodarone
O Atenolol
O Digoxin
O Flecainide
O Sotalol

A 49-year-old man is noted to have shortening of the QT interval on the ECG. Which drug is most

likely to be responsible?

Amiodarone

Atenolol
Digoxin CORRECT ANSWER

Flecainide
Sotalol

Effect of drugs on the QT interval


The cardiac glycosides (digoxin and ouabain) shorten the QT interval
Class la (eg disopyramide), class Ic (eg flecainide) and class III drugs (eg amiodarone and sotalol) all prolong the
QT interval

Beta blockers have a neutral effect on the QT interval but are effective at stabilising the QT in long QT syndromes

A 32-year-old lady is noted to have a loud first heart sound with reversed splitting of the second

heart sound on auscultation. Which cardiological diagnosis is she most likely to have?

Hypertrophic cardiomyopathy
Q Left bundle-branch block

Mitral stenosis

O Right bundle-branch block


O Wolff-Parkinson-White syndrome type B

A 32-year-old lady is noted to have a loud first heart sound with reversed splitting of the second

heart sound on auscultation. Which cardiological diagnosis is she most likely to have?

Hypertrophic cardiomyopathy
Left bundle-branch block

Mitral stenosis

Right bundle-branch block


Wolff-Parkinson-White syndrome type B CORRECT ANSWER

Splitting of the second heart sound


A loud first heart sound (S1) is typical of mitral stenosis if the valve is pliable, but splitting of the second heart
sound (S2) should be normal
S2 is caused by closure of the aortic valve (A2) followed by closure of the pulmonary valve (P2)
Reversed splitting of S2 is caused either by delayed A2 (eg left bundle-branch block (LBBB), aortic stenosis,
hypertrophic obstructive cardiomyopathy) or early P2 (eg Wolff-Parkinson-White (WPW) type B where the rightsided accessory pathway causes early RV depolarisation)
Right bundle-branch block causes wide splitting of S2 because it delays P2. S1 is soft if closure of the mitral valve
is delayed (eg LBBB, long PR), but loud if mitral or tricuspid closure is early (eg WPW type B)

A 60-year-old woman visits her GP complaining of increasing shortness of breath. She has also suffered one
fainting episode over the past month. Clinical examination reveals a BP of 132/82 mmHg, and an ejection systolic
murmur. ECG shows marked left ventricular hypertrophy with strain. Echocardiography shows a peak aortic
valve gradient of 90 mmHg and decreased left ventricular systolic function.

What is the correct management?


Aortic valvuloplasty

0 Anticoagulation
Regular out-patient review

O Routine aortic valve replacement (within 6 months)


O Urgent aortic valve replacement (within 8 weeks)

A 60-year-old woman visits her GP complaining of increasing shortness of breath. She has also suffered one
fainting episode over the past month. Clinical examination reveals a BP of 132/82 mmHg, and an ejection systolic

murmur. ECG shows marked left ventricular hypertrophy with strain. Echocardiography shows a peak aortic
valve gradient of 90 mmHg and decreased left ventricular systolic function.

What is the correct management?


Aortic valvuloplasty

Anticoagulation

Regular out-patient review


Routine aortic valve replacement (within 6 months)
Urgent aortic valve replacement (within 8 weeks) CORRECT ANSWER

Aortic valve replacement


Surgery for aortic valve replacement is indicated in symptomatic patients (angina, exertional breathlessness,
syncope) as the risk of sudden death increases dramatically with the onset of symptoms

Patients with a gradient of less than 25 mmHg have a 20% chance of needing surgical intervention within 15 years
Valvuloplasty is used only in patients with critical aortic stenosis who are unfit for surgery as the benefits are
usually short-lived

A 58-year-old man is having his drug therapy reviewed following a myocardial infarction.

Which one of the following has no proven benefit on mortality following myocardial infarction?

O Atorvastatin
Isosorbide mononitrate
O Ramipril

Timolol

O Tirofiban

A 58-year-old man is having his drug therapy reviewed following a myocardial infarction.

Which one of the following has no proven benefit on mortality following myocardial infarction?
Atorvastatin

Isosorbide mononitrate CORRECT ANSWER


Ramipril

Timolol
Tirofiban

Drug benefits post-myocardial infarction


The glycoprotein Ilb/11 la antagonist tirofiban (PRISM-PLUS), timolol (TIMI trials) and ramipril (AIRE) have all been
shown to reduce mortality following myocardial infarction
The recent MIRACL study showed that atorvastatin reduced cardiovascular events by 17% when given for three
months post-MI
Isosorbide mononitrate showed no benefit in the ISIS 4 study

A 70-year-old lady underwent mitral valve replacement surgery 2 years ago. She appeared to make a good
recovery initially, but now presents with infective endocarditis.

What causal organism would be most likely in her case?

O Escherichia coli
O Staphylococcus aureus
Staphylococcus epidermidis

O Streptococcus faecalis
Q Streptococcus viridans

A 70-year-old lady underwent mitral valve replacement surgery 2 years ago. She appeared to make a good

recovery initially, but now presents with infective endocarditis.

What causal organism would be most likely in her case?

Escherichia coli
Staphylococcus aureus
Staphylococcus epidermidis

Streptococcus faecalis
Streptococcus viridans CORRECT ANSWER

Organisms that cause infective endocarditis


Organisms causing infective endocarditis can be divided into three groups
native valve endocarditis
early prosthetic endocarditis (< 6 to 12 months post-surgery)
late prosthetic endocarditis (> 6 to12 months post-surgery)
Native valve endocarditis is mainly streptococcal, predominantly streptococcus viridans , although staphylococcus
aureus is also common and often affects previously normal valves

Early prosthetic endocarditis is most commonly due to staphylococcus epidermidis, with some gram-negative
organisms and fungi

Late prosthetic endocarditis is similar to native valve endocarditis (ie most commonly streptococcus viridans)
except with a higher incidence of staphylococcal infection

What is the plateau phase of the myocardial action potential mediated by?

O ATP-sensitive potassium current


O Rapid delayed rectifier potassium current
O Fast sodium inward current
O L-type calcium current
Slow calcium inward current

What is the plateau phase of the myocardial action potential mediated by?

ATP-sensitive potassium current


Rapid delayed rectifier potassium current

Fast sodium inward current

L-type calcium current


Slow calcium inward current CORRECT ANSWER

The myocardial action potential

Except in specialised regions of the heart such as the SA and AV nodes, the rapid upstroke of the cardiac
action potential results from a fast sodium inward current generated by the opening of voltage-sensitive
sodium channels
this current rapidly inactivates

The plateau of the action potential is maintained by


the slow calcium inward current, which activates at more depolarised potentials and inactivates more

slowly than the fast sodium current


an inward current generated by the sodium/calcium exchanger

Repolarisation is effected by outward potassium and chloride currents

ATP-sensitive potassium channels

The ATP-sensitive potassium channel mediates action potential shortening (and hence surface ECG changes)

during ischaemia
It is the site of action of potassium channel openers such as nicorandil and has been implicated in the
phenomenon of ischaemic preconditioning

Role of calcium

Activation of the contractile myofilaments occurs as a result of calcium binding to troponin C


The calcium is partly derived from calcium entry via L-type calcium channels during the action potential and

partly from calcium released from intracellular stores (chiefly the sarcoplasmic reticulum) in response to calcium
entry

What is the first-line treatment for a 50-year-old man with known poor left ventricular function, who
presents with a broad complex tachycardia at a rate of 150 bpm and a blood pressure of 120/70

mmHg?

Amiodarone
p-Blockers

Flecainide
Lidocaine
Verapamil

What is the first-line treatment for a 50-year-old man with known poor left ventricular function, who
presents with a broad complex tachycardia at a rate of 150 bpm and a blood pressure of 120/70

mmHg?
Amiodarone CORRECT ANSWER

p-Blockers

Flecainide
Lidocaine

Verapamil

Treating ventricular tachycardia


In the presence of poor left ventricular function, broad complex tachycardia is highly likely to be caused by
ventricular tachycardia (VT)
Verapamil may precipitate circulatory collapse in VT and is therefore contraindicated
In the presence of severe left ventricular dysfunction, negative inotropes such as lidocaine and p-blockers are
undesirable while flecainide may rarely cause degeneration of stable tachycardias to ventricular fibrillation

Amiodarone is, therefore, the most appropriate choice

An 18-year-old man with Marfan syndrome is reviewed in the cardiology clinic, after a screening ECG is found to

be abnormal, with left axis deviation and prominent Q waves in I, III, aVF and V3-V6.

Which cardiac abnormality is most likely to be found?

O Aortic regurgitation
Atrial septal defect

O Dilated cardiomyopathy
O Pulmonary regurgitation
O Persistent ductus arteriosus

An 18-year-old man with Marfan syndrome is reviewed in the cardiology clinic, after a screening ECG is found to

be abnormal, with left axis deviation and prominent Q waves in I, III, aVF and V3-V6.

Which cardiac abnormality is most likely to be found?

Aortic regurgitation CORRECT ANSWER


Atrial septal defect

Dilated cardiomyopathy

Pulmonary regurgitation
Persistent ductus arteriosus

Aortic regurgitation and Marfan syndrome


Marfan syndrome is characteristically associated with progressive aortic root dilatation leading to aortic
regurgitation and an increased risk of dissection

Other skeletal manifestations include


tall stature
scoliosis
chest wall malformations
high arched palate
lens dislocation

Mitral valve prolapse is also common but there is no association with other congenital malformations or
cardiomyopathy

A 67-year-old man with chronic heart failure is reviewed in terms of his drug therapy.

Which one of the following treatments has no proven mortality benefit?


O Bisoprolol

Digoxin

O Enalapril
O Nitrates and hydralazine
O Spironolactone

A 67-year-old man with chronic heart failure is reviewed in terms of his drug therapy.

Which one of the following treatments has no proven mortality benefit?


Bisoprolol
Digoxin CORRECT ANSWER

Enalapril
Nitrates and hydralazine

Spironolactone

Drug therapy for heart failure


Bisoprolol (CI BIS II), spironolactone (RALES), enalapril (CONSENSUS) and nitrates and hydralazine (V-HEFT)
have all been shown to improve mortality in chronic heart failure
Digoxin reduces the risk of death owing to heart failure but overall cardiovascular mortality is similar to that on
placebo, probably reflecting a small increase in the risk of arrhythmic death with digoxin therapy

What is the most likely lipid abnormality in a 48-year-old Asian man with type 2 diabetes who has
good glycaemic control?

O Elevated high-density lipoprotein (HDL)


Elevated low-density lipoprotein (LDL)

O Elevated LDL/elevated triglycerides


Low HDL/elevated LDL

O Low HDL/elevated triglycerides

What is the most likely lipid abnormality in a 48-year-old Asian man with type 2 diabetes who has
good glycaemic control?

Elevated high-density lipoprotein (HDL)


Elevated low-density lipoprotein (LDL)
Elevated LDL/elevated triglycerides
Low HDL/elevated LDL
Low HDL/elevated triglycerides CORRECT ANSWER

Lipid profile in the Asian population


Asians do not have classical LDL-related risk for ischaemic heart disease
Their profile includes low HDL and elevated triglycerides, meaning that measurement of LDL alone may
underestimate their risk

A 72-year-old man presents with 15 min of central crushing chest pain. ECG shows 0.5 mm ST elevation in leads
V1 and V2. You are in a peripheral hospital with no acute cardiac catheterisation lab.

What is the most appropriate treatment?

O Accelerated tissue plasminogen activator (tPA) + aspirin


Aspirin + heparin and repeat ECG in 15 min

O Heparin only
O No treatment and repeat ECG in 15 min
O Streptokinase + aspirin

A 72-year-old man presents with 15 min of central crushing chest pain. ECG shows 0.5 mm ST elevation in leads
V1 and V2. You are in a peripheral hospital with no acute cardiac catheterisation lab.

What is the most appropriate treatment?


Accelerated tissue plasminogen activator (tPA) + aspirin
Aspirin + heparin and repeat ECG in 15 min CORRECT ANSWER

Heparin only

No treatment and repeat ECG in 15 min


Streptokinase + aspirin

Drug treatment for a suspected coronary event


The criteria for thrombolysis are 1 mm ST elevation in two or more limb leads or 2 mm ST elevation in adjacent
chest leads, so thrombolysis is not indicated here
However, in the context of a good history of cardiac pain and borderline ECG, an acute coronary event should be
strongly suspected and aspirin and heparin given prophylactically, with a repeat ECG in 15 min

A 56-year-old man has known tricuspid regurgitation.

Which part of the jugular venous waveform is likely to be most prominent?

a wave
O c wave

v wave

O x descent

y descent

A 56-year-old man has known tricuspid regurgitation.

Which part of the jugular venous waveform is likely to be most prominent?

a wave
c wave

v wave CORRECT ANSWER


x descent
y descent

Jugular venous waveform in tricuspid regurgitation


Tricuspid regurgitation characteristically causes loss of the x descent in the jugular venous pressure causing fusion
of the c and v waves to produce 'giant' v waves, also known as 'cv' waves or 's! waves

A 56-year-old lady has a known ventricular septal defect. Which one of the following clinical signs

would most indicate the presence of established pulmonary hypertension?

O Loud systolic murmur

O Raised jugular venous pressure (JVP)


O Single loud second heart sound
O Systolic thrill
Displaced apex beat

A 56-year-old lady has a known ventricular septal defect. Which one of the following clinical signs

would most indicate the presence of established pulmonary hypertension?

Loud systolic murmur

Raised jugular venous pressure (JVP) CORRECT ANSWER


Single loud second heart sound

Systolic thrill
Displaced apex beat

Clinical signs of pulmonary hypertension


The systolic murmur and thrill of the ventricular septal defect are absent once Eisenmenger's complex has
developed
Under these circumstances there are just signs of pulmonary hypertension and cyanosis
Cardiomegaly may occur with Eisenmenger's complex because of right venticular enlargement but the left ventricle
is not usually significantly enlarged and the apex beat is not typically displaced
A single second heart sound is characteristically associated with Fallot's tetralogy where pulmonary stenosis
protects from developing pulmonary hypertension

A 30-year-old postman with hypertension but normally in good health presents to the Emergency Department
with sudden severe breathlessness and sweating. Chest examination reveals bilateral basal crackles. He
improves with diamorphine and frusemide (furosemide). ECG and cardiac enzymes are normal. He develops two
further episodes of pulmonary oedema which respond well to diuretics. Investigations in the follow-up clinic
reveal evidence of left ventricular hypertrophy on chest x-ray but with a preserved ejection fraction on
echocardiogram, and an elevated creatinine of 145 pmol/l.

What is the most likely cause of pulmonary oedema?

O Dilated cardiomyopathy
O Myocarditis
O Ischaemic heart disease
O Phaeochromocytoma

O Renal artery stenosis

A 30-year-old postman with hypertension but normally in good health presents to the Emergency Department
with sudden severe breathlessness and sweating. Chest examination reveals bilateral basal crackles. He

improves with diamorphine and frusemide (furosemide). ECG and cardiac enzymes are normal. He develops two
further episodes of pulmonary oedema which respond well to diuretics. Investigations in the follow-up clinic
reveal evidence of left ventricular hypertrophy on chest x-ray but with a preserved ejection fraction on
echocardiogram, and an elevated creatinine of 145 pmol/l.

What is the most likely cause of pulmonary oedema?


Dilated cardiomyopathy

Myocarditis
Ischaemic heart disease

Phaeochromocytoma
Renal artery stenosis CORRECT ANSWER

Renal artery stenosis


The differential diagnosis of acute pulmonary oedema includes
severe left ventricular dysfunction
paroxysmal arrhythmias
three-vessel or left main stem coronary disease
In the context of hypertension it also includes
renal artery stenosis
phaeochromocytoma
Renal artery stenosis is more common
This man is a postman and therfore would be expected to have reasonable exercise tolerance, given his age this
makes cardiomyopathy, myocarditis and ischaemic heart disease extremely unlikely
Renal artery stenosis in a 30-year-old is much more common than phaeochromocytoma

A 45-year-old man with a strong family history of ischaemic heart disease presents with atypical chest pains. ECG
exercise testing shows J-point depression of 1 mm with a heart rate of 120 bpm.

What is the most appropriate next step?

O Coronary angiography
O Dobutamine stress echocardiography
O Radionuclide myocardial perfusion scanning
O Reassure and discharge

q Repeat ECG exercise testing on anti-anginal medication

A 45-year-old man with a strong family history of ischaemic heart disease presents with atypical chest pains. ECG

exercise testing shows J-point depression of 1 mm with a heart rate of 120 bpm.

What is the most appropriate next step?


Coronary angiography

Dobutamine stress echocardiography


Radionuclide myocardial perfusion scanning

Reassure and discharge CORRECT ANSWER


Repeat ECG exercise testing on anti-anginal medication

J-point depression during ECG exercise test


J-point depression is a physiological response to an increase in heart rate and ST segments should, therefore, be
measured 80 ms post-J

J-point depression produces upward sloping ST depression which has little predictive value for coronary artery
disease

Horizontal and downward sloping segments are more predictive than upward sloping ones
In a young man with atypical chest pain, this exercise test would be reassuring and there would be no indication for
further investigation or treatment

A 56-year-old man presents with a cardiac rhythm disorder. Which one of the following scenarios

would be an indication for temporary transvenous cardiac pacemaker insertion?

Q Asymptomatic 2.8 s sinus pauses

A short period of complete heart block complicating inferior myocardial infarction (pre-thrombolysis)
with blood pressure 110/70 mmHg

q Asymptomatic complete heart block with broad complex ventricular complexes at 35 bpm
Mobitz II AV block complicating anterior myocardial infarction with blood pressure 110/70 mmHg

O Bifascicular block prior to aortic aneurysm repair

A 56-year-old man presents with a cardiac rhythm disorder. Which one of the following scenarios

would be an indication for temporary transvenous cardiac pacemaker insertion?

Asymptomatic 2.8 s sinus pauses


A short period of complete heart block complicating inferior myocardial infarction (pre-thrombolysis)

with blood pressure 110/70 mmHg


Asymptomatic complete heart block with broad complex ventricular complexes at 35 bpm
Mobitz II AV block complicating anterior myocardial infarction with blood pressure 110/70
mmHg

CORRECT ANSWER
Bifascicular block prior to aortic aneurysm repair

Need for a transvenous cardiac pacemaker


Asymptomatic sinus node disease, even with prolonged pauses, carries a low risk of sudden death and can usually
be managed without a temporary wire
Chronic complete heart block has a higher risk, but if asymptomatic and with a stable escape rhythm can also be
managed in most cases without a temporary wire

Bifascicular block carries a slightly higher risk of high-grade atrioventricular (AV) block but, in asysmptomatic
patients, this risk is sufficiently low to obviate the need for perioperative pacing
In myocardial infarction, AV block and even complicating Ml should be managed conservatively if asymptomatic
and causing no haemodynamic compromise
However, second- or third-degree heart block complicating anterior Ml requires pacing

A 40-year-old man is noted to have palmar crease xanthomas.

Which form of lipid disorder is most likely?


Familial hypercholesterolaemia
Q Familial mixed hypercholesterolaemia

Hyperchylomicronaemia

O Familial hypertriglyceridaemia
O Broad p disease

A 40-year-old man is noted to have palmar crease xanthomas.

Which form of lipid disorder is most likely?


Familial hypercholesterolaemia
Familial mixed hypercholesterolaemia

Hyperchylomicronaemia
Familial hypertriglyceridaemia

Broad p disease CORRECT ANSWER

Palmar crease xanthomas


Palmar crease xanthomas are pathognomonic of type III hyperlipidaemia also known as broad (3 disease
Familial hypercholesterolemia (type Ila hyperlipidaemia) has high total cholesterol and low-density lipoprotein
(LDL) values due to impaired LDL receptor activity
It is associated with
severe atherosclerosis and premature coronary heart disease

xanthomas
xanthelasmata
corneal arcus

Hyperchylomicronaemia is type I hyperlipidaemia due to lipoprotein lipase enzyme deficiency


Familial hypertriglyceridaemia is a form of type IV hyperlipidaemia

A 15-year-old patient undergoes echocardiography, which shows a right-sided aortic arch.

Which cardiac condition is most likely?

O Coarctation of the aorta


O Ebstein's anomaly
Q Tetralogy of Fallot

O Hypoplastic left ventricle


O Noonan syndrome

A 15-year-old patient undergoes echocardiography, which shows a right-sided aortic arch.

Which cardiac condition is most likely?


Coarctation of the aorta

Ebstein's anomaly
Tetralogy of Fallot CORRECT ANSWER

Hypoplastic left ventricle


Noonan syndrome

Right-sided aortic arch on Echo


Approximately 25% of patients with Fallot's tetralogy have a right-sided aortic arch
Coarctation is associated with bicuspid aortic valve and Noonan syndrome with pulmonary stenosis
Ebstein's anomaly is a congenital abnormality of the tricuspid valve which has an association with right-sided
accessory pathways

A 36-year-old old woman presents with a cerebral infarct following treatment for a deep vein thrombosis.

Cardiovascular examination is entirely normal.

What is the most likely underlying cardiac abnormality?


Partial anomalous pulmonary venous drainage

O Ostium primum atrial septal defect


O Ostium secundum
Common atrium

O Patent foramen ovale

A 36-year-old old woman presents with a cerebral infarct following treatment for a deep vein thrombosis.
Cardiovascular examination is entirely normal.

What is the most likely underlying cardiac abnormality?


Partial anomalous pulmonary venous drainage

Ostium primum atrial septal defect


Ostium secundum

Common atrium
Patent foramen ovale CORRECT ANSWER

Patent foramen ovale


The incidence and importance of patent foramen ovale (PFO) remain controversial but up to 25% of people have a
PFO which may allow passage of a thrombus from the venous to systemic circulation when the right heart

pressures are increased - characteristically with Valsalva or following a pulmonary embolus

PFO are not associated with clinical signs and cannot normally be identified on transthoracic
Use of agitated saline contrast during echo is helpful in identifying PFO

echo

Other notes
Other types of ASDs are much less common than PFO and abnormal clinical signs are usually present
Partial anomalous pulmonary venous drainage means that between one and three pulmonary veins open into the
right atrium rather than the left atrium

There is no increased risk of right-to-left shunting

A 70-year-old man undergoes successful DC cardioversion for atrial fibrillation (AF).

Which one of the following factors best predicts long-term maintenance of sinus rhythm following this
procedure?

O Age under 75 years


O Normal left ventricular function

q Warfarin therapy
O No alcohol intake
O AF duration less than 6 months prior to cardioversion

A 70-year-old man undergoes successful DC cardioversion for atrial fibrillation (AF).

Which one of the following factors best predicts long-term maintenance of sinus rhythm following this
procedure?
Age under 75 years

Normal left ventricular function


Warfarin therapy
No alcohol intake

AF duration less than 6 months prior to cardioversion CORRECT ANSWER

Maintenance of atrial fibrillation after cardioversion


Cardioversion has a much higher success rate in patients with structurally normal hearts but the left atrial size is a
better predictor than left ventricular function
AF is likely to be persistent where the left atrial dimension is > 5 cm
Age is much less important than the duration of AF
The success of cardioversion drops off significantly after 6 months of persistent AF and long-term sinus rhythm is
unlikely to be restored if AF has been persistent for more than 12 months

Alcohol is an important aetiological factor but less significant than AF duration


Warfarin is important to reduce stroke risk but does not help to restore or maintain sinus rhythm

A 60-year-old lady presents to the Cardiology Clinic. She has suffered increasing shortness of breath and

decreased exercise tolerance over the past few months. On examination her BP is 145/75 mmHg, pulse is 75/min,
atrial fibrillation. There are bilateral crackles on auscultation of the chest. You arrange for her to have an ECHO.

Which one of the following is most likely to lead to an increase in end-diastolic left ventricular dimension in
this patient?

0 Pulmonary stenosis
O Hypertrophic cardiomyopathy

0 Severe mitral regurgitation


O Pericardial effusion
Mitral stenosis

A 60-year-old lady presents to the Cardiology Clinic. She has suffered increasing shortness of breath and

decreased exercise tolerance over the past few months. On examination her BP is 145/75 mmHg, pulse is 75/min,
atrial fibrillation. There are bilateral crackles on auscultation of the chest. You arrange for her to have an ECHO.

Which one of the following is most likely to lead to an increase in end-diastolic left ventricular dimension in
this patient?

Pulmonary stenosis
Hypertrophic cardiomyopathy
Severe mitral regurgitation CORRECT ANSWER

Pericardial effusion
Mitral stenosis

Left ventricular dysfunction


Increased left ventricular (LV) dimensions are characteristic in dilated cardiomyopathy
Any other condition which causes increased left ventricular work (eg to overcome left ventricular outflow
obstruction in aortic stenosis) or increased left ventricular end-diastolic pressure (eg in end-stage mitral
regurgitation) will eventually cause LV dilatation
In hypertrophic cardiomyopathy, severe left ventricular hypertrophy prevents significant increase in intra-cavity
dimensions such as LV end-diastolic dimensions
In pericardial effusion, pericardial fluid prevents LV dilatation by external compression

Mitral and pulmonary stenosis do not affect left ventricular workload

A 38-year-old man of Chinese descent who smokes 60 cigarettes per day presents to his GP. He is developing

pain at rest in his legs, and is unable to walk more than a few yards owing to ischaemic pain. On examination

there is prolonged capillary refill and necrotic ulcers at the tips of his toes. There is also evidence of
thrombophlebitis.

What diagnosis fits best with this clinical picture?

O Buerger's disease
O Simple peripheral vascular disease
O Polyarteritis nodosa
O Familial hypercholesterolemia

O Temporal arteritis

A 38-year-old man of Chinese descent who smokes 60 cigarettes per day presents to his GP. He is developing
pain at rest in his legs, and is unable to walk more than a few yards owing to ischaemic pain. On examination

there is prolonged capillary refill and necrotic ulcers at the tips of his toes. There is also evidence of
thrombophlebitis.

What diagnosis fits best with this clinical picture?

Buerger's disease CORRECT ANSWER


Simple peripheral vascular disease

Polyarteritis nodosa
Familial hypercholesterolaemia
Temporal arteritis

Buerger's disease
Buerger's disease (thromboangiitis obliterans) is an occlusive inflammatory disease of small- to medium-sized
arteries of the upper and lower extremities.
Histopathology examination of affected arteries reveals fresh inflammatory thrombus within both small- and
medium-sized arteries and veins, with giant cells surrounding the thrombus

The disease is very closely associated with heavy smoking; continued smoking after diagnosis invariably leads to
a poor outlook, gangrene and multiple amputations

Prevalence is higher in men and people of Far-Eastern origin

Treatment

The main goal of therapy is elimination of tobacco smoking


Bypass surgery is of variable success owing to the distal nature of the occlusions
Sympathectomy may be useful in increasing distal blood flow and relieving pain

Amputation of gangrenous digits is frequently required

A 42-year-old painter presents to the Emergency Department with symptoms of vertigo, diplopia and gait
unsteadiness at the end of a busy afternoon painting the interior of a property. On examination there is a

markedly lower blood pressure in the left arm.

What diagnosis fits best with this clinical picture?

O Anterior circulation transient ischaemic attack


O Subclavian steal syndrome
O Vestibular neuronitis

O Posterior circulation transient ischaemic attack

q Unexplained cardiac arrhythmia

A 42-year-old painter presents to the Emergency Department with symptoms of vertigo, diplopia and gait

unsteadiness at the end of a busy afternoon painting the interior of a property. On examination there is a
markedly lower blood pressure in the left arm.

What diagnosis fits best with this clinical picture?


Anterior circulation transient ischaemic attack

Subclavian steal syndrome CORRECT ANSWER

Vestibular neuronitis
Posterior circulation transient ischaemic attack
Unexplained cardiac arrhythmia

Subclavian steal syndrome


Subclavian steal syndrome results from occlusion or stenosis of the proximal subclavian artery, leading to
decreased antegrade or retrograde flow in the ipsilateral vertebral artery

Symptoms

Many patients are asymptomatic


Upper extremity symptoms include fatigue, aching, coolness of the affected arm and some numbness
Neurological symptoms may include vertigo, diplopia, decreased vision, nystagmus and gait unsteadiness and
occur in around 25% of patients

Symptoms may be precipitated by extreme exercise on the affected side such as cricket bowling, use of an
underarm crutch or painting a wall

Pathophysiology
Subclavian steal produces symptoms by flow-related phenomena rather than embolic
When an atherosclerotic lesion in the proximal subclavian artery progresses to cause hemodynamically significant
stenosis, collateral vessels from the subclavian artery gradually enlarge

The upper extremity becomes dependent on these large collateral blood vessels that originate from the subclavian
artery distal to the obstruction

The collateral vessels serve as points of re-entry for blood flowing retrograde into the arm from the head, shoulder,
and neck, thereby providing the extremity with adequate perfusion

When the arm is exercised, the blood vessels dilate to enhance perfusion to the ischemic muscle, thus lowering
the resistance in the outflow vessels
Blood is siphoned from the head, neck, and shoulder through collateral vessels to supply this low-resistance
vascular bed, satisfying increased oxygen demand by the exercising muscles of the upper extremity
This results in posterior cerebral circulation neurological symptoms

Management
Appropriate imaging studies include non-invasive arterial flow studies, Doppler and arteriography
Most patients require no intervention, although surgical reconstruction may be required where symptoms are
severe

A 64-year-old woman presents with an episode of syncope while out shopping. On more direct questioning she
also reports a few episodes of fast palpitations, which she is able to tap out on her hand. She is discharged to

await an outpatient 24-h ECG Holter recording. Unfortunately she is readmitted after suffering a fit while in bed,
her husband felt her pulse at the time and claims that she was pulseless for a few seconds.

What diagnosis fits best with this clinical picture?

Paroxysmal atrial fibrillation


O Atrial flutter

Sick-sinus syndrome

O Multiple transient ischaemic attacks


O Epilepsy

A 64-year-old woman presents with an episode of syncope while out shopping. On more direct questioning she

also reports a few episodes of fast palpitations, which she is able to tap out on her hand. She is discharged to
await an outpatient 24-h ECG Holter recording. Unfortunately she is readmitted after suffering a fit while in bed,
her husband felt her pulse at the time and claims that she was pulseless for a few seconds.

What diagnosis fits best with this clinical picture?

Paroxysmal atrial fibrillation


Atrial flutter
Sick-sinus syndrome CORRECT ANSWER
Multiple transient ischaemic attacks

Epilepsy

Sick-sinus syndrome
Sick-sinus syndrome is characterised by
periods of sinus bradycardia
sinus arrest
a combination of sinoatrial or atrioventricular conduction defects
supraventricular tachycardias
In adults the disease is often associated with atherosclerosis, but may occur in the presence of a normal heart
Presentation may be with lightheadedness, syncope or palpitations, or patients may present with stroke
associated with atrial fibrillation

The cause is fibrosis or fatty infiltration of the sinus node, atrioventricular node, His bundle, or its branches
Treatment

Work-up includes ECG and ambulatory cardiac rhythm monitoring


A permanent pacemaker is the treatment of choice
Drug treatment of tachyarrhythmias may result in an increased risk of heart block

A 17-year-old youth is brought to the GP by his mother. He was previously seen 2 weeks earlier suffering from
acute pharyngitis. His teeth are in generally poor condition, but otherwise there is no previous medical history.
On examination he is febrile with a temperature of 38.2 C, and has a polyarthritis affecting his knees, ankles,

wrists and elbows. He also appears to have subcutaneous nodules over his elbows, and mitral regurgitation on
cardiovascular examination.

What diagnosis fits best with this clinical picture?

O Bacterial endocarditis
O Juvenile rheumatoid arthritis
O Scarlet fever
O Rheumatic fever

q Congenital valvular heart disease

A 17-year-old youth is brought to the GP by his mother. He was previously seen 2 weeks earlier suffering from
acute pharyngitis. His teeth are in generally poor condition, but otherwise there is no previous medical history.
On examination he is febrile with a temperature of 38.2 C, and has a polyarthritis affecting his knees, ankles,
wrists and elbows. He also appears to have subcutaneous nodules over his elbows, and mitral regurgitation on

cardiovascular examination.

What diagnosis fits best with this clinical picture?


Bacterial endocarditis
Juvenile rheumatoid arthritis
Scarlet fever
Rheumatic fever CORRECT ANSWER

Congenital valvular heart disease

Rheumatic fever
This patient's clinical condition is highly suggestive of rheumatic fever
Physical findings suggestive of rheumatic fever include
history of previous pharyngitis
fever
polyarthritis
carditis (including the mitral regurgitation murmur)
presence of subcutaneous extensor surface nodules
Laboratory testing suggestive of the diagnosis would include
a positive anti-streptolysin O titre (peaks at 4-5 weeks after a streptococcal throat infection)
raised erythrocyte sedimentation rate
C-reactive protein
A leucocytosis is also suggestive of rheumatic fever

Treatment
Acute treatment includes a course of penicillin to eradicate throat carriage of group A streptococci; where there is
carditis or arthritis, aspirin or prednisolone may be added, but specialist advice is advised
Erythromycin may be used in penicillin-allergic patients

A 65-year-old woman with a history of heavy smoking presents for review. She has woken during the early hours

of the morning for the second time with shortness of breath so bad that she had to fling open the windows. On
examination there are crackles in the lung bases, her chest X-ray shows bilateral fluffy perihilar shadowing. ECG
reveals small anterior Q waves and a sinus tachycardia of 105 bpm.

What diagnosis fits best with this clinical picture?

O Idiopathic pulmonary fibrosis


O Pulmonary embolus
Q Exacerbation of COPD

O Sarcoidosis
O Pulmonary oedema

A 65-year-old woman with a history of heavy smoking presents for review. She has woken during the early hours

of the morning for the second time with shortness of breath so bad that she had to fling open the windows. On
examination there are crackles in the lung bases, her chest X-ray shows bilateral fluffy perihilar shadowing. ECG
reveals small anterior Q waves and a sinus tachycardia of 105 bpm.

What diagnosis fits best with this clinical picture?


Idiopathic pulmonary fibrosis

Pulmonary embolus
Exacerbation of COPD
Sarcoidosis
Pulmonary oedema CORRECT ANSWER

Left ventricular failure


The history of paroxysmal nocturnal dyspnoea, chest X-ray suggestive of pulmonary oedema and ECG with
changes of a previous anterior myocardial infarction suggests that this woman is suffering from left ventricular
failure

A history of pink frothy sputum and distended neck veins on examination would also contribute to the diagnosis

Causes of pulmonary oedema


Causes of pulmonary oedema include
acute myocardial infarction
hypertensive heart failure
valvular disease
ventricular septal defect
cardiac tamponade
cardiac arrhythmias
endocarditis
myocarditis
cardiomyopathy

Echocardiography
Echocardiography is useful to determine the differential diagnoses, and provides information about
valvular disease
diastolic vs systolic dysfunction
ejection fraction
estimates of right-sided pressures

Management
The acute management of pulmonary oedema includes
oxygen therapy
intravenous furosemide
vasodilator therapy with iv nitrates

Many acute wards also have intermittent positive-pressure ventilation available, a useful adjunct to medical
therapy for left ventricular failure

An 18-year-old student who has never been vaccinated against measles presents to his GP with symptoms
suggestive of the disease. He is sent home and advised to rest, but later presents to the Emergency Department

with anterior chest pain that is worse on inspiration and relieved by sitting forward. On examination there appears
to be a rub on auscultation.

What diagnosis fits best with this clinical picture?

O Viral pleurisy
O Pericarditis
Myocardial ischaemia

0 Pneumothorax
Secondary bacterial pneumonia

An 18-year-old student who has never been vaccinated against measles presents to his GP with symptoms
suggestive of the disease. He is sent home and advised to rest, but later presents to the Emergency Department
with anterior chest pain that is worse on inspiration and relieved by sitting forward. On examination there appears
to be a rub on auscultation.

What diagnosis fits best with this clinical picture?


Viral pleurisy

Pericarditis CORRECT ANSWER


Myocardial ischaemia

Pneumothorax
Secondary bacterial pneumonia

Pericarditis
Pericarditis presents with anterior pleuritic chest pain, worse on inspiration and relieved by sitting forward

Associations
It is associated with a pericardial friction rub, which is best heard when the patient is upright and leaning forward
There may be associated cardiac tamponade, evidenced by tachycardia, low blood and pulse pressure and
distended neck veins

Origins
The origin of pericarditis may be
infectious (viral, bacterial or fungal)
inflammatory (eg rheumatoid, related to systemic lupus erythematosus, scleroderma or vasculitis)
drug-induced
myocardial infarction-related
postradiotherapy
uraemic
neoplastic
related to sarcoid
or to a host of other causes

Management and prognosis


For viral pericarditis, as in this case, limitation of activity is advised with additional pain relief using non-steroidals
and opiate-based agents such as codeine phosphate
In severe cases, oral prednisolone may be considered
Prognosis varies according to the underlying cause, but recurrence of pericarditis occurs in 10-15% of patients
with pericarditis within the first year

A 62-year-old man presents to the Emergency Department with persistent indigestion-like pain, retrosternal in
nature and radiating to his jaw. He is a heavy smoker of some 40 cigarettes per day and has been previously

treated for Barrett's oesophagus. On examination he is bradycardic at 55 bpm, has a blood pressure of 100/50
mmHg and looks unwell. ECG reveals ST elevation in leads II, III and aVF.

What diagnosis fits best with this clinical picture?

O Acute anterior myocardial infarction


O Non-Q wave myocardial infarction
Q Inferior myocardial infarction

O Recurrence of Barrett's oesophagus


O Unstable angina

A 62-year-old man presents to the Emergency Department with persistent indigestion-like pain, retrosternal in
nature and radiating to his jaw. He is a heavy smoker of some 40 cigarettes per day and has been previously
treated for Barrett's oesophagus. On examination he is bradycardic at 55 bpm, has a blood pressure of 100/50
mmHg and looks unwell. ECG reveals ST elevation in leads II, III and aVF.

What diagnosis fits best with this clinical picture?

Acute anterior myocardial infarction


Non-Q wave myocardial infarction

Inferior myocardial infarction CORRECT ANSWER

Recurrence of Barrett's oesophagus


Unstable angina

Acute myocardial infarction


This man has symptoms and signs of an acute myocardial infarction; changes in the inferior leads on ECG testing
confirm the diagnosis

Pain
Myocardial infarction is said to characteristically present with central crushing chest pain radiating to the arms and
jaw that is not pleuritic in character
However, it is important to note that large infarcts may be associated with a less typical pain distribution, and up to
20% of myocardial infarctions may not be associated with pain

Painless infarcts are commoner in the elderly and in patients with diabetes mellitus

Treatment
Acute treatment involves
oxygen therapy
nitrates
analgesia (including opiates and aspirin)
early angioplasty

Early postinfarct treatment includes the introduction of (3>-blockade and angiotensin-converting enzyme (ACE)
inhibition

A 43-year-old man presents with acute central chest pain radiating to his back. He is pale, sweaty and looks
extremely unwell. The admitting nurse notices that his blood pressures appear to be unequal when comparing
the left and right arms. Chest auscultation reveals aortic regurgitation, and on the ECG the complexes look small.

What diagnosis fits best with this clinical picture?

O Acute myocardial infarction


Pericarditis
Subacute bacterial endocarditis

O Aortic dissection
O Aortic regurgitation

A 43-year-old man presents with acute central chest pain radiating to his back. He is pale, sweaty and looks
extremely unwell. The admitting nurse notices that his blood pressures appear to be unequal when comparing

the left and right arms. Chest auscultation reveals aortic regurgitation, and on the ECG the complexes look small.

What diagnosis fits best with this clinical picture?


Acute myocardial infarction

Pericarditis
Subacute bacterial endocarditis

Aortic dissection CORRECT ANSWER


Aortic regurgitation

Aortic dissection
Aortic dissection may present with
hypertension or hypotension
unequal or absent pulses
aortic regurgitation
neurological abnormalities caused by vascular occlusion
Horner syndrome due to mass effect
cardiac tamponade caused by dissection into the pericardial sac
Predisposing factors include
hypertension
Marfan syndrome
congenital aortic valve abnormalities
syphilis infection
ECG may show left ventricular hypertrophy, or small complexes suggestive of cardiac tamponade
Chest X-ray may show mediastinal widening, but anteroposterior projection films overestimate the width of the
mediastinum anyway.

Management of type-A and type-B dissections


Dissections are generally classified into type-A dissections (involving the ascending aorta), and type-B dissections
(involving the descending aorta)

Type-A dissections usually require surgical intervention, although type-B dissections may be managed medically
Patients should be managed on the intensive care unit with aggressive management of blood pressure
For untreated patients, the mortality rate is said to approach 85%; even treated patients still have a mortality rate
approaching 20%

A 38-year-old man presents for review. His only previous history of note has been recurrent shoulder

subluxation. His main complaints are tiredness and increasing dyspnoea on exertion. The nursing clerk on
admission notes that he seems very tall and thin, his height is described as 1.93 m (6ft 4 inches). On examination
his blood pressure is 165/70 mmHg, he has left ventricular hypertrophy, a low-pitched apical diastolic murmur
and an early systolic apical ejection murmur.

What diagnosis fits best with this clinical picture?

O Mitral stenosis
Aortic regurgitation

O Mitral valve prolapse


O Aortic stenosis
O Infective endocarditis

A 38-year-old man presents for review. His only previous history of note has been recurrent shoulder

subluxation. His main complaints are tiredness and increasing dyspnoea on exertion. The nursing clerk on
admission notes that he seems very tall and thin, his height is described as 1.93 m (6ft 4 inches). On examination
his blood pressure is 165/70 mmHg, he has left ventricular hypertrophy, a low-pitched apical diastolic murmur
and an early systolic apical ejection murmur.

What diagnosis fits best with this clinical picture?


Mitral stenosis

Aortic regurgitation CORRECT ANSWER


Mitral valve prolapse

Aortic stenosis
Infective endocarditis

Aortic regurgitation
This man has a marfanoid habitus and is at risk of suffering aortic regurgitation

Aetiology
Aetiological factors involved in aortic regurgitation include
infective endocarditis
rheumatic heart disease
trauma with valvular rupture
congenital bicuspid aortic valve
myxomatous degeneration
syphilitic aortitis
systemic lupus erythematosus
aortic dissection
use of amphetamine slimming products

Symptoms
Symptoms of aortic regurgitation include
dyspnoea on exertion
syncope
chest pain
congestive heart failure

Investigation findings
Cardiac auscultation characteristically reveals displacement of the cardiac impulse downwards and to the left,
prominent S3 heard over the apex, a low-pitched apical diastolic rumble (Austin-Flint murmur) and an early systolic
apical ejection murmur
Chest X-ray may reveal left ventricular hypertrophy and aortic dilatation
Echocardiography reveals the coarse diastolic fluttering of the anterior mitral valve leaflet

Treatment
Surgical valve replacement is indicated in symptomatic patients with chronic aortic regurgitation who have
symptoms despite optimal medical management, and in acute aortic regurgitation where there is evidence of left

ventricular failure
Ideally, surgery should be considered before the ejection fraction falls to below 55%

An 18-year-old student is admitted from a night club in a state of collapse. On admission to the Emergency

Department his blood pressure is 90/45 mmHg, and he has a pulse of 190 bpm. ECG reveals a narrow complex
tachycardia, which is terminated with adenosine. ECG after termination of the tachycardia reveals a PR interval of

approximately 100 ms, and a slurred QRS complex with delta wave.

What diagnosis fits best with this clinical picture?

O Amphetamine overdose
Cocaine overdose

O Hypokalaemia-induced arrhythmia
Wolff-Parkinson-White syndrome

O Lown-Ganong-Levine syndrome

An 18-year-old student is admitted from a night club in a state of collapse. On admission to the Emergency

Department his blood pressure is 90/45 mmHg, and he has a pulse of 190 bpm. ECG reveals a narrow complex
tachycardia, which is terminated with adenosine. ECG after termination of the tachycardia reveals a PR interval of

approximately 100 ms, and a slurred QRS complex with delta wave.

What diagnosis fits best with this clinical picture?


Amphetamine overdose

Cocaine overdose

Hypokalaemia-induced arrhythmia
Wolff-Parkinson-White syndrome CORRECT ANSWER

Lown-Ganong-Levine syndrome

Wolff-Parkinson-White syndrome
Wolff-Parkinson-White (WPW) syndrome (due to accessory cardiac conduction pathway) presents with
paroxysmal tachycardias in 10% of patients aged 20-40 years, and 35% of sufferers aged over 60 years
Common types of arrhythmia at presentation include reciprocating tachycardia at 150-250 bpm (80%), atrial
fibrillation (15%) and atrial flutter (5%)

Thankfully, presentation with ventricular tachycardia is rare


Prevalence in the UK population is around 0.15%, being more frequent in males
Most WPW patients have a normal heart structure, but there may be associated mitral valve prolapse,
cardiomyopathy or Ebstein's anomaly in certain patients
ECG abnormalities are characterised by the presence of a PR interval < 120 ms and a QRS complex >120 ms with
slurred, slowly rising onset (delta wave)

The Lown-Ganong-Levine syndrome is characterised by a short PR interval and normal QRS complex on ECG

Treatment
Electrical cardioversion is the intervention of choice for narrow complex tachycardias in this situation
In the non-acute stage, radiofrequency ablation of the accessory pathway may be attempted

A 54-year-old man presents with an irregular tachycardia with a ventricular rate of around 130 bpm. He played in a

cricket match the previous day and consumed 28 units of alcohol on the evening of the match. On examination
his blood pressure is 95/50 mmHg.

What is the most likely diagnosis?

O Ventricular tachycardia
O Sick-sinus syndrome
Paroxysmal atrial fibrillation
O Atrial flutter

Sinus tachycardia

A 54-year-old man presents with an irregular tachycardia with a ventricular rate of around 130 bpm. He played in a
cricket match the previous day and consumed 28 units of alcohol on the evening of the match. On examination
his blood pressure is 95/50 mmHg.

What is the most likely diagnosis?


Ventricular tachycardia
Sick-sinus syndrome
Paroxysmal atrial fibrillation CORRECT ANSWER

Atrial flutter
Sinus tachycardia

Paroxysmal atrial fibrillation


This man has paroxysmal atrial fibrillation as evidenced by his irregular fast tachycardia
Episodes of tachycardia in this condition may occasionally be precipitated by an excess intake of alcohol or
caffeine

Other causes may be


acute myocardial infarction
atrial septal defect
or pre-excitation syndromes such as Wolff-Parkinson-White
Atrial flutter is associated with an absolutely regular rhythm of 150-220 bpm

Treatment
Standard therapy for atrial fibrillation of recent onset is electrical cardioversion, providing there are no
contraindications

Intravenous flecainide may be considered for chemical cardioversion in the absence of a history of ischaemic
heart disease; amiodarone is an acceptable alternative
Long-term prophylaxis with agents such as sotalol may be required

A 34-year-old man is brought in as an emergency by his wife. He has been unwell for a few days with severe

pharyngitis that is thought to be related to Epstein-Barr virus infection. On arrival in the Emergency Department
he is visibly dyspnoeic at rest, there is evidence of ankle swelling and bilateral crackles on auscultation of his

chest, his blood pressure is 100/55 mmHg and his pulse is 105 bpm. ECG reveals sinus tachycardia with non
specific ST-T wave changes. Troponin-T is elevated.

What diagnosis fits best with this clinical picture?

0 Acute myocardial infarction


O Pericarditis
O Cardiomyopathy
O Myocarditis
Subacute bacterial endocarditis

A 34-year-old man is brought in as an emergency by his wife. He has been unwell for a few days with severe
pharyngitis that is thought to be related to Epstein-Barr virus infection. On arrival in the Emergency Department

he is visibly dyspnoeic at rest, there is evidence of ankle swelling and bilateral crackles on auscultation of his
chest, his blood pressure is 100/55 mmHg and his pulse is 105 bpm. ECG reveals sinus tachycardia with non
specific ST-T wave changes. Troponin-T is elevated.

What diagnosis fits best with this clinical picture?


Acute myocardial infarction

Pericarditis
Cardiomyopathy
Myocarditis CORRECT ANSWER

Subacute bacterial endocarditis

Myocarditis
Causes
Causes of myocarditis may be
viral (eg coxsackievirus B, echovirus, poliovirus, adenovirus, mumps, HIV or Epstein-Barr virus)
bacterial (eg Staphylococcus aureus, Clostridium perfringens or Corynebacterium diphtheriae)
mycoplasma
fungal (eg Candida or Aspergillus spp)
parasitic (eg Trypanosoma cruzi, Trichinella , Echinococcus, amoeba or Toxoplasma spp)
rickettsial
spirochaetal (eg Lyme carditis)
Other possible causes are
rheumatic fever
drugs (eg cocaine, doxorubicin, sulphonamides, tetracycline, amphotericin B and 5-fluorouracil)
toxins such as carbon monoxide, lead or arsenic
systemic lupus erythematosus
sarcoidosis
or radiation

Investigations
The medical history may point to a possible cause of the carditis; laboratory testing may reveal
raised troponin-T levels
increased creatine kinase
increased erythrocyte sedimentation rate
or increased white blood cell count
Viral titres, cold agglutinins and Lyme disease titres may also help in determining aetiology
ECG often reveals sinus tachycardia with non-specific ST changes
Echocardiography reveals dilated and hypokinetic chambers with segmental wall motion abnormalities

Prognosis
Prognosis is dependent on aetiology, but the 5-year mortality rate may be as high as 50% in some cases

A 72-year-old woman presents with two syncopal episodes, and is brought to the Emergency

Department by her daughter. The second episode has occurred on a particularly hot day after a

family walk. She has a past history of hypertension and takes bendrofluazide. On admission to the
Emergency Department her blood pressure is 160/125 mmHg and there is an ejection systolic
murmur on auscultation of her chest that radiates to the carotids. What diagnosis best fits with this
clinical picture?

O Mitral regurgitation
O Hypertrophic cardiomyopathy
O Aortic stenosis
O Acute arrhythmia
O Dehydration due to diuretic use

A 72-year-old woman presents with two syncopal episodes, and is brought to the Emergency

Department by her daughter. The second episode has occurred on a particularly hot day after a

family walk. She has a past history of hypertension and takes bendrofluazide. On admission to the
Emergency Department her blood pressure is 160/125 mmHg and there is an ejection systolic
murmur on auscultation of her chest that radiates to the carotids. What diagnosis best fits with this
clinical picture?
Mitral regurgitation

Hypertrophic cardiomyopathy
Aortic stenosis CORRECT ANSWER

Acute arrhythmia

Dehydration due to diuretic use

Aortic stenosis
Aortic stenosis causes left ventricular outflow obstruction, which is manifest by a rough ejection systolic murmur,
best heard at the base of the heart, and transmitted to the carotids
As aortic stenosis becomes more severe, the sound of aortic valve closure begins to diminish in intensity
There is associated left ventricular hypertrophy, with narrowing of the pulse pressure in the later stages of aortic
stenosis

Symptoms
Symptoms commonly appear when the valve orifice decreases to less than 1 cm squared (normal orifice is 3 cm
squared)

Stenosis is considered severe when the orifice is less than 0.5 cm squared or the pressure gradient across the
valve is 50 mmHg or greater.
Symptoms of aortic stenosis include
angina

syncope (particularly exertional)


congestive heart failure
Gl bleeding may occur, as there is an association between aortic stenosis and haemorrhagic telangiectasia

Investigations
Investigations of choice are chest x-ray and echocardiography, with cardiac catheterisation in symptomatic
patients to assess the gradient across the valve

Treatment
Surgical valve replacement is the treatment of choice in appropriate patients.

An 82-year-old man was admitted to the Emergency Department from a local church service. He fainted and
another parishioner, who is a trained first-aider, reported that he was pulseless for a few seconds after the attack.
On examination his blood pressure was 165/95 mmHg (past history of hypertension), he had no murmurs on

auscultation of the chest and carotid auscultation was also normal. Outpatient 7-day ambulatory cardiac rhythm
monitoring was arranged, which is now reported as normal.

What diagnosis fits best with this presentation?

q Transient ischaemic attack


O Transient bradycardia
O Paroxysmal atrial fibrillation
O Simple syncope
Carotid sinus syndrome

An 82-year-old man was admitted to the Emergency Department from a local church service. He fainted and
another parishioner, who is a trained first-aider, reported that he was pulseless for a few seconds after the attack.
On examination his blood pressure was 165/95 mmHg (past history of hypertension), he had no murmurs on
auscultation of the chest and carotid auscultation was also normal. Outpatient 7-day ambulatory cardiac rhythm
monitoring was arranged, which is now reported as normal.

What diagnosis fits best with this presentation?


Transient ischaemic attack
Transient bradycardia

Paroxysmal atrial fibrillation


Simple syncope
Carotid sinus syndrome CORRECT ANSWER

Carotid sinus syndrome


Incidence

The incidence of carotid sinus syndrome is said to be around 10% in the adult population
This incidence increases with age, and men are affected twice as often as women
Presentation is rare below the age of 50 years
The incidence also increases in patients with hypertension, but often a definitive cause may not be identified

Investigations
Predisposing factors may include
head and neck tumours
neck surgery
significant lymphadenopathy
carotid body tumours
Physical examination in this case suggests that there is no significant cardiac pathology, and the 7-day Holter
monitor result makes arrhythmia less likely as a cause

Diagnosis and treatment


Supine carotid sinus massage with blood pressure and ECG monitoring is the diagnostic procedure
It should not be performed in patients with a history of cerebrovascular disease or carotid bruits, and should only
be applied to one artery at a time
The response may be cardioinhibitory with asystole for at least 3 s, or vasopressor with a drop of more than 30
mmHg (in the presence of symptoms) or more than 50 mmHg without symptoms
A mixed picture of a cardioinhibitory and vasopressor response may occur

Pacemaker insertion is recommended for patients with cardioinhibitory carotid sinus syndrome

A 62-year-old woman with a past history of rheumatic fever presents for review. In recent years she has been

well, but she underwent a dental extraction some 8 weeks ago. During the past 4 weeks she has suffered
intermittent fevers, chills and night sweats. On examination there is a pansystolic murmur, loudest at the apex.
Blood tests reveal a normochromic, normocytic anaemia and raised ESR.

Which organism is most likely to be responsible for this clinical picture?

O Staphylococcus epidermidis
Viridans streptococci

O Staphlococcus aureus
Candida albicans

O Enterococci

A 62-year-old woman with a past history of rheumatic fever presents for review. In recent years she has been

well, but she underwent a dental extraction some 8 weeks ago. During the past 4 weeks she has suffered
intermittent fevers, chills and night sweats. On examination there is a pansystolic murmur, loudest at the apex.
Blood tests reveal a normochromic, normocytic anaemia and raised ESR.

Which organism is most likely to be responsible for this clinical picture?


Staphylococcus epidermidis

Viridans streptococci CORRECT ANSWER

Staphlococcus aureus
Candida albicans

Enterococci

Subacute bacterial endocarditis

This patient has previous rheumatic fever (and would previously have received antibiotic cover for dental
extraction; however, there is no robust evidence that antibiotic prophylaxis reduces endocarditis risk)

From her presentation it is most likely that she now has subacute bacterial endocarditis

Organisms associated with subacute bacterial endocarditis include viridans streptococci, Streptococcus bovis.
enterococci and Staphylococcus aureus

Signs and symptoms

Patients may present with fever, chills or fatigue (said to occur in 25-80% of patients)
Heart murmurs may be absent in right-sided endocarditis
Embolic phenomena with peripheral manifestations may be found in up to 50% of patients
Other manifestations include finger clubbing, petechiae, Osier nodes, splinter haemorrhages and Janeway

lesions

Splenomegaly may also occur in association with subacute bacterial endocarditis

Investigations

It is crucial to collect at least three sets of blood samples for culture during the first 24 h, which should, if
possible, be taken before antibiotics are started
Normochromic, normocytic anaemia may also occur
The erythrocyte sedimentation rate (ESR) is elevated
A false-positive VDRL (syphilis test) may occur
Transthoracic echocardiography, with or without additional transoesophageal echo is indicated to confirm

diagnosis

A 72-year-old man was discharged following successful prosthetic aortic valve replacement. Apart from a small
Venflon abscess, which healed with appropriate dressings and cannula removal, his progress had been
unremarkable. Now, some 6 weeks later, he is brought to the Emergency Department by his wife, suffering from
malaise, fever and night sweats. On examination you can hear the murmur of his prosthetic heart valve. Blood
testing reveals mild anaemia and raised ESR. Transoesophageal echocardiography suggests the possibility of
vegetations.

Which one of the following regimens is the most appropriate initial choice of antibiotic therapy?
O Intravenous penicillin therapy

O Intravenous gentamicin therapy


O Intravenous penicillin and gentamicin
O Intravenous vancomycin, gentamicin and oral rifampicin
O Intravenous gentamicin and vancomycin

A 72-year-old man was discharged following successful prosthetic aortic valve replacement. Apart from a small
Venflon abscess, which healed with appropriate dressings and cannula removal, his progress had been
unremarkable. Now, some 6 weeks later, he is brought to the Emergency Department by his wife, suffering from
malaise, fever and night sweats. On examination you can hear the murmur of his prosthetic heart valve. Blood
testing reveals mild anaemia and raised ESR. Transoesophageal echocardiography suggests the possibility of
vegetations.

Which one of the following regimens is the most appropriate initial choice of antibiotic therapy?
Intravenous penicillin therapy
Intravenous gentamicin therapy
Intravenous penicillin and gentamicin

Intravenous vancomycin, gentamicin and oral rifampicin CORRECT ANSWER


Intravenous gentamicin and vancomycin

Prosthetic valve endocarditis


The choice of antibiotics to treat endocarditis should be guided by local policy, but in the case of possible

prosthetic valve endocarditis, regime D is the most appropriate


Early prosthetic valve endocarditis is usually caused by Staphylococcus epidermidis, occurring during the first 2
months following a valve replacement
Other causative organisms include Staphylococcus aureus, Gram-negative bacilli, diphtheroids and Candida
species
In patients with prosthetic valves, transthoracic echocardiography is less sensitive than transoesophageal echo
for detecting valve abnormalities
Unfortunately, medical therapy is rarely successful in prosthetic valve endocarditis, and surgical valve replacement
under antibiotic cover is usually required

Other notes
Regime C is the most appropriate initial therapy for non-prosthetic valve endocarditis

A 22-year-old student is admitted by ambulance from a local night club. He has no previous medical history of

note and is adopted so is unaware of his family history. Bystanders who have accompanied him say that he

suffered sudden collapse while dancing. Bouncers at the club claim that they couldn't feel a strong pulse during
his period of unconsciousness. On admission his blood pressure is 120/60 mmHg, and pulse is 80 bpm and
regular. ECG looks normal; corrected QT interval is 0.6 s.

Which one of the following diagnoses fits best with the patient's clinical picture?

O Simple syncope
O Long QT syndrome - mutation uncharacterised
Q Ecstasy overdose
Q Carotid sinus syndrome

Jervell and Lange-Nielsen (JLN) syndrome

A 22-year-old student is admitted by ambulance from a local night club. He has no previous medical history of
note and is adopted so is unaware of his family history. Bystanders who have accompanied him say that he
suffered sudden collapse while dancing. Bouncers at the club claim that they couldn't feel a strong pulse during
his period of unconsciousness. On admission his blood pressure is 120/60 mmHg, and pulse is 80 bpm and
regular. ECG looks normal; corrected QT interval is 0.6 s.

Which one of the following diagnoses fits best with the patient's clinical picture?

Simple syncope

Long QT syndrome - mutation uncharacterised CORRECT ANSWER

Ecstasy overdose
Carotid sinus syndrome

Jervell and Lange-Nielsen (JLN) syndrome

Long QT syndrome
This man's QT interval is prolonged; Jervell and Lange-Nielsen (JLN) syndrome is also associated with long QT,
but patients have deafness in addition to the cardiac rhythm abnormality
Episodes of severe QT prolongation and torsades de pointes ventricular tachycardia in congenital long QT
syndrome may be precipitated by increased adrenergic drive (such as that from dancing in a night club)

This patient is adopted, so that it may be possible that there is an unknown family history of sudden death
The molecular biology of long QT syndromes is heterogeneous, and a number of different mutations coding for
potassium or sodium channels may be responsible
Where specific mutations are identified, antiarrhythmic therapy may be specifically targeted to provide optimum
therapy
In patients who respond poorly to medical treatment, implantable defibrillator may be considered

A 26-year-old woman attends her GP for an insurance medical. Her previous medical history is unremarkable. On

examination, her BMI is 21, blood pressure is 105/62 mmHg, and auscultation of the heart reveals a mid systolic
click and a late systolic murmur (these findings being accentuated in the standing position).

What diagnosis fits best with this clinical picture?


Atrial septal defect

O Mitral regurgitation
O Mitral stenosis
Mitral valve prolapse

O Constrictive pericarditis

A 26-year-old woman attends her GP for an insurance medical. Her previous medical history is unremarkable. On

examination, her BMI is 21, blood pressure is 105/62 mmHg, and auscultation of the heart reveals a mid systolic
click and a late systolic murmur (these findings being accentuated in the standing position).

What diagnosis fits best with this clinical picture?


Atrial septal defect
Mitral regurgitation

Mitral stenosis
Mitral valve prolapse CORRECT ANSWER

Constrictive pericarditis

Mitral valve prolapse

Mitral valve prolapse is the posterior bulging of leaflets of the mitral valve in systole
Clinically, patients with mitral valve prolapse are often young females with a narrow anteroposterior (AP) chest
diameter, low body weight and low/normal blood pressure

Epidemiology

Mitral valve prolapse is thought to be present in around 4% of the population, with a higher incidence in females
Increased incidence is associated with:

autoimmune thyroid disease


Ehlers-Danlos syndrome
Marfan syndrome

pseudoxanthoma elasticum
pectus excavatum

Investigations

Cardiac auscultation reveals a mid to late systolic click, best heard at the apex, and a mid to late systolic

murmur
Echocardiography reveals bulging of the anterior and posterior mitral valve leaflets in systole
Associated embolic phenomena (stroke or transient ischaemic attack (TIA)) are rare

Complications

The incidence of complications of mitral valve prolapse is thought to be less than 1% per year, and treatment is
often not required

A 70-year-old woman had a history of dyspnoea and palpitations for six months. An electrocardiogram (ECG) at

that time showed atrial fibrillation. She was given digoxin, diuretics and aspirin. She now presents with two short
lived episodes of altered sensation in the left face, left arm, and leg. There is poor coordination of the left hand
but she tells you that this began around 6 months earlier. The echocardiogram (ECHO) was normal, as was a
computed tomography (CT) head scan.

Which one of the following is the most appropriate step in long-term management?

O Anticoagulation
O Carotid endarterectomy
O Clopidogrel
O Corticosteroid treatment
O No action

A 70-year-old woman had a history of dyspnoea and palpitations for six months. An electrocardiogram (ECG) at
that time showed atrial fibrillation. She was given digoxin, diuretics and aspirin. She now presents with two short
lived episodes of altered sensation in the left face, left arm, and leg. There is poor coordination of the left hand
but she tells you that this began around 6 months earlier. The echocardiogram (ECHO) was normal, as was a
computed tomography (CT) head scan.

Which one of the following is the most appropriate step in long-term management?
Anticoagulation CORRECT ANSWER

Carotid endarterectomy
Clopidogrel

Corticosteroid treatment
No action

Anticoagulation
Indications

Guidelines state that anticoagulation is indicated in patients with any one of:

prosthetic heart valve


prior history of rheumatic heart valve disease
prior history of stroke or transient ischaemic attack
age older than 75 years
hypertension or coronary artery disease with poor left ventricle (LV) function

Other risk factors that occur concurrently with atrial fibrillation and suggest a need for possible anticoagulation

include (clinicians look for two of these 'moderate' risk factors):

diabetes mellitus

age 65-75 years


coronary artery disease with normal LV function

A Cochrane analysis has suggested that most patients with atrial fibrillation should be considered for
anticoagulation unless there are specific reasons not to
In the case of this woman she has suffered at least two transient ischaemic attacks (TIAs), with some residual
poor damage to coordination in the left arm, so she now fits the criteria for anticoagulation
Carotid endarterectomy is indicated where there is symptomatic carotid stenosis
Clopidogrel would be indicated in TIA without atrial fibrillation
Corticosteroids may be considered in cases of cerebral oedema where there is significant mass effect

A 21-year-old woman has a history of palpitations and light-headedness. The electrocardiogram (ECG) shows a
short PR interval and inferior Q waves. Her symptoms improve with atenolol 25 mg/day, but she has had two
short episodes of similar symptoms in the previous 24 hours.

What is the long-term management of choice?

Anticoagulation
Oral amiodarone
Oral digoxin

Increase the dose of atenolol


Radiofrequency ablation

A 21-year-old woman has a history of palpitations and light-headedness. The electrocardiogram (ECG) shows a

short PR interval and inferior Q waves. Her symptoms improve with atenolol 25 mg/day, but she has had two
short episodes of similar symptoms in the previous 24 hours.

What is the long-term management of choice?

Anticoagulation
Oral amiodarone
Oral digoxin

Increase the dose of atenolol


Radiofrequency ablation CORRECT ANSWER

LGL and WPW syndromes


The short PR interval without delta wave suggests Long-Ganong-Levine (LGL) syndrome rather than WolfParkinson-White (WPW) syndrome
It is likely that the patient is suffering from short periods of supraventricular tachycardia, which result in her
palpitations and light-headedness

The management of WPW and LGL syndromes is similar: radiofrequency ablation is recommended for these
patients
Digoxin is not recommended, as it may result in an increased ventricular rate and worsen any circulatory
compromise during attacks of tachycardia
Long-term oral amiodarone therapy is not recommended in view of the age of this patient
Atenolol may be useful to manage ventricular rate during periods of tachycardia, but again is a suboptimal choice
for this patient in the long term

A 35-year-old woman presents with a history of intermittent light-headedness. Clinical examination

and 12-lead ECG are normal. Which one of the following, if present on a 24 h Holter ECG tracing,
would be the most clinically important?

Atrial premature beats

O Profound sleep-associated bradycardia


O Supraventricular tachycardia
Transient Mobitz type-1 atrioventricular block

O Ventricular premature beats

A 35-year-old woman presents with a history of intermittent light-headedness. Clinical examination

and 12-lead ECG are normal. Which one of the following, if present on a 24 h Holter ECG tracing,
would be the most clinically important?

Atrial premature beats


Profound sleep-associated bradycardia
Supraventricular tachycardia CORRECT ANSWER

Transient Mobitz type-1 atrioventricular block


Ventricular premature beats

Supraventricular tachycardia
Both atrial and ventricular premature beats are normal variants when seen on a 24 h Holter electrocardiogram
(ECG) tracing

Profound bradycardia may also occur during sleep and is a normal finding
Mobitztype-1 atrioventricular block carries less clinical significance than Mobitztype-2, because the risk of
progression to complete heart block is much lower

Thus, supraventricular tachycardia (SVT) carries the most clinical significance


Diagnosis of the underlying cause is based on the presence or absence of P-waves and P-wave morphology
Patients can be taught carotid sinus massage to avert SVTs at home, or adenosine can be used in non-asthmatic
patients for acute cardioversion to sinus rhythm
Class III antiarrythmics, such as sotalol, may be considered for prophylaxis

A 72-year-old man presents with an episode of collapse. He had experienced two similar episodes recently, each
lasting about one minute. Four years previously he suffered an anterior myocardial infarction. On examination he
was orientated and symptom-free with a regular pulse rate of 80 beats per minute (bpm), blood pressure 140/80
mmHg, and apex beat displaced to the left. There was an apical systolic murmur. There were no signs of trauma.
The electrocardiogram (ECG) shows sinus rhythm, Q waves, and ST segment elevation anteriorly without
reciprocal depression.

Which one of the following is the diagnosis?

O Acute anterior myocardial infarction


O Cerebrovascular accident
O Epileptic seizure
O Pulmonary embolism
O Ventricular tachycardia

A 72-year-old man presents with an episode of collapse. He had experienced two similar episodes recently, each
lasting about one minute. Four years previously he suffered an anterior myocardial infarction. On examination he
was orientated and symptom-free with a regular pulse rate of 80 beats per minute (bpm), blood pressure 140/80
mmHg, and apex beat displaced to the left. There was an apical systolic murmur. There were no signs of trauma.
The electrocardiogram (ECG) shows sinus rhythm, Q waves, and ST segment elevation anteriorly without
reciprocal depression.

Which one of the following is the diagnosis?


Acute anterior myocardial infarction
Cerebrovascular accident

Epileptic seizure
Pulmonary embolism
Ventricular tachycardia CORRECT ANSWER

Ventricular tachycardia
It is likely that this man has suffered a transient episode of ventricular tachycardia, which results in a period of
circulatory compromise that leads to the collapse

The persistent ST segment elevation in this case would not indicate acute myocardial infarction, but it is likely to
represent left ventricular aneurysm, a recognised complication of acute anterior myocardial infarction

Holter monitoring of his electrocardiogram (ECG) would be the investigation of choice - seven-day cardiac
monitors are now available and would indicate the best chance of capturing a period of ventricular tachycardia
The antiarrythmic of choice would be amiodarone, although this patient should probably be considered for an
implantable defibrillator once the diagnosis is confirmed

A 58-year-old male patient has suffered from a recent acute inferior myocardial infarction 3 days ago. He initially
recovered well and has been transferred to the recovery ward, but has become acutely unwell with a hypotensive
episode. There is a pansystolic murmur, which is accentuated by inspiration, along the lower left sternal border. A
Swan-Ganz catheter is inserted and the following noted: right atrial pressure is 12 (very high); calculated left
atrial pressure is 2 (low normal).

Which one of the following is the likely cause?

Right heart failure

O Left heart failure


O Mitral regurgitation
Tricuspid regurgitation

O Aortic regurgitation

A 58-year-old male patient has suffered from a recent acute inferior myocardial infarction 3 days ago. He initially
recovered well and has been transferred to the recovery ward, but has become acutely unwell with a hypotensive
episode. There is a pansystolic murmur, which is accentuated by inspiration, along the lower left sternal border. A
Swan-Ganz catheter is inserted and the following noted: right atrial pressure is 12 (very high); calculated left
atrial pressure is 2 (low normal).

Which one of the following is the likely cause?

Right heart failure


Left heart failure
Mitral regurgitation

Tricuspid regurgitation CORRECT ANSWER


Aortic regurgitation

Tricuspid regurgitation
Tricuspid regurgitation may occur in post-myocardial infarction, in association with:
cor pulmonale
rheumatic heart disease
infective endocarditis
carcinoid syndrome
Ebstein anomaly
other congenital abnormalities of the atrioventricular valves

Signs and symptoms


Regurgitation gives rise to high right atrial pressures (as seen here)
Physical signs include a large jugular venous cardiovascular wave and a pulsatile liver that pulsates in systole
A right ventricular impulse may be felt at the left sternal edge and there is a blowing pansystolic murmur

Other notes
Severe tricuspid regurgitation may require valve repair, or rarely replacement
Another consideration with this type of presentation post-myocardial infarction is pulmonary embolus: a high
proportion of those patients who die post-myocardial infarction do so because of thrombo-embolic disease

A 32-year-old man is recently diagnosed with ankylosing spondylitis. Echocardiogram shows a valvular
abnormality.

Which one of the following is the most likely diagnosis?

O Mitral regurgitation
O Aortic regurgitation
O Mitral stenosis
O Aortic stenosis

O Tricuspid stenosis

A 32-year-old man is recently diagnosed with ankylosing spondylitis. Echocardiogram shows a valvular
abnormality.

Which one of the following is the most likely diagnosis?


Mitral regurgitation
Aortic regurgitation CORRECT ANSWER

Mitral stenosis
Aortic stenosis

Tricuspid stenosis

Chronic aortic regurgitation


Causes and associations
Rheumatic heart disease
Syphilis
Seronegative arthritides, including:
ankylosing spondylitis
severe hypertension
congenital bicuspid aortic valve
aortic endocarditis
Marfan syndrome
osteogenesis imperfecta

Signs

Bounding or collapsing pulse


Eponymous signs that accompany that the bounding or collapsing pulse:
Quincke sign

De Musset sign
Duroziez sign

Signs

Bounding or collapsing pulse


Eponymous signs that accompany that the bounding or collapsing pulse:
Quincke sign
De Musset sign

Duroziez sign

Investigations

Chest X-ray

Electrocardiograph (ECG)
Echocardiogram
Cardiac catheterisation to assess the degree of regurgitation

Indications
Valve replacement is indicated before the appearance of significant left ventricular failure, as valve replacement
before the onset of symptoms is associated with a much more favourable prognosis

A patient presents with shortness of breath and ankle swelling. An echocardiogram has been ordered to
determine the left ventricular ejection fraction.

Which one of the following echocardiography modes is the most appropriate?

O M-mode
A-mode

O Modern transthoracic
Continuous wave

O Power wave

A patient presents with shortness of breath and ankle swelling. An echocardiogram has been ordered to
determine the left ventricular ejection fraction.

Which one of the following echocardiography modes is the most appropriate?


M-mode

A-mode
Modern transthoracic CORRECT ANSWER

Continuous wave

Power wave

Echocardiography
Transthoracic echocardiography
Modern transthoracic echocardiography combines real-time two-dimensional imaging of the myocardium and
valves with information about velocity and direction of blood flow obtained by Doppler and colour flow mapping
It is non-invasive, and a complete examination can be performed in most patients in less than 30 min

M-mode echocardiography
M-mode echocardiography has preceded modern two-dimensional imaging
Unlike two-dimensional imaging, which uses a series of sweeps across the heart, M-mode uses a single static
beam of very frequent ultrasound pulses
The narrow beam is analogous to a vertical mineshaft passing through various layers of rock
Displayed in real time, this results in reflections from cardiac structures being displayed as horizontal lines with
superficial structures at the top of the screen and the deeper structures at the bottom
These data are interpretable when one knows which structure each line represents, and the technique has
excellent spatial resolution

A 28-year-old man presents with a 2-year history of increasing dyspnoea with strenuous exertion.

Hypertrophic cardiomyopathy is diagnosed. Which is the most appropriate screening method for
his brother?

O Computed tomography (CT) scan


O Exercise tolerance test
Ventilation-perfusion scan
Q Echocardiography

Genetic screening

A 28-year-old man presents with a 2-year history of increasing dyspnoea with strenuous exertion.

Hypertrophic cardiomyopathy is diagnosed. Which is the most appropriate screening method for
his brother?
Computed tomography (CT) scan
Exercise tolerance test

Ventilation-perfusion scan
Echocardiography CORRECT ANSWER

Genetic screening

Hypertrophic cardiomyopathy
Hypertrophic cardiomyopathy is usually familial, with autosomal dominant transmission
The diagnosis of hypertrophic cardiomyopathy is based upon the demonstration of unexplained myocardial
hypertrophy, which is best done using two-dimensional echocardiography

The diagnosis requires that measurements of wall thickness exceed two standard deviations for sex-, age-, and
size-matched populations
In practice, in an adult of normal size, the presence of a left ventricular myocardial segment of 1.5 cm or greater in
thickness, in the absence of a recognised cause, is usually considered to be diagnostic

Less stringent criteria should be applied to first-degree relatives of an affected individual, where the probability of
carrying the disease gene increases from 1:500 to 1:2

A patient with left ventricular failure undergoes echocardiography.

Which one of the following is the correct formula for calculating the ejection fraction (EF)?

O EF= [end-diastolic volume(EDV) - end-systolic volume (ESV)]/EDV


EF = [end-diastolic volume(EDV) - end-systolic volume (ESV)]/heart rate (HR)
EF = [heart rate (HR) * end-diastolic volume (EDV)]/end-systolic volume (ESV)
EF = [heart rate (HR)

* end-systolic volume (ESV)]/end-diastolic volume (EDV)


EF = [end-systolic volume (ESV) - end-diastolic volume (EDV)]/EDV

A patient with left ventricular failure undergoes echocardiography.

Which one of the following is the correct formula for calculating the ejection fraction (EF)?

EF = [end-diastolic volume(EDV)

- end-systolic volume (ESV)]/EDV CORRECT ANSWER

EF = [end-diastolic volume(EDV) - end-systolic volume (ESV)]/heart rate (HR)


EF = [heart rate (HR) * end-diastolic volume (EDV)]/end-systolic volume (ESV)
EF = [heart rate (HR)

* end-systolic volume (ESV)]/end-diastolic volume (EDV)

EF = [end-systolic volume (ESV) - end-diastolic volume (EDV)]/EDV

Ejection fraction (EF) is calculated using the following equation:

EF = [end-diastolic volume (EDV) - end-systolic volume (ESV)]/EDV


In essence it is the difference in the volume of the volume between when the ventricle is full of blood, (end of diastole),
vs when it is most empty of blood, (then end of systole), expressed as a percentage. Normal individuals usually have
an ejection fraction between 50% and 65%.

A 69-year-old man has been admitted to the Emergency Department with syncope. He feels hot,

complains of nausea and then faints. His electrocardiogram (ECG) is normal. His brother suffers

from adult-onset epilepsy. Which one of the following is the investigation most likely to reveal the
underlying diagnosis?

O Electroencephalogram (EEG)
24-h ECG
Computed tomography (CT) of the brain

Q Echocardiography

Tilt test

A 69-year-old man has been admitted to the Emergency Department with syncope. He feels hot,

complains of nausea and then faints. His electrocardiogram (ECG) is normal. His brother suffers

from adult-onset epilepsy. Which one of the following is the investigation most likely to reveal the
underlying diagnosis?

Electroencephalogram (EEG)

24-h ECG
Computed tomography (CT) of the brain

Echocardiography

Tilt test CORRECT ANSWER

The tilt test


The development of tilt testing has allowed the study of the pathophysiology of neurocardiogenic syncope

Procedure
The patient is strapped to a tilt-table and is tilted, head upright, usually at 70 degrees for up to 45 min
Protocols that use additional provocation with isoprenaline or nitrates are also commonly used
Blood pressure and cardiac rhythm are monitored throughout the tilt test

Patient responses
In neurocardiogenic syncope, the patient classically maintains normal blood pressure initially, until the sudden
onset of syncope is associated with severe hypotension and bradycardia, often preceded by tachycardia

These features resolve with return to the supine posture


Some patients have a mainly vasodepressor response, with hypotension and little change in heart rate, whereas
others have a marked cardioinhibitory response, with severe bradycardia or asystole of several seconds' duration
However, most patients exhibit a mixed response, and those patients with marked cardioinhibition also have a
preceding vasodepressor response

This is an important observation when treatment is considered: permanent pacing to maintain cardiac rhythm may
not cure all symptoms, because falls in blood pressure may still occur even when bradycardia is prevented

In this case the factor of the family history of adult-onset epilepsy is irrelevant given the clinical picture of syncope

A 62-year-old patient presents with atrial fibrillation of unknown duration.

Which drug may slow his ventricular rate over a prolonged period but is unlikely to result in cardioversion?
O Adenosine
Amlodipine

O Digoxin
O Flecanide
O Amiodarone

A 62-year-old patient presents with atrial fibrillation of unknown duration.

Which drug may slow his ventricular rate over a prolonged period but is unlikely to result in cardioversion?
Adenosine
Amlodipine

Digoxin CORRECT ANSWER

Flecanide
Amiodarone

Digoxin

Digoxin has inotropic actions based on inhibition of cardiac Na+/K+ ATPase

The antiarrhythmic activity appears to be mediated predominantly through vagal stimulation


Digoxin is used to slow ventricular rate in atrial fibrillation
Adenosine will reveal underlying tachycardia but is unlikely to result in cardioversion versus flecainide

Which one of the following statements is most indicative of myocardial ischaemia?

O Associated shortness of breath


Claudication
O Dizziness

Radiation to jaw

O Relief by glyceryl trinitrate

Which one of the following statements is most indicative of myocardial ischaemia?

Associated shortness of breath


Claudication

Dizziness

Radiation to jaw CORRECT ANSWER


Relief by glyceryl trinitrate

Angina
The pain of angina usually radiates out from the chest - the commonest sites of radiation include the following:
the neck and throat (causing a feeling of choking, strangulation or suffocation)
the jaw (and may be interpreted as toothache or problems with dentures)
down one or both arms - this is usually felt down the inside, under the axilla to the inner two fingers (by
contrast, muscular pain usually runs over the shoulder and down the outside of the arm)

Other sites include the abdomen, the back, and areas of previous injury
Angina is often misinterpreted as indigestion

Which one of the following is the best clinical marker of the severity of aortic stenosis?

Character of apex beat

O Character of carotid pulse


O Character of S2
Intensity of murmur
O Pulse rate

Which one of the following is the best clinical marker of the severity of aortic stenosis?

Character of apex beat


Character of carotid pulse
Character of S2 CORRECT ANSWER

Intensity of murmur
Pulse rate

Aortic stenosis
Physical findings of aortic stenosis may include a narrow pulse pressure, especially when stroke volume
decreases, and a slow-rising, small-volume carotid pulse
However, the poorly compliant arterial wall may mask these abnormalities, so that the carotid pulse appears
relatively normal
The cardiac apex impulse is forceful and sustained, but this finding may be masked by kyphosis (in which the
anteroposterior diameter of the chest is increased)

Heart sounds
The first heart sound is soft
The aortic component of the second heart sound is also soft; it may be inaudible when stenosis is severe and the
valve is heavily calcified
Reverse splitting of the second heart sound may occur in patients with left ventricular failure
A fourth heart sound is common but disappears in one-quarter of elderly patients who develop atrial fibrillation
Ejection sounds are rare because the valve cusps are immobile

A 55-year-old man who has sustained an acute Ml subsequently present with heart failure. As well as other

treatments the cardiologist has recommended that a p-blocker be commenced.

According to currently available evidence, which of the following p-blockers would be most appropriate?
Celiprolol

O Labetalol
O Bisoprolol
Propranolol

O Sotalol

A 55-year-old man who has sustained an acute Ml subsequently present with heart failure. As well as other

treatments the cardiologist has recommended that a p-blocker be commenced.

According to currently available evidence, which of the following p-blockers would be most appropriate?
Celiprolol

Labetalol
Bisoprolol CORRECT ANSWER

Propranolol

Sotalol

Beta-blockers

Beta-blockers may produce benefit in heart failure by blocking sympathetic activity


Bisoprolol and carvedilol reduce mortality in any grade of stable heart failure
Treatment should be initiated by those experienced in the management of heart failure
According to currently available evidence (see NICE guidelines on cardiac failure, 2003), bisoprolol, metoprolol
sustained release and carvedilol have shown the most useful effects

At present metoprolol is not licensed in the UK for this indication and so carvedilol or bisoprolol are the preferred
choices

A 67-year-old woman is found to have a small pericardial effusion located posteriorly on routine
echocardiography. There is no haemodynamic compromise, she has no past medical history of note.

Which one of the following is the most appropriate next step in her management?

q Diagnostic tap
O Mammography
O Tuberculosis screen
O Reassure
Right heart catheter examination

For how long should the warfarin be continued?

O 4 weeks
O 6 months

O 1 year

O 3 years
Stop with immediate effect

A 67-year-old woman is found to have a small pericardial effusion located posteriorly on routine

echocardiography. There is no haemodynamic compromise, she has no past medical history of note.

Which one of the following is the most appropriate next step in her management?
Diagnostic tap
Mammography

Tuberculosis screen
Reassure CORRECT ANSWER

Right heart catheter examination

Pericardial effusion
Once the diagnosis of pericardial effusion has been made, it is important to determine whether the effusion is
creating significant haemodynamic compromise

Asymptomatic patients without haemodynamic compromise, even with large pericardial effusions, do not need to
be treated with pericardiocentesis unless there is a need for fluid analysis for diagnostic purposes (eg, in acute

bacterial pericarditis, tuberculosis and neoplasias)


A previous history of TB or history of TB exposure may prompt screening for the presence of active disease

During preoperative assessment, a 67-year-old woman is found to have a small pericardial effusion located
posteriorly on routine echocardiography. Electrocardiogram (ECG) is entirely normal.

Which one of the following is the most appropriate next step in her management?

O Cardiac catheterisation
O Reassure
O Pericardiocentesis

O Diuretics
O Computed tomography (CT) of the heart

During preoperative assessment, a 67-year-old woman is found to have a small pericardial effusion located
posteriorly on routine echocardiography. Electrocardiogram (ECG) is entirely normal.

Which one of the following is the most appropriate next step in her management?
Cardiac catheterisation
Reassure CORRECT ANSWER

Pericardiocentesis

Diuretics
Computed tomography (CT) of the heart

Pericardial effusion
Once the diagnosis of pericardial effusion has been made, it is important to determine whether the effusion is
creating significant haemodynamic compromise

Asymptomatic patients without haemodynamic compromise, even with large pericardial effusions, do not need to
be treated with pericardiocentesis unless there is a need for fluid analysis for diagnostic purposes (eg in acute
bacterial pericarditis, tuberculosis and neoplasias)

An elderly man is admitted with syncope. He also complains of shortness of breath and is
diagnosed as having aortic stenosis. Which one of the following conditions when associated with
aortic stenosis would indicate a poor prognosis?

0 Aortic regurgitation
O Left ventricular failure

0 Electrocardiography (ECG) changes


O Endocarditis
Valvular calcification

An elderly man is admitted with syncope. He also complains of shortness of breath and is
diagnosed as having aortic stenosis. Which one of the following conditions when associated with
aortic stenosis would indicate a poor prognosis?

Aortic regurgitation

Left ventricular failure CORRECT ANSWER

Electrocardiography (ECG) changes


Endocarditis
Valvular calcification

Aortic stenosis
Symptomatic
The prognosis of symptomatic aortic stenosis is poor, with a 50% survival of only 1 to 2 years
Approximately half of the deaths are due to relentless haemodynamic deterioration, and the remainder are sudden
and unexpected

Asymptomatic
The prognosis of asymptomatic but haemodynamically severe aortic stenosis is somewhat better
However, older patients with a peak velocity of 4 m/s or more across the aortic valve are likely to become
symptomatic in a period of 2 years or less

A 62-year-old patient with underlying ischaemic heart disease had two transient episodes of loss of
consciousness but feels fine at present. Both episodes were preceded by a feeling of dizziness and 'vision going
black', and witnesses report that the subject went very pale and then collapsed, lying motionless for a few
seconds before making a rapid recovery. No abnormal movements were seen during the period of
unconsciousness and there were no external signs of a head injury. Examination reveals a BP of 135/75 mmHg,
pulse is 70/min and regular. There are bibasal crackles on auscultation of the chest. Routine bloods are
unremarkable, and a CXR reveals cardiomegaly.

Which one of the following investigations should you order next?

Echocardiography

O Computed tomography (CT) of the head


24-h electrocardiogram (ECG)

O Cardiac catheterisation
O Treadmill test

A 62-year-old patient with underlying ischaemic heart disease had two transient episodes of loss of
consciousness but feels fine at present. Both episodes were preceded by a feeling of dizziness and 'vision going
black', and witnesses report that the subject went very pale and then collapsed, lying motionless for a few
seconds before making a rapid recovery. No abnormal movements were seen during the period of
unconsciousness and there were no external signs of a head injury. Examination reveals a BP of 135/75 mmHg,
pulse is 70/min and regular. There are bibasal crackles on auscultation of the chest. Routine bloods are
unremarkable, and a CXR reveals cardiomegaly.

Which one of the following investigations should you order next?

Echocardiography
Computed tomography (CT) of the head
24-h electrocardiogram (ECG) CORRECT ANSWER
Cardiac catheterisation

Treadmill test

The patient presenting with transient episodes of loss of


consciousness
The key in assessing any episode of loss of consciousness is a detailed history including eye-witness descriptions
This is necessary to try to clinically distinguish between the many different possible aetiologies of such an
occurrence

In this case the pre-syncopal symptoms, as well as the brief nature of the attack, pallor, lack of convulsions and
prior cardiac history are in favour of either neurogenic syncope or an arrhythmia

Cardiac syncope
Loss of consciousness of cardiac origin may result from abnormalities of heart rhythm, due to extremes of rate,
either fast or slow, or from some major disturbance of cardiovascular function, with resultant reduced cerebral
perfusion
The importance in establishing the diagnosis of cardiac syncope is the associated adverse prognosis, which may
be improved with appropriate treatment
The probability of cardiac syncope is increased in the presence of structural cardiovascular disease identified from
the history, clinical examination or investigation

Syncope is defined as a transient loss of consciousness with the loss of postural tone, and is most commonly due

to cardiovascular mechanisms resulting in reduced cerebral perfusion

It is a common presentation, resulting in 1-2% of Emergency Department visits and up to 6% of hospital


admissions
The cause is often initially uncertain, and assessment must first differentiate syncope from other causes of loss of
consciousness, in particular epileptic seizures

The next priority is to identify high-risk patients


Documentation of cardiac rhythm during syncope is desirable, but is difficult to obtain because of the intermittent
and usually infrequent nature of the symptom

Holter monitoring is unlikely to record the rhythm during an episode, but may provide evidence of lesser degrees of
abnormality, which may support a diagnosis such as sinoatrial dysfunction

A 65-year-old woman with severe heart failure presents with increasing shortness of breath. Her current

pharmacological treatment consists of an angiotensin-converting enzyme (ACE) inhibitor, loop diuretic and pblocker. There is only mild ankle swelling and bilateral basal crackles on auscultation of the chest. Her BP is
142/82 mmHg.

Which one of the following is the most appropriate management?

O Add digoxin
O Add spironolactone
O Stop B-blocker
O Stop ACE inhibitor

q Add simvastatin

A 65-year-old woman with severe heart failure presents with increasing shortness of breath. Her current
pharmacological treatment consists of an angiotensin-converting enzyme (ACE) inhibitor, loop diuretic and pblocker. There is only mild ankle swelling and bilateral basal crackles on auscultation of the chest. Her BP is
142/82 mmHg.

Which one of the following is the most appropriate management?


Add digoxin
Add spironolactone CORRECT ANSWER

Stop -blocker
Stop ACE inhibitor

Add simvastatin

Treatment of heart failure


The positive inotropic effects of cardiac glycosides can be useful in reducing symptoms (mainly breathlessness) in
patients already taking diuretics and angiotensin-converting enzyme (ACE) inhibitors
However, they have no effect on mortality and must be used with caution in patients such as this who are prone to
disturbances of potassium

Low-dose spironolactone has been shown to have positive effects on cardiovascular mortality in patients with endstage heart failure

Large outcome studies with carvedilol and bisprolol have also shown a mortality benefit in heart failure
Guidance in severe fluid overload is to temporarily discontinue the beta-blocker, but it is probably not necessary
here
http://www.nejm.Org/doi/full/10.1056/NEJM 19990902341 1001

A 22-year-old cocaine addict presents with central crushing chest pain after apparently snorting 3 lines of the

drug. He is pale and sweaty. His blood pressure is 180/110 mmHg. ECG shows anterior ST elevation consistent
with myocardial infarction.

Which one of the following is the most appropriate treatment?

O Thrombolysis
Heparin
O Percutaneous coronary intervention
Naloxone

O Glycoprotein 2b/3a inhibitors

A 22-year-old cocaine addict presents with central crushing chest pain after apparently snorting 3 lines of the
drug. He is pale and sweaty. His blood pressure is 180/110 mmHg. ECG shows anterior ST elevation consistent

with myocardial infarction.

Which one of the following is the most appropriate treatment?

Thrombolysis
Heparin

Percutaneous coronary intervention CORRECT ANSWER


Naloxone
Glycoprotein 2b/3a inhibitors

Cocaine-induced myocardial ischaemia


Cocaine use has recently been implicated as a cause of unstable angina
Three possible mechanisms by which cocaine induces myocardial ischaemia are:
increased myocardial oxygen demand
decreased myocardial oxygen supply secondary to vasospasm or coronary thrombosis
direct myocardial toxicity
Documented cocaine use should not be considered to rule out underlying significant coronary artery disease
(CAD), as the drug may precipitate coronary vasospasm or acute myocardial infarction in the patient with

atherosclerotic CAD
Where urgent angioplasty is available, this is preferable to thrombolysis as outcome studies show it to be superior

A 16-year-old girl presents to the Emergency Department with a collapse and palpitations after

attending her end-of-term school disco. The only medication history of note includes a recent

antibiotic prescription for an infected toe. Past medical history includes allergy to penicillin. Family
history reveals that her mother died suddenly at the age of 34 when the daughter was 3 years old.
One aunt and one uncle have also passed away suddenly. Electrocardiogram (ECG) reveals sinus

rhythm in the Emergency Department, but the QT interval is prolonged at 550 ms (corrected). Which
one of the following conditions is most likely to be related to the patient's collapse?
Wolff-Parkinson-White type A

Wolff-Parkinson-White type B
Congenital long QT syndrome
Lown-Ganong-Levine syndrome

O Ebstein's anomaly

A 16-year-old girl presents to the Emergency Department with a collapse and palpitations after

attending her end-of-term school disco. The only medication history of note includes a recent

antibiotic prescription for an infected toe. Past medical history includes allergy to penicillin. Family
history reveals that her mother died suddenly at the age of 34 when the daughter was 3 years old.
One aunt and one uncle have also passed away suddenly. Electrocardiogram (ECG) reveals sinus

rhythm in the Emergency Department, but the QT interval is prolonged at 550 ms (corrected). Which
one of the following conditions is most likely to be related to the patient's collapse?
Wolff-Parkinson-White type A
Wolff-Parkinson-White type B

Congenital long QT syndrome CORRECT ANSWER


Lown-Ganong-Levine syndrome

Ebstein's anomaly

Congenital long QT syndrome


The QT prolongation and history of sudden death in the family suggests the possibility of congenital long QT
syndrome

QT interval can be prolonged in association with a number of medications, including:


erythromycin (prescribed for the patient's foot)
ketoconazole
antihistamines
antiarrhythmics
The condition is associated with torsades de pointes ventricular tachycardia
Lange-Nielsen syndrome is one syndrome of QT prolongation, which has been described in association with
congenital deafness; Romano-Ward syndrome is associated with normal hearing

Lown-Ganong-Levin syndrome
The Lown-Ganong-Levine syndrome (LGL) is usually considered in a class of pre-excitation syndromes that
includes the Wolff-Parkinson-White syndrome (WPW), LGL and Mahaim-type pre-excitation

Theories proposed to explain LGL have centered around the possible existence of intranodal or paranodal fibers
that bypass all or part of the atrioventricular (AV) node
Criteria for LGL include PR interval less than or equal to 0.12 s (120 ms), normal QRS complex duration and
occurrence of supraventricular tachycardia but not atrial fibrillation or atrial flutter

You review a 68-year-old woman who presents with a sudden episode of collapse while taking
communion in church. This has been her third syncopal episode. Past medical history of note
includes recently diagnosed severe hypertension, for which her GP has commenced enalapril
therapy. On examination her blood pressure is 160/130 mmHg, she has left ventricular hypertrophy
on clinical examination and a loud ejection systolic murmur. Auscultation of the chest reveals
bibasilar crackles consistent with mild heart failure. Which one of the following is the definitive
investigation of choice for this patient?
O Chest X-ray

O Electrocardiogram (ECG)
O Echocardiogram
O Cardiac catheterisation
24 h Holter monitor

You review a 68-year-old woman who presents with a sudden episode of collapse while taking
communion in church. This has been her third syncopal episode. Past medical history of note
includes recently diagnosed severe hypertension, for which her GP has commenced enalapril
therapy. On examination her blood pressure is 160/130 mmHg, she has left ventricular hypertrophy
on clinical examination and a loud ejection systolic murmur. Auscultation of the chest reveals
bibasilar crackles consistent with mild heart failure. Which one of the following is the definitive
investigation of choice for this patient?
Chest X-ray

Electrocardiogram (ECG)
Echocardiogram
Cardiac catheterisation CORRECT ANSWER

24 h Holter monitor

Aortic stenosis
This patient is suffering from symptomatic aortic stenosis, as evidenced by the history of syncope, hypertension,
left ventricular hypertrophy and harsh ejection systolic murmur

Although echocardiography will aid in diagnosis, gradient across the aortic valve may be underestimated because
of the possibility of multiple echo signals and co-existent left ventricular dysfunction

As such, cardiac catheterisation is the definitive investigation, as it allows for more accurate estimation of valve
gradient and characterisation of co-existent coronary artery disease, which may require intervention at the same

time

You are called urgently to review a 54-year-old man who has developed acute onset pulmonary oedema some 36
h after his myocardial infarction. On arrival you note that his blood pressure is 95/50 mmHg with a pulse of 100
bpm regular, and a pansystolic murmur is noted. There are crackles on auscultation of the chest consistent with
heart failure.

Which one of the following represents the next investigation of choice in this man?

O Troponin I
O Troponin T

Urgent chest X-ray

Referral for angiography


Urgent echocardiogram

You are called urgently to review a 54-year-old man who has developed acute onset pulmonary oedema some 36
h after his myocardial infarction. On arrival you note that his blood pressure is 95/50 mmHg with a pulse of 100
bpm regular, and a pansystolic murmur is noted. There are crackles on auscultation of the chest consistent with
heart failure.

Which one of the following represents the next investigation of choice in this man?
Troponin I
Troponin T

Urgent chest X-ray


Referral for angiography
Urgent echocardiogram CORRECT ANSWER

Acute pulmonary oedema


The timing of this man's deterioration coupled with a murmur of mitral regurgitation and acute pulmonary oedema
suggests the onset of papillary muscle dysfunction, or even rupture

Echocardiogram is the investigation of choice to demonstrate the mitral regurgitation


Management involves the use of vasodilators such as sodium nitroprusside and the use of angiotensin-converting
enzyme (ACE) inhibition if tolerated, (unlikely in this case due to hypotension)

Inotropic support with drugs such as dopamine or dobutamine may also be required
The case should be discussed with cardiothoracic surgical colleagues to assess suitability for surgical repair,
although this should be postponed until after haemodynamic stabilisation if possible, owing to the high risks of periinfarct surgical intervention

A 32-year-old woman who is known to be 17 weeks' pregnant presents for review. She has periods of paroxysmal

supraventricular tachycardia (SVT) and on this occasion has a ventricular rate of 165 bpm and a blood pressure of 105/50
mmHg, feeling faint and unwell.

Which one of the following antiarrhythmics would be the most appropriate prophylaxis for her?
Metoprolol

O Amiodarone
O Digoxin

O Phenytoin
O Propafenone

A 32-year-old woman who is known to be 17 weeks' pregnant presents for review. She has periods of paroxysmal

supraventricular tachycardia (SVT) and on this occasion has a ventricular rate of 165 bpm and a blood pressure of 105/50
mmHg, feeling faint and unwell.

Which one of the following antiarrhythmics would be the most appropriate prophylaxis for her?
Metoprolol CORRECT ANSWER

Amiodarone
Digoxin

Phenytoin
Propafenone

Paroxysmal supraventricular tachycardia


This patient has paroxysmal supraventricular tachycardia (SVT)
Although digoxin slows the ventricular rate in patients with chronic atrial fibrillation, it does not maintain sinus
rhythm in patients with paroxysmal tachycardia

Amiodarone is known to be teratogenic and is contraindicated in pregnancy


Guidelines suggest that metoprolol is potentially the most appropriate option for SVT in pregnancy

You are asked to review a 19-year-old woman who presents with increasing shortness of breath on
exercise. She is from a travelling family and has rarely encountered medical care. On examination
she appears of short stature with extra skin folds around her neck, and appears to have failure of
secondary sexual development. Her blood pressure is raised at 165/100 mmHg. She reports that her
legs feel tired all the time and she has occasional chest pain on exercising. Which one of the

following cardiac diagnoses fits best with her clinical condition?

O Pulmonary stenosis
Mitral regurgitation

O Coarctation of the aorta


O Aortic regurgitation

Hypertrophic obstructive cardiomyopathy

You are asked to review a 19-year-old woman who presents with increasing shortness of breath on
exercise. She is from a travelling family and has rarely encountered medical care. On examination

she appears of short stature with extra skin folds around her neck, and appears to have failure of
secondary sexual development. Her blood pressure is raised at 165/100 mmHg. She reports that her
legs feel tired all the time and she has occasional chest pain on exercising. Which one of the
following cardiac diagnoses fits best with her clinical condition?

Pulmonary stenosis
Mitral regurgitation
Coarctation of the aorta CORRECT ANSWER

Aortic regurgitation

Hypertrophic obstructive cardiomyopathy

Turner syndrome
Given this patient's clinical picture it appears that she has Turner syndrome
Turner syndrome is associated with coarctation of the aorta, and bicuspid aortic stenosis may also co-exist

Signs and symptoms


Symptoms include:
vigorous pulsation in the neck or throat
hypertension
tired legs or intermittent claudication on running
left ventricular failure
angina pectoris
aortic rupture
Physical signs to note include:
radiofemoral pulse delay
development of collateral vessels
left ventricular failure
ejection systolic murmur
Electrocardiogram (ECG) shows left ventricular hypertrophy, and right bundle branch block is common

A 41-year-old man with a family history of sudden death presents to casualty with a second

episode of collapse. On this occasion he is referred to the Cardiology Department for review.

Echocardiography reveals asymmetrical septal hypertrophy, abnormal systolic motion of the


anterior mitral valve leaflet and narrowing of the left ventricular outflow tract. The 24 h
electrocardiogram (ECG) monitoring as an outpatient reveals several periods of non-sustained
ventricular tachycardia. Which one of the following would be most appropriate for the management
of his arrhythmia?

O Oral flecainide 100 mg daily


Oral amiodarone 200 mg tds

O Oral amiodarone 200 mg daily


Implantable cardioverter defibrillator

O Phenytoin 100 mg po daily

A 41-year-old man with a family history of sudden death presents to casualty with a second

episode of collapse. On this occasion he is referred to the Cardiology Department for review.

Echocardiography reveals asymmetrical septal hypertrophy, abnormal systolic motion of the


anterior mitral valve leaflet and narrowing of the left ventricular outflow tract. The 24 h
electrocardiogram (ECG) monitoring as an outpatient reveals several periods of non-sustained
ventricular tachycardia. Which one of the following would be most appropriate for the management
of his arrhythmia?
Oral flecainide 100 mg daily
Oral amiodarone 200 mg tds
Oral amiodarone 200 mg daily

Implantable cardioverter defibrillator CORRECT ANSWER

Phenytoin 100 mg po daily

Hypertrophic obstructive cardiomyopathy

Given this clinical history, this patient has a diagnosis of hypertrophic obstructive cardiomyopathy

He has non-sustained ventricular tachycardias and is at significant risk of sudden death


Although myomectomy will impact on symptoms, it won't affect the risk of arrhythmia
Management in the past would have been medical therapy with amiodarone, but recent studies have demonstrated
superior efficacy for implantable cardioverter defibrillators (ICDs)

Given that long-term use of amiodarone is associated with significant morbidity, ICDs are taking over as
management of choice

A 72-year-old man presents for an extraction of three teeth under local anaesthesia. He has a past history of
rheumatic heart disease. Mitral stenosis has been identified, but the rotten teeth are being removed before valve
replacement. He is allergic to penicillin.

Which one of the following would be the most appropriate antibiotic regime for him?
Amoxicillin 3 g po 1 hour before procedure

O No prophylaxis necessary
O Vancomycin 1 g po 1 hour before procedure
Ciprofloxacin 1 g po 1 hour before procedure

O Augmentin 1 g po before procedure

A 72-year-old man presents for an extraction of three teeth under local anaesthesia. He has a past history of

rheumatic heart disease. Mitral stenosis has been identified, but the rotten teeth are being removed before valve
replacement. He is allergic to penicillin.

Which one of the following would be the most appropriate antibiotic regime for him?
Amoxicillin 3 g po 1 hour before procedure
No prophylaxis necessary

CORRECT ANSWER

Vancomycin 1 g po 1 hour before procedure


Ciprofloxacin 1 g po 1 hour before procedure

Augmentin 1 g po before procedure

Alternatives to penicillin
After an extensive evidence review, the most recent NICE guidelines do not recommend antibiotic prophylaxis for a
number of routine procedures including dental extraction
Their conclusion is based on the fact that they were unable to determine any evidence which suggested that
antibiotic prophylaxis prevented the development of endocarditis

This advice includes patients with rheumatic heart disease and those who have undergone valve replacement
http://www.nice.org.uk/cg064

A 45-year-old man attends for review. He has been suffering increasing shortness of breath over the past

few years. He is a non-smoker who drinks 20 units per week of alcohol and has no significant past
cardiovascular history. Now he presents with what seems to have been a transient ischaemic attack (TIA),
with weakness and co-ordination problems affecting his left side, which have resolved over the past 24
hours. On examination blood pressure is 142/95 mmHg and he is in sinus rhythm. There is no opening
snap, but there is a diastolic murmur, which changes in character according to posture. Bloods are
unremarkable, including C-reactive protein (CRP), which is in the normal range. Which one of the
following diagnoses fits best with this clinical picture?

O Right atrial myxoma


Left atrial myxoma

O Aortic stenosis
Mitral stenosis

O Mitral regurgitation

A 45-year-old man attends for review. He has been suffering increasing shortness of breath over the past

few years. He is a non-smoker who drinks 20 units per week of alcohol and has no significant past
cardiovascular history. Now he presents with what seems to have been a transient ischaemic attack (TIA),
with weakness and co-ordination problems affecting his left side, which have resolved over the past 24
hours. On examination blood pressure is 142/95 mmHg and he is in sinus rhythm. There is no opening
snap, but there is a diastolic murmur, which changes in character according to posture. Bloods are
unremarkable, including C-reactive protein (CRP), which is in the normal range. Which one of the
following diagnoses fits best with this clinical picture?

Right atrial myxoma


Left atrial myxoma CORRECT ANSWER
Aortic stenosis

Mitral stenosis
Mitral regurgitation

Left atrial myxoma

This patient has suffered a transient ischaemic attack (TIA)? most likely owing to embolus from an intra-cardiac

cause
One possible clinical explanation could be mitral stenosis, left atrial enlargement and atrial fibrillation, leading to
clot formation within the atrium, but this patient is in sinus rhythm, there is no opening snap on auscultation, and
the murmur changes in character with posture
This suggests the possibility of another cause, and left atrial myxoma would fit the bill
Myxoma can occur in any cardiac chamber, but occurs most commonly in the left atrium
It is a gelatinous, friable tumour, which leads to transient signs of mitral stenosis that occur only if the tumour
approaches the mitral valve orifice

Signs and symptoms

There is no opening snap


There may be an early diastolic plop as the tumour prolapses through the mitral valve
X-ray may show calcification within the tumour if it is long standing

Treatment and follow-up

Definitive treatment involves surgical excision; recurrence rate is extremely low, but follow-up is recommended
for a period of 5 years

Other notes

Right atrial myxomas are more rare and difficult to identify clinically; there may be evidence of multiple
pulmonary infarcts due to formation of emboli

A 75-year-old man is referred for total hip replacement. He has a history of hypertension and angina

and has suffered a myocardial infarction some 8 years earlier. Current medication includes atenolol

50 mg daily, ramipril 10 mg daily, aspirin 75 mg daily and isosorbide dinitrate 60 mg. Blood
pressure at the preoperative assessment was 160/80 mmHg, but the patient maintains that his
readings with the GP have been normal. He last had an exercise test some 3 years earlier and
managed 8 min with no significant electrocardiogram (ECG) changes. Which one of the following
investigations in addition to standard assessment would be most appropriate for the preoperative
assessment of this patient?

0 Repeat exercise ECG test


O Routine echocardiogram
O

"Tcm MIBI SPECT scan

O Stress ECG
Magnetic resonance angiography

A 75-year-old man is referred for total hip replacement. He has a history of hypertension and angina

and has suffered a myocardial infarction some 8 years earlier. Current medication includes atenolol
50 mg daily, ramipril 10 mg daily, aspirin 75 mg daily and isosorbide dinitrate 60 mg. Blood
pressure at the preoperative assessment was 160/80 mmHg, but the patient maintains that his
readings with the GP have been normal. He last had an exercise test some 3 years earlier and
managed 8 min with no significant electrocardiogram (ECG) changes. Which one of the following

investigations in addition to standard assessment would be most appropriate for the preoperative
assessment of this patient?
Repeat exercise ECG test

Routine echocardiogram CORRECT ANSWER

"Tcm MIBI SPECT scan


Stress ECG

Magnetic resonance angiography

Preoperative assessment
From the history given it appears that this man has relatively stable angina and there seems little value to be
gained from further imaging of his coronary arteries

Routine echocardiogram would add information about left ventricular function and point out any valvular disease
Given possible haemodynamic changes during total hip replacement, significant left venticule (LV) impairment or
valvular disease may impact significantly on operative prognosis

Blood pressure
The patient's blood pressure does appear labile and it would be worthwhile to document that his normal blood
pressure is within acceptable limits
If he has significant white coat hypertension, when he attends for surgery he may well be cancelled if his blood
pressure is too high
For this reason, 24 h outpatient blood pressure recording would seem sensible

A 64-year-old woman suffers from frequent and painful urinary tract infections. After her third

course of antibiotics in the past 6 months she is advised by the GP to take cranberry juice
supplements. Significant past medical history of note includes hypertension, for which she takes
ramipril and bendroflumethiazide, and hypercholesterolemia, for which she takes simvastatin.
There is also a history of paroxysmal atrial fibrillation, for which she takes warfarin and
amiodarone. Which one of her medications is most likely to interact with the cranberry juice?
O Simvastatin
O Amiodarone

O Bendroflumethiazide
O Warfarin
O Ramipril

A 64-year-old woman suffers from frequent and painful urinary tract infections. After her third

course of antibiotics in the past 6 months she is advised by the GP to take cranberry juice
supplements. Significant past medical history of note includes hypertension, for which she takes
ramipril and bendroflumethiazide, and hypercholesterolaemia, for which she takes simvastatin.
There is also a history of paroxysmal atrial fibrillation, for which she takes warfarin and
amiodarone. Which one of her medications is most likely to interact with the cranberry juice?
Simvastatin

Amiodarone
Bendroflumethiazide

Warfarin CORRECT ANSWER


Ramipril

Warfarin and cranberry juice


Cranberry juice contains a number of bioflavinoids, some of which are thought to cause inhibition of the
cytochrome p450 2C9 isoenzyme, which is responsible for warfarin metabolism
The warfarin interaction was given as a Committee for Safety of Medicines (CSM) warning in 2003 and features
prominently in the MRCP examination

Other notes
Metabolism of simvastatin is inhibited by grapefruit juice

A 63-year-old smoker is admitted with nausea, sweating and central crushing chest pain. A 12-lead

ECG reveals ST elevation in leads II, III and aVF. Which coronary artery is most likely to have been

affected in this case?


Circumflex artery

0 Left anterior descending artery


Right coronary artery
O Obtuse marginal artery

O Posterolateral artery

A 63-year-old smoker is admitted with nausea, sweating and central crushing chest pain. A 12-lead

ECG reveals ST elevation in leads II, III and aVF. Which coronary artery is most likely to have been
affected in this case?
Circumflex artery
Left anterior descending artery

Right coronary artery CORRECT ANSWER


Obtuse marginal artery

Posterolateral artery

Blood supply to the heart


The right coronary artery generally supplies:
the right ventricle
the posterior third of the interventricular septum
the inferior wall of the left ventricle
a portion of the posterior wall of the left ventricle
Occasionally the posterior interventricular septum may be supplied by a branch of the left circumflex artery: a socalled left-dominant circulation
The left anterior descending and left circumflex coronary arteries arise at the left main coronary artery bifurcation
and supply:

the anterior left ventricle


the bulk of the interventricular septum (anterior two-thirds)
the apex
the lateral and posterior left ventricular walls

A 25-year-old man was found by his family at home having suffered a cardiac arrest. He was previously well, apart

from well-controlled type 1 diabetes controlled with a basal bolus insulin regime. His family followed the
ambulance and ask if they can be in the resuscitation room. After 20 min of repeated resuscitation cycles, he has
remained in asystole. The table below gives the results of blood gas analyses:
pH

7.01

P02

8.4 kPa

PC02

3.9 kPa

Bicarb 10 mmol/l

Which one of the following is the most appropriate person to make the decision to discontinue
resuscitation?

Q Emergency Department consultant

On-call medical consultant

O Parents of the patient


O Patient's fiancee
Resuscitation team leader

A 25-year-old man was found by his family at home having suffered a cardiac arrest. He was previously well, apart

from well-controlled type 1 diabetes controlled with a basal bolus insulin regime. His family followed the
ambulance and ask if they can be in the resuscitation room. After 20 min of repeated resuscitation cycles, he has
remained in asystole. The table below gives the results of blood gas analyses:
pH

7.01

P02

8.4 kPa

PC02

3.9 kPa

Bicarb 10 mmol/l

Which one of the following is the most appropriate person to make the decision to discontinue
resuscitation?

Emergency Department consultant


On-call medical consultant

Parents of the patient


Patient's fiancee
Resuscitation team leader CORRECT ANSWER

Resuscitation decisions
Clear, appropriate communication is a key component of resuscitation
Although it may be appropriate to allow the presence of relatives within the resuscitation room, they do not have
the authority to continue or discontinue resuscitation, although they do need to be informed of progress
Although the experience of the Emergency Department or on-call medical consultants may be useful in gaining
advice, the resuscitation team leader is usually a senior anaesthetist or physician in their own right, and qualified
to decide on discontinuing resuscitation

If there is any doubt, the resuscitation team leader can discuss the situation with the consultant on call

A 71-year-old woman with a history of one previous myocardial infarction presents to the Emergency Department
She has sudden onset shortness of breath and palpitations, which happened after her dinner a couple of hours
earlier. A previous ECG from clinic a month earlier shows sinus rhythm. Medication includes ramipril 10 mg daily,
amlodipine 10 mg daily and aspirin 75 mg. On examination her blood pressure is 100/60 mmHg, her pulse is 140
bpm irregular and she has evidence of left ventricular failure. Clinical results are given in the table below:

Hb

14.0 g/dl

WCC

6.7 x 109/1

PLT

190 x 109/1

Na+

140 mmol/l

K+

5.0 mmol/l

Creatinine 130 (jmol/l


ECG

fast atrial fibrillation, lateral ST depression

Which one of the following is the most appropriate medication to control her atrial fibrillation?

Digoxin
O Amiodarone
Flecainide

O Sotalol
O Verapamil

Which one of the following is the most appropriate medication to control her atrial fibrillation?

Digoxin
Amiodarone CORRECT ANSWER

Flecainide

Sotalol
Verapamil

Treatment of atrial fibrillation


Flecainide, although effective at cardioverting atrial fibrillation, is contraindicated in patients with a history of
ischaemic heart disease, as the CAST post infarct trial demonstrated increased mortality in patients treated with
flecainide
Sotalol and verapamil are negatively inotropic and likely to worsen left ventricular failure
Digoxin would be effective at slowing the ventricular rate, but given that the patient was in sinus rhythm 1 month

earlier, it would be a reasonable objective to attain sinus rhythm again


As such, iv loading with amiodarone would appear to be the most appropriate option in this patient

A 78-year-old woman is admitted by ambulance from home. She was found lying on the floor by her home help
after suffering a fall. She has a history of hypertension managed with ramipril 10 mg po daily. On examination her
temperature is 30.0C, and her BP is 100/50 mmHg, with a pulse of 52 bpm. She has a fractured left neck of
femur. Clinical results are given in the table below:

Hb

14.5 g/dl

WCC

4.5

PLT

192 x 109/1

Na+

143 mmol/l

K+

5.3 mmol/l

>=

109/1

Creatinine 195 nmol/l

Which one of the following ECG features is most characteristic of moderate to severe hypothermia?

O Long QT interval
O Short PR interval
Second degree heart block
Complete heart block

O J waves

Which one of the following ECG features is most characteristic of moderate to severe hypothermia?

Long QT interval
Short PR interval
Second degree heart block

Complete heart block


J waves CORRECT ANSWER

Moderate to severe hypothermia


Although varying degrees of heart block may be seen in association with hypothermia, J waves are said to be most
characteristic of moderate to severe hypothermia

J waves are best seen in the left chest leads and are described as a dome or hump in the terminal portion of the
QRS complex

The size of the J wave is correlated with the degree of hypothermia


Slow atrial fibrillation may also be seen as core temperature falls
Death from ventricular arrhythmias is common in moderate to severe hypothermia

A 70-year-old woman presents to the preoperative orthopaedic clinic before hip replacement. She has suffered a
myocardial infarction 4 years earlier and is managed with aspirin 75 mg daily, ramipril 10 mg daily and
atorvastatin 40 mg daily. There is no history of angina, but she is only able to walk around 50 yards due to hip
pain. On examination she looks well, and her BP is 145/80 mmHg with a pulse of 75 bpm. Clinical results are given
in the table below:

Hb

14.0 g/dl

WCC

5.9 >= 109/1

PLT

180 x 109/1

Na+

140 mmol/l

K+

5.0 mmol/l

Creatinine 130 (jmol/l

Which one of the following is the most appropriate investigation to assess her suitability for surgery from
the point of view of her cardiovascular status?
12-lead ECG

q Treadmill stress test


O Echocardiogram

O Dobutamine stress echo


O Cardiac angiography

Which one of the following is the most appropriate investigation to assess her suitability for surgery from
the point of view of her cardiovascular status?
12-lead ECG
Treadmill stress test

Echocardiogram
Dobutamine stress echo CORRECT ANSWER
Cardiac angiography

Dobutamine stress echo


Dobutamine stress echo simulates the effect of exercise on the heart in patients who are unable to undertake a
stress test

Dobutamine is given via iv infusion, and ECG monitoring with echocardiography is undertaken both at rest and at
the point of maximal stimulation

Patients are recommended to discontinue ft-blockade if possible for around 3 days before the procedure, as the
negatively inotropic and chronotropic effects of

-blockade

can blunt the effects of the dobutamine infusion

12 lead ECG will not provide any information on cardiac performance under ischaemic stress
Treadmill exercise test is not recommended because in view of this patient's orthpaedic condition she is highly
unlikely to be able to exercise well enough to generate ischaemia

Cardiac angiography would be an alternative if for some reason Dobutamine stress ECHO was unavailable

A 45-year-old man was diagnosed with new onset atrial fibrillation after visiting his GP complaining

of palpitations. An ECG confirmed atrial fibrillation with a ventricular rate of 85 bpm, and an ECHO
did not reveal any significant structural heart disease. On advice of the hospital he was given low
molecular weight heparin and stabilised on warfarin, with an INR of 2.5. You arrange for him to be
cardioverted a few weeks later, and the procedure is successful. According to current guidelines,
how long is it recommended to continue his warfarin therapy?

For life
O For 1 week

For 72 h
O For 4 weeks

O For 6 months

A 45-year-old man was diagnosed with new onset atrial fibrillation after visiting his GP complaining

of palpitations. An ECG confirmed atrial fibrillation with a ventricular rate of 85 bpm, and an ECHO
did not reveal any significant structural heart disease. On advice of the hospital he was given low

molecular weight heparin and stabilised on warfarin, with an INR of 2.5. You arrange for him to be
cardioverted a few weeks later, and the procedure is successful. According to current guidelines,
how long is it recommended to continue his warfarin therapy?

For life
For 1 week
For 72 h
For 4 weeks CORRECT ANSWER

For 6 months

Warfarinisation
Guidelines published in 2006 recommend warfarinisation for at least 3 weeks pre-cardioversion and for 4 weeks
post-cardioversion
The aim is for a target international normalised ratio (INR) of 2.5, although this can be allowed to drift up to 3 a few
days before the procedure to minimise any risks of cancellation due to inadequate anticoagulation

The period of 4 weeks post procedure is recommended because there is a high relapse rate in the first few weeks

You review a 26-year-old woman who attends the cardiology clinic with her husband. They wish to
start a family, but they have been referred by their GP, as he is worried that the woman has a
history of heart disease. Which one of the following cardiovascular conditions is an absolute

contraindication to pregnancy?

0 Mitral valve prolapse


O Previous repaired patent ductus arteriosus
O Atrial septal defect
O Primary pulmonary hypertension
Bicuspid aortic valve

You review a 26-year-old woman who attends the cardiology clinic with her husband. They wish to
start a family, but they have been referred by their GP, as he is worried that the woman has a
history of heart disease. Which one of the following cardiovascular conditions is an absolute
contraindication to pregnancy?

Mitral valve prolapse


Previous repaired patent ductus arteriosus
Atrial septal defect

Primary pulmonary hypertension CORRECT ANSWER

Bicuspid aortic valve

Pregnancy
Contraindications
Although increased monitoring is recommended in patients with mitral valve prolapse or congenital bicuspid aortic
valve, neither are absolute contraindications to pregnancy
Small atrial septal defects often remain undetected for many years, and previously repaired patent ductus
arteriosis should not impair pregnancy
Primary pulmonary hypertension rapidly worsens in pregnancy, however, and patients are advised not to get
pregnant

A 58-year-old man with multiple dental problems presents to the Emergency Department. Apart
from an abscess on his toe, for which he has been receiving flucloxacillin, he has been relatively

well. On examination he has splinter haemorrhages and looks anaemic. You detect an aortic

systolic murmur. Echocardiogram is suggestive of aortic valve endocarditis, and blood cultures
confirm Streptococcus viridans. In addition to iv benzylpenicillin, which one of the following
antibiotics would you prescribe?

q Ceftriaxone
O Gentamicin
O Azithromycin
O Vancomycin
Ciprofloxacin

A 58-year-old man with multiple dental problems presents to the Emergency Department. Apart

from an abscess on his toe, for which he has been receiving flucloxacillin, he has been relatively
well. On examination he has splinter haemorrhages and looks anaemic. You detect an aortic
systolic murmur. Echocardiogram is suggestive of aortic valve endocarditis, and blood cultures
confirm Streptococcus viridans. In addition to iv benzylpenicillin, which one of the following
antibiotics would you prescribe?
Ceftriaxone

Gentamicin CORRECT ANSWER

Azithromycin
Vancomycin
Ciprofloxacin

Streptococcus viridans endocarditis


The standard regime for suspected viridans endocarditis would be benzylpenicillin iv together with gentamicin 1
mg/kg/day

Ceftriaxone is an alternative in stable patients and has the advantage of being given once per day

In patients who are penicillin allergic, vancomycin is a viable alternative


The most likely route for his infection is via dental infection
As long as Streptococcus viridans endocarditis is appropriately managed, cure rates approach 98%

A 23-year-old woman presents to the GP complaining of palpitations. She says these are rapid and when she gets
them she feels lightheaded and sick. They tend to come on without warning, but have occurred when she has
been out dancing with friends, and after a game of squash, and on one occasion at rest. On examination she
looks well; her BMI is 21, pulse 70 bpm regular, BP 122/70 mmHg. Blood test results are shown in the table below:

Hb

13.1 g/dl

WCC

5.4 x 109/1

PLT

251 x 109/1

Na+

139 mmol/l

K+

4.0 mmol/l

Creatinine 75 jjmol/l

Which one of the following investigations is most likely to help with the diagnosis?

O Tilt table test


O Continuous loop recorder
24 h Holter monitor
O 3 day Holter monitor

12-lead ECG

Which one of the following investigations is most likely to help with the diagnosis?

Tilt table test


Continuous loop recorder CORRECT ANSWER

24 h Holter monitor

3 day Holter monitor

12-lead ECG

Continuous loop recorder


Although a 12-lead ECG may reveal evidence of structural heart disease leading to changes in the resting ECG, or
an aberrant pathway such as that found in Wolf-Parkinson-White syndrome, more often than not it will be normal

A Holter monitor may also be in place during a period where no palpitations occur
By contrast, a continuous loop recorder can be activated by the patient during symptoms and therefore carries the
greatest chance of recording the arrhythmia

You review a 28-year-old woman with palpitations. On examination you suspect that there is
splitting of the first heart sound. Her BP is 123/80 mmHg, pulse 70 bpm regular, and her chest is
clear. There are no other cardiovascular findings. You arrange a 12-lead ECG. Which one of the

following parts of the ECG is most closely associated with the first heart sound?

O P wave

T wave
O S wave

O R wave
U wave

You review a 28-year-old woman with palpitations. On examination you suspect that there is
splitting of the first heart sound. Her BP is 123/80 mmHg, pulse 70 bpm regular, and her chest is

clear. There are no other cardiovascular findings. You arrange a 12-lead ECG. Which one of the
following parts of the ECG is most closely associated with the first heart sound?

P wave
T wave
S wave
R wave

U wave

CORRECT ANSWER

R wave
The first heart sound occurs most closely in association with the R wave
Splitting of the first heart sound may occur in atrioventricular septal defects (AVSDs) or in conditions such as
Ebstein anomaly
In patients with AVSD there is usually left axis deviation, prominent P waves and prolongation of the P-R interval
Echocardiography and cardiac catheterisation follow to determine the extent of the defect

A 58-year-old man with a history of hypertension, managed with ramipril 10 mg daily, and 40 pack years of
cigarette smoking presents to the Emergency Department after a collapse at work. Neurological examination
reveals a left-sided hemiplegia. Clinical results are given in the table below:
Hb

13.8 g/dl

WCC

5.4 x 109/1

PLT

192 x 109/1

Na+

139 mmol/l

K+

4.9 mmol/l

Creatinine 149 |jmol/l

CT head

no intracerebral haemorrhage identified

What is the time limit after the onset of symptoms up to which IV thrombolysis should be administered?

0 1h
O 3h
O 4.5 h

O 6h
O 12 h

What is the time limit after the onset of symptoms up to which IV thrombolysis should be administered?
1h

3h
4.5 h CORRECT ANSWER
6h

12 h

Thrombolysis guidelines
Although 3 h was the initial time limit recommended in 2004 guidelines from the RCP, the newer guidelines have
proposed a limit of 4.5 h
This takes into account all currently available data on the risk-benefit ratio of treatment within this timeframe
Despite an increase in haemorrhagic transformation of their stroke, the group receiving thrombolysis in the NINDs
study had a lower rate of death or severe disability
Both the SIGN and RCP guidelines recommend that thrombolysis for stroke should only be undertaken in a
specialist stroke thrombolysis unit

A 62-year-old man with two previous myocardial infarctions and a history of left ventricular failure controlled with
ramipril and furosemide presents to his GP with palpitations. On examination his BP is 100/72 mmHg, pulse 95

bpm atrial fibrillation (AF), with bibasal crackles consistent with heart failure. Clinical results are given in the table
below:
Hb

12.1 g/dl

WCC

5.4 x 109/1

PLT

234 x 109/1

Na+

140 mmol/l

K+

5.0 mmol/l

Creatinine 130 Mmol/l

Echo

dilated left atrium and left ventricle

Which one of the following would be the most appropriate agent to control his AF?
Diltiazem

O Sotalol
O Amiodarone
O Digoxin
O Verapamil

Which one of the following would be the most appropriate agent to control his AF?
Diltiazem
Sotalol

Amiodarone

Digoxin CORRECT ANSWER


Verapamil

Control of atrial fibrillation


Verapamil, diltiazem and sotalol are all to a greater or lesser extent negatively inotropic and may worsen cardiac
failure

Given that the patient's systolic blood pressure is only 100, any further reduction in cardiac output is likely to
further worsen his BP

Amiodarone is useful for chemical cardioversion and as such is not the best choice here
Digoxin is less useful for rate control in atrial fibrillation (AF) than calcium antagonists or ft-blockers, but is the most
appropriate choice here as it does improve symptoms in patients with cardiac failure and, given the enlarged left
atrium, successful cardioversion is unlikely

A 54-year-old woman with a history of atrial fibrillation presents with left face and arm weakness consistent with a
stroke. On examination her BP is 162/82 mmHg, with a pulse of 85 bpm, irregular. Clinical results are given in the

table below:

Hb

12.1 g/dl

WCC

5.4

PLT

175 x 109/1

Na+

140 mmol/l

K+

5.0 mmol/l

109/1

Creatinine 105 (jmol/l


CT head

no evidence of intracerebral haemorrhage

Six hours post stroke you are considering anticoagulation or antiplatelet therapy. Which one of the
following would be most appropriate?

O Full iv heparinisation
Low molecular weight heparin and commence warfarin treatment
Q Alteplase
Aspirin

O Streptokinase

Six hours post stroke you are considering anticoagulation or antiplatelet therapy. Which one of the
following would be most appropriate?

Full iv heparinisation
Low molecular weight heparin and commence warfarin treatment
Alteplase
Aspirin CORRECT ANSWER

Streptokinase

Stroke
This patient is outside the window of the NINDS study within which thrombolysis is recommended
Meta-analysis suggested strong evidence of benefit up to 3 h, and guidelines now support use of thrombolysis up
to 4.5 h after the onset of symptoms

Additionally, early anticoagulation with heparin has been shown to increase the risk of intracerebral haemorrhage,
without having a significant impact on the risk of long-term disability or death
As such, commencement of aspirin is the most appropriate option, with anticoagulation at a later stage
A (somewhat arbitrary) delay of 2 weeks after acute stroke is recommended before starting warfarin for AF, to
minimise the risk of haemorrhagic complications

A 30-year-old man is being investigated for hypertension. A combination of BPs estimated by colour flow Doppler
and measured values are listed in the table below:

LV

200/10 mmHg

Ascending aorta

200/70 mmHg

Right arm

190/70 mmHg

Right femoral artery 110/70 mmHg

Which one of the following is the most likely diagnosis?


Coarctation of aorta

O Left subclavian artery stenosis


O Aortic regurgitation
O Aortic stenosis
HOCM

A 30-year-old man is being investigated for hypertension. A combination of BPs estimated by colour flow Doppler
and measured values are listed in the table below:

LV

200/10 mmHg

Ascending aorta

200/70 mmHg

Right arm

190/70 mmHg

Right femoral artery 110/70 mmHg

Which one of the following is the most likely diagnosis?


Coarctation of aorta CORRECT ANSWER
Left subclavian artery stenosis
Aortic regurgitation
Aortic stenosis

HOCM

Coarctation of aorta
This picture would be typical of coarctation distal to the origin of the right brachiocephalic artery
Cardiac catheterisation is the next most appropriate step in further characterising the pressure gradient across the
coarctation, MRI scanning
Prognosis after coarctation is dependent on speed of diagnosis
In those diagnosed after the age of 35 years, survival to age 50 is only 20%, whereas diagnosis and treatment as
a child is associated with a survival above 90%

A 53-year-old woman who has had chemotherapy for metastatic breast cancer 6 months earlier comes to the
clinic complaining of shortness of breath on exertion. Her BP is 125/78 mmHg, her pulse is 94 bpm and her apex
beat is displaced to the anterior axillary line. Clinical results are given in the table below:

Hb

11.9 g/dl

WCC

5.0

PLT

190 x 109/1

Na+

140 mmol/l

K+

4.5 mmol/l

109/1

Creatinine 160 (jmol/l


CXR

cardiomegaly, increased shadowing consistent with mild pulmonary oedema at both bases

Which one of the following chemotherapeutic agents is most likely to be responsible for this patient's
symptoms?

O Doxorubicin
O Docetaxel

O Cisplatin
Bleomycin
Q Carbiplatin

Which one of the following chemotherapeutic agents is most likely to be responsible for this patient's
symptoms?
Doxorubicin CORRECT ANSWER

Docetaxel
Cisplatin

Bleomycin
Carbiplatin

Doxorubicin
Doxorubicin can be associated with cardiac failure, reduced left ventricular ejection fraction and tachyarrhythmias

Other notes
Docetaxel may also be associated with arrhythmias and cardiac failure, although the likelihood is less than with
doxorubicin

Platinum-based chemotherapies are associated with nerve damage


Bleomycin is associated with interstitial pneumonitis

A 24-year-old man from a travelling family who has shunned regular medical follow-up comes to the clinic

complaining of shortness of breath and chest pain. On examination his BP is 145/82 mmHg, he is cyanosed and

short of breath at rest.

You review his catheterisation results, given in the table below:


Pressure RV

110/0 mmHg

Pressure LV

90/0 mmHg

LV oxygen saturation 88%

Given the probable clinical diagnosis, which one of the following is the most likely finding on clinical
examination?
O A diastolic murmur

G Persistent hypoxia despite maximal oxygen therapy

O Tapping apex beat


O Broad P waves on ECG

Decreased pulmonary vasculature on CXR

A 24-year-old man from a travelling family who has shunned regular medical follow-up comes to the clinic

complaining of shortness of breath and chest pain. On examination his BP is 145/82 mmHg, he is cyanosed and
short of breath at rest.

You review his catheterisation results, given in the table below:


Pressure RV

110/0 mmHg

Pressure LV

90/0 mmHg

LV oxygen saturation 88%

Given the probable clinical diagnosis, which one of the following is the most likely finding on clinical
examination?

A diastolic murmur
Persistent hypoxia despite maximal oxygen therapy CORRECT ANSWER

Tapping apex beat


Broad P waves on ECG
Decreased pulmonary vasculature on CXR

Persistent hypoxia
This man has a right to left shunt, with right ventricular pressure greater than the left
There is clear mixing of deoxygenated and oxygenated blood within the left ventricle, as evidenced by the left
ventricular oxygen saturation of 88%
The most likely diagnosis is a long-standing ventricular septal defect (VSD)
The typical murmur seen is a holosystolic murmur, and the apex beat is usually displaced

P waves are tall on the ECG, a sign typical of that seen with right atrial overload due to pulmonary hypertension
Because of pulmonary hypertension, increased pulmonary vasculature markings are normally seen on the chest Xray (CXR)

A 42-year-old patient who has a history of paroxysmal atrial fibrillation (AF) has been treated with warfarin. The AF
has now resolved after successful DC cardioversion. Clinical results are given in the table below:

Hb

13.1 g/dl

WCC

4.9

PLT

294 x 109/1

Na+

139 mmol/l

K+

4.8 mmol/l

109/1

Creatinine 100 jjmol/l


TSH

2.1 U/l

Echo

normal sized left atrium, no significant valvular disease

For how long should the warfarin be continued?

O 4 weeks
O 6 months
O 1 year

O 3 years
Stop with immediate effect

For how long should the warfarin be continued?


4 weeks CORRECT ANSWER

6 months
1 year

3 years
Stop with immediate effect

Warfarin and atrial fibrillation


This patient has benign findings on echocardiography, with normal left atrial size
It is therefore likely that he has a reasonable chance of remaining in sinus rhythm
NICE guidelines on the management of atrial fibrillation, most recently reissued in 2006, recommend continuing
warfarin therapy for a minimum of 4 weeks (see
http://www.nice.org.uk/Guidance/CG36/QuickRefGuide/pdf/English)

Where the risk of recurrence is high, or there are multiple failed cardioversions, long-term warfarin therapy is
advised

A 32-year-old man presents to the clinic with shortness of breath, which is particularly bad when he goes jogging.
He has recently increased his exercise to try and reduce his weight. On a couple of occasions he has also
noticed some chest discomfort, which has caused him to stop exercising. On examination his BP is 150/88
mmHg, and he has a double apical impulse. On auscultation there is a harsh midsystolic murmur, which is
loudest between the apex and the left sternal border. Clinical results are given in the table below:
Hb

13.0 g/dl

WCC

4.8

PLT

199 x 109/1

Na+

140 mmol/l

K+

5.0 mmol/l

109/1

Creatinine 100 Mmol/l


ECG

LVH and widespread Q waves

Which one of the following is most directly correlated with increased risk of sudden death?
Increased left ventricular outflow tract gradient

O Presence of mitral regurgitation


O Degree of septal hypertrophy
O Presence of atrial fibrillation
O Systolic anterior motion

Which one of the following is most directly correlated with increased risk of sudden death?
Increased left ventricular outflow tract gradient

Presence of mitral regurgitation


Degree of septal hypertrophy CORRECT ANSWER

Presence of atrial fibrillation


Systolic anterior motion

Sudden cardiac death


A number of studies have attempted to examine potential correlations between clinical features of hypertrophic
obstructive cardiomyopathy (HOCM) and increased risk of sudden cardiac death
J Am Coll Cardiol, 2003; 41:994-996, doi: 10.1016/S0735-1097(02)03003-6 is a good review of the relevant data
Of the options given, degree of ventricular septal hypertrophy appears to be most strongly linked to increased risk
of sudden death

Other publications also point to LVH per se as being a risk factor

Other notes
Patients with HOCM usually die from arrhythmias, and previous ventricular tachycardia is thus strongly predictive
of the risk of sudden death

Increased outflow tract gradient appears to be related to symptoms such as shortness of breath and angina
Myomectomy, taking tissue from the interventricular septum, appears to improve these symptoms

A 42-year-old man with the features of congenital myotonic dystrophy comes to see you for review

He has suffered from mild intellectual impairment, frontal balding typical of the disease and

increasing muscle weakness with increased muscle tone over the past few years. Most recently he
has suffered from a number of episodes of syncope. On examination his BP is 129/70 mmHg and
his pulse 55 bpm; there are no other significant findings on cardiovascular examination. Which of
the following ECG findings might you most commonly expect to see in this case?

O Short PR interval
PR prolongation
O Long QT syndrome

Bifasicular block

O Left bundle branch block

A 42-year-old man with the features of congenital myotonic dystrophy comes to see you for review

He has suffered from mild intellectual impairment, frontal balding typical of the disease and
increasing muscle weakness with increased muscle tone over the past few years. Most recently he
has suffered from a number of episodes of syncope. On examination his BP is 129/70 mmHg and
his pulse 55 bpm; there are no other significant findings on cardiovascular examination. Which of
the following ECG findings might you most commonly expect to see in this case?
Short PR interval
PR prolongation CORRECT ANSWER

Long QT syndrome
Bifasicular block
Left bundle branch block

Congenital myotonic dystrophy


PR prolongation is the commonest feature seen in association with congenital myotonic dystrophy
Varying conduction defects may be seen, including:
right bundle branch block
left bundle branch block
bifasicular block
The syncopes seen here may be related to periods of complete heart block, and a 72 h Holter monitor would be the
next most logical investigation in an attempt to capture these

The other possibility would be short runs of ventricular tachycardia (VT), as myotonic dystrophy also increases the
risk of paroxysms of VT (again, it would be hoped that these would be captured on a Holter monitor)

A 50-year-old man with no previous cardiovascular history comes to the Emergency Department after referral
from his GP. He attended the GP surgery with palpitations, which were extremely rapid and irregular. It is now
0900 h and he tells you the palpitations began the previous morning after a heavy drinking session with a friend
from work. There is no history of smoking, cardiovascular disease or previous myocardial infarction; he plays
squash twice per week and cycles to work. On examination his BP is 125/77 mmHg and his pulse is 140 bpm,
irregular. He is not in cardiac failure. Clinical results are given in the table below:

Hb

13.1 g/dl

WCC

4.9

PLT

210

Na+

139 mmol/l

K+

4.7 mmol/l

x
x

109/1
109/1

Creatinine 120 Mmol/l


CXR

no cardiomegaly, no LVF

ECG

fast atrial fibrillation, no Q waves

Which of the following is the most appropriate therapy to chemically cardiovert him?
Adenosine

O Bisoprolol
O Digoxin
O Flecainide
O Verapamil

Which of the following is the most appropriate therapy to chemically cardiovert him?
Adenosine
Bisoprolol
Digoxin

Flecainide CORRECT ANSWER


Verapamil

Flecainide
In the UK the drugs most commonly used for cardioversion of atrial fibrillation are flecainide and amiodarone
Successful cardioversion is reported in up to 90% of patients given iv flecainide
Flecainide should be avoided in patients with a previous ischaemic cardiovascular history, as the CAST trial
suggested that mortality was increased in patients given flecanide post myocardial infarction
http://www.nejm.Org/doi/full/10.1056/NEJM199103213241201

Other notes
Digoxin does not cardiovert, and bisoprolol and verapamil are more usually employed to maintain sinus rhythm after
successful electrical cardioversion

As we have a very clear history of duration of atrial fibrillation and no evidence of structural heart disease,
successful cardioversion is likely to be possible

An 18-year-old man comes to the Emergency Department because he has suffered a severe syncopal attack
while playing a game of squash. His opponent tells you that he collapsed and took a few minutes to recover.
Apparently this was the second episode, the first having occurred after a strenuous period of exercise at the
swimming pool. Of note is the fact that his father died of a cardiac arrest at the age of 32. On examination he
looks fit, his BP is 132/78 mmHg and his pulse is 70 bpm, sinus rhythm. Clinical results are given in the table
below:
Hb

12.8 g/dl

WCC

5.0

PLT

182 x 109/1

Na+

139 mmol/l

K+

4.8 mmol/l

109/1

Creatinine 120 pmol/l


ECG

sinus rhythm but QT interval 0.51 s

24h tape

paroxysmal AF on 2 occasions

Which one of the following agents should be given for rhythm control in this case?
Adenosine

O Flecainide
Verapamil

O Amiodarone
O Metoprolol

Which one of the following agents should be given for rhythm control in this case?
Adenosine
Flecainide
Verapamil

Amiodarone
Metoprolol CORRECT ANSWER

Long QT syndrome
Beta-blockers
Beta-blocking agents are the drugs of choice for rhythm control in long QT syndrome
They decrease conduction through the atrioventricular (AV) node and have negatively inotropic and chronotropic
effects

Other options
If patients continue to have rhythm disturbances on f$-blockade, cervical sympathectomy is one treatment option,
or they may be referred for implantable cardioverter defibrillator (ICD)

Lifestyle changes, such as avoiding competitive or particularly intensive sporting activity, may also be
recommended

Other agents, particularly amiodarone, lengthen the cardiac action potential and may increase the risk of torsades
de pointes ventricular tachycardia (VT)

An 18-year-old student is admitted to the Emergency Department after a collapse in a night club. He has no
recollection of the incident, was assisted by his friends and had begun to regain consciousness by the time the
ambulance had arrived. On direct questioning in the Emergency Department he admits to two previous syncopal
episodes. He denies elicit drug use. On examination his BP is 123/72 mmHg, and his pulse is 72 regular. Clinical
results are given in the table below:

Hb

13.2 g/dl

WCC

5.3 x 109/1

PLT

199 x 109/1

Na

142 mmol/l

4.6 mmol/l

Creatinine 90 pmol/l
ECG

sinus rhythm, QT interval 0.52 s

A defect in which one of the following ion channels is the most likely cause of his symptoms?

O Magnesium
Sodium

Potassium
Chloride
Calcium

A defect in which one of the following ion channels is the most likely cause of his symptoms?

Magnesium

Sodium
Potassium CORRECT ANSWER

Chloride
Calcium

Ion channel defects


LQT 1, 2 and 3 mutations account for 45, 45 and 7% of cases of long QT syndrome, respectively
Both LQT1 and 2 mutations are associated with defective potassium transport, leading to a decrease in potassium
outflow and more prolonged depolarisation
LQT8 is associated with defective calcium channel transport; often patients also have associated congenital heart
disease and behavioural disorders

LQT3 mutation is associated with a gain of function mutation in sodium channels

A 72-year-old man who visited his GP suffering from an infection 2 days earlier is admitted to the Emergency
Department after suffering a collapse at the supermarket. His wife tells you that he suffered from a myocardial
infarction some 6 years ago, but has otherwise been relatively well, taking aspirin, ramipril and atorvastatin as
regular medications. She doesn't know the type of antibiotics he has been taking. On examination his BP is
120/71 mmHg and he is drowsy. His pulse is 70 bpm and regular. Clinical results are given in the table below:

Hb

13.1 g/dl

WCC

5.1

PLT

232

Na+

140 mmol/l

K+

4.2 mmol/l

Creatinine

123 fjmol/l

Short runs of torsades seen on the monitor

x
x

109/1
109/1

Which one of the following is the most likely causative antibiotic?

O Oxytetracycline
Metronidazole

O Co-amoxiclav
Cefalexin

O Clarithromycin

Which one of the following is the most likely causative antibiotic?


Oxytetracycline
Metronidazole

Co-amoxiclav
Cefalexin
Clarithromycin CORRECT ANSWER

Antibiotics
Both macrolides such as erythromycin, and clarithromycin, and quinolones such as ciprofloxacin and olfloxacin may
lead to QT prolongation, which pre-disposes to the development of torsades des pointes VT
The problem may be exacerbated by co-administration with CYP-P450 inhibitors such as ketoconazole
Initial management involves withdrawal of the potential offending agent and electrolyte assay to exclude potential
exacerbating factors such as hypomagnesaemia

A 55-year-old man with a history of mitral regurgitation and atrial fibrillation is warfarinised. His INR

is therapeutic at 2.0. He needs to undergo pre-planned tooth extraction under local anaesthesia.

How would you manage him before the procedure?


Stop warfarin for 2 days

O Stop warfarin, start LMWH


Stop warfarin, start unfractionated heparin

O Stop warfarin start aspirin


O Maintain warfarin at the therapeutic dose

A 55-year-old man with a history of mitral regurgitation and atrial fibrillation is warfarinised. His INR

is therapeutic at 2.0. He needs to undergo pre-planned tooth extraction under local anaesthesia.

How would you manage him before the procedure?


Stop warfarin for 2 days
Stop warfarin, start LMWH
Stop warfarin, start unfractionated heparin
Stop warfarin start aspirin

Maintain warfarin at the therapeutic dose CORRECT ANSWER

Surgical treatment during warfarin therapy


For patients on short-term warfarin treatment it is recommended that patients wait for treatment until after they
have discontinued their period of warfarin therapy
For those on long-term warfarin therapy, British Haematological Society guidelines suggest that as long as the
international normalised ratio (INR) is not above 2.0, the procedure may take place in the standard way
UK Medicines Information (UKMI) recommendations are more relaxed, suggesting that dental procedures may take

place as long as the INR is less than 4.0

A 67-year-old man attends the cardiology clinic. He has been suffering some angina-type chest

pain on going out in the cold air and is worried that he might have coronary artery disease. There
is a past medical history of smoking 20 cigarettes per day and hypertension which is managed with
ramipril 10 mg daily. His GP has sent an ECG which appears to show that he is in left bundlebranch block. Which one of the following would you expect to hear on auscultation?

O Loud first heart sound, reversed splitting of the second heart sound
O Soft first heart sound, fixed splitting of the second heart sound
O Soft first heart sound, reversed splitting of the second heart sound
O Soft first heart sound, normal second heart sound
O Loud first heart sound, normal second heart sound

A 67-year-old man attends the cardiology clinic. He has been suffering some angina-type chest

pain on going out in the cold air and is worried that he might have coronary artery disease. There

is a past medical history of smoking 20 cigarettes per day and hypertension which is managed with
ramipril 10 mg daily. His GP has sent an ECG which appears to show that he is in left bundlebranch block. Which one of the following would you expect to hear on auscultation?

Loud first heart sound, reversed splitting of the second heart sound
Soft first heart sound, fixed splitting of the second heart sound
Soft first heart sound,reversed splitting of the second heart sound CORRECT ANSWER
Soft first heart sound, normal second heart sound

Loud first heart sound, normal second heart sound

Auscultation findings of left bundle-branch block


Left bundle-branch block results in the left ventricle depolarising from cell-to-cell conduction via the right ventricle,
rather than via the normal pathway
This results in reversed splitting of the second heart sound, in other words split in expiration and single in
inspiration
Additionally the first heart sound tends to be softer than usual
Left bundle-branch block in this age group is most likely to be ischaemic in origin, hence his chest pain almost
certainly warrants further investigation, especially given his history of smoking and hypertension

You are working in the chemical pathology laboratory and receive a sample request for analysis of
BNP. You don't have any clinical details on the form apart from 'chest pain'. You plan to ring the
SHO who requested the test for further details. In which one of the following situations is BNP
most likely to be normal?

O Unstable angina
O Constrictive pericarditis
Pulmonary embolus

Q Acute myocardial infarction

Acute mitral valve rupture

You are working in the chemical pathology laboratory and receive a sample request for analysis of
BNP. You don't have any clinical details on the form apart from 'chest pain'. You plan to ring the
SHO who requested the test for further details. In which one of the following situations is BNP
most likely to be normal?
Unstable angina CORRECT ANSWER

Constrictive pericarditis

Pulmonary embolus
Acute myocardial infarction
Acute mitral valve rupture

B-type natriuretic peptide


B-type natriuretic peptide (BNP) is secreted in response to raised intra-cardiac pressures, primarily owing to
volume distension, and leads to increased sodium excretion and decreased systemic vascular resistance
Both acute myocardial infarction and acute mitral valve rupture may result in volume distension, leading to elevated
levels of BNP
Constrictive pericarditis may also lead to raised intracavity pressures and hence elevated BNP
Large pulmonary embolus produces raised right-sided cardiac pressures and thus may lead to elevated BNP

Unstable angina does not usually lead to altered levels of BNP

A 62-year-old woman suddenly deteriorates 2 days after receiving tPA for an acute myocardial

infarction. She complained of severe shortness of breath during the course of the afternoon and
when the nurses examined her, her O2 saturation was only 91% on oxygen delivered via a non-re-

breather. On examination her BP is 105/70 mmHg, with a pulse of 105 bpm regular. She has an
apical systolic murmur and marked left ventricular failure. Which one of the following is the most
likely cause?

O Acute VSD
Acute ASD

O Pericardial tamponade
O LV wall rupture

O Papillary muscle rupture

A 62-year-old woman suddenly deteriorates 2 days after receiving tPA for an acute myocardial

infarction. She complained of severe shortness of breath during the course of the afternoon and
when the nurses examined her, her O2 saturation was only 91% on oxygen delivered via a non-re-

breather. On examination her BP is 105/70 mmHg, with a pulse of 105 bpm regular. She has an
apical systolic murmur and marked left ventricular failure. Which one of the following is the most
likely cause?
Acute VSD

Acute ASD
Pericardial tamponade

LV wall rupture
Papillary muscle rupture CORRECT ANSWER

Papillary muscle rupture


This patient has papillary muscle rupture
Out of the possible muscles which may rupture, the posteromedial papillary muscle is twice as likely to rupture as
the anterolateral one because the posteromedial papillary muscle is supplied by the right coronary artery only,
whereas the anterolateral papillary muscle receives supply from both the left anterior descending and left
circumflex arteries

The murmur is consistent with mitral regurgitation which in turn leads to acute left ventricular failure

Management
Management centres on decreasing afterload in an attempt to stabilise patients before undergoing valvular surgery
Sodium nitroprusside is the usual therapy of choice where blood pressure allows, as this can be closely titrated to
systolic BP

A 70-year-old man presents with severe tearing back and chest pain which came on very suddenly.

He has a past medical history of hypertension for which he takes ramipril 10 mg daily, amlodipine 5
mg, and he smokes 30 cigarettes per day. On examination he is in severe pain, his BP is 155/85
mmHg, he has bilateral upgoing plantars and 4/5 weakness affecting left-ankle dorsiflexion. He
appears to have a pericardial rub. Which one of the following features is most suggestive of
dissecting aortic aneurysm?

O The pattern of pain described


O Hypertension
O Bilateral upgoing plantars
O Left lower limb signs

O Pericardial rub

A 70-year-old man presents with severe tearing back and chest pain which came on very suddenly.

He has a past medical history of hypertension for which he takes ramipril 10 mg daily, amlodipine 5
mg, and he smokes 30 cigarettes per day. On examination he is in severe pain, his BP is 155/85
mmHg, he has bilateral upgoing plantars and 4/5 weakness affecting left-ankle dorsiflexion. He
appears to have a pericardial rub. Which one of the following features is most suggestive of
dissecting aortic aneurysm?
The pattern of pain described CORRECT ANSWER

Hypertension
Bilateral upgoing plantars
Left lower limb signs

Pericardial rub

Diagnosing dissecting aortic aneurysm


The acute onset of severe tearing back and chest pain is very typical of dissecting aortic aneurysm
It is impossible to tell whether his neurological signs seen are new or old, and a pericardial rub or hypertension are
more likely to be associated with other causes of chest pain

Painless aortic dissection only occurs in around 10% of patients, and is more common in patients who have
connective tissue disorders such as Marfan syndrome

Other notes
Upper limb neurological signs are more likely to be associated with thoracic aortic dissection, lower limb signs may
be commoner in anterior spinal artery dissection or thrombosis

A 72-year-old man was admitted with an acute anterior myocardial infarction. He has chronic renal impairment,
with a recent creatinine recorded at 148 pmol/litre. Medication included ramipril, atorvastatin and indapamide for
the treatment of hypertension. He was taken straight to the angiography suite where he received stenting of a left
main-stem stenosis. You are asked to see him after about 30 hours as the nurses feel he is deteriorating. On
examination his BP is 149/84 mmHg, his pulse is 75 bpm and regular. His legs look dusky in colour, particularly
his right big toe which looks blue in colour. He has splinter haemorrhages affecting toenails on both feet. There is
a loud left femoral bruit. The table below contains the investigation results.

Hb

13.2 g/dl

WCC

5.0

PLT

190 x 109/litre

Na+

141 mmol/litre

K+

5.9 mmol/litre

109/litre

Creatinine 630 pmol/litre

Urine

blood ++, protein +

Which one of the following is the most likely diagnosis?

O Renal vein thrombosis

q Acute tubular necrosis


O Renal artery stenosis
O Cholesterol embolism
O Femoral artery embolism

Which one of the following is the most likely diagnosis?


Renal vein thrombosis
Acute tubular necrosis
Renal artery stenosis

Cholesterol embolism CORRECT ANSWER

Femoral artery embolism

Cholesterol embolism
Risk factors for cholesterol embolism after coronary artery instrumentation include increased age (> 60 years),
hypertension, cerebral vascular disease and aorto-iliac arterial disease

Management
Further vascular procedures, anti-coagulant and thrombolytic therapies are not of value in the management of the
condition

Patients should be dialysed during the acute period as they may recover a limited amount of renal function

Prognosis
Unfortunately the prognosis of cholesterol embolism is very poor: where multiple organs are involved mortality may
approach 90% at 3 months

A 21-year-old woman presents to the clinic with symptoms of increased shortness of breath and decreased
exercise tolerance. She used to be a keen hockey player when at school but is now virtually unable to even walk
to the bus stop without becoming short of breath. On examination she looks tired and slightly short of breath at
rest. Her BP is elevated at 145/92 mmHg. Echocardiogram showed increased right atrial size and elevated right
arterial pressure by Doppler. Cardiac catheterization results were as follows:

O2 saturation SVC 74%


O2 saturation RA 82%
O2 saturation RV 82%
O2 saturation LA 91%
O2 saturation LV 91%
Which one of the following is the most likely diagnosis?

O Ostium primum atrial septal defect


O Secundum atrial septal defect
Patent ductus arteriosus

O Pulmonary stenosis
O Tricuspid regurgitation

A 21-year-old woman presents to the clinic with symptoms of increased shortness of breath and decreased
exercise tolerance. She used to be a keen hockey player when at school but is now virtually unable to even walk
to the bus stop without becoming short of breath. On examination she looks tired and slightly short of breath at
rest. Her BP is elevated at 145/92 mmHg. Echocardiogram showed increased right atrial size and elevated right
arterial pressure by Doppler. Cardiac catheterization results were as follows:

O2 saturation SVC 74%


O2 saturation RA 82%
O2 saturation RV 82%
O2 saturation LA 91%
O2 saturation LV 91%
Which one of the following is the most likely diagnosis?
Ostium primum atrial septal defect

Secundum atrial septal defect CORRECT ANSWER

Patent ductus arteriosus

Pulmonary stenosis

Tricuspid regurgitation

Secundum atrial septal defect


Secundum defects may often be diagnosed in patients entering early adulthood
The difference in saturations between the SVC and the right ventricle indicates that there is a left to right shunt of
oxygenated blood

Patients with secundum defects tend to be slim and to not suffer from cyanosis
Therapy of choice is via delivery of a catheter device to close the defect, or surgical closure if the defect is
particularly large

A 52-year-old man is admitted to the intensive therapy unit with left ventricular failure post-

myocardial infarction. Despite prompt activity including angioplasty within a few minutes of the

onset of chest pain, his systolic BP on admission to the unit was only 80 mmHg, with a pulse of
105 bpm. Auscultation of the chest revealed crackles up to the mid zones on both sides consistent
with cardiac failure. The team decide to insert an intra-aortic balloon pump timed to coincide with
the dicrotic notch. Which one of the following does the dicrotic notch refer to?

O Aortic valve opening


O Aortic valve closure

q Mitral valve opening


O Mitral valve closure
O Pulmonary valve closure

A 52-year-old man is admitted to the intensive therapy unit with left ventricular failure post-

myocardial infarction. Despite prompt activity including angioplasty within a few minutes of the
onset of chest pain, his systolic BP on admission to the unit was only 80 mmHg, with a pulse of
105 bpm. Auscultation of the chest revealed crackles up to the mid zones on both sides consistent
with cardiac failure. The team decide to insert an intra-aortic balloon pump timed to coincide with
the dicrotic notch. Which one of the following does the dicrotic notch refer to?
Aortic valve opening
Aortic valve closure CORRECT ANSWER

Mitral valve opening


Mitral valve closure

Pulmonary valve closure

Effect of intra-aortic balloon pump


The dicrotic notch refers to a secondary upstroke in the downward part of the pulse wave, which corresponds with
closure of the aortic valve

Intra-aortic balloon pumps deflate during systole which then increases forward blood flow because of the reduction
in afterload
The pump then re-inflates during diastole, increasing the blood flow to coronary arteries

A 72-year-old woman is admitted for assessment after two episodes of collapse over the past few months. She
has been managed by her GP for many years for hypertension and is currently treated with indapamide and
amlodipine. On examination she looks a little thin. Her BP is 175/125 mmHg. Auscultation of the chest reveals an
ejection systolic murmur. She has minor crackles at both lung bases. The table below contains the investigation
results.

Hb

13.0 g/dl

WCC

5.3

PLT

194 x 109/litre

Na+

140 mmol/litre

K+

4.2 mmol/litre

Creatinine

145 nmol/litre

109/litre

Cardiac catheterisation gradient of 50 mmHg across the valve

Which one of the following would most influence your decision to refer this patient for aortic valve
replacement?
Gradient of 50 mmHg
O Presence of left ventricular hypertrophy

Presence of symptoms

O Presence or absence of valvular calcification


O Her relatively young age

Which one of the following would most influence your decision to refer this patient for aortic valve
replacement?
Gradient of 50 mmHg

Presence of left ventricular hypertrophy


Presence of symptoms CORRECT ANSWER

Presence or absence of valvular calcification


Her relatively young age

Valve replacement
While in gradient terms her valvular stenosis is on the cusp of the severe category, elective valve replacement is
generally not recommended in the absence of symptoms
If there is co-existent coronary artery disease, then elective valve replacement may be undertaken at the same
time as coronary artery bypass graft surgery

Elevated age is not a barrier to aortic valve replacement, functional status prior to surgery is much more important
In selected patients who are unfit to undergo surgery, balloon valvuloplasty may provide symptomatic relief for a
period of 6-12 months

A 73-year-old woman is admitted for pacemaker insertion because of a number of syncopes and

periods of complete heart block identified on 72 hour ECG. She receives a DDDR pacemaker. What
does the R stand for?

O Rate limiting
Rate modulated

O Repolarising

O Rate enhancing
O Rate reducing

A 73-year-old woman is admitted for pacemaker insertion because of a number of syncopes and

periods of complete heart block identified on 72 hour ECG. She receives a DDDR pacemaker. What

does the R stand for?

Rate limiting
Rate modulated CORRECT ANSWER

Repolarising

Rate enhancing
Rate reducing

DDDR pacemakers
DDDR stands for dual-chamber paced, dual-chamber sensed, dual response, rate-modulated device
In other words, the activity of the pacemaker is varied according to the background heart rate
Dual-chamber pacing devices are less likely than ventricular-pacing-only devices to lead to pacemaker syndrome
in the context of atrial fibrillation, which are associated with increased risk of atrioventricular dysyncrony

A 32-year-old woman is admitted in an unconscious state after an overdose of a large number of amitriptyline
tablets. It is thought that she took them between 7 and 8 pm and was not found by her partner until he returned

from a bar some 3 hours later. When you see her she has already been intubated by the Emergency Department

consultant. Her BP is 100/70 mmHg and she has a sinus tachycardia of 100 bpm. While you are watching the
monitor you can see she is suffering from short unsustained runs of ventricular tachycardia. The table below
contains the investigation results.

pH

7.29

P02

8.1 kPa

pco2

4.9 kPa

hco3-

13 mmol/litre

Which one of the following is the most appropriate way to initially manage the short runs of VT?

O Normal saline infusion


O Magnesium infusion
O Amiodarone infusion

O Adenosine bolus
O iv Sodium bicarbonate

Which one of the following is the most appropriate way to initially manage the short runs of VT?
Normal saline infusion
Magnesium infusion

Amiodarone infusion
Adenosine bolus

iv Sodium bicarbonate CORRECT ANSWER

Management of ventricular tachycardia


Whilst magnesium may be a useful option to control ventricular tachycardia (VT) in cases of severe tricyclic
antidepressant (TCA) overdose, iv sodium bicarbonate is the initial therapy of choice
Increasing the pH to the range of 7.45-7.55 has been shown to decrease the QRS interval, stabilise arrhythmias
and increase blood pressure
Other case series state that phenytoin may be useful in correcting conduction defects in this patient population
Conventional class IA, IC, II and III antiarrhythmic agents should be avoided

A 54-year-old man with a history of smoking and hypertension presents to the Emergency Department with
central crushing chest pain, nausea and sweating. On examination his BP is 104/70 mmHg, his pulse 85 bpm and
regular, and he looks pale, grey and sweaty. There are no murmurs on auscultation but he has crackles at both
lung bases consistent with heart failure. The table below contains the investigation results.
Hb

12.8 g/dl

WCC

5.9 x 109/litre

PLT

190 x 109/litre

Na+

141 mmol/litre

K+

5.0 mmol/litre

Creatinine 110 (jmol/litre


ECG

ST elevation V1-V4, ST depression II, III and aVL

Which one of the following is the most likely finding on angiography?


Total occlusion of right coronary artery

O 70% stenosis of left anterior descending artery


O Total occlusion of left anterior descending artery

O 70% stenosis of left circumflex artery


O Total occlusion of left circumflex artery

Which one of the following is the most likely finding on angiography?


Total occlusion of right coronary artery
70% stenosis of left anterior descending artery

Total occlusion of left anterior descending artery CORRECT ANSWER


70% stenosis of left circumflex artery

Total occlusion of left circumflex artery

ST elevation anterior myocardial infarction


This man is suffering from an ST elevation anterior myocardial infarction and most likely has an occlusion of his left
anterior descending artery due to plaque rupture

Management
Management of choice is primary percutaneous coronary intervention (PCI), as early as possible after the onset of
chest pain
Studies have now confirmed that primary PCI is superior to thrombolysis with rates of arterial patency in more than
90% of procedures, and lower rates of bleeding complications

ECG
Left circumflex or right coronary artery occlusions would be expected to result in either lateral or inferior ECG
changes

A 50-year-old man presents to the cardiology clinic for review. His brother died suddenly of a

cardiac arrest while playing Sunday league soccer, and was found on post-mortem to have an
underlying diagnosis of hypertrophic obstructive cardiomyopathy (HOCM). Which one of the

following is the most appropriate way to screen the patient?

Exercise ECG

Electrocardiography studies
Transoesophageal echo

Radionucleotide scanning
Resting 12 lead ECG

A 50-year-old man presents to the cardiology clinic for review. His brother died suddenly of a
cardiac arrest while playing Sunday league soccer, and was found on post-mortem to have an

underlying diagnosis of hypertrophic obstructive cardiomyopathy (HOCM). Which one of the


following is the most appropriate way to screen the patient?
Exercise ECG

Electrocardiography studies
Transoesophageal echo CORRECT ANSWER

Radionucleotide scanning

Resting 12 lead ECG

Screening for hypertrophic obstructive cardiomyopathy


Two-dimensional echocardiography is diagnostic for hypertrophic obstructive cardiomyopathy (HOCM), with
transoesophageal echo delivering the best views

Echocardiography findings are as follows


elevated flow velocity across the left ventricular (LV) outflow tract is seen
diastolic dysfunction with reduced left ventricular compliance

systolic anterior motion of the anterior mitral valve leaflet


asymmetrical septal hypertrophy
Radionucleotide imaging may show reversible perfusion defects, but these are not diagnostic of the condition
Cardiac catheterisation follows echocardiography, to assess the degree of LV outflow tract obstruction, LV
anatomy and coronary artery anatomy

A 67-year-old woman presents with syncope. She has suffered two or three episodes of collapse during the past
6 months, the most recent while attending church on a Sunday morning. She has a history of hypertension which
is currently managed with ramipril and bendroflumethiazide and dyslipidaemia treated with simvastatin. On
examination her pulse is 40 bpm and BP is 100/50 mmHg. Her chest is clear and heart sounds are normal. You
notice irregular cannon waves on examination of the JVP. The table below shows the investigation results.

Hb

12.1 g/dl

WCC

7.4

PLT

203 x 109/litre

Na+

139 mmol/litre

K+

4.9 mmol/litre

109/litre

Creatinine 149 (jmol/litre

Which one of the following is the most likely diagnosis?

Complete heart block


Mobitz type 2 heart block

O Sinus bradycardia
O Junctional rhythm
O Ventricular bigeminy

Which one of the following is the most likely diagnosis?


O Complete heart block CORRECT ANSWER
Mobitz type 2 heart block

Sinus bradycardia

Junctional rhythm
Ventricular bigeminy

Complete heart block


Cannon waves occur when the right atrium contracts against a closed tricuspid valve, and these occur irregularly
in complete heart block
This diagnosis is entirely in accordance with the presentation with bradycardia, relative hypotension and syncope
Management in this case would be an ECG to confirm the diagnosis and then likely referral for insertion of
permanent pacemaker
Cannon waves are also seen in conjunction with ventricular tachycardia

A 46-year-old man is admitted with a tachycardia. He has no previous medical history of note, but admits to
excessive use of alcohol and caffeine associated with a particularly stressful period at work during his job as a
bond trader. On examination his BP is 122/80 mmHg, his pulse is 180 bpm. His chest is clear and there are no
signs of cardiac failure. The table below shows the investigation results.

Hb

12.1 g/dl

WCC

5.6

PLT

190 x 109/litre

Na+

139 mmol/litre

K+

4.8 mmol/litre

109/litre

Creatinine 110 (jmol/litre


ECG

Narrow complex tachycardia, rate 180 bpm

He has tried swallowing of crushed ice to no effect, 6 and 6 mg of adenosine iv with no effect. Which one
of the following is the most appropriate, next management step?

O iv 12 mg adenosine
O iv amiodarone loading
O iv atenolol
O iv verapamil

O iv flecainide

He has tried swallowing of crushed ice to no effect, 6 and 6 mg of adenosine iv with no effect. Which one
of the following is the most appropriate, next management step?
iv 12 mg adenosine CORRECT ANSWER

iv amiodarone loading
iv atenolol
iv verapamil

iv flecainide

Management of tachycardia
It is appropriate to try 12 mg iv adenosine before moving on to an alternative therapy
Verapamil or short acting Q> blockers, such as esmolol, are both appropriate alternative agents for achieving sinus
rhythm

Flecainide is useful for chemical cardioversion of paroxysmal atrial fibrillation in patients with no history of
underlying ischaemic heart disease; iv amiodarone is a reasonable alternative

A 30-year-old man presents with 1 hour of central crushing chest pain. He admits to regular cocaine use,
including on the evening that he presents to the Emergency Department. Other history of note includes smoking
10 cigarettes per day and a family history of mixed hyperlipidaemia. On examination his BP is 220/120 mmHg, but
this falls to 180/80 mmHg after diamorphine. He has a sinus tachycardia of 110 bpm. He has been given 300 mg
of aspirin by the ambulance crew. The table below shows the investigation results.

Hb

13.8 g/dl

WCC

5.9 x 109/litre

PLT

211 x 109/litre

Na+

141 mmol/litre

K+

4.9 mmol/litre

Creatinine 110 (jmol/litre


ECG

Anterior ST elevation consistent with acute myocardial infarction

Which one of the following is the most appropriate way to manage him?

O Abciximab
Percutaneous coronary intervention
Q Low molecular-weight heparin
Streptokinase

rtPA

Which one of the following is the most appropriate way to manage him?
Abciximab

Percutaneous coronary intervention CORRECT ANSWER

Low molecular-weight heparin


Streptokinase

rtPA

Cocaine-associated chest pain


Guidelines suggest that patients with evidence of acute STEMI should be managed in the same way as those
without history of cocaine abuse; as such, percutaneous transluminal coronary angioplasty (PTCA) is the most
appropriate option
Some notable exceptions do exist however, mainly around use of Q> blockade, which is not recommended in
patients with a history of cocaine abuse, as it may worsen coronary artery vasospasm

Instead, nitrates or calcium-channel blockade are recommended as alternatives


With respect to use of low molecular-weight heparin or llbllla antagonists, the American Heart Association
guidelines endorse their use in a clinically appropriate situation despite the fact that no randomised controlled trial
evidence exists (http://circ.ahajournals.org/cgi/content/full/117/14/1897)

A 45-year-old man with previously diagnosed type 2 diabetes presents to the Emergency Department with severe
central chest pain, nausea and sweating. He was riding his bike in an attempt to lose weight when the chest pain
began and in total it lasted for 25 minutes. He also smokes 15 cigarettes per day and is hypertensive, managed
with ramipril and amlodipine. On examination his BP is 155/95 mmHg, he is pale, sweaty and anxious. He has
been given 300 mg of aspirin by a passer-by. The table below shows the investigation results.

Hb

13.2 g/dl

WCC

5.9 x 109/litre

PLT

209 x 109/litre

Na+

141 mmol/litre

K+

4.9 mmol/litre

Creatinine 130 [jmol/litre


Glucose

17.1 mmol/litre

ECG

Inferior ST elevation

Which one of the following is the most appropriate intervention?

O Low molecular-weight heparin


Alteplase

0 Streptokinase
Percutaneous coronary intervention

O Abciximab

Which one of the following is the most appropriate intervention?


Low molecular-weight heparin
Alteplase

Streptokinase
Percutaneous coronary intervention CORRECT ANSWER
Abciximab

Percutaneous coronary intervention


Studies have shown that in patients with acute STEMI, percutaneous coronary intervention (PCI) is superior to
thrombolysis

As such PCI is recommended here above both streptokinase and alteplase


Whilst some acute trusts do not yet have access to an acute angioplasty service, development of this has been
made a priority for the next few years

A 70-year-old man with a history of extensive acute myocardial infarction 4 years earlier, comes to the hospital with his
wife. He has suffered four episodes of collapse over the past 6 months, the most recent that morning, when his wife
witnessed slurred speech, confusion and weakness of his right arm and leg. On examination he has no chest pain, his
BP is 145/82 mmHg and he is not in cardiac failure. His apex beat is displaced to the left. The neurological features
have resolved. The table below shows the investigation results.
Hb

12.1 g/dl

WCC

5.9

PLT

187 x 109/litre

Na+

142 mmol/litre

K+

5.1 mmol/litre

109/litre

Creatinine 148 pmol/lotre


ECG

ST elevation in the anterior leads

Which one of the following is the most appropriate way to diagnose his primary underlying pathology?
O Monitor on the CCU
O Thrombolyse with tPA
O Refer for immediate PCI
C Arrange an urgent CT head

C Arrange a cardiac MRI

A 70-year-old man with a history of extensive acute myocardial infarction 4 years earlier comes to the hospital with his
wife. He has suffered four episodes of collapse over the past 6 months, the most recent that morning, when his wife
witnessed slurred speech, confusion and weakness of his right arm and leg. On examination he has no chest pain, his
BP is 145/82 mmHg and he is not in cardiac failure. His apex beat is displaced to the left. The neurological features
have resolved. The table below shows the investigation results.
Hb

12.1 g/dl

WCC

5.9

PLT

187 x 109/litre

Na+

142 mmol/litre

K+

5.1 mmol/litre

109/litre

Creatinine 1 48 pmol/lotre
ECG

ST elevation in the anterior leads

Which one of the following is the most appropriate way to diagnose his primary underlying pathology?
Monitor on the CCU

Thrombolyse with tPA


Refer for immediate PCI

Arrange an urgent CT head


Arrange a cardiac MRI CORRECT ANSWER

Diagnosis of an aneurysm
Persistent ST elevation in the absence of chest pain, in a patient who has a history of previous anterior myocardial
infarction raises the possibility of left ventricular aneurysm

Cardiac MRI is an effective way to image the aneurysm non-invasively, and would be the preferred initial
investigation

The presence of multiple transient ischaemic attacks raises the possibility of thrombus formation
Hence anticoagulation may be considered here, with possible referral for surgical excision of aneurysm

A 67-year-old man Is referred to the cardiology clinic with angina, progressive heart failure and two episodes
of syncope. He has a history of hypertension managed with ramipril and indapamide, and suffered an inferior
myocardial infarction some 4 years ago. On examination his BP is 125/105 mmHg, and he has a soft ejection
systolic murmur loudest at the right second intercostal space. He has evidence of LVH and there are bilateral
inspiratory crackles on auscultation of the chest consistent with LVF. The table below shows the
investigation results.
Hb

12.4 g/dl

WCC

6.1

PLT

208

Na+

140 mmol/litre

K+

4.3 mmol/litre

x
x

109/litre
109/litre

Creatinine 185 pmol/litre

Which one of the following is likely to be the most significant problem driving his symptoms?
O Coronary artery disease
O Mitral regurgitation
O Aortic stenosis
O Cardiac arrhythmias
O Chronic renal failure

A 67-year-old man Is referred to the cardiology clinic with angina, progressive heart failure and two episodes
of syncope. He has a history of hypertension managed with ramipril and indapamide, and suffered an inferior
myocardial infarction some 4 years ago. On examination his BP is 125/105 mmHg, and he has a soft ejection
systolic murmur loudest at the right second intercostal space. He has evidence of LVH and there are bilateral
inspiratory crackles on auscultation of the chest consistent with LVF. The table below shows the
investigation results.
Hb

12.4 g/dl

WCC

6.1

PLT

208

Na+

140 mmol/litre

K+

4.3 mmol/litre

x
x

109/litre
109/litre

Creatinine 185 pmol/litre

Which one of the following is likely to be the most significant problem driving his symptoms?
Coronary artery disease

Mitral regurgitation
Aortic stenosis CORRECT ANSWER
Cardiac arrhythmias
Chronic renal failure

Aortic stenosis
The triad of angina, left ventricular failure and syncope is classical with respect to aortic stenosis

Confounding factors
Two confounders exist
in the elderly the more high frequency components of aortic stenosis may be heard best at the apex, the so
called Gallavardin phenomenon
and the components of the murmur may be softened in situations where cardiac output is reduced

Management
Given this man has evidence of coronary artery disease he may well have co-existent reduced cardiac output
Hence he requires assessment of both aortic valve and coronary artery status, with combined valve replacement
and coronary artery bypass graft surgery likely to be the most appropriate way to manage him

A 52-year-old male is undergoing exercise tolerance testing for coronary artery disease screening

after suffering indigestion type pain whilst playing squash with a workmate. He reaches stage II of
the Bruce protocol when his BP is 210/100 mmHg and heart rate is 170 bpm. ECG changes are
noted. Which one of the following is the strongest indicator of underlying arterial disease for
stopping the test?
O His BP of 210/100 mmHg

O His heart rate


O 2 mm ST depression in the lateral leads
O Patient request
Ventricular ectopics on the monitor

A 52-year-old male is undergoing exercise tolerance testing for coronary artery disease screening

after suffering indigestion type pain whilst playing squash with a workmate. He reaches stage II of
the Bruce protocol when his BP is 210/100 mmHg and heart rate is 170 bpm. ECG changes are
noted. Which one of the following is the strongest indicator of underlying arterial disease for
stopping the test?

His BP of 210/100 mmHg


His heart rate

2 mm ST depression in the lateral leads CORRECT ANSWER


Patient request
Ventricular ectopics on the monitor

Discontinuation of exercise tolerance testing


Hypertension of greater than 250/1 15 mmHg is usually considered a reason to discontinue the test
A drop of more than 10 mmHg blood pressure in the presence of other evidence of ischaemia is also a reason for
discontinuing the exercise test

In the presence of an achieved heart rate of 170 bpm, ST depression of 2 mm is an entirely appropriate reason for
discontinuing the test

Ventricular ectopics, rather than sustained ventricular tachycardia are acceptable, and the test need not be
stopped for these

A 62-year-old man presents to the clinic with Increasing shortness of breath. He has a history of smoking 10
cigarettes per day and hypertension for which he takes ramipril 10 mg daily. On examination his BP is 152/87
mmHg, his pulse is 75 bpm and there is reversed splitting of the second heart sound. There are no signs of
cardiac failure. The table below contains the investigation results.
Hb

13.2 g/dl

WCC

5.6

PLT

209

Na+

139 mmol/litre

K+

4.5 mmol/litre

x
x

109/litre

109/litre

Creatinine 132 pmol/litre

Which one of the following is the most likely diagnosis?


O Atrial septal defect
O Left bundle-branch block
O Mitral regurgitation
O Right bundle-branch block
O Ventricular septal defect

A 62-year-old man presents to the clinic with increasing shortness of breath. He has a history of smoking 10
cigarettes per day and hypertension for which he takes ramipril 10 mg daily. On examination his BP is 152/87
mmHg, his pulse is 75 bpm and there is reversed splitting of the second heart sound. There are no signs of
cardiac failure. The table below contains the investigation results.
Hb

13.2 g/dl

WCC

5.6

PLT

209

Na+

139 mmol/litre

K+

4.5 mmol/litre

109/litre

109/litre

Creatinine 132 pmol/litre

Which one of the following is the most likely diagnosis?

Atrial septal defect


Left bundle-branch block CORRECT ANSWER

Mitral regurgitation
Right bundle-branch block

Ventricular septal defect

Splitting of the second heart sound

Reversed splitting of the second heart sound occurs when the sound from the closure of the pulmonary valve
occurs prior to that from the closure of the aortic valve

Causes include

aortic stenosis
left bundle-branch block

hypertrophic obstructive cardiomyopathy

Fixed splitting of the second heart sound where aortic valve closure occurs before the pulmonary, and which
doesn't vary with inspiration, occurs with an atrial or ventricular septal defect

A 52-year-old woman presents to the clinic with shortness of breath and angina. She has a history

of hypertension and type 2 diabetes for which she takes metformin and BD Mixed Insulin. On

examination her BP is 155/92 mmHg. You notice reversed splitting of the second heart sound and
bibasal crackles on auscultation of the chest consistent with cardiac failure. Which one of the
following is the most likely finding on ECG?

Right bundle-branch block

0 QT prolongation
P mitrale

O P pulmonale
O Left bundle-branch block

A 52-year-old woman presents to the clinic with shortness of breath and angina. She has a history

of hypertension and type 2 diabetes for which she takes metformin and BD Mixed Insulin. On
examination her BP is 155/92 mmHg. You notice reversed splitting of the second heart sound and
bibasal crackles on auscultation of the chest consistent with cardiac failure. Which one of the
following is the most likely finding on ECG?

Right bundle-branch block


QT prolongation

P mitrale
P pulmonale
Left bundle-branch block CORRECT ANSWER

Reversed splitting of the second heart sound occurs when closure of the pulmonary valve occurs before the aortic
valve

Conditions associated with reversed splitting of the second heart sound include left bundle-branch block,
hypertrophic obstructive cardiomyopathy and aortic stenosis

Fixed splitting of the second heart sound is associated with an atrial septal or ventricular septal defect

You are designing a study for a new agent that may reduce myocardial necrosis after myocardial
infarction. The agent is specifically thought to reduce early myonecrosis. Which one of the
following enzymes is most appropriate to measure early myocardial necrosis?
Glycogen phosphorylase isoenzyme BB (GPBB)

O Myoglobin
O Creatinine kinase
Troponin

O Lactate dehydrogenase

You are designing a study for a new agent that may reduce myocardial necrosis after myocardial
infarction. The agent is specifically thought to reduce early myonecrosis. Which one of the
following enzymes is most appropriate to measure early myocardial necrosis?
Glycogen phosphorylase isoenzyme BB (GPBB) CORRECT ANSWER

Myoglobin
Creatinine kinase

Troponin
Lactate dehydrogenase
YOUR ANSWER WAS INCORRECT

The Answer

Comment on this Question

Enzyme measurements in early myocardial necrosis


Glycogen phosphorylase (GP) exists in a number of isoforms, but GPBB exists in heart and brain tissue
During a period of ischaemia GPBB is released and is elevated 1-3 h after the event
Myoglobin levels become significantly elevated 2 h after ischaemia, for CK the level may not be markedly elevated
until 4 h or more after an infarct
LDH is a late marker of myocardial infarction which remains elevated for a few days after infarct
Troponin is the current gold standard marker for myocardial infarction, although levels only become elevated after 6

h; conventional practice is to measure levels at 6 and 12 h after a period of chest pain

Next question

i
tag this question

gPasTest

* *

previous question

next question

back to menu

finish session

2014 PasTest Ltd. Egerton Court. Parkgate Estate.. Knutsford. Cheshire, United Kingdom WA16 8DX. Tel: +44 (0) 1565 752000. Ema

A 74-year-old man presents with acute onset palpitations, ECG showed regular tachycardia with a ventricular
rate of 150. He has a history of hypertension treated with ramipril and amlodipine, but has no other
significant past medical history. He smokes 5 cigars per day. On examination his BP is 110/70 mmHg; his
pulse is very rapid, at least 130 bpm. The table below contains investigation results.
Hb

12.9 g/dl

WCC

6.7 x 109/litre

PLT

291

Na+

141 mmol/litre

K+

5.3 mmol/litre

109/litre

Creatinine 141 pmol/litre


ECG

regular narrow complex tachycardia with ventricular rate of 150 bpm; saw-tooth pattern particularly evident
in leads II, III and aVF

Which one of the following rhythms is it most likely to be?


O Idioventricular tachycardia with dissociated AV response
O Atrial flutter
O Atrial fibrillation
O AV nodal re-entrant tachycardia
O Atrioventricular re-entrant tachycardia

A 74-year-old man presents with acute onset palpitations, ECG showed regular tachycardia with a ventricular rate

of 150. He has a history of hypertension treated with ramipril and amlodipine, but has no other significant past
medical history. He smokes 5 cigars per day. On examination his BP is 110/70 mmHg; his pulse is very rapid, at

least 130 bpm. The table below contains investigation results.

Hb

12.9 g/dl

WCC

6.7 x 109/litre

PLT

291 x 109/litre

Na+

141 mmol/litre

K+

5.3 mmol/litre

Creatinine 141 pmol/litre


regular narrow complex tachycardia with ventricular rate of 150 bpm; saw-tooth pattern particularly evident in
leads II, III and aVF

ECG

Which one of the following rhythms is it most likely to be?


Idioventricular tachycardia with dissociated AV response
Atrial flutter CORRECT ANSWER

Atrial fibrillation

AV nodal re-entrant tachycardia


Atrioventricular re-entrant tachycardia

YOUR ANSWER WAS INCORRECT

Atrial flutter
The saw tooth pattern which is particularly well characterised in leads II, III and aVF is typical of atrial flutter with
2:1 block
It is the second most common tachyarrhythmia after atrial fibrillation, prevalence in the 65-90 year age group
being around 0.5-1%

Treatment
Electrical cardioversion may be attempted; otherwise amiodarone is the drug of choice for chemical cardioversion
In patients in whom cardioversion is unsuccessful, ventricular rate may be controlled with non-dihydropyridine
calcium channel blockers such as verapamil or diltiazem, or cardioselective

blockers such as metoprolol

Next question

i
Easy

T-

tag this question

PasTest
Dnfurfl*d to )OUi tu((u

previous question

or
next question

back to menu

finish session

2014 PasTest Ltd. Egerton Court, Parkgate Estate, Knutsford, Cheshire, United Kingdom WA16 8DX. Tel: +44 (0) 1565 752000. Email: e

An 82-year-old man presents to the Cardiology Clinic with syncopal attacks. His GP has been treating him for
worsening hypertensive heart failure. He takes furosemide, 80 mg and ramipril, 10 mg, with the recent addition of a
small dose of spironolactone. Around 2 wee ks ago he presented with an angina attack to the Emergency Department
after getting chest pain whilst out shopping at the supermarket. On examination his BP is 165/122 mmHg. His pulse is
92 bpm, heart sounds reveal a systolic murmur loudest in the aortic area, but no ejection click. He has bibasal crackles
on auscultation of the chest. The table below contains investigation results.
Hb

12.9 g/dl

WCC

5.9

PLT

189 x 109/litre

Na+

1 38 mmol/litre

K+

5.4 mmol/litre

Creatinine

201 pmol/litre

109/litre

Chest X-ray Bilateral basal infiltrates consistent with fluid, and cardiomegaly

Which one of the following is the most likely cause of his aortic stenosis?
O Subacute bacterial endocarditis
O Rheumatic fever
O Bicuspid aortic valve
O Malformed tricuspid aortic valve
O Senile degenerative aortic stenosis

An 82-year-old man presents to the Cardiology Clinic with syncopal attacks. His GP has been treating him for
worsening hypertensive heart failure. He takes furosemide, 80 mg and ramipril, 10 mg, with the recent addition of

a small dose of spironolactone. Around 2 weeks ago he presented with an angina attack to the Emergency

Department after getting chest pain whilst out shopping at the supermarket. On examination his BP is 165/122
mmHg. His pulse is 92 bpm, heart sounds reveal a systolic murmur loudest in the aortic area, but no ejection
click. He has bibasal crackles on auscultation of the chest. The table below contains investigation results.

Hb

12.9 g/dl

WCC

5.9

PLT

189 x 109/litre

Na+

138 mmol/litre

K+

5.4 mmol/litre

Creatinine

201 (jmol/litre

109/litre

Chest X-ray Bilateral basal infiltrates consistent with fluid, and cardiomegaly

Which one of the following is the most likely cause of his aortic stenosis?
Subacute bacterial endocarditis

Rheumatic fever
Bicuspid aortic valve

Malformed tricuspid aortic valve


Senile degenerative aortic stenosis CORRECT ANSWER

YOUR ANSWER WAS INCORRECT

Comment on this Question

The Answer

Senile degenerative aortic stenosis


Senile degenerative aortic stenosis involves progressive calcification of the valve leaflets, in response to long
standing haemodynamic stress

It represents the commonest cause of aortic valve replacement, usually presenting after the age of 75
Diabetes and hypercholesterolemia are risk factors for development of the lesion
Patients with senile degenerative aortic stenosis may well have co-existent coronary artery disease, in which
case coronary artery bypass surgery can be carried out at the same time as valve replacement

Other notes

Rheumatic fever related aortic stenosis tends to present earlier in patients in their sixties
Congenital bicuspid aortic stenosis presents in patients in the 40-50 year age bracket

Next question

Easy

tag this question

previous question

PasTest"
im
:
'

or
next question

back to menu

finish session

2014 PasTest Ltd. Egerton Court, Parkgate Estate. Knutsford, Cheshire. United Kingdom WA16 8DX. Tel: +44 (0) 1565 752000. Email: e

A 55-year-old man has infective endocarditis. He has refused dental work for a number of years and presented with
night sweats and lethargy to his GP. Subsequent blood cultures demonstrated a S. viridans bacteraemia. On
examination he is pyrexial, 37.8 C, his BP is 110/70 mmHg, pulse is 95 bpm. He has splinter haemorrhages on
examination of his fingers. Auscultation reveals a murmur consistent with mitral regurgitation. His chest is clear. The
table below contains investigation results.

Hb

10.7 g/dl

WCC

13.1 109/litre

PLT

211

Na+

1 39 mmol/litre

K+

4.9 mmol/litre

Creatinine

1 39 pmol/litre

ESR

72 mm/h

109/litre

ECG on admission PR interval 180 ms, otherwise no significant changes

Echocardiogram

mitral regurgitation with evidence of vegetations

Which one of the following would be considered an urgent indication for surgical intervention after 12
days of antibiotic therapy?
O Increasing PR interval
O Systolic murmur after 10 days of antibiotics
O Increasing fever
O Raised ESR
O Mobile vegetation > 15mm in size

A 55-year-old man has infective endocarditis. He has refused dental work for a number of years and presented

with night sweats and lethargy to his GP. Subsequent blood cultures demonstrated a S. viridans bacteraemia. On
examination he is pyrexial, 37.8 C, his BP is 110/70 mmHg, pulse is 95 bpm. He has splinter haemorrhages on
examination of his fingers. Auscultation reveals a murmur consistent with mitral regurgitation. His chest is clear.
The table below contains investigation results.
Hb

10.7 g/dl

WCC

13.1 x 109/litre

PLT

211

Na+

139 mmol/litre

K+

4.9 mmol/litre

Creatinine

139 pmol/litre

ESR

72 mm/h

109/litre

ECG on admission PR interval 180 ms, otherwise no significant changes

Echocardiogram

mitral regurgitation with evidence of vegetations

Which one of the following would be considered an urgent indication for surgical intervention after 12 days

of antibiotic therapy?

Increasing PR interval CORRECT ANSWER


Systolic murmur after 10 days of antibiotics

Increasing fever
Raised ESR
Mobile vegetation > 15mm in size
YOUR ANSWER WAS INCORRECT

Comment on this Question

The Answer

Surgical intervention post-infection


Endocarditis

Increase of the PR interval suggests extension of the endocarditic infection into the myocardium
This also raises the possibility of abscess formation, particularly given the length of time of antibiotic treatment so
far, as such urgent surgical referral is indicated
Rupture into the pericardium is an indication for same day surgical intervention
Delahaye et al., (2004) Heart, 90: 618-620

Other indications
Other indications for urgent surgical intervention include
mitral regurgitation or aortic regurgitation with heart failure
septal perforation
valvular obstruction
Particularly large vegetations, (>15mm) are also an indication for surgical assessment, (although not necessarily
for urgent intervention)

Next question

tag this question

previous question

ilk PasTest'
: <*'

or
next question

back to menu

finish session

2014 PasTest Ltd. Egerton Court, Parkgate Estate, Knutsford, Cheshire, United Kingdom WA16 8DX. Tel: +44 (0) 1565 752000. Email: e

A 39-year-old woman comes to the Emergency department with a severe "crushing" frontal headache, which she
describes as the worst she has ever had. She has suffered increasing headaches in the morning over the past 3
months, but puts this down to stress as she is undergoing investigations for hypercalcaemia. You understand her
mother died from a stroke at a young age. On examination her BP is 190/100 mmHg, pulse is 85/min and regular. She
looks anxious. There is evidence of hypertensive retinopathy on fundoscopy.
Investigations;

Hb

12.9 g/dl

WCC

9.1 x 109/1

PLT

203x109/1

Na+

138 mmol/l

K+

3.9 mmol/l

Creatinine

110 micromol/l

Calcium

3.05 mmol/l

Which of the following is likely to be the most appropriate step in managing her blood pressure?

O Labetolol
O Hydralazine

O Phenoxybenzamine
O Ramipril

G Sodium nitroprusside

A 39-year-old woman comes to the Emergency department with a severe "crushing" frontal headache, which she
describes as the worst she has ever had. She has suffered increasing headaches in the morning over the past 3
months, but puts this down to stress as she is undergoing investigations for hypercalcaemia. You understand
her mother died from a stroke at a young age. On examination her BP is 190/100 mmHg, pulse is 85/min and
regular. She looks anxious. There is evidence of hypertensive retinopathy on fundoscopy.
Investigations;

Hb

12.9 g/dl

WCC

9.1 x 109/1

PLT

203x109/1

Na+

138 mmol/l

K+

3.9 mmol/l

Creatinine

110 micromol/l

Calcium

3.05 mmol/l

Which of the following is likely to be the most appropriate step in managing her blood pressure?

Labetolol

Hydralazine

Phenoxybenzamine CORRECT ANSWER

Ramipril
Sodium nitroprusside

YOUR ANSWER WAS INCORRECT

The Answer

Comment on this Question

The answer is Phenoxybenzamine


Whilst sodium nitroprusside and labetolol are both used in the management of accelerated hypertension, the
clues in this patient are the family history of death from a stroke and the hypercalcaemia, which might be
pointers to multiple endocrine neoplasia Type 2 (MEN2). Phenoxybenzamine to achieve alpha blockade is
therefore the treatment of choice. Agents which achieve an element of beta blockade, such as labetolol, are
not recommended until complete alpha blockade has been achieved.

Next question

i
tag this question

previous question

or
next question

back to menu

finish session

jlfePasTest*
'

Rv.i.(..i..iioyn. >.<.%

2014 PasTest Ltd. Egerton Court, Parkgate Estate, Knutsford, Cheshire, United Kingdom WA16 8DX. Tel: +44 (0) 1565 752000. Email: ei

A 45-year-old man is admitted to the Emergency department from an office party after complaining of a severe frontal
headache, the worst he has ever had. On further questioning he admits to headaches on most mornings of the week,
"like a hangover". On examination his BP is 190/100 mmHg, pulse is 84/min and regular. He has grade 4 hypertensive
retinopathy and bibasal crackles on auscultation of the chest.
Investigations;

Hb

11.9 g/dl

WCC

9.9x109/1

PLT

192 x 109/1

Na+

1 38 mmol/l

K+

4.2 mmol/l

Creatinine

182 micromol/l

Renal ultrasound

Bilateral small kidneys

Which of the following is the optimal treatment?

O IV sodium nitroprusside
O Oral ramipril
O IV labetalol
O Oral amlodipine
O Oral indapamide

A 45-year-old man is admitted to the Emergency department from an office party after complaining of a severe
frontal headache, the worst he has ever had. On further questioning he admits to headaches on most mornings
of the week, "like a hangover". On examination his BP is 190/100 mmHg, pulse is 84/min and regular. He has
grade 4 hypertensive retinopathy and bibasal crackles on auscultation of the chest.
Investigations;

Hb

11.9 g/dl

WCC

9.9x109/1

PLT

192 x 109/1

Na+

138 mmol/l

K+

4.2 mmol/l

Creatinine

182 micromol/l

Renal ultrasound

Bilateral small kidneys

Which of the following is the optimal treatment?

IV sodium nitroprusside CORRECT ANSWER


Oral ramipril

IV labetalol
Oral amlodipine
Oral indapamide

YOUR ANSWER WAS INCORRECT

The Answer

Comment on this Question

The answer is IV sodium nitroprusside


This man has malignant hypertension. Out of the two IV options given, nitroprusside is preferred as
treatment since labetalol may be associated with a greater reduction in cerebral blood flow, and as such may

precipitate cerebral ischaemia. The oral options are not preferred in this situation as the overall reduction in
BP is not as great as that seen with IV therapy, and titration is potentially less precise.

Next question

i
tag this question

ilk PasTest'
:

previous question

or
next question

back to menu

finish session

2014 PasTest Ltd. Egerton Court, Parkgate Estate, Knutsford, Cheshire, United Kingdom WA16 8DX. Tel: +44 (0) 1565 752000. Email: e

A 61-year-old man is referred to the falls clinic after suffering a second syncopal episode in the past few months.
He apparently collapsed in the local tesco's with very little warning. Past history of note includes erectile
dysfunction for which he was prescribed a PDE-5 inhibitor, and according to his wife he has become a little
unsteady on his feet and vague over the past few months. On examination his BP is 122/60 mmHg, falling to
90/50 mmHg on standing. There is postural instability coupled with an ataxic gait when you get him to walk
accross the consulting room. You notice nystagmus on examination of his cranial nerves.

Which of the following is the most appropriate initial therapy for his blood pressure?

O Fludrocortisone
O Midodrine
O Support stockings
O Stop the PDE-5 inhibitor
O Increased salt in the diet

A 61-year-old man is referred to the falls clinic after suffering a second syncopal episode in the past few months.
He apparently collapsed in the local tesco's with very little warning. Past history of note includes erectile
dysfunction for which he was prescribed a PDE-5 inhibitor, and according to his wife he has become a little
unsteady on his feet and vague over the past few months. On examination his BP is 122/60 mmHg, falling to
90/50 mmHg on standing. There is postural instability coupled with an ataxic gait when you get him to walk
accross the consulting room. You notice nystagmus on examination of his cranial nerves.

Which of the following is the most appropriate initial therapy for his blood pressure?

Fludrocortisone
Midodrine

Support stockings CORRECT ANSWER

Stop the PDE-5 inhibitor


Increased salt in the diet
YOUR ANSWER WAS INCORRECT

Comment on this Question

The Answer

The answer is Support stockings


This man has features of multi-system atrophy, and autonomic dysfunction is the most likely cause of his
postural hypotension. Non-drug interventions include use of support stockings which may significantly
reduce the risk of syncopal episodes. This can be followed by low dose fludrocortisone and midodrine in
resistant cases. It's unlikely that stopping the PDE-5 inhibitor will help, and increasing salt in his diet will only
have a minor effect versus the other options.

Next question

tag this question

previous question

i' PasTest'
'

or
next question

back to menu

finish session

2014 PasTest Ltd. Egerton Court, Parkgate Estate, Knutsford, Cheshire, United Kingdom WA16 8DX. Tel: +44 (0) 1565 752000. Email: e

A 54-year-old man is moved to the catheter lab for urgent PTCA after infero-posterior myocardial infarction. The

nurses ask you to see him as he has hypotension, (BP 90/50 mmHg). His JVP is markedly elevated, pulse is
65/min and regular. The chest is clear with no signs of heart failure, and there are no murmurs. ECG confirms
inferoposterior changes consistent with an acute STEMI.

Which of the following is the most likely diagnosis?


O Aortic dissection

Left ventricular dysfunction

O Right ventricular dysfunction


O Ventricular free wall rupture
O Mitral regurgitation

A 54-year-old man is moved to the catheter lab for urgent PTCA after infero-posterior myocardial infarction. The
nurses ask you to see him as he has hypotension, (BP 90/50 mmHg). His JVP is markedly elevated, pulse is
65/min and regular. The chest is clear with no signs of heart failure, and there are no murmurs. ECG confirms
inferoposterior changes consistent with an acute STEMI.

Which of the following is the most likely diagnosis?

Aortic dissection
Left ventricular dysfunction
Right ventricular dysfunction CORRECT ANSWER

Ventricular free wall rupture


Mitral regurgitation
YOUR ANSWER WAS INCORRECT

Comment on this Question

The Answer

The answer is Right ventricular dysfunction

Approximately 10% of patients with an acute infero-posterior infarct present with significant right ventricular
dysfunction. In this case RV output is redcued leading to systemic hypotension and a markedly elevated JVP.
Given the infarct is in the inferior territory, it seems he is also unable to mount a tachycardia to maintain his
BP. Management involves maintaining adequate RV filling pressures, (with CVP monitoring to reduce the risk
of overload), and the use of inotropes such as Dobutamine if required.

LV dysfunction, ventricular free wall rupture and mitral regurgitation are associated with acute left ventricular
failure. Aortic dissection in the absence of worsening pain spreading to the back or an extension of ECG
changes is unlikely.

Next question

tag this question

previous question

or
next question

back to menu

finish session

PasTest'

2014 PasTest Ltd. Egerton Court, Parkgate Estate. Knutsford, Cheshire, United Kingdom WA16 8DX. Tel: +44 (0) 1565 752000. Email: c

A 67-year-old man comes to the clinic for review. He has suffered episodes of central chest pain on 3 occasions over
the past year, which have occured twice when he went out on a cold morning in the winter, and once when he was
carrying a tree trunk which had been chopped down in his garden. On all of the occasions the pain lasted for a few
minutes and then subsided spontaneously when he rested. He smokes 5 cigarettes per day and has a history of
hypertension managed with Ramipril 10mg daily. He is also taking Aspirin 75mg for primary prevention. On examination
his BP is 135/72 mmHg, pulse is 72/min and regular. His BMI is 25.
Investigations:

Hb

13.1 g/dl

WCC

8.9x109/1

PLT

203x109/1

Na+

138 mmol/l

K+

4.9 mmol/l

Creatinine

100 micromol/l

Total cholesterol

6.2 mmol/l

Glucose

6.2 mmol/l (fasting)

Which of the following is the optimal management with respect to ischaemic cardiovascular event prevention?

C Atenolol
C Atorvastatin
O Clopidogrel

Isosorbide dinitrate

A 67-year-old man comes to the clinic for review. He has suffered episodes of central chest pain on 3 occasions
over the past year, which have occured twice when he went out on a cold morning in the winter, and once when
he was carrying a tree trunk which had been chopped down in his garden. On all of the occasions the pain lasted
for a few minutes and then subsided spontaneously when he rested. He smokes 5 cigarettes per day and has a
history of hypertension managed with Ramipril 10mg daily. He is also taking Aspirin 75mg for primary prevention.
On examination his BP is 135/72 mmHg, pulse is 72/min and regular. His BMI is 25.
Investigations;

Hb

13.1 g/dl

WCC

8.9x109/1

PLT

203x109/1

Na+

138 mmol/l

K+

4.9 mmol/l

Creatinine

100 micromol/l

Total cholesterol

6.2 mmol/l

Glucose

6.2 mmol/l (fasting)

Which of the following is the optimal management with respect to ischaemic cardiovascular event prevention?

Atenolol

Atorvastatin CORRECT ANSWER

Clopidogrel
Isosorbide dinitrate
Metformin

YOUR ANSWER WAS INCORRECT

The Answer

Comment on this Question

The answer is Atorvastatin


This patient's symptoms are consistent with chronic stable angina. With respect to cardiovascular risk
factors he should of course be encouraged to stop smoking, and to exercise and lose weight if possible
given that he has impaired fasting glucose. Metformin in this situation is not indicated. Whilst Atenolol or
Isosorbide dinitrate may reduce the risk of future attacks of angina, they are not effective in reducing the risk
of future ischaemic events compared to Atorvastatin, the correct answer here. Clopidogrel is used in

combination with aspirin in the management of ACS and post coronary artery stenting.

Next question

i
tag this question

Pasfiest

previous question

or
next question

back to menu

finish session

2014 PasTest Ltd. Egerton Court, Parkgate Estate. Knutsford, Cheshire United Kingdom WA16 8DX. Tel: +44 (0) 1565 752000. Email: e

A 55-year-old woman with manic depressive psychosis, obesity and hypertension comes to the clinic. Her GP is

unsure of the optimal way to manage her blood pressure. On examination in the clinic her BP is 155/95 mmHg,
pulse is 78/min and regular. Her BMI is 36. Investigations reveal a fasting glucose of 6.3 mmol/l.

Which of the following is the optimal way to treat her hypertension bearing in mind she takes

Lithium?

O Atenolol
O Bendroflumethiazide
O Felodipine
O Ramipril
Valsartan

A 55-year-old woman with manic depressive psychosis, obesity and hypertension comes to the clinic. Her GP is
unsure of the optimal way to manage her blood pressure. On examination in the clinic her BP is 155/95 mmHg,

pulse is 78/min and regular. Her BMI is 36. Investigations reveal a fasting glucose of 6.3 mmol/l.

Which of the following is the optimal way to treat her hypertension bearing in mind she takes
Lithium?

Atenolol
Bendroflumethiazide
Felodipine CORRECT ANSWER

Ramipril
Valsartan

YOUR ANSWER WAS INCORRECT

Comment on this Question

The Answer

The answer is Felodipine

Prescribing anti-hypertensives in conjunction with Lithium therapy is difficult as a number of agents increase
the risk of toxicity. These include Thiazides, ACE inhibitors and Angiotensin receptor blockers. This leaves
us with the option of using a beta blocker or the Dihydropyridine calcium antagonist. Out of the two, beta
blockers are known to worsen glucose intolerance and given that her BMI is very elevated and she has

impaired fasting glucose, Atenolol is therefore not the favoured option. This leaves Felodipine as the
favoured option for treating her blood pressure.

Next question

tag this question

previous question

PasTest'
vm.%%

or
next question

back to menu

finish session

2014 PasTest Ltd. Egerton Court, Parkgate Estate, Knutsford, Cheshire. United Kingdom WA16 8DX. Tel: +44 (0) 1565 752000. Email: enqi

A 58-year-old man returns to the Emergency Department with 30 mins of central chest pain radiating to the
upper part of his left arm. He suffered an inferior territory NSTEMI some 6 months earlier with a small
troponin rise to 1.2 and current cardiovascular medication includes Lisinopril, Bisoprolol, Aspirin and

Atorvastatin. His pain is relieved by 2 sprays of GTN. Physical examination is unremarkable.

Investigations.

Hb

13.1 g/dl

WCC

8.1 X109/I

PLT

213x109/1

Na+

138 mmol/l

K+

4.3 mmol/l

Creatinine

102 micromol/'l

HbA1c

55 mmol/mol (7.2%)

Random glucose 6.7 mmol/l

12 lead ECG

normal sinus rhythm, inferior Q waves

Which of the following is the most appropriate way to manage him?


G Prophylactic LMW Heparin and observe overnight
O Thrombolysis
O Treatment dose LMW Heparin and observe overnight
O Treatment dose Fondaparinux, Aspirin and Clopidogrel

G Urgent angiography

A 58-year-old man returns to the Emergency Department with 30 mins of central chest pain radiating to the upper
part of his left arm. He suffered an inferior territory NSTEMI some 6 months earlier with a small troponin rise to 1.2

and current cardiovascular medication includes Lisinopril, Bisoprolol, Aspirin and Atorvastatin. His pain is
relieved by 2 sprays of GTN. Physical examination is unremarkable.

Investigations;

Hb

13.1 g/dl

WCC

8.1 x 109/1

PLT

213x109/1

Na+

138 mmol/l

K+

4.3 mmol/l

Creatinine

102 micromol/l

HbA1c

55 mmol/mol (7.2%)

Random glucose 6.7 mmol/l


normal sinus rhythm, inferior Q waves

12 lead ECG

Which of the following is the most appropriate way to manage him?

Prophylactic LMW Heparin and observe overnight


Thrombolysis
Treatment dose LMW Heparin and observe overnight
Treatment dose Fondaparinux, Aspirin and Clopidogrel CORRECT ANSWER

Urgent angiography
YOUR ANSWER WAS INCORRECT

The Answer

Comment on this Question

The answer is Treatment dose Fondaparinux, Aspirin and Clopidogrel

Despite the fact this patient has no acute changes on his ECG, he should be managed aggressively for ACS,
as it is quite possible that the 6hr troponin will be elevated. Given that he has had an NSTEMI 6 months
earlier, he is at significant risk of a further infarct and should be considered for angiography, electively within
a few weeks if the pain settles on this occasion, or as an inpatient if the pain fails to settle.

Next question

i
tag this question

ilk PasTest'
:

previous question

or
next question

back to menu

finish session

2014 PasTest Ltd. Egerton Court, Parkgate Estate, Knutsford, Cheshire, United Kingdom WA16 8DX. Tel: +44 (0) 1565 752000. Email: enc

A 36-year-old man is admitted to the Emergency Department with central crushing chest pain, sweating,
tachycardia and anxiety. He admits to heavy use of cocaine and smokes 30 cigarettes per day. On examination
his BP is 165/85 mmHg, pulse is 95/min and regular. His ECG shows anterolateral ST depression.

A 6hr troponin is elevated at 3.1 microg/l

How would you classify his myocardial infarction?

O Type 1
O Type 2

Type 3
O Type 4

Type 5

A 36-year-old man is admitted to the Emergency Department with central crushing chest pain, sweating,
tachycardia and anxiety. He admits to heavy use of cocaine and smokes 30 cigarettes per day. On examination
his BP is 165/85 mmHg, pulse is 95/min and regular. His ECG shows anterolateral ST depression.
A 6hr troponin is elevated at 3.1 microg/l

How would you classify his myocardial infarction?

Type 1
Type 2 CORRECT ANSWER

Type 3
Type 4
Type 5

YOUR ANSWER WAS INCORRECT

The Answer

Comment on this Question

The answer is Type 2


The universal definition of Ml subclassified Ml types in 2012. This would fit the classification for a Type 2 Ml,
myocardial infarction secondary to an ischaemic imbalance, (i.e. in this case related to severe vasospasm

because of the cocaine abuse).


Other types of Ml are listed below

Type 1: Spontaneous Ml
Type 3: Death due to Ml
Types 4 and 5: Ml due to a procedure
http://www.escardio.org/guidelines-surveys/esc-guidelines/Pages/universal-definition-myocardial-

infarction.aspx

Next question

i
tag this question

previous question

4
next question

back to menu

finish session

#%Paslest"
rvdkivd to youi mk*v%

2014 PasTest Ltd. Egerton Court, Parkgate Estate. Knutsford, Cheshire, United Kingdom WA16 8DX. Tel: +44 (0) 1565 752000. Email: en

A 62-year-old man with a history of CKD 4 renal impairment is admitted to the Emergency Department after a
prolonged episode of gastroenteritis. His wife proudly tells you that even whilst he has been unwell she has
continued to give him his diuretics and Lisinopril. On examination his BP is 90/60 mmHg, pulse is 90/min and
regular. He looks very dehydrated. You arrange urgent investigations, the results of which are shown below:

pH

7.21

K+

7.2 mmol/l

Na+

139 mmol/l

Bicarbonate 15 mmol/l

Creatinine

585 micromol/l

Urea

23.1 mmol/l

Which of the following would you expect to see on the 12 lead ECG?
O Inverted P waves
O J waves
O Peaked T waves
O ST depression
O U waves

A 62-year-old man with a history of CKD 4 renal impairment is admitted to the Emergency Department after a
prolonged episode of gastroenteritis. His wife proudly tells you that even whilst he has been unwell she has
continued to give him his diuretics and Lisinopril. On examination his BP is 90/60 mmHg, pulse is 90/min and
regular. He looks very dehydrated. You arrange urgent investigations, the results of which are shown below:

PH

7.21

K+

7.2 mmol/l

Na+

139 mmol/l

Bicarbonate 15 mmol/l
Creatinine

585 micromol/l

Urea

23.1 mmol/l

Which of the following would you expect to see on the 12 lead ECG?

Inverted P waves
J waves
Peaked T waves CORRECT ANSWER

ST depression

U waves
YOUR ANSWER WAS INCORRECT

Comment on this Question

The Answer

The answer is Peaked T waves


Hyperkalaemia is associated with:

Peaked T waves
Prolongation of the PR interval
Widening of the QRS
Reduced or loss of the P wave

Severe hyperkalaemia is associated with a sine wave pattern on the ECG and ultimately asystole if treatment

is not instigated. In this situation urgent treatment would include IV calcium under cardiac monitoring, IV
insulin and dextrose, and potentially nebulised beta agonists.
Inverted P waves are associated with abnormal atrial conduction, J waves with hypothermia. ST depression
and U waves are seen in patients with hypokalaemia.

Next question

tag this question

previous question

next question

back to menu

I PasTest

't&.WJ*??

INmIm

lo )ui WKM$

finish session

2014 PasTest Ltd. Egerton Court. Parkgate Estate, Knutsford, Cheshire, United Kingdom WA16 8DX. Tel: +44 (0) 1565 752000. Email: ei

A 72-year-old man is reviewed in the falls clinic after 4 previous episodes of syncope. Carotid sinus
hypersensitivity was suspected, and he had a profound bradycardic response to carotid sinus massage. He has

no significant past medical history apart from mild hypertension which is managed with Indapamide. On
examination his BP is 142/72 mmHg, pulse is 70/min and regular. Routine bloods are unremarkable.
Which of the following is the most appropriate intervention?

O Dual chamber pacemaker


O Fludrocortisone
O Fluoxetine
O Midodrine
O Stop Indapamide

A 72-year-old man is reviewed in the falls clinic after 4 previous episodes of syncope. Carotid sinus
hypersensitivity was suspected, and he had a profound bradycardic response to carotid sinus massage. He has
no significant past medical history apart from mild hypertension which is managed with Indapamide. On
examination his BP is 142/72 mmHg, pulse is 70/min and regular. Routine bloods are unremarkable.

Which of the following is the most appropriate intervention?


Dual chamber pacemaker CORRECT ANSWER
Fludrocortisone
Fluoxetine
Midodrine

rn\'inf U

Jl

Stop Indapamide

.f U

"V"- S\*f >

M~I

|-i-u-j-W-oJI

r.

t_0

.*

YOUR ANSWER WAS INCORRECT


.> Mf \ iA

J*

|-| i

The Answer
The answer is Dual chamber pacemaker

In bradycardic carotid sinus hypersensitivity, the optimal intervention given the number of falls this man has
suffered is dual chamber pacing. Sertraline and Fluoxetine may be useful in patients who fail to respond to
pacing. Midodrine, (an alpha receptor agonist) and Fludrocortisone are useful for hypotensive carotid sinus
hypersensitivity. His BP is at the higher end of the normal range, as such there is no reason to discontinue
the Indapamide.

Next question

i
tag this question

previous question

next question

back to menu

finish session

jlfePasTest*

d*i1h

to

2014 PasTest Ltd. Egerton Court, Parkgate Estate, Knutsford, Cheshire, United Kingdom WA16 8DX. Tel: +44 (0) 1565 752000. Email: en

A 72-year-old man presents to the Cardiology Clinic for review. He has a history of central chest ache when he
goes out in the cold to walk the dog over the past 18 months. He also reports a minor chest "niggle" when he
walks up a steep hill near his home. He smokes 10 cigarettes per day and has done so for the past 50 years. He
has hypertension with a BP of 155/90 mmHg on treatment, and an LDL cholesterol of 3.9 mmol/l. Medication

includes Ramipril, Aspirin and Bisoprolol.

Which of the following is the optimal next step?

O Angiography
O Exercise test
Isosorbide dinitrate
Q Myocardial perfusion scan
Nebivulol

A 72-year-old man presents to the Cardiology Clinic for review. He has a history of central chest ache when he
goes out in the cold to walk the dog over the past 18 months. He also reports a minor chest "niggle" when he
walks up a steep hill near his home. He smokes 10 cigarettes per day and has done so for the past 50 years. He
has hypertension with a BP of 155/90 mmHg on treatment, and an LDL cholesterol of 3.9 mmol/l. Medication

includes Ramipril, Aspirin and Bisoprolol.

Which of the following is the optimal next step?

Angiography
Exercise test

Isosorbide dinitrate CORRECT ANSWER

Myocardial perfusion scan


Nebivulol

YOUR ANSWER WAS INCORRECT

Comment on this Question

The Answer

The answer is Isosorbide dinitrate


NICE guidance suggests in the presence of stable angina, with a typical history, coronary artery investigation

is not needed if the presence of coronary artery disease (CAD) is predicted to be 90% or higher. In this case,
given his typical history, age, smoking, hypercholestrolaemia and hypertension, his chest pain is almost
certainly related to CAD, as such the next step is therapy for his angina. Nebivulol is indicated for the
treatment of hypertension and heart failure. If coronary artery disease is suspected and the predicted risk of

CAD is less than 90%, myocardial perfusion scanning or angiography would be investigations of choice.
http://guidance.nice.org.uk/CG95/QuickRefGuide/pdf/English

Next question

tag this question

previous question

ilk PasTest'
:

or
next question

back to menu

finish session

2014 PasTest Ltd. Egerton Court, Parkgate Estate, Knutsford, Cheshire, United Kingdom WA16 8DX. Tel: +44 (0) 1565 752000. Email: enqu

A 62-year-old woman is treated with NSAIDs and long term Methotrexate for rheumatoid arthritis. She presents to
the clinic for review and complains of increasing nausea and indigestion. An additional finding is increased BP
(now 157/72 mmHg), and she is started on new medications for both problems by her doctor. A recent GFR is
measured at 28ml/min.

Which of the following medications should be used with caution in this situation?

Amlodipine

Q Bisoprolol

Gaviscon

O Omeprazole
O Ranitidine

A 62-year-old woman is treated with NSAIDs and long term Methotrexate for rheumatoid arthritis. She presents to
the clinic for review and complains of increasing nausea and indigestion. An additional finding is increased BP
(now 157/72 mmHg), and she is started on new medications for both problems by her doctor. A recent GFR is
measured at 28ml/min.

Which of the following medications should be used with caution in this situation?

Amlodipine
Bisoprolol

Gaviscon

Omeprazole CORRECT ANSWER


Ranitidine

YOUR ANSWER WAS INCORRECT

Comment on this Question

The Answer

The answer is Omeprazole


Methotrexate is a substrate for the OAT-1 renal transporter and levels of methotrexate are therefore affected
by decreased renal function. OAT-1 inhibitors include drugs such as Probenecid, and therefore should not

be used in conjunction with Methotrexate. Omeprazole is also known to affect clearance of Methotrexate, this
interaction is not thought to be via OAT-1, but is thought to be related to inhibition of breast cancer
resistance protein, which is responsible for Methotrexate transport.
The other options given are not thought to affect Methotrexate levels. In this situation with a GFR of 28ml/min
an alginate preparation or Ranitidine may therefore be better options for initial therapy for indigestion in this
situation.

Next question

tag this question

lljPasTest'
S ; *'

previous question

or
next question

back to menu

finish session

2014 PasTest Ltd. Egerton Court. Parkgate Estate. Knutsford, Cheshire, United Kingdom WA16 8DX. Tel: +44 (0) 1565 752000. Email: er

A 62-year-old man is reviewed the day after admission with an non ST elevation myocardial infarction (NSTEMI).
He has a history of smoking, hypertension and diabetes mellitus, and suffered a previous anterior NSTEMI 1 year

earlier. On this occasion he has inferior T wave inversion and his troponin rose to 5.2 at the 12hrs point. His 6
months CV risk is assessed at 4.5%.

Which of the following is the most appropriate next step with respect to investigations?

O Angiography within 96 hrs


Angiography within 6 weeks
O Exercise test
No further investigations needed at this stage

O Stress echo

A 62-year-old man is reviewed the day after admission with an non ST elevation myocardial infarction (NSTEMI).

He has a history of smoking, hypertension and diabetes mellitus, and suffered a previous anterior NSTEMI 1 year
earlier. On this occasion he has inferior T wave inversion and his troponin rose to 5.2 at the 12hrs point. His 6
months CV risk is assessed at 4.5%.

Which of the following is the most appropriate next step with respect to investigations?

Angiography within 96 hrs CORRECT ANSWER


Angiography within 6 weeks
Exercise test
No further investigations needed at this stage
Stress echo

YOUR ANSWER WAS INCORRECT

Comment on this Question

The Answer

The answer is Angiography within 96hrs


NICE guidance is very clear in this matter.

" Offer coronary angiography (with follow-on PCI if indicated) within 96 hours of first admission to hospital to
patients who have an intermediate or higher risk of adverse cardiovascular events (predicted 6-month
mortality above 3.0%) if they have no contraindications to angiography (such as active bleeding or
comorbidity). Perform angiography as soon as possible for patients who are clinically unstable or at high
ischaemic risk."
Patients who have a 6 month risk below 3% are suitable for conservative management. Exercise testing and
stress echocardiography are best reserved where the diagnosis of ischaemia related chest pain is in doubt.
http://egap.evidence.nhs.uk/unstable-angina-and-nstemi-cg94/guidance#assessment-of-a-patients-risk-of-

future-adverse-cardiovascular-events

Next question

tag this question

previous question

i' PasTest'
'

or
next question

back to menu

finish session

2014 PasTest Ltd. Egerton Court, Parkgate Estate, Knutsford, Cheshire, United Kingdom WA16 8DX. Tel: +44 (0) 1565 752000. Email: er

A 51-year-old man who has a history of smoking and Type 2 diabetes controlled with Metformin monotherapy
comes to the Emergency Department with 30 minutes of central crushing chest pain radiating to both arms.
Examination reveals a BP of 105/70 mmHg and a pulse of 95/min and regular. There are bilateral basal
crackles on auscultation of his chest although he is able to lie flat.

Investigations:

Hb

13 1 g/dl

WCC

10.1 X109/I

PLT

203 X109/I

Na+

137 mmol/l

K+

4.5 mmol/l

Creatinine

104 micromol/l

Glucose

18.2 mmol/l

ECG

Anterior ST elevation >2mm

CXR

No mediastinal widening

Which of the following is the most appropriate intervention whilst awaiting percutaneous coronary
intervention (PCI)?
C Asprin, Clopidogrel. LMW heparin

O Aspirin, Clopidogrel, Fondaparinux


O Aspirin, Clopidogrel, unfractionated heparin
O Aspirin, Clopidogrel, Bivalirudin

C Aspirin, Ticagrelor, Abciximab

A 51-year-old man who has a history of smoking and Type 2 diabetes controlled with Metformin monotherapy
comes to the Emergency Department with 30 minutes of central crushing chest pain radiating to both arms.

Examination reveals a BP of 105/70 mmHg and a pulse of 95/min and regular. There are bilateral basal crackles
on auscultation of his chest although he is able to lie flat.
Investigations;

Hb

13.1 g/dl

WCC

10.1 x 109/1

PLT

203x109/1

Na+

137 mmol/l

K+

4.5 mmol/l

Creatinine

104 micromol/l

Glucose

18.2 mmol/l

ECG

Anterior ST elevation >2mm

CXR

No mediastinal widening

Which of the following is the most appropriate intervention whilst awaiting percutaneous coronary
intervention (PCI)?

Asprin, Clopidogrel, LMW heparin


Aspirin, Clopidogrel, Fondaparinux
Aspirin, Clopidogrel, unfractionated heparin

Aspirin, Clopidogrel, Bivalirudin CORRECT ANSWER


Aspirin, Ticagrelor, Abciximab

YOUR ANSWER WAS INCORRECT

The Answer

Comment on this Question

The answer is Aspirin, Clopidogrel, Bivalirudin

This trio of medications is now the recommendation for patients whilst awaiting primary PCI (2013 NICE
guidelines on acute Ml). If PCI cannot be performed within 120 minutes of a diagnosis of STEMI being made,
then thrombolysis is recommended as an alternative. Fondaparinux is the anti-coagulant of choice in
patients diagnosed with NSTEMI. Ticagrelor can be used as an alternative anti-platelet option post STEMI and
then continued for up to 12 months. Ilbllla inhibitors are not recommended as part of routine therapy for

patients with STEMI.


http://www.nice.org.uk/nicemedia/live/14208/64410/64410.pdf

Next question

i
tag this question

ilk PasTest'
: <*'

previous question

or
next question

back to menu

finish session

2014 PasTest Ltd. Egerton Court, Parkgate Estate, Knutsford, Cheshire, United Kingdom WA16 8DX. Tel: +44 (0) 1565 752000. Email: enq

A 61-year-old woman with a history of ischaemic heart disease and COPD comes to the Emergency Department. She
tells you that she has suffered a number of episodes of palpitations and thinks she is about to faint. In the past few
days she has started a course of erythromycin for an exacerbation of COPD. Examination reveals a BP of 105/60, pulse
is 75 and regular. There is coarse wheeze on auscultation of the chest. Whilst you are listening to her chest you notice
a self-terminating short period (15 seconds) of torsades de pointes.
Investigations (venous blood gas result);

Hb

12.9 g/dl

WCC

11.2x109/1

PLT

281 x 109/1

Na+

1 37 mmol/l

K+

3.9 mmol/l

Bicarbonate 23 mmol/l
Creatinine

121 micromol/1

Which of the following is the most appropriate intervention?

O IV Lignocaine
O IV Magnesium
O IV Isoprenaline
O IVAmiodarone
O IV Potassium

A 61-year-old woman with a history of ischaemic heart disease and COPD comes to the Emergency Department.
She tells you that she has suffered a number of episodes of palpitations and thinks she is about to faint. In the
past few days she has started a course of erythromycin for an exacerbation of COPD. Examination reveals a BP

of 105/60, pulse is 75 and regular. There is coarse wheeze on auscultation of the chest. Whilst you are listening
to her chest you notice a self-terminating short period (15 seconds) of torsades de pointes.

Investigations (venous blood gas result);

Hb

12.9 g/dl

WCC

11.2 X 109/1

PLT

281 X 109/1

Na+

137 mmol/l

K+

3.9 mmol/l

Bicarbonate 23 mmol/l

Creatinine

121 micromol/l

Which of the following is the most appropriate intervention?

IV Lignocaine
IV Magnesium CORRECT ANSWER

IV Isoprenaline
IV Amiodarone
IV Potassium

YOUR ANSWER WAS INCORRECT

Comment on this Question

The Answer

The answer is IV Magnesium

It is likely this woman has a degree of QT prolongation as a result of her ischaemic heart disease, and that
this has been further exacerbated by the use of erythromycin. In this situation IV Magnesium is of value in
preventing further episodes of torsades, even if serum magnesium levels are normal. Conventional anti
arrhythmics such as Amiodarone and Lignocaine worsen the likelihood of torsades in this situation. IV
Isoprenaline to keep the ventricular rate above 90/min is an interim option when overdrive pacing is planned
for resistant torsades.
Drugs known to increase the risk of torsades include class la and III antiarrhythmic agents, Erythromycin,
Ketoconazole, Tricyclic antidepressants and antipsychotics.

Next question

tag this question

previous question

illPasTest"
'

nv<ji(..udio>cur>u(.-

or
next question

back to menu

finish session

2014 PasTest Ltd. Egerton Court, Parkgate Estate, Knutsford, Cheshire, United Kingdom WA16 8DX. Tel: +44 (0) 1565 752000. Email: enq

A 49-year-old woman comes to the Cardiology Clinic for review. She has been investigated for palpitations and is
coming to the clinic to get her results. A past history of asthma managed with high dose Seretide and Monteleukast is
noted. She is now a non-smoker. Examination in the clinic reveals a BP of 135/80 mmHg, pulse is 75/min, sinus rhythm.

Her chest is clear apart from some mild wheezing.

Investigations:

Hb

13.5 g/dl

WCC

7.4x109/1

PLT

197 x 109/1

Na+

1 38 mmol/l

K+

4.5 mmol/l

Creatinine 110 micromol/l


ECHO

Normal ejection fraction, no structural heart disease


Multiple episodes of AF. the longest for a period of 15 minutes during the night

24hrtape

Which of the following is the most appropriate intervention?


C Amiodarone
O Bisoprolol

G Digoxin
O Flecainide

G Verapamil

A 49-year-old woman comes to the Cardiology Clinic for review. She has been investigated for palpitations and is
coming to the clinic to get her results. A past history of asthma managed with high dose Seretide and
Monteleukast is noted. She is now a non-smoker. Examination in the clinic reveals a BP of 135/80 mmHg, pulse is
75/min, sinus rhythm. Her chest is clear apart from some mild wheezing.

Investigations;

Hb

13.5 g/dl

WCC

7.4x109/1

PLT

197x109/1

Na+

138 mmol/l

K+

4.5 mmol/l

Creatinine 110 micromol/l


ECHO

Normal ejection fraction, no structural heart disease


Multiple episodes of AF, the longest for a period of 15 minutes during the night

24hr tape

Which of the following is the most appropriate intervention?

Amiodarone
Bisoprolol

Digoxin
Flecainide CORRECT ANSWER

Verapamil

YOUR ANSWER WAS INCORRECT

Comment on this Question

The Answer

The answer is Flecainide

Whilst Bisoprolol remains an option in COPD (since studies show that the effect on lung function is very
modest and potentially sub-clinical), in severe asthma it is more appropriate to trial alternatives first.
Flecainide is a class 1C option in patients without structural or ischaemic heart disease and is therefore the
first choice here. Due to long-term adverse effects Amiodarone would remain a second or third line agent
after Flecainide, then Bisoprolol. Digoxin and Verapamil are most effective as rate control agents.

Next question

tag this question

previous question

iSiPasTest'
;

Dv<iii

to

or
next question

back to menu

finish session

2014 PasTest Ltd. Egerton Court. Parkgate Estate Knutsford, Cheshire, United Kingdom WA16 8DX. Tel: +44 (0) 1565 752000. Email: enc

A 65-year-old man presents to the Emergency Department with a history of 40 minutes of central crushing
chest pain radiating to his left arm some 8hrs earlier. He suffered a previous inferior Ml some 3 years

earlier, has hypertension managed with two oral agents, and smokes 5 cigars per day. Examination reveals a
BP of 105/60 mmHg, pulse is 75/min and regular. There are bilateral basal crackles on auscultation of the
chest.

Investigations:

Hb

13.1 g/dl

WCC

9.1 X109/I

PLT

203 X109/I

Na+

137 mmol/l

K+

4.3 mmol/l

Creatinine 132 micromol/l


Glucose

9.3 mmol/l

Troponin

1.5

ECG

Infero-lateral T wave inversion

Which of the following is the most appropriate next step?

G Aspirin, Clopidogrel, Fondaparinux


O Aspirin, Prasugrel, LMW heparin

G Aspirin, Clopidogrel, Bivalirudin

O Aspirin, Clopidogrel, LMW heparin


G Aspirin, Prasugrel, Fondaparinux

A 65-year-old man presents to the Emergency Department with a history of 40 minutes of central crushing
chest pain radiating to his left arm some 8hrs earlier. He suffered a previous inferior Ml some 3 years earlier,
has hypertension managed with two oral agents, and smokes 5 cigars per day. Examination reveals a BP of
105/60 mmHg, pulse is 75/min and regular. There are bilateral basal crackles on auscultation of the chest.

Investigations;

Hb

13.1 g/dl

WCC

9.1 X109/I

PLT

203x109/1

Na+

137 mmol/l

K+

4.3 mmol/l

Creatinine 132 micromol/l


Glucose

9.3 mmol/l

Troponin

1.5

ECG

Infero-lateral T wave inversion

Which of the following is the most appropriate next step?

Aspirin, Clopidogrel, Fondaparinux CORRECT ANSWER

Aspirin, Prasugrel, LMW heparin


Aspirin, Clopidogrel, Bivalirudin
Aspirin, Clopidogrel, LMW heparin
Aspirin, Prasugrel, Fondaparinux

YOUR ANSWER WAS INCORRECT

The Answer

Comment on this Question

The anwer is Aspirin, Clopidogrel, Fondaparinux

This patient has an elevated troponin indicating an NSTEMI and minor ECG changes. The fact he is now
pain free and has been for the last 8hrs, means that angiography in the next 24hrs is unlikely and for this

reason Aspirin, Clopidogrel and Fondaparinux is the recommended regimen. Bivalirudin is considered in
patients likely to undergo angiography within 24hrs. Prasugrel is considered for STEMI and where
patients have a history of diabetes mellitus.
http://www.nice.org.uk/nicemedia/live/12949/47924/47924.pdf

Average

tag this question

tflEMest'
.*

o*diuti to >oui *wKti

* *

previous question

next question

o r

back to menu

finish session

2014 PasTest Ltd. Egerton Court. Parkgate Estate Knutsford. Cheshire. United Kingdom WA16 8DX. Tel: +44 (0) 1565 752000

A 72-year-old woman presents to the Rapid Access Chest Pain Clinic with central chest pain which comes on when she
is walking her dog on a cold day, and when she carries her hoover up two flights of stairs at home to do the cleaning.
Her episodes of pain have worsened considerably over the last 2 months. She is a non-smoker who has a history of
hypertension for which she takes ramipril 5mg daily and has Type 2 diabetes for which she takes Metformin 1g daily.
On examination her BP is 135/70 mmHg, pulse is 80/min and regular. Her chest is clear.
Investigations:

Hb

13.0 g/dl

WCC

7.2x109/1

PLT

271 x 109/1

Na+

138 mmol/l

K+

4.3 mmol/l

Creatinine 110 micromol/l


ECG

Small inferior Q waves, nil else of note

Which of the following is the most appropriate next step?


C Angiography
O Exercise ECG
O Myocardial perfusion scan

C Stress ECHO
O Start a long-acting nitrate

A 72-year-old woman presents to the Rapid Access Chest Pain Clinic with central chest pain which comes on
when she is walking her dog on a cold day, and when she carries her hoover up two flights of stairs at home to
do the cleaning. Her episodes of pain have worsened considerably over the last 2 months. She is a non-smoker
who has a history of hypertension for which she takes ramipril 5mg daily and has Type 2 diabetes for which she
takes Metformin 1g daily. On examination her BP is 135/70 mmHg, pulse is 80/min and regular. Her chest is clear
Investigations;

Hb

13.0 g/dl

WCC

7.2 x 109/1

PLT

271 x 109/1

Na+

138 mmol/l

K+

4.3 mmol/l

Creatinine 110 micromol/l


ECG

Small inferior Q waves, nil else of note

Which of the following is the most appropriate next step?

Angiography CORRECT ANSWER


Exercise ECG

Myocardial perfusion scan


Stress ECHO
Start a long-acting nitrate
YOUR ANSWER WAS INCORRECT

The Answer

Comment on this Question

The answer is Angiography

NICE guidelines recommend that when likelihood of coronary artery disease is >90%, angiography is the most

appropriate next step. In this case, for a woman over 70 who has risk factors with typical symptoms,
likelihood >90% should be assumed. When estimated risk is between 61 and 90%, angiography is also
recommended. For 30-60% likelihood functional imaging should take place, and for 10-29% CT calcium
scoring. Assuming stable angina and progressing straight to therapy without investigation is not
recommended under current guidelines.
http://www.nice.org.uk/nicemedia/live/12947/47938/47938.pdf

Next question

<

tag this question

previous question

or
next question

back to menu

finish session

#%Paslest"
nv.i

to

2014 PasTest Ltd. Egerton Court, Parkgate Estate. Knutsford, Cheshire, United Kingdom WA16 8DX. Tel: +44 (0) 1565 752000. Email:

A 55-year-old woman, who is a non-smoker, presents with rib pain. A bone scan shows multiple lesions highly
suggestive of metastases. Clinical examination is normal apart from unilateral axillary lymphadenopathy.
Excision biopsy of an affected lymph node shows adenocarcinoma.

Which investigation should be prioritised in finding the site of the primary?

O Ca125
Chest X-ray

O Gastroscopy
Mammography

O Colonoscopy

A 55-year-old woman, who is a non-smoker, presents with rib pain. A bone scan shows multiple lesions highly

suggestive of metastases. Clinical examination is normal apart from unilateral axillary lymphadenopathy.
Excision biopsy of an affected lymph node shows adenocarcinoma.

Which investigation should be prioritised in finding the site of the primary?


Ca125
Chest X-ray

Gastroscopy
Mammography CORRECT ANSWER

Colonoscopy

YOUR ANSWER WAS INCORRECT

The Answer

Comment on this Question

Finding primary tumours


Cancers of an unknown primary site account for around 3-10% of all neoplasms
Common sites of presentation include bones, lymph nodes, lungs and liver

If the site of presentation is the axillary lymph node(s) this can often be as a result of an occult breast primary, and
therefore mammography should be the first examination of choice
If the mammogram is negative, the other investigations listed may identify alternative occult sites
Identifying the primary is useful even in metastatic disease as it guides treatment and gives an idea of prognosis

Next question

i
Easy

tag this question

ilk PasTest'
'

[><! ..t.d to

mum

* *

previous question

next question

t
back to menu

finish session

2014 PasTest Ltd. Egerton Court Parkgate Estate, Knutsford, Cheshire, United Kingdom WA16 8DX. Tel: +44 (0) 1565 752000. Email: enq

A 58-year-old businessman presents with bilateral leg weakness that has suddenly become worse over the last
12 hours. Some 10 months ago he had a lobar resection for a stage-ll squamous-cell carcinoma followed by

radiotherapy and adjuvant chemotherapy. He has had a dull ache in his lower back over the past three months,
but put this down to arthritis from when he used to play rugby as a young man. On examination there is reduced

power and altered sensation in both legs.

Which one of the following is the most likely cause of the current problem?
Q Peripheral neuropathy secondary to carcinomatosis

O Paraneoplastic myelopathy
O Spinal cord compression as a result of vertebral metastases
O Secondary spinal tumour deposit
Spinal tuberculosis

A 58-year-old businessman presents with bilateral leg weakness that has suddenly become worse over the last
12 hours. Some 10 months ago he had a lobar resection for a stage-ll squamous-cell carcinoma followed by
radiotherapy and adjuvant chemotherapy. He has had a dull ache in his lower back over the past three months,
but put this down to arthritis from when he used to play rugby as a young man. On examination there is reduced
power and altered sensation in both legs.

Which one of the following is the most likely cause of the current problem?
Peripheral neuropathy secondary to carcinomatosis
Paraneoplastic myelopathy
Spinal cord compression as a result of vertebral metastases CORRECT ANSWER

Secondary spinal tumour deposit


Spinal tuberculosis

YOUR ANSWER WAS INCORRECT

The Answer

Comment on this Question

Bilateral leg weakness


Cause
The sudden onset of bilateral leg weakness and loss of sensation is highly suggestive of spinal cord
compression, which is an oncological emergency

Presentation of spinal cord compression


Other common presenting symptoms are back pain, urinary retention and constipation
The most common cancers giving rise to spinal cord compression are
breast
bronchus
prostate
multiple myeloma
a high-grade, non-Hodgkin's lymphoma
Although the patients are often known to have metastatic disease, it can sometimes be a presenting feature

Investigations in spinal cord compression


Investigations should include a plain radiograph of the spine (which may be normal) and magnetic resonance
imaging

Treatment of spinal cord compression


Treatment should be instigated immediately with IV dexamethasone
Neurosurgery or radiotherapy should follow

Other notes

The time course is too rapid for a peripheral neuropathy


There is no history to suggest spinal TB

Paraneoplastic myelopathy is rare and usually associated with small-cell lung cancer
A secondary spinal tumour deposit would present in a similar fashion but is less common

Next question

i
Easy

tag this question

ilk PasTest'
:

previous question

next question

back to menu

finish session

2014 PasTest Ltd. Egerton Court, Parkgate Estate, Knutsford, Cheshire, United Kingdom WA16 8DX. Tel: +44 (0) 1565 752000. Email: er

A 66-year-old woman with a history of ischaemic heart disease presents with an acute onset of breathlessness
and chest pain. Earlier that day she had commenced adjuvant chemotherapy (5-fluorouracil and folinic acid) for a
completely resected caecal carcinoma.

Which one of the following is the most likely precipitating factor?

O Rib metastasis
Side-effect of folinic acid

O Pericardial effusion due to metastases


O Side-effect of 5-fluorouracil
O Stress related

A 66-year-old woman with a history of ischaemic heart disease presents with an acute onset of breathlessness
and chest pain. Earlier that day she had commenced adjuvant chemotherapy (5-fluorouracil and folinic acid) for a
completely resected caecal carcinoma.

Which one of the following is the most likely precipitating factor?


Rib metastasis

Side-effect of folinic acid

Pericardial effusion due to metastases


Side-effect of 5-fluorouracil CORRECT ANSWER

Stress related
YOUR ANSWER WAS INCORRECT

The Answer

Comment on this Question

5-Fluorouracil
5-Fluorouracil can sometimes precipitate angina attacks in individuals with ischaemic heart disease and it can also
cause tachyarrhythmias
The complications should be managed in the usual way and should resolve reasonably quickly once the drug is
stopped due to its relatively short half-life
Depending on the severity, further cycles may be tried at reduced doses or alternative drugs should be given

l\

Other notes
Folinic acid (which is usually given with 5-fluorouracil) is not cardiotoxic
Metastases would be less likely given the history and the fact that the tumour was completely resected
Stress could obviously be a precipitating factor but other causes should be excluded first

Next question

i
Easy

tag this question

PasTest
rNMiict.dtoyou.vi.

previous question

next question

O
back to menu

finish session

2014 PasTest Ltd. Egerton Court. Parkgate Estate. Knutsford, Cheshire. United Kingdom WA16 8DX. Tel: +44 (0) 1565 752000. Email: enqui

In cancer therapy, what is the rationale behind using combinations of chemotherapeutic agents
rather than single agents?

O Fewer side-effects occur in combination therapy


O Combination therapy can be given over a shorter period of time
Combination therapy decreases the chances of drug resistance developing

O Metastases are less common in combination therapy


Combination therapy is less likely to result in long-term toxicity

In cancer therapy, what is the rationale behind using combinations of chemotherapeutic agents
rather than single agents?

Fewer side-effects occur in combination therapy


Combination therapy can be given over a shorter period of time
Combination therapy decreases the chances of drug resistance developing CORRECT
ANSWER

Metastases are less common in combination therapy

Combination therapy is less likely to result in long-term toxicity


YOUR ANSWER WAS INCORRECT

Comment on this Question

The Answer

Combination chemotherapy
There are two main reasons for using combinations of chemotherapeutic agents
different drugs exert their effects through different mechanisms, so carefully combining them will increase the
number of tumour cells killed in each cycle
secondly, there may be an even greater effect with drugs that are synergistic
As cells might be killed through a number of different mechanisms they are less likely to develop resistance
Drug combinations have to be chosen carefully otherwise the side-effects and long-term toxicity could be
potentially more severe, eg if two drugs both had significant hepatotoxicity
Combination therapy may not affect the duration of therapy and sometimes it turns out to be less effective, so
metastases would not necessarily be less common

Next question

i
tag this question

- k PasTest
lo

MH

+ +

previous question

next question

Q
back to menu

finish session

2014 PasTest Ltd. Egerton Court, Parkgate Estate. Knutsford, Cheshire, United Kingdom WA16 8DX. Tel: +44 (0) 1565 752000. Email: en

A 72-year-old heavy smoker presents with shortness of breath and haemoptysis. On examination you notice

some facial swelling. You suspect a bronchial neoplasm.

What other clinical sign would it be particularly important to look for if you were suspecting SVC
obstruction?
Venous dilatation over the anterior chest wall

Q Supraclavicular or cervical lymphadenopathy

Finger clubbing

O Horner's syndrome
O Central cyanosis

A 72-year-old heavy smoker presents with shortness of breath and haemoptysis. On examination you notice

some facial swelling. You suspect a bronchial neoplasm.

What other clinical sign would it be particularly important to look for if you were suspecting SVC
obstruction?
Venous dilatation over the anterior chest wall

CORRECT ANSWER

Supraclavicular or cervical lymphadenopathy

Finger clubbing
Horner's syndrome
Central cyanosis
YOUR ANSWER WAS INCORRECT

Comment on this Question

The Answer

Superior vena cava obstruction


It seems very likely that there is an underlying bronchial neoplasm in this 72-year-old smoker with shortness of
breath and haemoptysis
However, the facial swelling should alert you to the possibility of a superior vena cava obstruction (SVCO)
In this respect, venous dilatation over the anterior chest wall is another sign that might be present
SVCO is an oncological emergency and needs to be treated rapidly with steroids once the diagnosis is confirmed
Thereafter, treatment depends on the underlying cause
About 70% of cases of SVCO are due to lung cancer, but other malignant causes include lymphomas, leukaemias
and germ-cell tumours

Other notes
The other answers describe clinical signs that might all occur in patients with lung cancer, but they would be less
useful in deciding whether or not an SVCO was present

Next question

Easy

tag this question

tPasTest'
Dvdi(<il*d lo )oui w(c*u

previous question

or
next question

back to menu

finish session

2014 PasTest Ltd. Egerton Court. Parkgate Estate.. Knutsford. Cheshire, United Kingdom WA16 8DX. Tel: +44 (0) 1565 752000. Email: enqi

A 57-year-old woman is referred to you as an emergency with severe nose bleeds and skin petechiae. You
suspect she has thrombocytopenia. She is currently receiving second-line carboplatin-based chemotherapy for a

relapsed ovarian carcinoma. There is no organomegaly.

Which one of the following is the most likely cause of the thrombocytopenia?

Myelosuppression
Q Carcinomatosis

Disseminated intravascular coagulation

O Secondary leukaemia
O Myelofibrosis

A 57-year-old woman is referred to you as an emergency with severe nose bleeds and skin petechiae. You
suspect she has thrombocytopenia. She is currently receiving second-line carboplatin-based chemotherapy for a
relapsed ovarian carcinoma. There is no organomegaly.

Which one of the following is the most likely cause of the thrombocytopenia?

Myelosuppression CORRECT ANSWER


Carcinomatosis

Disseminated intravascular coagulation


Secondary leukaemia
Myelofibrosis

YOUR ANSWER WAS INCORRECT

The Answer

Comment on this Question

Side-effects of chemotherapy
Platinum-based chemotherapy is the mainstay of treatment for ovarian cancer, the fifth commonest cancer in
women

It is usually given as part of a combination therapy in first-line treatment, but may be given again at relapse if there
was a reasonable duration of initial response

The main side-effects of cisplatin are


nausea and vomiting
neuropathy
deafness
renal failure
mild bone marrow suppression
Carboplatin, on the other hand, causes a more severe myelosuppression, particularly thrombocytopenia
Myelosuppression is also more common in individuals who have already had previous courses of myelotoxic
chemotherapy

Other notes
Carcinomatosis and disseminated intravascular coagulation are both possible differential diagnoses, although they
are less likely

The time frame for secondary leukaemia is too short, and, while myelofibrosis can present with thrombocytopenia,
it is typically associated with splenomegaly

Next question

i
Easy

tag this question

PasTest"
im
:
'

* *

previous question

next question

back to menu

finish session

2014 PasTest Ltd. Egerton Court, Parkgate Estate. Knutsford, Cheshire United Kingdom WA16 8DX. Tel: +44 (0) 1565 752000. Email: em

A 45-year-old woman who is currently 4 weeks into a course of postoperative radiotherapy for locally advanced

cervical carcinoma is admitted with abdominal pain and diarrhoea.

Which one of the following is the most likely cause of the clinical picture?

O Complication of surgery
O Radiation enteritis
O Bowel obstruction
O Local malignant infiltration
Q Bowel perforation

A 45-year-old woman who is currently 4 weeks into a course of postoperative radiotherapy for locally advanced

cervical carcinoma is admitted with abdominal pain and diarrhoea.

Which one of the following is the most likely cause of the clinical picture?
Complication of surgery

Radiation enteritis CORRECT ANSWER


Bowel obstruction
Local malignant infiltration
Bowel perforation
YOUR ANSWER WAS INCORRECT

Comment on this Question

The Answer

Radiation enteritis
Radiation enteritis is a radiation-induced inflammation of the bowel and is a function of the volume of bowel
irradiated and the radiation dose
The acute type occurs during therapy and manifests as ileitis, colitis or proctitis with abdominal pain and diarrhoea
Usually conservative management with stool softeners suffices
Late radiation enteritis occurs 6-24 months after therapy and may present with colicky abdominal pain and
intermittent diarrhoea

Other notes
The clinical picture is unlikely to be due to a surgical complication given the time frame, and is less suggestive of
bowel obstruction or perforation
Local malignant infiltration to the bowel is most likely to present with obstruction

Next question

Easy

tag this question

PasTest"
im
: *'
'

.itd 10

u( i

or
previous question

next question

back to menu

finish session

2014 PasTest Ltd. Egerton Court, Parkgate Estate. Knutsford, Cheshire. United Kingdom WA16 8DX. Tel: +44 (0) 1565 752000. Email: enc

A 40-year-old man presents with symptoms and signs of anaemia. He has also recently had recurrent infections

that have taken longer than usual to resolve, and he has frequent nose bleeds. There is no organomegaly. Some
15 years ago he had chemotherapy and radiotherapy for Hodgkin's disease.

Which haematological condition would you be particularly concerned about?

O Relapse of Hodgkin's disease


O A new primary lymphoma
O A secondary acute myeloid leukaemia
O Multiple myeloma
Q Myelofibrosis

A 40-year-old man presents with symptoms and signs of anaemia. He has also recently had recurrent infections
that have taken longer than usual to resolve, and he has frequent nose bleeds. There is no organomegaly. Some
15 years ago he had chemotherapy and radiotherapy for Hodgkin's disease.

Which haematological condition would you be particularly concerned about?


Relapse of Hodgkin's disease
A new primary lymphoma

A secondary acute myeloid leukaemia CORRECT ANSWER


Multiple myeloma

Myelofibrosis

YOUR ANSWER WAS INCORRECT

The Answer

Comment on this Question

Secondary acute myeloid leukaemia


Secondary acute myeloid leukaemia (AML) is a late complication of the chemotherapy and radiotherapy given in

Hodgkin's disease
Secondary AML has a poor prognosis and is often refractory to treatment

Other notes
The presentation, which is suggestive of bone marrow failure, would be unusual for a lymphoma, although the
marrow can be involved in advanced disease
The patient would be rather young for multiple myeloma
The absence of an enlarged spleen makes a diagnosis of myelofibrosis unlikely

Next question

<

tag this question

previous question

is PasTest'
m
.ll'ijlo

!**

or
next question

back to menu

finish session

2014 PasTest Ltd. Egerton Court. Parkgate Estate. Knutsford, Cheshire, United Kingdom WA16 8DX. Tel: +44 (0) 1565 752000. Email: enqi

A relative of a patient of yours who has metastatic cancer asks your opinion about a new experimental cancer

treatment that works by cutting off the tumour blood supply.

He is referring to which group of drugs?

O Angiogenesis inhibitors
O Interferons

0 Monoclonal antibodies
O Taxanes

0 Matrix metalloproteinase inhibitors

A relative of a patient of yours who has metastatic cancer asks your opinion about a new experimental cancer
treatment that works by cutting off the tumour blood supply.

He is referring to which group of drugs?

Angiogenesis inhibitors CORRECT ANSWER


Interferons

Monoclonal antibodies
Taxanes
Matrix metalloproteinase inhibitors

YOUR ANSWER WAS INCORRECT

The Answer

Comment on this Question

Anti-cancer drugs
Angiogenesis inhibitors
Angiogenesis inhibitors have been shown to treat cancer in mice by halting tumour growth and stopping the
formation of new blood vessels in these tumours

For a tumour to grow it must first induce the creation of new blood vessels by a process called 'angiogenesis', and
if this is halted then the tumour cannot grow
Trials of this type of drug are currently underway in humans

Interferons
Interferons are used in immunotherapy for certain cancers such as kidney cancer and myeloma
The main side-effects are malaise and flu-like symptoms

Monoclonalantibodies
Monoclonal antibodies are artificial antibodies against a particular target (the antigen) and are produced in the
laboratory

Monoclonal antibodies are used in cancer diagnosis and therapy


A monoclonal antibody called rituximab (MabThera) can be useful in the treatment of non-Hodgkin's lymphoma,
while trastuzumab (Herceptin) is useful against certain breast cancers

Taxanes
Taxanes are well-established chemotherapeutic agents that block cell division by inhibiting formation of the mitotic
spindle
Taxanes are used extensively in the treatment of breast and ovarian cancer

Matrix metalloproteinases
Matrix metalloproteinases inhibit the proteases that cancer cells produce to facilitate detachment from the primary
tumour, invasion of the bloodstream and growth at distant sites

They are currently being tested in clinical trials

Next question

Easy

tag this question

lljPasTest'
S ; *'

or
previous question

next question

back to menu

finish session

2014 PasTest Ltd. Egerton Court. Parkgate Estate. Knutsford, Cheshire, United Kingdom WA16 8DX. Tel: +44 (0) 1565 752000. Email: enqi

An elderly man is brought to the Emergency Department by paramedic ambulance after a collapse.
He is known to have ischaemic heart disease. Peripheral pulses are absent and ECG reveals widecomplex tachycardia. He has apparently been conversant with the ambulance staff, but is now very
sleepy. You manage to elicit a carotid pulse which seems to be around 200 bpm, and the electronic

BP machine measures his BP as 70/40 mmHg. Immediate management consists of which one of the
following?

0 Programmed stimulation
O Thrombolysis with TPA or streptokinase
O A bolus dose of intravenous lidocaine
O DC cardioversion
Intravenous phenylephrine and carotid sinus pressure

An elderly man is brought to the Emergency Department by paramedic ambulance after a collapse.
He is known to have ischaemic heart disease. Peripheral pulses are absent and ECG reveals widecomplex tachycardia. He has apparently been conversant with the ambulance staff, but is now very

sleepy. You manage to elicit a carotid pulse which seems to be around 200 bpm, and the electronic
BP machine measures his BP as 70/40 mmHg. Immediate management consists of which one of the
following?

Programmed stimulation
Thrombolysis with TPA or streptokinase
A bolus dose of intravenous lidocaine
DC cardioversion CORRECT ANSWER

Intravenous phenylephrine and carotid sinus pressure


YOUR ANSWER WAS INCORRECT

The Answer

Comment on this Question

Ventricular tachycardia
The patient has ventricular tachycardia and is haemodynamically unstable
This has caused loss of consciousness

Treatment
Immediate treatment consists of cardioversion followed by a suitable anti-arrhythmic such as lidocaine IV or
amiodarone and correction of any electrolyte imbalance
Myocardial infarction (Ml) may be the cause of the ventricular tachycardia, hence treatment for Ml would be
indicated if required
Programmed stimulation may only be carried out when the patient is stable

A 60-year-old man with unstable angina on long-term digoxin was being monitored on the ward

with telemetry, when the monitor displayed a tachycardia of 180 beats per minute. The printout
showed discrete normal morphology P waves before each QRS complex and there was an

acceleration in the rate after initiation of the arrhythmia. The QRS width was 0.12 s. Which one of
the following is the most likely arrhythmia?

O Automatic supraventricular tachyarrhythmias


AV nodal re-entrant tachycardia
Bypass tract-mediated macroentrant tachycardia

O Intra-atrial re-entry
O Ventricular tachycardia

A 60-year-old man with unstable angina on long-term digoxin was being monitored on the ward

with telemetry, when the monitor displayed a tachycardia of 180 beats per minute. The printout
showed discrete normal morphology P waves before each QRS complex and there was an
acceleration in the rate after initiation of the arrhythmia. The QRS width was 0.12 s. Which one of
the following is the most likely arrhythmia?
Automatic supraventricular tachyarrhythmias CORRECT ANSWER

AV nodal re-entrant tachycardia


Bypass tract-mediated macroentrant tachycardia
Intra-atrial re-entry
Ventricular tachycardia
YOUR ANSWER WAS INCORRECT

The Answer

Comment on this Question

Diagnosing automatic supraventricular arrhythmia


Automatic supraventricular arrhythmias characteristically show a warm-up phenomenon, ie the rate accelerates
after its initiation

AV nodal re-entrant tachycardia, bypass tract-mediated macroentrant tachycardia and intra-atrial re-entry are all
types of re-entrant supraventricular arrhythmias

In AV nodal re-entrant tachycardia, P waves are usually of abnormal morphology (inverted); in contrast, there are
discrete P waves in intra-atrial re-entry but there is no warm-up phenomenon
The normal QRS width rules out a ventricular tachycardia

A 30-year-old man with known hypertrophic obstructive cardiomyopathy (HOCM) presents to the Emergency
Department with an episode of witnessed collapse: a passer-by provided initial resuscitation when he felt no

pulse and the man was making no effort to breathe. On admission he is unwell with pulse rate of 160 bpm, blood
pressure 70/40 mmHg and decreased conscious level. ECG confirms ventricular tachycardia. Sinus rhythm is
restored with a DC shock.

What would be the most appropriate strategy for the long term?

O Amiodarone
O Automatic implantable cardioverter defibrillator
O Dual-chamber pacemaker
O Sotalol
Verapamil

A 30-year-old man with known hypertrophic obstructive cardiomyopathy (HOCM) presents to the Emergency
Department with an episode of witnessed collapse: a passer-by provided initial resuscitation when he felt no

pulse and the man was making no effort to breathe. On admission he is unwell with pulse rate of 160 bpm, blood
pressure 70/40 mmHg and decreased conscious level. ECG confirms ventricular tachycardia. Sinus rhythm is
restored with a DC shock.

What would be the most appropriate strategy for the long term?
Amiodarone
Automatic implantable cardioverter defibrillator CORRECT ANSWER

Dual-chamber pacemaker
Sotalol
Verapamil

YOUR ANSWER WAS INCORRECT

The Answer

Comment on this Question

Long-term management of hypertrophic obstructive


cardiomyopathy after cardiac arrest
This man has survived an out-of-hospital cardiac arrest and therefore an automatic implantable cardioverter
defibrillator (AICD) is warranted

Overall, patients with HOCM have an annual mortality rate of around 1%

Risk factors
Identifying those at greatest risk of sudden cardiac death (SOD) is challenging
Several factors have been identified that are associated with an increased risk
maximum wall thickness > 30 mm
non-sustained ventricular tachycardia on a 48-hour tape
a history of SCD in a relative under 45 years of age and a history of syncope
resting, left ventricular outflow-tract gradient > 30 mmHg
abnormal blood-pressure response to exercise

Although a single risk factor does not, on its own, have a particularly high positive-predictive accuracy, the
presence of two or more risk factors does identify a much higher risk population

Treatment
Dual-chamber pacing, (B-blockers or verapamil may be used to reduce symptoms in patients with a left ventricular
outflow-tract obstruction

A 60-year-old woman presents with exertional dyspnoea on rushing upstairs or going to the local

shops. She is in sinus rhythm at 80 bpm and her blood pressure is 160/80 mmHg, without evidence
of fluid overload. She is already taking the maximum dose of an angiotensin-converting enzyme
inhibitor and frusemide 40 mg once daily for long-standing hypertension. Echo confirms
significantly impaired left ventricular systolic function (ejection fraction 35%). Her coronary arteries
and renal function are normal. What additional medication should be considered for symptomatic
and prognostic benefit?

O Amiodarone
Amlodipine

O Bisoprolol
O Isosorbide mononitrate
O Losartan

A 60-year-old woman presents with exertional dyspnoea on rushing upstairs or going to the local
shops. She is in sinus rhythm at 80 bpm and her blood pressure is 160/80 mmHg, without evidence

of fluid overload. She is already taking the maximum dose of an angiotensin-converting enzyme
inhibitor and frusemide 40 mg once daily for long-standing hypertension. Echo confirms
significantly impaired left ventricular systolic function (ejection fraction 35%). Her coronary arteries
and renal function are normal. What additional medication should be considered for symptomatic
and prognostic benefit?

Amiodarone
Amlodipine
Bisoprolol CORRECT ANSWER

Isosorbide mononitrate
Losartan
YOUR ANSWER WAS INCORRECT

The Answer

Comment on this Question

Treating chronic heart failure

This woman has chronic heart failure (CHF) with compatible symptoms and objective evidence of left ventricular

dysfunction at rest

There is overwhelming evidence for the long-term prognostic and symptomatic benefit of an angiotensin-converting
enzyme (ACE) inhibitor in patients with CHF
Recent trials support the prognostic and symptomatic benefit of certain (3-blockers in CHF patients in addition to
ACE inhibitors

In the UK, bisoprolol and carvedilol are the two licensed agents for use in this condition
trials of both bisoprolol and carvedilol suggest a reduction of around 30% in mortality, particularly in patients
with severe cardiac failure
Isosorbide mononitrate and amlodipine are safe to use in patients with CHF either for symptomatic treatment of
angina or associated hypertension
there is no evidence that they influence outcome

A 54-year-old man is 48 h post-myocardial infarction. You are asked to review him as he is suffering worsening

cardiac failure. On examination he has a pansystolic murmur, loudest at the apex.

Which one of the following complications of his myocardial infarction is most likely to have occurred?

O Ventricular septal defect


O Atrial septal defect

0 Acute mitral regurgitation


O Acute pulmonary regurgitation

O Ventricular rupture

A 54-year-old man is 48 h post-myocardial infarction. You are asked to review him as he is suffering worsening

cardiac failure. On examination he has a pansystolic murmur, loudest at the apex.

Which one of the following complications of his myocardial infarction is most likely to have occurred?
Ventricular septal defect
Atrial septal defect
Acute mitral regurgitation CORRECT ANSWER

Acute pulmonary regurgitation


Ventricular rupture

YOUR ANSWER WAS INCORRECT

Comment on this Question

The Answer

Acute mitral regurgitation


Acute mitral regurgitation associated with myocardial infarction may occur due to ruptured chordae tendineae
Other causes of mitral regurgitation include:
papillary muscle dysfunction
infective endocarditis
rheumatic heart disease
idiopathic myxomatous valve degeneration
left atrial myxoma
systemic lupus erythematosus (SLE)
drugs (fenfluramine and dexfenfluramine)
The investigation of choice is echocardiography, which may identify left atrial and left ventricular dilatation and
confirm the diagnosis of chordae tendineae rupture

Mitral regurgitation associated with chordal rupture in Ml may be catastrophic and require emergency surgery for
valve replacement
Acute medical management involves treatment with angiotensin-converting enzyme (ACE) inhibition, diuretic
therapy and possible anticoagulation

The prognosis for patients with mitral regurgitation is generally good, except in the post-myocardial infarction
situation

(Next question

i
Easy

tag this question

Dwdi(Hld
n<ii(.ii><Jtc.youi
lo

previous question

next question

back to menu

finish session

2014 PasTest Ltd. Egerton Court, Parkgate Estate, Knutsford, Cheshire, United Kingdom WA16 8DX. Tel: +44 (0) 1565 752000. Email: enq

A 21-year-old woman comes to the clinic for review, having returned early from a summer job as an assistant in a
diving school because of an episode of decompression sickness. Her only past history of note is persistent
migraine despite a range of medical therapies, and she takes no regular medications. Her BP is 120/80 mmHg,
pulse is 65/min and regular. There are no murmurs and her chest is clear. Her BMI is 21. Routine bloods are

unremarkable.

Which of the following is the most useful investigation to find the cause of her symptoms?

O Contrast echocardiography
O CT head
O CXR

O MRA cerebral circulation

q Transcranial doppler

A 21-year-old woman comes to the clinic for review, having returned early from a summer job as an assistant in a

diving school because of an episode of decompression sickness. Her only past history of note is persistent
migraine despite a range of medical therapies, and she takes no regular medications. Her BP is 120/80 mmHg,

pulse is 65/min and regular. There are no murmurs and her chest is clear. Her BMI is 21. Routine bloods are

unremarkable.

Which of the following is the most useful investigation to find the cause of her symptoms?
Contrast echocardiography CORRECT ANSWER

CT head
CXR

MRA cerebral circulation


Transcranial doppler
YOUR ANSWER WAS INCORRECT

The Answer

Comment on this Question

The answer is Contrast echocardiography

Persistent migraine is a recognised presentation of patent foramen ovale (PFO), the potential underlying
diagnosis here. PFO is also associated with increased risk of decompression sickness because of the
propensity for nitrogen bubbles to pass through the defect. After obtaining optimal views of the atrial septum
on transthoracic or transesophageal echocardiography, a bolus of agitated saline is injected into an

antecubital vein. Subsequently, microbubbles appear in the right atrium. The study is deemed positive for
PFO if the microbubbles appear in the left atrium within 3 cardiac cycles of their appearance in the right
atrium. Transcranial doppler can establish the presence of a shunt, but it does not establish the location of
the shunt.

owe

fMem&li

Ijh

AvjA H Mvira?>

%\JI

You might also like